Download as doc, pdf, or txt
Download as doc, pdf, or txt
You are on page 1of 235

Income Tax in Zimbabwe

Table of Contents
Income Tax in Zimbabwe....................................................................1
CONTENTS..............................................................................1
Abbreviations..........................................................................1
Abbreviations..........................................................................1
Chapter 1....................................................................................2
Introduction.................................................................................2
Synopsis.................................................................................2
Income Statement for the year ended 31 December 1999..........................4
Chapter 2....................................................................................5
Gross Income................................................................................5
Synopsis.................................................................................5
Chapter 3...................................................................................11
Source of Gross Income..................................................................11
Introduction..........................................................................11
Chapter 4...................................................................................21
Capital and Revenue Accruals...........................................................21
Chapter 5...................................................................................35
Items Specifically Included in Gross Income..........................................35
Chapter 6...................................................................................48
Exemptions.................................................................................48
Chapter 7...................................................................................52
Deductions: The General Formula......................................................52
Chapter 8...................................................................................70
Deductions: Specific......................................................................70
Chapter 9...................................................................................85
Deductions: Specific: Capital Allowances.............................................85
Chapter 10................................................................................102
Determination of Income Tax Payable...............................................102
Chapter 11................................................................................121
Farmers...................................................................................121
Chapter 12................................................................................133
Miners.....................................................................................133
Chapter 13................................................................................141
Insurance Companies....................................................................141
Chapter 14................................................................................146
Partnerships..............................................................................146
Chapter 15................................................................................150
Estates and Trusts.......................................................................150
Chapter 16................................................................................160
Pension and Benefit Funds:............................................................160
Contributions to and Amounts from..................................................160

1
Chapter 17................................................................................164
Tax Avoidance............................................................................164
Chapter 18................................................................................174
Sundry Technical Matters..............................................................174
Chapter 19................................................................................179
Withholding Taxes, etc.................................................................179
Chapter 20................................................................................187
Double Taxation Agreements..........................................................187
Chapter 21................................................................................194
Administration, Tax Collection, etc..................................................194
Table of cases
Indexes
Addendum

Abbreviations

AITR Australian Income Tax Reports


All ER All England Law Reports
Appellate Division South African unless otherwise stated
ATR Australian Tax Reports
CGTA Capital Gains Tax Act [Chapter 23:01] (Zimbabwe)
CIR Commissioner of Inland Revenue (South Africa unless otherwise stated)
CIT Collector of Income Tax, Botswana
CLR Commonwealth Law Review (Australia)
COT Commissioner of Taxes (Zimbabwe)
CTBR Commonwealth Taxation Board of Review (Australia)
EDA Estate Duty Act [Chapter 23:03] (Zimbabwe)
FA Finance Act [Chapter 23:04] (Zimbabwe)
FC Federal Supreme Court of the Federation of Rhodesia and Nyasaland
FC of T Federal Commissioner of Taxation (Australia)
Gazette Government Gazette (Zimbabwe)
GN Government Notice (Zimbabwe)
High Court Zimbabwe unless otherwise stated
HL House of Lords
IRC Inland Revenue Commissioners (United Kingdom)
ITA Income Tax Act [Chapter 23:06] (Zimbabwe)
ITC Income Tax Case (South African or Zimbabwean Special Court)
KBI As for CIR (South Africa)
Minister Minister of Finance, or nominee, Zimbabwe
SA South African Law Reports
SATC South African Tax Cases
SI Statutory Instrument (Zimbabwe)
SIR As for CIR (South Africa)
Special Court Special Court for Hearing Income Tax Appeals (Zimbabwe unless otherwise stated)
Supreme Court Supreme Court of Zimbabwe, except where otherwise stated
(T) Transvaal Provincial Division of the Supreme Court of South Africa
TC Tax Case Reports (United Kingdom)
TLR Times Law Reports (United Kingdom)

2
Chapter 1
Introduction
Synopsis
1.1 The Income Tax Act and its administration
1.2 The Capital Gains Tax Act
1.3 Legal precedents
1.4 Determination of taxable income
1.1 The Income Tax Act and its administration
Income tax is levied in terms of the Income Tax Act [Chapter 23:06]. In terms of section 7 of the Act the
rates of tax are fixed for each year of assessment, ending on 31 December, by the “charging Act”, namely
the Finance Act [Chapter 23:04] as amended by annual Finance Acts. In some years more than one of the
latter has been passed. Such annual Finance Acts are also the principal means by which amendments to the
Income Tax Act are effected. The term “Finance Act” or “principal Finance Act” used in this book refers to
Chapter 23:04, as amended.
The Income Tax Act is interpreted and administered by the Commissioner of Taxes (shortly to be by the
Commissioner-General, an official of the Zimbabwe Revenue Authority) whose office is in Harare. The
duties of obtaining returns from taxpayers and issuing assessments are carried out by Assistant
Commissioners at branches in Harare, Bulawayo, Gweru, Mutare and Masvingo. The collection of tax,
once it has been assessed, is the responsibility also of the latter, though the term “Collector” is used for
convenience in this book in such context. All employees in the Department of Taxes are subject to strict
secrecy with regard to information obtained in the course of their duties.
Policy matters relating to changes in the legislation rest with the Minister.
The Act deals with both technical and administrative matters and encompasses, despite its title, various
withholding and other taxes, certain of which are also dealt with in this book.
1.2 The Capital Gains Tax Act
Although most of the book is devoted to the question of liability arising under the various taxes imposed by
the Income Tax Act, capital gains tax is considered in chapter 22. The above comments regarding
administration apply.
1.3 Legal precedents
The Income Tax Act has its roots in the original Southern Rhodesian Ordinance of 1918, which was based
largely on the South African and United Kingdom Acts in force at that time. Although the statutes have, of
course, undergone substantial change, certain basic principles remain common. There are similarities also
between the Zimbabwe and Australian Acts.
Although the rulings of foreign courts are not binding on the Zimbabwe courts, the common ancestry of the
Acts encourages our courts to accord considerable persuasive value to judgments from those countries. The
South African courts tend to give similar weight to judgments issued by the Zimbabwe courts and this has
resulted in the published law reports of tax cases covering both countries. The majority of cases referred to
hereafter are those reported in volumes termed South African Tax Cases, containing judgments from the
courts of South Africa, Zimbabwe and other Southern African states. Where this is so, the originating court
is not always shown.
This book does not seek to list every relevant case. Sufficient cases are quoted to illustrate a principle or
clarify an interpretation, or to assist the reader in reaching a conclusion on facts with which he is concerned
as compared with those which were before the court. In choosing those which should be cited, leading
cases which have stood the test of time have, naturally, been included, but many of those heard in more
recent years are also cited as indicating current thought. Some are analysed in detail while brief references
to others are intended as a lead towards further research.
It is most desirable that students realise the importance of precedents. Legislation alone can never cover the
wide variety of circumstances which arise and, where a topic is so covered, there will always be
terminology requiring interpretation. Further, despite the increase in sophistication in the financial and
commercial fields, Zimbabwe’s tax legislation has remained relatively undeveloped in that it continues to
rely largely on general principles, particularly in areas such as the capital or revenue nature of receipts and
expenditure. It is largely for this reason that resort to case law becomes necessary on so many occasions.
There are a number of matters on which, some years ago, the Commissioner published his instructions to
assessors, constituting in some instances a recapitulation of the legislation and in others his application of

3
it. Although his instructions do not carry the force of law, and could be challenged in court, they are a
helpful guide to taxpayers. They are published as “departmental practices”, a term which will be used in the
following pages. Regrettably practices relating to income tax as may have been adopted in recent years
have not been published for general information, nor have any in relation to capital gains tax since its
introduction. (It is understood that this situation may shortly be rectified.)
1.4 Determination of taxable income
In Zimbabwe income tax is technically not levied on profits, as in some countries, but on taxable income as
defined in the Act. In general, the nature and the source of the income determine whether or not it is
taxable. The identity and country of residence of the recipient are often irrelevant.
Thus, generally, all receipts (not of a capital nature) arising from a source within Zimbabwe are taxed,
irrespective of the residential status of the taxpayer. Income arising from sources outside Zimbabwe is
taxable only if its source is “deemed” by the legislation to be within Zimbabwe.
Most items of expenditure may be deducted from income. Capital expenditure is, subject to limited
exceptions, not allowed as a deduction. The cost of most fixed assets, used for trade purposes, is deductible
only by way of annual allowances, designed to spread such cost over a period.
Almost without exception the rules for taxability and deductibility are the same for both individuals and
companies (including private business corporations incorporated under the Private Business Corporations
Act [Chapter 24:11]).
The method of ascertaining taxable income may be summarised as follows:
(a) determine the total gross receipts from all Zimbabwe and deemed Zimbabwe sources;
(b) eliminate receipts of a capital nature;
(c) eliminate income which is specifically exempted from tax;
(d) deduct permitted expenditure and allowances.
The early chapters of the book deal with the problems involved in ascertaining taxable income, in the same
order as the above summary, i.e. gross income, capital receipts, exempt income, and permitted deductions.
Accounting profit is, of course, arrived at in a similar way. For practical purposes, therefore, if accounts are
prepared, the accounting profit is used, instead of the gross income, as the starting point for determining
taxable income. When information is given in the form of an income statement the amount of taxable
income can be calculated more conveniently by adding to the net profit all disallowable expenses reflected
in such statement, and deducting therefrom any non-taxable items (for example most foreign and capital
receipts). Differences between the profit as reflected in the income statement and the eventual taxable
income could arise as follows.
(i) Factors causing the taxable income to exceed the accounting profit
(a) Fines and certain donations are not normally allowed as deductions for tax purposes, and should
be added back to the accounting profit.
(b) Depreciation of fixed assets is not a permitted deduction and should be replaced by the capital
allowances, if any, which may be claimable.
(c) Most expenses of a capital nature as may have been written off in the income statement should
also be added back.
(ii) Factors causing the accounting profit to exceed the taxable income
(a) To promote certain types of investment, particular allowances are granted. These take such forms
as accelerated allowances on machinery and are claimed in lieu of depreciation.
(b) The profit may include exempt accruals such as dividends from Zimbabwe companies.To illustrate
this common method of determining taxable income the following is an example of an individual small
trader. The technical aspects of the adjustments will be dealt with in later chapters.

Income Statement for the year ended 31 December 1999


Gross profit on trading 174 100
Bad debts (trade) recovered 200
Dividends on Zimbabwe shares 1 500
Interest on loans to other individuals 1 100
$176 900
Bad debts (trade) written off 725
Expenses (fully allowable)22 500
Fines 150
Insurance of trading stock 2 175

4
Interest on loan (used to purchase trading stock) 11 200
Interest on loan (used to purchase the shares) 1 900
Loss on sale of vehicle 5 000
Wages 57 200
Net profit 76 050
$176 900
Income tax computation
Accounting profit 76 050
Add
Interest on loan to purchase shares 1 900
Fines 150
Loss on sale of vehicle 5 000 7 050
83 100
Less
Dividends (exempt) 1 500
Taxable income $81 600

Notwithstanding the convenience of this short-cut, the fact is that the Income Tax Act technically rewrites
the accounts. As was said, in the context of the deductibility of expenditure, in one of the judgments in
Sub-Nigel Ltd v CIR (1948) 15 SATC 381 at 389:
“(T)he court is not concerned with deductions which may be considered proper from an accountant’s point
of view or from the point of view of a prudent trader, but merely with the deductions which are permissible
according to the language of the Act . . . . Regard must therefore be had to the Act and the Act alone in
order to ascertain whether the deductions sought to be made . . .  are permissible.”
Thus, while accounting principles are generally followed, in the event of a conflict the Act prevails.
Chapter 2
Gross Income
Synopsis
2.1 Definition of gross income
2.2 The total amount
2.3 Received . . . in any year of assessment
2.4 Accrued . . . in any year of assessment
2.5 Received or accrued
2.6 A person
2.7 Or deemed to have been received by or to have accrued to or in favour of a person
2.1 Definition of gross income
The starting point for the determination of the taxable income of a taxpayer is the definition of gross
income in section 8(1) of the Income Tax Act.
The definition states initially that the term means:
– the total amount;
– received by or accrued to or in favour of;
– a person;
– or deemed to have been received by or to have accrued to or in favour of a person;
– in any year of assessment;
– from a source within or deemed to be within Zimbabwe.
It then eliminates most receipts and accruals proved by the taxpayer to be of a capital nature, but identifies
those which are to fall within the definition regardless of whether they are of a capital nature or not.
Each of the above elements is discussed in this and succeeding chapters.
2.2 The total amount
Section 2 of the Act defines “amount” as money or any other property, corporeal or incorporeal, having an
ascertainable money value.
The calculation of gross income must begin with a review of the total amount which includes non-
monetary items having an ascertainable money value. Probably the commonest examples of the latter are
those received by an employee from his employer. These are among the topics dealt with in chapter 5.

5
An exchange of one item for another, however, constitutes a realisation: Royal Insurance Co. Ltd v Stephen
(1928) 14 TC 22 and Westminster Bank Ltd v Osler (1933) 17 TC 381. It is considered that, in the case of
items held on revenue account, this results in the accrual of an amount and that liability would arise on the
difference between the value of the item received and the cost of the item exchanged for it.
2.3 Received . . . in any year of assessment
(a) A person will have received an amount even if it is paid not to him personally but to his agent, or,
for example, if it is banked on his behalf.
A person does not “receive” money which, for example, he has stolen: COT v G (1981) 43 SATC
159. Nor does he “receive” in the sense used in the Act, money which has been lent to him: CIR v Genn &
Co. (Pty) Ltd (1955) 20 SATC 113.
In ITC 1545 (1992) 54 SATC 464 the taxpayer was nevertheless held to have “received” amounts
from sales under a scheme which, because it constituted a lottery, meant that transactions were void ab
initio. (He made profits also from the purchase and sale of stolen diamonds. Since he was aware of the fact
that they were stolen his conduct virtually amounted to theft. It was common cause, however, that, unlike
the position in G’s case, the proceeds of his sales constituted “receipts”.)
A similar finding as to the meaning of “received” occurred in ITC 1624 (1997) 59 SATC 373.
The taxpayer, an agent, was entitled to recover various disbursements which it (a South African close
corporation) had made on behalf of principals. It defrauded one by claiming and recovering an amount in
excess of the amount disbursed. This was reflected in its financial statements for the year as income but a
deduction was also claimed in recognition of its obligation to restore what it had unlawfully taken. The
court held that since the amount in question had been paid to the taxpayer as part of its business receipts it
indeed formed part of its business income. (No deduction was allowed, however, having regard to the
inexplicable inaction of the principal, and the fact that at the time the latter had not discovered the fraud or
given any intimation of an intention to institute proceedings.)
(b) There have been a number of cases which have determined liability on an amount which has been
received, as distinct from one which has accrued. These have usually been concerned with deposits.
In Brookes Lemos Ltd v CIR (1945) 14 SATC 295, it was held that deposits, charged by the company on
glass containers of products supplied to customers, formed part of the company’s gross income. The
following is an extract from one of the judgments:
“(The company) contended that the deposits were not receipts because amounts which are received as
security or amounts which are held in trust for someone or for some purpose are not ‘received’, in the sense
in which that word is used in the definition. If such amounts are really received as trust moneys, of which
the recipient is not the beneficial owner but merely a trustee, then doubtless (the company’s) contention is
correct, but that position does not exist here.”
In ITC 675 (1949) 16 SATC 238, the court held that, since deposits received from prospective purchasers
of day-old chicks to be delivered in the future were not refundable at the instance of the depositor, such
deposits represented income in the hands of the seller.
This contrasts with C v COT (1984) 46 SATC 57. The company, which operated a petrol station,
was obliged to pay cash for its bulk supplies. It accordingly instituted a system whereby it required those
of its regular customers, who wished to continue to be supplied on monthly credit, to deposit with it a sum
equal to one-twelfth of their estimated annual purchases. The deposits were paid into the company’s bank
account and utilised by it. They were not reflected in customers’ monthly statements or utilised against
their monthly purchases but were credited to respective customers in the company’s books. The deposits
were held not to form part of the company’s income. They were more in the nature of loans than of
payments received in advance for fuel to be supplied.
2.4 Accrued . . . in any year of assessment
As will appear below, while many disputes have centred on the question of whether an accrual has
taken place in one year as opposed to another, others have concerned the question of whether there has
been an accrual to the particular taxpayer.
(a) In terms of section 10(7) an amount is deemed to have accrued in the year of assessment in which
the taxpayer becomes entitled to it, despite its being only due and payable to him in a future year.
Nevertheless, in the case of sales of property, both movable and immovable, the deeming provision is made
subject to sections 17 and 18 of the Act. Accordingly profits on hire-purchase and instalment-credit sales
are spread, for tax purposes, over the years in which the instalments become due and payable by the
purchaser (see 8.29). Foreign exchange profits also continue to be deferred (see 5.18). There are also many

6
common instances in practice where the Commissioner continues to accept accruals on, for example, a day-
by-day basis.
The impact of section 10(7) is thus on income from activities other than the above. Where,
however, a taxpayer’s entitlement to an amount remains conditional at a year-end it is submitted, on the
basis of the principle in Mooi v SIR (1971) 34 SATC 1, that there is no accrual in that year.
(b) Various other circumstances in which the meaning of “accrued” has been examined, and where the
findings are, it is submitted, valid, notwithstanding the provisions of section 10(7), are as follows.
In Building Contractors v COT (1941) 12 SATC 182, the appellants had contracted (to construct buildings)
subject to 10% of the contract price being retained by the customer for three months pending the remedying
of any faults which might develop. It was held that the amount retained accrued to the contractor only after
the expiry of that period. The case thus constitutes authority regarding the year of taxability of retention
monies which, commonly, are subject to such a clause. The principle is the same as that in Mooi’s case,
referred to above.
The principle was applied again in ITC 1557 (1993) 55 SATC 218, where it was held that bus-fare
subsidies, receipt of which by a transport operator was subject to approval of an audit certificate, accrued
only when such condition had been fulfilled.
A distinction from these cases was drawn, however, in ITC 1645 (1999) 61 SATC 31. There the
agreements of sale of the taxpayer’s products contained a clause granting a rebate if payment was received
within a stipulated period. It was thus only at the latter point that the customer became unconditionally
entitled to the rebate. The taxpayer was held to be liable in the year of assessment on the total amount
invoiced and not to be entitled to deduct an estimate of rebates payable in the following year in the event of
prompt settlement.
The case of ‘A’ v COT (1988) 50 SATC 159 concerned a subsidiary company which supplied goods to its
parent which then retailed them. Previously supplies had been effected on a consignment basis but for
accounting reasons the system was changed with the effect, the court found, that the company was selling
to its parent at cost price plus 25%. At the end of the year the company granted an allowance in recognition
of the mark-up on goods unsold by the parent at that date (the relevant entry later being reversed as the
goods would be retailed in the following year). The court found that the full price, including the mark-up
on the unsold goods, had nevertheless accrued to the company and was unaffected by the entry of the
allowance in its books.
The result of the above was that, from the point of view of the group as a whole, income tax liability arose
on goods which had not yet been sold. This had not been so under the earlier consignment relationship.
In ITC 1634 (1998) 60 SATC 235 the taxpayer company carried on business as a shipping,
clearing and forwarding agent and as manager of terminals and container transport. In invoicing its
customers for its services it incorporated disbursements which it was entitled to recover from them.
Occasionally the suppliers of the corresponding facilities or services to the company failed to issue it with
an invoice. Under the company’s standard accounting procedures the latter amounts, as remained
outstanding after two years, were transferred from suspense account to profit. The court held that these
amounts constituted an accrual or, in any event, a recoupment.
In C : SARS v Cape Consumers(Pty) Ltd (1999) 61 SATC 91 the operations of a buy-aid organisation, in
the form of the company, generated a surplus being the difference between what its members paid to it and
what it paid to suppliers of goods. Such surplus, after deduction of the organisation’s operational expenses,
was distributed to the buyers/members. Other amounts were earned by way of discounts and investment
income which were credited to a buyers’ reserve fund. The court found that the surplus had been held in
trust for the buyers/members and that there had been no accrual to the company.
In ITC 1669 (1999) 61 SATC 479 three corporate bodies which were processors of raw hides had set up a
company – the taxpayer – to undertake the operations. The company invoiced each of the three parties, for
the tanning of each batch of hides supplied by them, at rates fixed by it. Since the intention had been that
the company should not make a profit it then granted rebates which it reflected in its financial accounts as
deductions arising from the purchase of materials. The court held that the Commissioner had been correct
in including the invoiced amounts in the taxpayer’s gross income; the position would have been different
had the invoices been raised on a provisional basis and been subject to revision when the processing costs
were known. (It was found also that there was no legal obligation on the company to repay any part of the
costs paid by the three parties for the processing of hides.)
See also ITC 1618 (1997) 59 SATC 290 and ITC 1670 (2000) 62 SATC 34, considered at 17.4.

7
(c) If income has accrued to a person, the method of assessment cannot be influenced by what
happens to the income after accrual. Thus, if income has accrued, it is taxable even if it has been given
away or returned. This situation was held to prevail in ITC 1346 (1982) 44 SATC 31, in circumstances
where an employee, who resigned during a period of study leave, was obliged to repay an amount of salary
which had already accrued.
In Cactus Investments (Pty) Ltd v CIR (1999) 61 SATC 43 the taxpayer company had devised a scheme
with a view to making interest-bearing investments without attracting liability to income tax. The system
used was that taxable interest was exchanged for non-taxable dividends by way of cession and counter-
cession. The question was whether the interest had been ceded before it had accrued, the agreements
relating to the deposits, etc., having provided for accrual at the end of each relevant period.
The court held that common law principles had to be applied; unless the parties otherwise agree a lender of
money becomes entitled to the right to receive interest, despite it being receivable on a stipulated future
date, as soon as he has made the loan funds available to the borrower. Thus a stipulation in an agreement
that interest would accrue on a date after the date on which it had legally accrued had no effect; the right to
claim interest accrued to the company on the days on which the investments were made and was not
affected by the subsequent cessions.
This South African decision is in line with the provisions of section 10(7) of the Zimbabwe Act, to
which reference is made above.
There is, however, a distinction between the disposal of income after it has accrued and the
disposal of a right to income which has not accrued. This is clearly shown in a number of instances,
including the leading case of Hiddingh v CIR (1941) 11 SATC 205, and Taxpayer v COT Botswana
(1980) 43 SATC 118.
In the latter the facts were that an individual, upon assuming salaried employment, ceded to his employer
all future director’s fees thereafter becoming payable to him by the employer. It was held that, the taxpayer
having ceded his future rights, there was no accrual to him and that he was not taxable on the fees. There
was a similar result in ITC 1415 (1986) 48 SATC 179. The taxpayer was a minister of religion whose
salary was payable by his church on the 17th of each month. To augment his congregation’s funds he
periodically renounced part of his salary, written notice of his renunciation being given to the cashier
before the 17th. The court found that, because the taxpayer had no right to claim any money before the
17th, and because renunciation had always been effected prior to that date, no accrual had arisen in relation
to the amounts in dispute. In ITC 1378 (1983) 45 SATC 230, concerning the cession of the right to
dividends, it was held that a cession of a right even for a limited period was valid.
It is essential, however, that all steps be taken to create a valid cession of a right. Any failure to
meet these requirements may result in a finding that income remains taxable in the original hands, as
occurred in ITC 1411 (1986) 48 SATC 35.
Further, if a cession fell within those provisions of the Act which counter tax avoidance, the
Commissioner could set it aside or take alternative action: see chapter 17.
A cession which is not of the right to the income, but merely as security for a debt, similarly will
not operate to relieve the original taxpayer from liability: Moodie v CIR, Transkei, and Another; CIR,
Transkei, and Another v Moodie and Another (1993) 55 SATC 164.
2.5 Received or accrued
Where gross income accrues to a taxpayer in one year of assessment, but is received by him in another, the
above words, subject to what is said below, give the Commissioner the right to levy tax in either year. This
was recognised in principle in CIR v Delfos (1933) 6 SATC 92, though there is, as observed there, “a
‘necessary implication’ that the same amount shall not be taxed twice in the hands of the same taxpayer”.
In practice the Commissioner adopts the accruals basis, but there have been exceptions where his
imposition of liability on the year of receipt has been upheld, as in Maguire v COT (1966) 28 SATC 146
and G v COT (1972) 34 SATC 62.
On the basis, however, of views expressed in SIR v Silverglen Investments (Pty) Ltd (1968) 30 SATC 199,
the Commissioner does not have an unrestricted power to choose the year of assessment. It is submitted
that where accruals are disclosed he is bound to use that basis, in which event the receipts basis will arise
only in cases of earlier non-disclosure of accrual, as in the cases of Maguire and, possibly, G.
2.6 A person
“Person” is defined in section 2 as including:
– a company;
– a body of persons corporate or unincorporate (not being a partnership);

8
– a local or like authority;
– a deceased or insolvent estate; and
– in relation to income the subject of a trust to which no beneficiary is entitled, the trust.
This is in addition to its ordinary meaning of an individual.
There is no residential qualification in this definition. Every individual wherever resident, every company
wherever registered, and every estate and trust wherever formed, falls within the ambit of the Act. Whether
they will be assessed depends, of course, on whether they have taxable income from a source within or
deemed to be within Zimbabwe. A person and his nominee are deemed to be one person (section 2(2)).
Although the point is not part of the definition it is observed here that a company in liquidation is also a
(potentially taxable) person: Rhodesia Metals Ltd (in liquidation) v COT (1940) 11 SATC 244, referred to
more recently in van Zyl NO v CIR (1997) 59 SATC 105 at 113.
2.7 Or deemed to have been received by or to have accrued to or in favour of a person
2.7.1 Income not paid over (s 10(1))
Income is deemed to have accrued to a person despite its:
– having been invested, accumulated or otherwise capitalised by him; or
– having not been actually paid over to him, but remaining due and payable to him; or
– having been (merely) credited to an account or reinvested or accumulated or capitalised or
otherwise dealt with in his name or on his behalf.
This ensures that income which is either being withheld from a taxpayer (such as in the case of trusts: see
chapter 15) or invested or accumulated on his behalf, remains potentially taxable in his hands. Since such
income would in any event already have constituted an actual accrual to a taxpayer, reinforced by the
“entitlement” test referred to at 2.4, this deemed accrual provision appears to achieve little.
2.7.2 Partnership income (s 10(2))
The date of accrual of a partner’s profits, and other aspects of the taxation of partnerships, are considered in
chapter 14.
2.7.3 Income of a taxpayer’s minor child (s 10(3))
(a) Income accruing to a minor child from a donation, settlement or other disposition made by one of
his parents is taxable in that parent’s hands, even though the income flowing from the donation is
accumulated or expended for the benefit of the child. The provision applies only while the child is a minor,
i.e. under 18 years of age and unmarried. When the child becomes of age, any subsequent income flowing
from the donation, etc., is taxable in the child’s hands.
(See also, however, 21.5.4.)
As to the meaning of “donation”, etc., see Ovenstone v SIR (1980) 42 SATC 55.
(b) If a minor receives income in his own right, such as wages for services rendered, he, and not his
parent, is taxable on such income. If a minor genuinely renders services to his parent, who pays him
appropriate wages, such wages are taxable in the minor’s hands.
(c) A question which arises is whether the section applies only to income from the donation and not to
income earned from the use of that income. In CIR v Widan (1954) 19 SATC 341 it was held, contrary to
the decision in Kohler v CIR (1949) 16 SATC 312, that income on income continued to be taxable in the
hands of the donor parent.
Both of these cases concerned investment income which was reinvested. It is submitted that, if the
child used the original investment income to carry on, for example, business operations, the income from
the latter would be taxable in his hands, since the link with his parent’s donation would have been broken.
A further aspect of the scope of the (equivalent South African) section arose in C : SARS v
Woolidge (2000) 62 SATC 1. There the taxpayer had sold shares to two discretionary inter vivos trusts of
which he, his minor children and other family members were beneficiaries. Payment of the purchase price
was subject to agreement between the taxpayer and the trustees and, although it was within the taxpayer’s
discretion to charge interest on the outstanding price, he had not done so. The court found that the sale did
not fall within the terms “donation”, etc., but that the taxpayer’s failure to charge interest made him liable
under the section.
2.7.4 Minor child’s income flowing from donations made reciprocally (s 10(4))
The section would, among other things, counter tax avoidance schemes by means of which taxpayers
otherwise circumvent the provisions of section 10(3). It provides, briefly, that if:
(i) a child becomes entitled to income in pursuance of a donation, etc., made by a third party,
i.e. a person other than his parent; and

9
(ii) the parent or near relative of the child has made a donation to that third party (or his near
relative)
the child’s income will be taxable in the hands of the parent.
(See also, however, 21.5.4.)
2.7.5 Income postponed by stipulation of maker of donation, settlement or other disposition (s 10(5))
Tax avoidance is prevented in the circumstances specified. These are where a person makes a donation,
etc., (commonly to a trust) for the purpose of divesting himself of the right to the income from the donated
assets but at the same time withholding such income from the beneficiaries until the happening of some
event.
The withheld income is deemed to remain that of the donor if:
(a) he (or a near relative) has power to control the ultimate devolution of the income; or
(b) any of the funds or income could devolve on or be lent to the donor (or his spouse, or one of their
deceased estates, or a company controlled by any of them).
The “event” referred to may be fixed or contingent. It may be a specified date, a marriage, a death, the
attainment of a certain age or, on the authority of Hulett v CIR (1944) 13 SATC 58 and ITC 1033 (1959)
26 SATC 73, the exercise by a trustee of a discretionary power to pay income to a beneficiary.
Freely distributable income is not affected.
The provisions are important in the context of trusts where the trustee is granted a discretion, or where
distribution must await the happening of some other event. A donor could find himself remaining taxable
on income despite having divested himself of the assets on which it accrues. (See also, however, 21.5.4.)
It appears that these rules apply even where a tax avoidance motive is absent. The above is the situation
only during the donor’s lifetime. In the case of a trust continuing after his death the position is considered
in chapter 15.
2.7.6 Income accruing under rights/powers retained by maker of donation, settlement or other
disposition (s 10(6))
If a donor reserves to himself the right to confer the income from a donation, settlement or other disposition
on some other person, he is taxable on the income arising from the donation, etc., for so long as he retains
that power.
As in the case of section 10(5), considered above, trusts constitute the area which, during the donor’s
lifetime, is the most likely to be affected by the provisions. Again they appear to apply equally in the
absence of a tax avoidance motive.
(See also, however, 21.5.4.)
Chapter 3
Source of Gross Income
Synopsis
3.1 From a source within Zimbabwe
3.2 Or deemed to be within Zimbabwe
Introduction
The terms of the definition of “gross income” are such that, as indicated earlier, only amounts from a
source within or deemed to be within Zimbabwe fall under it. No other amounts may be taxed. Zimbabwe’s
two-pronged basis means that, while its taxing power generally rests on the income being from a true
source in Zimbabwe, foreign-sourced income also may attract liability but only if it falls within the specific
provisions relating to deemed source.
3.1 From a source within Zimbabwe
3.1.1 General
The Act contains no definition of “source” and guidance must therefore be sought from the large number of
cases in which the meaning has been considered.
There is general acceptance that, as a matter of principle, the test laid down by Watermeyer, CJ, in CIR v
Lever Bros and Unilever Ltd (1946) 14 SATC 1 at 8, is authoritative, namely that:
“ .... the source of receipts, received as income, is not the quarter whence they come, but the originating
cause of their being received as income and that this originating cause is the work which the taxpayer does
to earn them, the quid pro quo which he gives in return for which he receives them.”
The judgment continued:

10
“The work which he does may be a business which he carries on, or an enterprise which he undertakes, or
an activity in which he engages and it may take the form of personal exertion, mental or physical, or it may
take the form of employment of capital either by using it to earn income or by letting its use to someone
else. Often the work is some combination of these.”
It follows that, just as “the quarter whence they come” is irrelevant, so too is the matter of whether receipts
are remitted to Zimbabwe.
Reference to a number of decided cases is made in the following pages. It is dangerous to generalise with
regard to source and each case turns on its particular facts. The decisions nevertheless are practical
illustrations of the workings of what could otherwise be an abstract principle.
It is repeated that Zimbabwe’s taxing power may be expanded by the deeming provisions of the Act; it may
on the other hand, as will be seen later, be restricted under a double taxation agreement.
Various types of income are now considered.
3.1.2 Annuities
The source of an annuity is the place where the formal act giving rise to the annuity is performed,
commonly the conclusion of a contract or the creation of a trust through the execution of a will. In the case
of a contractual annuity the source is the place where the contract is concluded: Boyd v CIR (1951) 17
SATC 366. The same goes for a will trust, the source of the annuity being the place where the will is
executed and regardless of the original source of the trust’s income from which the annuity is paid: ITC
826 (1956) 21 SATC 189 and COT v R (1966) 28 SATC 115. The principle which emerges in the latter
circumstances is that the creation of the trust is the originating cause of the annuity.
See also 3.2.8 regarding deemed source.
3.1.3 Business operations: sales
(a) There will usually be little difficulty in concluding that where business operations are carried on in
a particular country, and the taxpayer’s capital is employed there, such country will constitute the source of
the taxpayer’s profits. A taxpayer’s activities, which are the originating cause of a receipt, do not always,
however, all occur in the same place. The question may arise also of the country in which the contract was
concluded. Problems such as these are examined in the following cases.
Overseas Trust Corporation Ltd v CIR (1926) 2 SATC 71
A finance and investment company carried on the business in South Africa of buying and selling shares. A
parcel of shares was sold through brokers in Germany. The court held that the source of the profit on the
latter was South Africa, the grounds being that the company did not carry on business in Germany or
employ any of its capital there. Profit had been earned on the capital paid for the shares and such capital
had been employed in South Africa. According to the court: “Had the company carried on part of its
business in Germany by buying and selling shares there the position would have been different, but it
carried on no business in Germany ...”.
CIR v Black (1957) 21 SATC 226
A stockbroker who carried on business in Johannesburg, where he lived, earned profits from sharedealing
in London. The court found that there was a distinct business of buying and selling shares in London and
that accordingly the profit on such transactions was not from a source in South Africa. The contrast may be
seen between these facts and those in the Overseas Trust Corporation case.
M Ltd v COT (1958) 22 SATC 27
A company carried on copper-mining operations in the then Federation of Rhodesia and Nyasaland, where
it was managed and controlled. Proceeds from sales of exported copper arose in London. Sums surplus to
immediate requirements were employed from time to time by the company’s financial agent there in the
purchase of short-term United Kingdom Government securities for resale at a profit. It was held that the
source of the profit on sales of securities was London, on the grounds that the activities conducted there
constituted the originating cause of such receipts. It was observed that, where a taxpayer company has
funds surplus to the requirements of its main business, and uses them temporarily in a profit-making
scheme distinct from that business, it is carrying on a separate activity, and that, if such activity is
conducted entirely at a place outside the location of the main business, the profits are located at that place.
CIR v Epstein (1954) 19 SATC 221
The taxpayer carried on business in South Africa as an agent for foreign firms and was associated with a
partnership which carried on business in Argentina. For his services in procuring asbestos in South Africa,
for the partnership, the taxpayer received a share of the profits on the sale of the asbestos in Argentina by
the partnership. The court applied the “activities” test and held that, because all of the taxpayer’s activities,

11
which resulted in his receiving a share of the profits, were carried on in South Africa, such share was
derived from a source in South Africa.
Essential Sterolin Products (Pty) Ltd v CIR (1993) 55 SATC 357
The company, having developed a medicine to a particular stage in South Africa, derived consideration for
the granting, outside South Africa, of a sub-licence under which, in the event of the taxpayer being unable
to manufacture to a further stage outside South Africa, another party was entitled to do so. The
consideration was held not to be from a South African source.
The same conclusion was reached in ITC 1585 (1995) 57 SATC 81, in which the taxpayer company, which
had processed mineral concentrates in South Africa on behalf of the owners, earned commissions on further
work on the materials performed overseas by other parties.
(b) The cases referred to in (a) above concerned circumstances in which the courts pin-pointed one
country, as opposed to another, as the source of the income.
It appears from ITC 1103 (1967) 29 SATC 35, however, that it is possible for the source of
income to be found partly in one country and partly in another.
ITC 1103 was concerned with actual source, in the context of cross-border operations. The taxpayer
company, which manufactured flour in Zimbabwe, sold some through a branch in Zambia, transfer being
effected at manufacturing cost plus railage. The Commissioner determined the company’s taxable income
as 10% of the factory cost of the flour supplied. The court set aside this determination as being irregular
since the company had not been called upon to submit proposals. The matter was accordingly remitted to
the Commissioner for reassessment.
Despite such an inconclusive result, which arose from a failure to follow proper procedures in
terms of section 19, referred to below, the court’s findings on the matter of principle were valuable,
including the following observations, based on various authorities, at 40-41:
“I ... conclude ... that some profit at least ... derived from the sale of goods manufactured in one country and
sold by the same taxpayer in another country, accrues from sources in the country of manufacture.”:
“ .... some part of the accruals derived from the sale of flour (in Zambia) had its originating cause in the
company’s productive activities (in Zimbabwe)”:
“ .... I consider that the business (in Zambia) was responsible for part of the overall profit.”:
“ .... the Commissioner is entitled to ascertain and determine what part of the accruals (in Zambia)
represented taxable income derived from manufacturing and associated activities (in Zimbabwe) and so
from sources (in Zimbabwe).”
This case fell outside the scope of section 23(2) (considered below) since the company was
resident in Zimbabwe. It indicates nevertheless that while, in these circumstances, the Act does not
specifically authorise an apportionment of profit between countries, allocations of profit, which may well
amount to apportionment under another name, may arise in the application of the concept of source (and in
the procedural application of section 19).
This approach conflicts with that adopted at about the same time in Transvaal Associated Hide and
Skin Merchants v COT Botswana (1967) 29 SATC 97, which was decided on the criterion that the sole
source was the country in which the dominant activity was considered to have taken place. Some resolution
of the difficulty may be found in Zimbabwe’s expanding network of double taxation agreements, in which
the concept of the attribution of profits commonly involves apportionment ( see 20.2.1(a)).
(c) Section 19 covers any trade of a person (other than insurance business) which “extends to a
country other than Zimbabwe and the Commissioner is satisfied that it is impossible or impracticable to
ascertain the taxable income derived by such person from sources in Zimbabwe in the manner otherwise
provided in this Act....”. Such person is required to submit proposals for the determination of his taxable
income in some alternative manner.
If the Commissioner finds such proposals unacceptable, or if none are submitted, he is empowered
to determine the taxable income, or assessed loss.
(d) The operations of non-residents are covered by section 23(2). This is in fact a “deemed source”
section and although for the most part such topic is dealt with at 3.2 below, it is, in the case of business
profits, convenient to consider it here. The section relates to any non-resident person who “produces,
grows, mines, creates, manufactures, fabricates, improves, packs, preserves or constructs in whole or in
part, anything within Zimbabwe, and exports the same without (prior sale)”. He is “deemed to have derived
from a source within Zimbabwe a taxable income corresponding to the proportionate part of any profit
ultimately derived from the sale or disposal thereof outside Zimbabwe”. The provisions of section 19 then
apply for procedural purposes.

12
The case of Mining and Manufacturing Co. v COT (1944) 13 SATC 146 concerned a company which was
registered and resident in South Africa and which mined asbestos in Zimbabwe. The mineral extracted was
reduced to a form known as cobbed asbestos. This was railed to the company’s factory in South Africa
where it was used in the manufacture of asbestos cement for sale. The Zimbabwean asbestos represented
25% of the total used at the factory, and asbestos as such formed 20% in bulk of the content of the cement.
The Commissioner taxed the company on a proportion of the profit from the sale of its finished products,
based on the ratio of the Zimbabwean raw materials content. The court held, however, that regard could not
be had to the finished products as these stemmed from the South African manufacturing operations. The
Zimbabwean taxable income had to be determined by reference to the fair market value of the cobbed
asbestos exported from Zimbabwe, less the cost of its production.
The decision throws light on the application of section 23(2) which specifically authorises
apportionment.
(e) Where a trader merely purchases goods in one country, without conducting any other activity
there, and sells in another, the question of the source of his profit is, in the absence of a clear precedent, one
of some difficulty. The greater body of opinion, however, appears to favour the country of sale. This
submission rests on remarks, albeit obiter, in separate judgments in the Transvaal Associated Hide case,
quoting from Epstein’s case; both of these are referred to above. On this assumption a trader operating a
business of purchasing and selling goods in Zimbabwe, and selling also through a branch in another
country, would not be liable in Zimbabwe on profits from the latter. Exceptions may arise under the
concept of deemed source (see 3.2.2). Further, such trader would be liable in Zimbabwe if his foreign sales
were effected other than through a branch; he would then be in a situation of trading with the other country
as opposed to trading in it.
3.1.4 Directors’ fees
The courts have consistently treated the services of a director, in his capacity as such, as having been
rendered at the head office of the company. If, therefore, the head office of a company is in Zimbabwe, the
fees are derived from a Zimbabwe source: ITC 106 (1927) 3 SATC 336 and ITC 250 (1932) 7 SATC 46.
If, however, a director renders services to his company in a capacity other than that of a director, earning
for example a salary, the source of such receipts is determined in accordance with the rules for services
generally, which are considered at 3.1.11.
Directors’ fees from a company whose head office is outside Zimbabwe may still be liable here by reason
of being deemed to be from a source within Zimbabwe: see 3.2.4.
3.1.5 Dividends
The source of dividends from a company is the country in which such company is incorporated and its
register of shareholders is situated: Boyd v CIR (1951) 17 SATC 366. The fact that part of the profits of the
company paying the dividend may have been from a source in another country is irrelevant.
In the case of a branch register, a dividend will still be from a source in the country in which the paying
company is incorporated, and the principal register situated, if, in terms of the law of the country of
incorporation, the branch register is deemed to be part of the principal register: Lamb v CIR (1955) 20
SATC 1.
See also 3.2.7 regarding deemed source.
3.1.6 Immovable property: sales
The cases of Liquidator, Rhodesia Metals Ltd v COT (1938) 9 SATC 363 and Rhodesia Metals Ltd (in
liquidation) v COT (1940) 11 SATC 244 concerned the source of the profit made on the sale by the
company of its principal asset which consisted of mining claims in Zimbabwe. The company was registered
and resident in England where it had its board of directors, organisation, connections and equipment. The
court held that the source of the profit was Zimbabwe. Regard was had to the fact that the sole business
operation of the company had been the purchase of the immovable property and that the company had
never ventured any part of its capital except on such property. The claims had been acquired and developed
in Zimbabwe for the very purpose of obtaining the particular receipt.
3.1.7 Interest
The leading authority is the Lever Bros case, to which reference was made at the beginning of this chapter.
There it was held that the originating cause of interest accruing from the lending of money is not the loan
but the services rendered by the lender to the borrower, i.e. the supply of credit, for which the borrower
pays interest. In that case, although the borrower was resident in South Africa, and used the borrowed funds
in and paid the interest from that country, the funds were made available, and thus the creditor’s services
rendered, outside South Africa and the agreements were entered into outside South Africa. It was held that

13
the interest did not accrue from a source within South Africa. (The court, having reached this conclusion,
did not have to decide the positive source.)
It appears, in the light of the above, that the question of the source of interest may turn on an otherwise
insignificant factor; that if a foreign investor transfers loan funds to Zimbabwe the source of his interest
will be this country whereas if he supplies the credit outside, leaving the borrower here to effect transfer,
the interest will be from a non-Zimbabwe source.
The approach in the Lever Bros case differs from that in the earlier case of COT v William Dunn and Co.
Ltd (1918) 32 SATC 33, because the circumstances were different. William Dunn was not concerned with
a loan account. The company was registered and carried on business in England, where it purchased goods
and whence it invoiced and shipped them to South African customers. The company also charged mora
interest on outstanding proceeds. It was held (as in Lever Bros) that the interest did not accrue from a
source in South Africa, but the test applied in this case was that, accepting that the interest had been earned
through the employment of the company’s capital, such capital had been employed in its business and such
business had been conducted in England.
See also 3.2.7 regarding deemed source.
3.1.8 Partnership profits
See chapter 13.
3.1.9 Rentals
A distinction must firstly be drawn between immovable and movable property.In the case of immovable
property, such as a building, it is submitted that the source of rentals receivable will always be the country
in which the building is situated. It is that country in which the owner of the building has employed his
capital, and the originating cause of his income is his making available the use in that country of such
building.
In the case of movables the rule may vary according to the circumstances. It is likely that in most instances
the source of rentals will be the country in which the owner of the hired assets carries on his business.
Thus, where a person operating a car-hire business in Zimbabwe enters into a contract under which a
vehicle is used for a limited period outside the country, his income from such contract will still be from a
source in Zimbabwe.
There are instances, however, where the country where the asset is used by the customer may constitute
the source of the rentals. Special circumstances obtained in, for example, COT v British United Shoe
Machinery (SA) (Pty) Ltd (1964) 26 SATC 163. The company which was registered, managed and
controlled in South Africa, entered into a lease agreement under which it leased out equipment in the
former Federation of Rhodesia and Nyasaland, to be used in a footwear undertaking. The court held that the
source of the rentals was situated in the Federation as it was the equipment, and not the capital invested
therein, which generated the income, and that the source was thus where the equipment was productively
employed. It was made clear, however, that the case turned on its own facts and that significant features
included the nature of the property and the existence of long leases (varying between five and ten years).
3.1.10 Royalties
In Millin v CIR (1928) 3 SATC 170 the facts were that Mrs Millin wrote books in South Africa which were
published in England under contracts negotiated there. The court held that, since the exercise of her wits
and labour had produced the royalties, and since these had been exercised in South Africa, the source of the
royalties was South Africa. The contention that the source was London, where her copyright had been
employed, was rejected.
It is submitted that the same principle will apply to income from patent rights and similar accruals to
inventors.
There appears to be no conclusive guide to source where royalties are derived by a person who is not the
original author or inventor as where, for example, another person acquires the rights by cession.
The rule in Millin’s case is of no application in the case of royalties receivable in respect of, for example,
the extraction of minerals or the use of patent rights. The decision in the Rhodesia Metals cases, referred to
at 3.1.6 would be likely to apply.
As regards patents, etc., see also 3.2.9 regarding deemed source.
3.1.11 Income from services rendered
(a) In the case of services rendered in the course of employment the originating cause of the income is
the services, regardless of the place where the contract is concluded, or payment made. The source is
located at the place where the services are rendered. This has been so held in a long line of Special Court
cases and in COT v Shein (1958) (FC) 22 SATC 12.

14
Thus in the case of an individual from another country, who comes to work in Zimbabwe as an
employee, the source of his remuneration will be Zimbabwe notwithstanding that such remuneration, or
part of it, is receivable outside this country. Zimbabwe would, however, probably not be the source if the
services here were merely occasional or casual.
Where an individual is employed to work specifically in businesses both inside and outside
Zimbabwe his remuneration would have to be apportioned, probably on a time basis: ITC 77 (1927) 3
SATC 72. The terms of the contract of service may thus become crucial.
(b) In the case of services rendered in the course of carrying on a business or profession the courts
have, it is submitted, approached the matter along lines similar to those in “business-type” circumstances
such as those referred to at 3.1.3(a) above.
In ITC 1102 (1967) 29 SATC 28, for example, a Zimbabwean lessor of equipment for social and other
functions entered into contracts for functions outside Zimbabwe. The taxpayer was required both to supply
equipment in the usual way and also to render additional services such as catering. The court held that the
latter contracts constituted a separate and distinct business activity in the other country where the source
was, accordingly, held to be located. (It appears that the conclusion regarding non-Zimbabwe source would
have been the same even in the absence of the additional services.)
There was a similar outcome in ITC 1104 (1967) 29 SATC 46. A commercial diver working in the
Zambezi river at Kariba took his equipment from his Harare base to the Zambian bank of the river and set
up a small temporary office there. Diving operations were performed on both sides of the international
border. Only profits from operations on the Zimbabwe side, calculated on a time-apportioned basis, were
held to be from a source in this country.
A different result arose in ITC 1127 (1969) 31 SATC 114. A Harare-based film-producing company carried
out contracts for the making of films which involved a very substantial proportion of work on location
outside the country. The source of the fees earned was found to be Zimbabwe; the originating cause was the
performance of the contract to produce the film and not (despite the acknowledged necessity of the work)
the performance of a contract to do certain filming elsewhere. (The latter principle has been widely applied
by the courts in instances where work performed outside a particular country has been characterised as
merely preliminary or preparatory to the performance of a contract inside the country.)
(c) See also 3.2.3 and 3.2.4 regarding deemed source.
3.1.12 Shares: sales
The cases of Overseas Trust Corporation Ltd v CIR (1926) 2 SATC 71 and CIR v Black (1957) 21 SATC
226, and also, in relation to profits on the sale of Government securities, M Ltd v COT (1958) 22 SATC 27,
have been considered at 3.1.3.
The Zimbabwe case of D v COT, unreported but referred to in ITC 1395 (1985) 47 SATC 123, at 129,
concerned the source of profit on shares in South African companies. An individual conducted speculative
activities in a portfolio comprising both Zimbabwe and South African shares. He and his wife lived in
Zimbabwe where they took all the decisions affecting the shares, usually in consultation with a local
stockbroker. It was found that the taxpayer’s general activity of investment-dealing was, for practical
purposes, carried on in this country. Although transactions affecting the South African shares took place in
South Africa these transactions were essentially linked to the overall enterprise and were an integral and
inseparable part of it. There was, in short, only one business. The source of the profit was held to be
Zimbabwe.
3.1.13 Income from trade marks, “know-how”, etc.
It appears that the true source is the country where the recipient carries on the activity, or employs the
assets, giving rise to such income.
In United Aircraft Corporation v FC of T, 2 AITR 458, an American manufacturer of aircraft engines
permitted an Australian company to manufacture and sell engines in Australia, using designs which were
the property of the American company. Amounts received by the latter were held not to be from a source in
Australia; the company carried on no business operations and had no industrial or other property there.
(See, however, 3.2.9 regarding deemed source.)
In different circumstances, in ITC 1491 (1991) 53 SATC 115 the taxpayer company, based in South Africa,
carried on business as a supplier, to various franchise licence holders, of material in kit form, equipment
and expertise in the reconditioning of household baths and other sanitary ware.
The company, in expanding its activities, had established an office in London and concluded there an
agreement under which it sold franchise rights to operate in the United Kingdom a business utilising its
formulae, etc., and its name.

15
The court found that the taxpayer carried on business in the United Kingdom and, apparently as a separate
matter of principle, that there is no difference between the letting of movable assets and the grant of a right
to use a trade mark and secret processes, the latter constituting the use of capital in the form of incorporeal
property. The source of the income, being the activity of the company, or the property over which it had
rights, or a combination of the two, was thus situate in the United Kingdom.

(See also Essential Sterolin Products (Pty) Ltd v CIR (1993) 55 SATC 357.)
3.2 Or deemed to be within Zimbabwe
3.2.1 General
As indicated earlier, some of the “deemed source” provisions bring to account amounts which, because
their true source is outside Zimbabwe, would otherwise not be liable. Other such provisions ensure that
liability arises on amounts the true source of which is doubtful.
As in the case of true Zimbabwe source it must be remembered (see 3.1.1) that Zimbabwe’s taxing power
may be restricted under a double taxation agreement.
3.2.2 Business operations: sales of goods (s 12(1)(a))
An amount accruing under any contract made in Zimbabwe for the sale of goods is deemed to be from a
source in this country. This applies regardless of whether the goods have been or are to be delivered inside
or outside Zimbabwe.
The effect of this is that a trader who, for example, conducts his business entirely outside this country, with
the result that the true source of his profit is outside Zimbabwe, will be deemed nevertheless to have
derived income from a source in Zimbabwe if it arises from a contract, for the sale of goods, made in this
country. Such circumstances are, of course, rare.
3.2.3 Income from services rendered (s 12(1)(b))
(a) Receipts for any services rendered in the carrying on in Zimbabwe of any trade, irrespective of
where or by whom payment is made, are deemed to be from a source in Zimbabwe. “Trade” is defined in
section 2(1) as including, among other things, any profession, activity or occupation.
As will probably be appreciated, once a taxpayer is found to have received an amount for services
rendered in the carrying on in Zimbabwe of a trade, it is possible that such amount is from a true source in
Zimbabwe (see 3.1.11(b)). The Commissioner then has no need to rely on the deemed source provisions of
section 12(1)(b). Liability in Zimbabwe would have arisen in any event. Alternatively the Commissioner
may fail to establish Zimbabwe as the true source, yet succeed in establishing it as the deemed source.
In ITC 1102 and ITC 1104, referred to at 3.1.11(b) (concerning hiring and diving operations
respectively) the “Zambian” fees which were the subject of the dispute were found neither to be from a true
source, nor deemed to be from a source, in Zimbabwe. By contrast in ITC 1127, concerning film
production, the Commissioner succeeded under both tests.
Other instances where the Commissioner has relied on the deemed source test only, and has
succeeded, include the following
ITC 56 (1926) 2 SATC 178. An accountant, practising in South Africa, performed audit work outside the
country. The latter services were held to have been performed in the carrying on of his trade within South
Africa. The result would, it seems clear, have been different had he maintained a branch office in the other
country.
ITC 1105 (1967) 29 SATC 116. A taxpayer, carrying on business as an insurance assessor in Zimbabwe,
carried out loss investigations in Zambia and Malawi, embodying his findings in reports prepared in his
Harare office. His fees from the investigations were found to have been earned in the carrying on of his
trade in Zimbabwe.
The Commissioner failed, however, in ITC 1585 (1995) 57 SATC 81: see 3.1.3(a). The court
observed, at 89-90, that the corresponding section of the South African Act “.... was intended to apply
where a taxpayer has a trade within the Republic and where the taxpayer renders services or does work or
labour outside the Republic as part of that trade, and was not intended to apply in the situation where the
taxpayer has two distinct businesses, one within the Republic and one outside the Republic.”
The approach of the courts has been to ascertain whether there is what has been characterised as
the “necessary close link” between the disputed services and the country whose authorities are seeking to
tax the income; only when that link exists can it be said that the services, despite having been performed
outside, were performed in the course of carrying on a trade within that country.
Once such a link is established, lengthy absence from Zimbabwe does not assist the taxpayer
since, unlike in section 12(1)(c), considered below, there is no reference to a time limit.

16
(b) The above examples refer either to companies or to self-employed individuals. In view of the wide
definition of “trade”, however, instances have arisen where the authorities have sought to apply the
provisions of section 12(1)(b) to an employee. Despite the deletion of the term “employment” from the
definition, and the substitution of “activity”, it appears that decisions such as the following would continue
to apply.
In ITC 1130 (1969) 31 SATC 148, and ITC 1330 (1981) 43 SATC 65, salesmen based in Harare and
travelling to neighbouring countries to effect sales were held to be taxable in Zimbabwe on their entire
earnings. There was the necessary close link between their earnings for services rendered outside, and the
carrying on of their trade (of employment) inside, this country.
In ITC 1088 (1966) 28 SATC 202, however, in which the managing director of a holding company in
Zimbabwe visited subsidiary companies in Zambia and Malawi in his capacity as chairman or managing
director of those companies, the section was held to be inapplicable.
The case of K v COT (1993) 55 SATC 276 concerned earnings of the taxpayer and his wife respectively
(all of which, since the case arose prior to the introduction of separate taxation, fell to be considered in his
hands). They were chartered accountants employed by a firm practising in Zimbabwe and were selected to
participate in England in an international programme which was intended to benefit both firms and staff.
The court found that the husband’s salary from the Zimbabwean firm was deemed to have accrued to him
from a source in this country in terms of section 12(1)(b). There was the necessary close link between the
work which he did outside Zimbabwe and the carrying on of his “trade” within Zimbabwe; he participated
in the programme in order to further his career. In the case of his wife it had transpired that she was
ineligible for the programme. Though she had accompanied her husband she was employed on non-
auditing work by the programme’s co-ordinating body. It was found that there was no such close link and
that section 12(1)(b) was inapplicable.
3.2.4 Services rendered by an employee during a temporary absence from Zimbabwe (s 12(1)(c))
In addition to section 12(1)(b), a person who is ordinarily resident in Zimbabwe, and who receives
remuneration for services rendered outside as an employee, is deemed to have received the income from a
source within Zimbabwe, unless his total absences from the country exceed 183 days during the year of
assessment.
If the time limit is satisfied, liability under this provision is mandatory for an employee and, unlike the
position under section 12(1)(b), does not depend on a link with services rendered in Zimbabwe. Further, it
applies equally to services rendered to Zimbabwean and foreign employers.
Since a director is by definition included in the term “employee”, fees earned for attending meetings in
other countries are taxable in the hands of a director who is ordinarily resident in this country.
Where an employee derives additional income from work performed outside his terms of employment, and
in the absence of control or supervision, it may be possible for him to establish, as in ITC 1174 (1972) 34
SATC 135, that the amount accrues to him in a capacity other than that of an employee, and that it
therefore falls outside the terms of section 12(1)(c). It might still, of course, fall within section 12(1)(b).
3.2.5 Services rendered to the Zimbabwe Government (s 12(l)(d))
Remuneration is deemed to be from a Zimbabwe source regardless of where the services are rendered
unless the person is not ordinarily resident in Zimbabwe. The same rule applies to Government pensions.
3.2.6 Services rendered: pensions and annuities (s 12(l)(e))
A deemed Zimbabwe source again arises in respect of any pension or annuity for services rendered by the
taxpayer within or deemed to be within Zimbabwe, regardless of whether he has remained ordinarily
resident in Zimbabwe at the time of receipt.
A (non-Government) pension or annuity is apportioned in the ratio of the services performed within and
outside Zimbabwe. This applies, however, only where the pension or annuity is received by the person
who rendered the services. Thus a widow receiving a widow’s pension from a non-Zimbabwe fund would
not be taxable in Zimbabwe since she would not have rendered any services in this country.
3.2.7 Interest and dividends (s 12(2))
If a person is ordinarily resident in Zimbabwe at the time interest or dividends accrue to him on foreign
securities (the term “securities” being widely defined in section 2(1)) such income is deemed to be
from a Zimbabwe source. (The section extends also to interest or dividends which, if they were from a
Zimbabwe source, would be deemed to accrue to the taxpayer by virtue of section 10 of the Act.)
Foreign dividends attract a separate rate of tax in the hands of the recipients: see chapter 10.
With regard to a non-resident recipient of interest, potential liability to Zimbabwe income tax will arise
only if this country is the true source (see 3.1.7). There is no provision deeming a Zimbabwe source on the

17
grounds that, for example, the borrower is ordinarily resident here. This contrasts with the position for
withholding tax: see 19.4.
3.2.8 Annuities (s 12(3))
A foreign-sourced annuity purchased by a person who, at the time of purchase, was ordinarily resident in
Zimbabwe, is deemed to be from a source in this country.
Annuities by way of donations, inheritances, etc., thus fall outside this provision. Only if their true source is
in Zimbabwe (see 3.1.2) do they constitute “gross income”.
3.2.9 Income from trade marks, “know-how”, etc. (s 12(4))
The section deems a Zimbabwe source in relation to any receipt for the use in Zimbabwe of: (a) any patent,
design, trade mark, copyright, model, plan, secret process or formula, etc.; or (b) any motion picture film or
television film, etc.
This relates to: (i) a grant of permission to use; and (ii) related “know-how”; and applies regardless of
where the patent, etc., was produced, or the “know-how” imparted, or where payment is to be made. The
provision would, in the latter context, overcome the decision in the United Aircraft Corporation case
referred to at 3.1.13.
The deeming of a source to be in Zimbabwe has effect, of course, only in relation to an amount, commonly
a royalty, which is otherwise gross income. A receipt of a capital nature, such as for the permanent
surrender of patent rights or trade marks, or, as considered at 4.14, of “know-how”, would not fall within
the provision. The important words above, in this context, are “for the use ... of”.
3.2.10 Recoupments (s 12(5))
Any amounts recovered outside Zimbabwe, which otherwise fall within section 8(1) ( i) or (j), are deemed
to be from a source in Zimbabwe. Thus a sale outside Zimbabwe of an asset, such as a vehicle, on which
capital allowances had been granted, could give rise to a taxable recoupment.
3.2.11 Business operations: sales (s 23(2))
See 3.1.3.
Chapter 4
Capital and Revenue Accruals
Synopsis
Excluding any amount of a capital nature
4.1 Introduction
4.2 Basic principles
4.3 Original intention
4.4 Change of intention
4.5 Mixed intentions
4.6 Fixed and floating capital
4.7 Damages and compensation
4.8 Restraint of trade
4.9 Insurance proceeds
4.10 Gambling and hobbies
4.11 Isolated transactions
4.12 Inheritances, gifts and prizes
4.13 Sale of goodwill
4.14 Proceeds of “know-how”
4.15 Exchange rates: profits and losses
4.16 Options
4.17 Illegal trade
4.1 Introduction
The area of tax law in which probably the largest number of disputes has occurred is the distinction
between those receipts and accruals which are income and those which are capital. The latter are excluded
from “gross income”, as defined in section 8 of the Income Tax Act, unless they are specifically included
by further provisions of the Act: see chapter 5. The matter is accordingly considered in some detail below.
A further aspect of the definition is that it requires that, for an amount to be excluded, it must be “proved by
the taxpayer” (see 21.4.2) to be of a capital nature.

18
In view of the provisions of the Capital Gains Tax Act [Chapter 23:01] (see chapter 22) the classification of
a particular receipt or accrual as capital and not revenue does not mean that it will necessarily be tax free. It
will be free of income tax but, if the receipt or accrual results in a capital gain falling within the scope of
the latter Act, the gain will attract capital gains tax. Almost invariably proof of the capital nature of a
receipt should be advantageous.
4.2 Basic principles
(a) The term “receipts of a capital nature” is not defined in the Income Tax Act. As to its
interpretation there are many decided cases, usually concerning sales of assets (tangible and intangible)
since amounts arising from the use of assets are almost invariably of a revenue nature. In this connection it
was said in ITC 1543 (1992) 54 SATC 446 at 448: “Whether a particular receipt or accrual is of a capital
nature or is income must be determined in the light of the facts and circumstances of the particular case.
There is no single infallible test for settling that question.”
(b) It is submitted, however, that in the context of purchases and sales of assets the single most
important factor in any approach to the distinction between capital and revenue receipts is the question of
the taxpayer’s intention.
Where, for example, a person purchases fixed property, if the intention at the time of purchase was
to re-sell at a profit when a suitable opportunity arose, the proceeds of the sale will, prima facie, be taxable.
If the original intention was to achieve an enhanced continuing income, the proceeds of an eventual sale
will, prima facie, be regarded as a capital receipt arising from the disposal of a fixed asset. Other factors
may affect the issue but, as was said in Elandsheuwel Farming (Edms) Bpk v SBI (1978) 39 SATC 163 at
181:
“In the determination of the question into which of these two classes a particular transaction falls, the
intention of the taxpayer, both at the time of acquiring the asset and at the time of its sale, is of great, and
sometimes decisive, importance ....”.
It follows that, subject to what is said at 4.12, fortuitous receipts and accruals, such as those by
way of gift, inheritance or prize, in respect of which no question of the recipient’s intention arises, will
generally be of a capital nature. The same goes for receipts from hobbies (see 4.10) since the profit-making
intention would have been absent. (Further considerations arise if the donee or heir, etc., incorporates the
asset received into a trade conducted by him. Probably the commonest instances occur in the case of
farmers. These are examined at 11.2(j).)
It follows also, in view of the subjective approach to be adopted, that the capital or revenue nature
of a receipt can never be determined by reference to the capital or revenue nature of the corresponding
expenditure in the hands of the payer.
Intention, or purpose, must not, however, be taken to be the same thing as contemplation. Profit on
resale of, for example, a picture which had been purchased with the intention of resale at a profit would
clearly attract liability. This would not be so, however, had the picture been purchased for private
enjoyment, i.e. despite the purchaser contemplating that if resale should occur at a future date a profit might
result. This approach has been followed in a number of cases including COT v Levy (SR) (1952) 18
SATC 127 at 133 and ITC 1608 (1997) 59 SATC 63 at 67-8. (See also the comments at 4.5 regarding
mixed intentions.)
(c) In virtually every instance where the courts have had to consider whether an accrual is of a
revenue or capital nature judgment has been given in relation to the character of the amount in toto.
Apportionment was, however, in relation to a receipt in restraint of trade, considered appropriate in Tuck v
CIR (1988) 50 SATC 98. This concept is relatively new and it remains to be seen in what contexts the
courts will apply it. Indeed in ITC 1616 (1997) 59 SATC 272 at 280 it was stated, in relation to a
sale of property, that:
“Profits can only be of a revenue or capital nature ….”.
(d) The question may turn on the taxpayer’s original intention, or he may have had a change of
intention or may have acquired with mixed intentions. Each of these is now considered.
4.3 Original intention
4.3.1  The practical problem arises of how one is to determine the intention with which an item was
acquired. As was said in COT v Hepker (1933) 6 SATC 87 at 91:
“The question here is whether the appellant bought this land as an investment or with a view to selling it at
a profit and the answer depends upon what his intention was when he bought it. What his intention was
seems to me to be entirely a question of fact.”
Another summary of the approach is in AB v COT(SR) (1921) 1 SATC 77 at 78:

19
“His objects must be ascertained from expressed intention and reasonable inferences from his acts.”
Such regard, both to the taxpayer’s statement of his intentions, and to the general probabilities and
surrounding circumstances, is indicated also in ITC 1185 (1972) 35 SATC 122 at 123-4 and ITC 1509
(1992) 54 SATC 18 at 26-28.
4.3.2  A phrase which is often used in the cases is “in pursuance of a scheme of profit-making” as in
Overseas Trust Corporation Ltd v CIR (1926) 2 SATC 71 at 75:
“Where an asset was realised at a profit as a mere change of investment there was no difference in character
between the amount of enhancement and the balance of the proceeds. But where the profit was, in the
words of an eminent Scottish judge ‘.... a gain made by an operation of business in carrying out a scheme
for profit making’ then it was revenue derived from capital productively employed, and must be income.”
In CIR v George Forest Timber Co Ltd (1924) 1 SATC 20 the company was a dealer in timber. It had
purchased a forest together with the ground on which it stood, felled the trees and converted them into
timber. It employed its capital in its business of dealing in timber to produce a profit. The proceeds of the
sale of such timber were held to be income. A direct contrast arose in ITC 1494 (1991) 53 SATC 206. The
farmer in that case had cut and sold timber from a natural plantation of yellow-wood trees on his land. He
established that the sales had been necessitated by the occurrence of wastage and rotting of the trees, which
were not self-renewing and that there was no scheme of profit making. The proceeds were found to be of a
capital nature.
In African Life Investment Corporation (Pty) Ltd v CIR (1969) 31 SATC 163 at 174, Steyn CJ observed as
follows, quoting from the judgment in a similar case in a CTBR report:
“Having regard to the nature, the objects and the operations of the company, I am satisfied that the
continual and substantial dealing in investments was an essential feature of its business. Of necessity it had
to deal in shares and to that end some of its officers constantly watched the share market and, on their
advice and that of outside experts, securities were varied whenever it was considered to be to the advantage
of the company to vary them. As part of its business operations the company bought and sold shares in a
wide variety of companies ... I consider that the profits under consideration in this case were made in the
normal course of the company’s operations and, in consequence, are assessable income ...”
(In the case of an individual, aspects such as large-scale share-owning, and a close watch on and
assiduous “farming” of the portfolio, similarly resulted in taxability of profits, in CIR v Nussbaum (1996)
58 SATC 283.)
Another useful approach to the “scheme of profit making” test is that, as stated in CIR v Pick ’n Pay
Employee Share Purchase Trust (1992) 54 SATC 271 at 280, it “really means that receipts or accruals bear
the imprint of revenue if they are not fortuitous but designedly sought for and worked for”.
4.3.3  There is of course a wide variety of factors which have been taken by the courts as indicative of a
taxpayer’s intention.
(a) Acceptance of an unsolicited offer to buy property not actually on the market (as in Constantia
Heights v SIR (1979) 41 SATC 77 at 95) or disposing of it because of expropriation, will (if it was not
already held as trading stock) be a strong indication that the property was originally acquired to be held as
an investment and has continued to be so held. If so, any profit derived from the disposal will not be subject
to income tax.
If the asset has been in the taxpayer’s possession for only a short period of time and continuous
efforts are made to sell, this will be a strong indication that the asset has been acquired with the intention of
disposing of it at a profit, as in ITC 1432 (1988) 50 SATC 70 and ITC 1482 (1990) 52 SATC 298, both of
which concerned profits on sales of shares. In ITC 1277 (1978) 40 SATC 204 at 208 the court, referring to
a short time-factor, said: “In the absence of an acceptable explanation it could be a point of decisive
significance.”
(b) In ITC 1194 (1973) 35 SATC 227 an investment company disposed of shares because a
disproportionate part of its portfolio was in one concern; this was held to justify a finding that the profit
was capital.
The taxpayer in ITC 1391 (1985) 47 SATC 66, however, failed to establish that a sale of shares in a mining
company was effected in order to balance a portfolio of capital investments. On the facts the taxpayer’s
intention at the time of purchase was found to have been profitable resale.
In CIR v Tod (1983) 45 SATC 1, the argument that sales of shares were effected in order to balance the
portfolio was accepted in respect of some sales but others were held to be on revenue account.
(c) The method of financing the purchase of an asset can also indicate the intention associated with a
transaction. If a short-term loan is used for purchasing an asset, rather than the taxpayer using his own

20
funds, it may be an indication that the asset was acquired for speculative purposes. As was said in
Strathmore Holdings (Pty) Ltd v CIR (1959) 22 SATC 203:
“The appellant was handicapped by a lack of capital from the outset. The fact that it had to resort to large-
scale borrowing of money might have been a vital consideration if the appellant had possessed revenue-
producing assets out of which to pay the interest on its loans. The fact that it had no revenue-producing
assets might not have been important either if it had not had to pay a substantial amount of interest
annually. Taken in conjunction, however, these two factors alone are sufficient to warrant the inference that
the appellant acquired the...shares with a view to a profitable resale and not with the object of holding them
as an investment.”
Shortage of funds similarly undermined the taxpayers’ contentions regarding profits on sales of
property in ITC 1541 (1992) 54 SATC 408 and ITC 1597 (1996) 58 SATC 27. Unforeseen liquidity
problems arising during the period of ownership will, however, not cause a sale to be adjudged to be part of
a profit-making scheme if the other elements pointing to a capital nature are present: Berea Park Avenue
Properties (Pty) Ltd v CIR (1995) 57 SATC 167.
(d) There are also other reasons why, where a profitable resale occurs shortly after acquisition, it may
still be possible for the seller to prove that the asset was acquired as an investment and not speculatively.
An individual taxpayer might, for example, sell due to an unforeseen change in family or (as in ITC
1580 (1994) 56 SATC 275) financial circumstances. See also CIR v Middelman (1990) 52 SATC 323.
(e) Possession of an asset for a long time, although it may be an indication of intention, will not in
itself be sufficient to outweigh an initial and continuing intention to make a profit by reselling the asset. A
profit-making intention was found in, for example, T v COT (1978) 40 SATC 179, in which the period was
11 years, and in Durban North Traders Ltd v CIR (1956) 21 SATC 85 (18 years) and ITC 862 (1958) 22
SATC 301 (over 50 years).
(f) As indicated earlier, each transaction is looked at individually. As was stated in T v COT (1978)
40 SATC 179 at 183:
“An investment-holding company may engage in a scheme of profit making in respect of particular assets
just as an investment-dealing company may hold and own capital assets.”
Where a subsidiary company is an acknowledged dealer in shares it is still possible for the holding
company to establish that its own periodic sales are on capital account: ITC 1354 (1982) 44 SATC 118.
In ITC 1638 (1998) 60 SATC 423 the taxpayer, a sixty-year-old builder and property developer,
had erected blocks of maisonettes, the rentals from which formed his sole source of income. His stated
purpose was to provide a form of pension for his old age. His involvement in further property development
forced him to sell certain of the maisonettes in order to comply with funding requirements. The profit on
sale was accepted as being of a capital nature.
In ITC 1509 (1992) 54 SATC 18 an individual with a long history of buying and selling shares on
the Johannesburg Stock Exchange established that a particular purchase had been made as a long-term
investment and that its sale, which had yielded substantial profit, had been caused by its intended retention
being frustrated by circumstances. The court held that it would be incorrect to adopt the approach of “once
a share-dealer always a share-dealer”.
The same principle was adopted in Berea Park Avenue Properties (Pty) Ltd v CIR (1995) 57
SATC 167 in relation to property sales, where it was observed that a land-jobber may have other property
as an investment which is therefore not part of his trading stock. It also did not follow that all the business
affairs of (in this case) the taxpayer-company’s shareholders were interrelated and interdependent.
(g) Shares may be found to have been purchased on capital account if they were acquired with a view
to the furtherance of other business operations of the taxpayer. In CIR v Nedbank Ltd (1986) 48 SATC 73,
the bank had purchased shares in a corporation with a view to securing the banking business of the latter
and others. When this did not materialise it sold the shares. The court held that the resultant profit was of a
capital nature.
(h) Where an asset is acquired as trading stock the profit on resale is likely to be taxable even if the
sale arises from intervening factors.
This accords with the principle stated in CIR v Niko (1940) 11 SATC 124. The stock had originally been
acquired with the intention of resale and this intention was thus carried out, despite the sale having been
effected after the cessation of business. In ITC 1350 (1982) 44 SATC 54, a township development
company found itself prevented by legislation from proceeding with the development of a portion of its
land. It was held still to be taxable on the profit despite the resale having been effected in an involuntary
manner. The above result arose in ITC 1547 (1993) 55 SATC 19 in relation to profit on expropriation of

21
land held by a property developer, and in ITC 1659 (1999) 61 SATC 239 in relation to profit on resale of
land effected after a development (and resale) scheme had failed to come to fruition.
This situation must, however, be distinguished from that which arises in other instances of factors
intervening, such as those giving rise to a change of intention: see 4.4.
(i) The importance of establishing the character of the original intention may be seen also from the
facts in Matla Coal Ltd v CIR (1986) 48 SATC 223.
The company possessed coal rights which were to be exploited through the earning of royalties (which
would, of course, have been taxable). It instead sold its rights, to the same party, for an amount representing
the computed present value of the royalties which would otherwise have been earned. The consideration
remained of a capital nature because the company proved that the rights had been held by it as income-
producing capital assets.
4.3.4  The above cases concern disposals of either shares or property, since these comprise the commonest
areas of investment and speculation. The same principles apply, however, to other assets, such as coins,
metals and precious stones. In South Africa, in the latter context, sales of Krugerrands have resulted in a
series of cases coming before the courts.
In one of the more recent of these, CIR v Nel (1997) 59 SATC 349, the taxpayer was able to convince the
court that his acquisition of the coins had been made with the intention of retaining them, for possible
inheritance by his children, and that his sale (of 80 out of 250) arose from unforeseen personal
circumstances.
In a number of other instances, however, South African taxpayers have been unsuccessful in appeals in
relation to profits on such sales. So also were an auctioneer, in relation to diamond sales, in ITC 1608
(1997) 59 SATC 63, and in the United Kingdom, a professional actor, in Wisdom v Chamberlain (1968) 45
TC 92, in relation to sales of silver purchased as a hedge against devaluation.
A feature of this area of tax law is that assets such as coins frequently have no economic utility and are
income-producing assets only through sale. Thus despite the taxpayer’s success in Nel’s case this must
surely remain a vulnerable area for sellers generally. Nevertheless it appears that the “economic non-
utility” argument gives way to the familiar one of intention.
4.3.5  Further circumstances arose in Realisation Co v COT (1950) 17 SATC 139. There it was held that, if
a person would have been able to dispose of certain assets at a profit without attracting tax (i.e. the profit
would have been of a capital nature in his particular circumstances) a company formed for the sole purpose
of acquiring those assets and realising them could also do so without attracting tax. This Zimbabwe case
was successfully relied on by the appellants in a South African case (ITC 1244 (1976) 38 SATC 7) in
which the court distinguished the situation from that where a company’s business was to deal in shares at a
profit. The Appellate Division also recognised the concept of a “realisation company” in SIR v Struben
Minerals (Pty) Ltd (1966) 28 SATC 248, and again in Berea West Estates (Pty) Ltd v SIR (1976) 38 SATC
43, but in the latter, at 62, there was a qualification:
“It is, of course, clear that if a realisation company, or any other company, so conducts its affairs that it can
be said to be carrying on the trade or business of making profits from the sale of the land, using the latter as
its stock-in-trade, the profits will be `revenue derived from capital productively employed’, and taxable.”.
The concept was again followed, more recently, in ITC 1481 (1990) 52 SATC 285.
4.3.6  In addition to what has been said on intention there are, in the case of a company, several matters
over and above the usual test which must be taken into account. For instance in Lace Proprietary Mines Ltd
v CIR (1938) 9 SATC 349 the following aspects were considered relevant to the enquiry as to whether the
company’s sale was effected in pursuance of a pre-existing scheme of profit making: (i) the name of the
company; (ii) its objects; (iii) its activities; (iv) its policy; (v) the circumstances of acquisition of the asset
in question; and (vi) the circumstances of the realisation.
With regard specifically to the objects of a company the clause in its Memorandum of Association should
be treated with caution. The court in CIR v Strathmore Exploration and Management Ltd (1956) 20 SATC
375 at 388 quoted with approval the following extract from the judgment in the Australian case of
Ruhamah Property Company Ltd v FC of T, 41 CLR 148:
“In our opinion the authorities show that the objects and powers of the company contained in its
memorandum ... are not decisive of the question whether the sale was an operation in carrying out a scheme
of profit making, but that a consideration of all matters advanced by the company was relevant to a
determination of that question.”
In Natal Estates Ltd v SIR (1975) 37 SATC 193, to which more detailed reference is made below, there
was confirmation at 215 that, where a company has among its objects both dealing and holding, a profit

22
made on the realisation of assets will not necessarily be income. The court referred to “the well-known
practice in South Africa of framing objects clauses in very wide terms”.
A company’s intention is in any event formed by the persons who are in effective control of the company:
Constantia Heights (Pty) Ltd v SIR (1979) 41 SATC 77 and ITC 1543 (1992) 54 SATC 446.
4.4 Change of intention
(a) The original intention of the taxpayer is not necessarily the end of the matter. It is not uncommon
to change one’s intention and, if this has occurred, any profit, which could otherwise have been capital,
may be held to be of a revenue nature. As will appear below, this requires something more than simply a
change of heart, resulting in a decision to sell, and indeed something more than taking steps to sell to best
advantage. The danger to the taxpayer lies in going too far.
As was observed in John Bell & Co (Pty) Ltd v SIR (1976) 38 SATC 87 at 103:
“... my understanding of the cases that deal with the matter is that a mere change of intention to dispose of
an asset hitherto held as capital does not per se subject the resultant profit to tax. Something more is
required in order to metamorphose the character of the asset and so render its proceeds gross income. For
example, the taxpayer must already be trading in the same or similar kinds of assets, or he then and there
starts some trade or business or embarks on some scheme for selling such assets for profit and, in either
case, the asset in question is taken into or used as his stock-in-trade.”
(b) A number of decisions are illustrative of the doctrine of realising to best advantage.
An early case is CIR v Stott (1928) 3 SATC 253 where the taxpayer, wishing to acquire a particular piece
of land, had to comply with the seller’s requirements that he take a larger area. The taxpayer subdivided the
surplus into plots which he resold. His profit was held to be of a capital nature. The same principle was
applied in CIR v Paul (1956) 21 SATC 1.
This approach has been followed more recently, as in ITC 1348 (1982) 44 SATC 46, where a company,
having owned a block of 30 flats for some years, decided to sell, having regard to the poor returns by way
of rentals. The sales were effected by sectional title. Again the profit was found to be of a capital nature.
The same result arose in Berea Park Avenue Properties (Pty) Ltd v CIR (1995) 57 SATC 167, where it was
observed that, while in many cases a sale by sectional title will indicate that a trade has been embarked
upon, this is not necessarily so. In ITC 1436 (1988) 50 SATC 122 the taxpayer was also successful, again
in relation to sales of flats, and even overcame the fact that he had originally had the building constructed
to sectional title requirements.
In ITC 1471 (1990) 52 SATC 96 the taxpayer was also successful. The case concerned a farmer on whose
land lay quantities of building sand. In response to approaches by building contractors the taxpayer
permitted them to remove sand, according to their requirements, at a price per cubic metre. The
Commissioner contended that, although the farm itself was a capital asset, there had been a change of
intention by the taxpayer with regard to the sand which had then become the subject of his trading
operations. The court rejected the latter contention, finding that the taxpayer’s actions were instead akin to
a disposal of his land; the sand was part of the land and was not its “fruit”.
In C: SARS v van Blerk (2000) 62 SATC 131, however, which similarly was concerned with sales of sand
from a farming property, the court considered, having regard to the manner in which the sales were
conducted on a regular basis over a number of years, that the taxpayer’s operations had the characteristics
of trading in the commodity. The proceeds were accordingly held to be of a revenue nature.
The facts in ITC 740 (1951) 18 SATC 219 were that a farmer was statutorily obliged to allow underground
water supplies, which were self-renewing, to be abstracted. Compensation per gallon was held to be
revenue.
In C: SARS v Knuth and Industrial Mouldings (Pty) Ltd (2000) 62 SATC 65 the individual concerned,
being the owner of 50% of the share capital of a company, wished to acquire the balance. Because he was
unable to raise the necessary finance by alternative means he, acting on advice, entered into an elaborate
scheme (producing other tax advantages and) involving the sale at a profit of his existing holding which
nevertheless reverted to him. The profit was held to be of a capital nature on the grounds that his
shareholding had been acquired as a capital asset and that its use in the scheme did not change such
character.
(c) A change of intention which went too far is illustrated in the leading case of Natal Estates Ltd
v SIR (1975) 37 SATC 193, where the following excerpt, from COT v Booysens Estates Ltd (1918) 32
SATC 10, was quoted with approval at 216:
“It is quite a well-settled principle in dealing with questions of income tax that, when the owner of an
ordinary investment chooses to realise it, and obtains a greater price than he originally acquired it at, the

23
enhanced price is not profit ... assessable to income tax. But it is equally well-established that enhanced
values obtained from realisation or conversion of securities may be so assessable where what is done is not
merely a realisation or change of investment, but an act done in what is truly the carrying on or carrying out
of business.”.
The Natal Estates judgment then deals with the tests needed to show whether what was, on the
face of it, an ordinary investment of surplus funds, had been converted into a profit-making business. It was
held to be important that there should be “proof of some special acts which in the ordinary experience of
men shows that the taxpayer has conceived some scheme for profit-making and has made it his business to
carry it out”.
The facts were that the company had originally acquired land for agricultural purposes and had for some
years conducted the business of growing sugar-cane. When it came to sell the land it engaged in the
“special acts” referred to, namely preparatory intensive planning and organisation, lavish expenditure in
creating a luxury township, and construction of certain houses to “set the tone”. On the totality of the facts,
it was found at 222 that the company “was doing much more than merely realising a capital asset to the
best advantage in a businesslike manner; and that by any canons of commerce, it had gone beyond that
field: it had crossed the Rubicon and committed itself on a grand scale to the course and business of selling
land for profit, using the land as its stock-in-trade.”.
Its profits were therefore taxable.
(The court raised but did not decide the question of the taxable or non-taxable character of that
part of the enhancement of value which had probably occurred before the change of intention. The point
was decided against the taxpayer, however, in Land Dealing Company v COT (1959) 22 SATC 310 where
it was held that it was not possible to separate such accretion.)
(d) It is possible for the converse to apply, i.e. for assets acquired on revenue account to be sold on
capital account.
In CIR v Richmond Estates (Pty) Ltd (1955) 20 SATC 355 the circumstances were that the taxpayer
company’s intention had changed in respect of certain properties which had originally been acquired for
resale. Because of a restriction by law on the acquisition of stands in areas including that in question, it was
decided no longer to retain the stands for resale, but to erect and let buildings thereon. Some three years
later, uncertainty about the effects of new legislation created fear of a possible decline in the value of the
properties. The company subsequently decided to sell its stands in the area. The Appellate Division
accepted that the relevant profits were of a capital nature, not subject to tax.
Similarly in CIR v Modified Investments (Pty) Ltd (1981) 43 SATC 257, a profit on shares which had been
held as trading stock during the lifetime of the company’s controlling shareholder, and which were sold by
the executors who had meanwhile converted the company to an investment-holding company, was held to
be of a capital nature.
(e) As regards a shift from capital to revenue account, the question has arisen as to the position of a
company the decisions of which are determined by the intentions of its major shareholders. That this may
vary is illustrated by the following.
In Elandsheuwel Farming (Edms) Bpk v SBI (1978) 39 SATC 163, a company, which had originally
acquired land as a capital asset, some years later came totally under the control of speculators who caused
the land to be sold at a profit. It was held, though only by a majority, that the change in shareholding
resulted in a change of policy whereby the land became held as trading stock, and that the profit was
taxable.
A different result arose in ITC 1406 (1986) 48 SATC 12. There again a company had acquired a property
as an investment. Ten years later a new shareholder acquired a 50% shareholding, his intention being that
the property be sold. The company’s profit on the sale effected after a further two years was held to be of a
capital nature (despite the then shareholders personally having earlier had taxable property deals). In CIR v
Malcomess Properties (Isando) (Pty) Ltd (1991) 53 SATC 153 the taxpayer company also was successful.
The company, having owned a property as a capital asset, decided to sell. Its lessee, with a view to ensuring
long-term tenure, devised a scheme whereby the company’s shares were owned by the lessee by the time
the sale was effected. The company’s profit on the sale of the property was held to remain of a capital
nature.
The converse circumstances may, of course, arise where substituted shareholders seek to establish
that a company’s asset which was originally acquired on trading account has, with their arrival, become the
subject of a change of intention. The contention that a subsequent disposal was on capital account may well

24
fail, as in Greenband Properties (Pty) Ltd v CIR (1981) 43 SATC 151, ITC 1507 (1991) 53 SATC 425 and
ITC 1541 (1992) 54 SATC 408.
The Greenband case is authority also for the view that, in determining whether property which
was acquired as trading stock has become a fixed capital asset, the principles are the same as in a converse
enquiry whether land acquired as capital has become trading stock.
4.5 Mixed intentions
A taxpayer may have had mixed intentions when purchasing an asset. In such a case the approach is to
establish the predominant factor which influenced the purchase. In COT (SR) v Levy (1952) 18 SATC 127
it was found that when the taxpayer purchased shares his main intention was to hold them as an investment,
although he considered the possibility of the shares appreciating in value and that a profit could be made.
Schreiner JA stated at 135-6:
“It seems to me that where the purposes of an individual taxpayer are mixed the only course, on principle
as well as for practical reasons, is to seek and give effect to the dominant factor operating to induce him to
effect the purchase”; …. “ ... unless one were to hold, what the legislature could not have intended, that the
taxpayer must exclude the slightest contemplation of a profitable resale of the property, it seems to me that
the only test to apply is that of the main or dominant purpose.”
If the taxpayer cannot be said to have had a dominant intention, but was concerned only to make a profit
out of the asset, either by retaining and working it or by selling it, he will be taxable on the profit if he sells.
This has been so held in numerous judgments including those in Durban North Traders Ltd v CIR (1956)
21 SATC 85, and the Appellate Division cases of Barnato Holdings Ltd v SIR (1978) 40 SATC 75, SIR v
Rile Investments (Pty) Ltd (1978) 40 SATC 135 and Ropty (Edms) Bpk v SBI (1981) 43 SATC 141. In the
Durban North Traders case the company had acquired land and erected buildings thereon with the intention
of letting them for as long as possible and of selling if a good profit could be made. The court, in deciding
that the profit was of a revenue nature, characterised the company’s activities as “alternative ways of
carrying on its business which was a single business of turning land to account.” In Rile’s case the
intentions at the time of purchase were obscure and may have been mixed. The company nevertheless
established that it had subsequently decided to treat the property in question as a capital asset.
Similarly, in COT v P (1978) 40 SATC 215 a mining engineer had the intention of working the surface
materials on mining claims with a concurrent intention of selling the claims to a mining house which would
have the resources to mine below the surface. He was held to be taxable on the profit from selling the
claims.
In Malan v KBI (1983) 45 SATC 59, however, the taxpayer was a member of a consortium of individuals
which owned the entire shareholding in a township-development company. When 51% of the shareholding
was sold in order to raise additional funds to continue the financing of the development, the taxpayer
succeeded in establishing (i) that this was done only with reluctance, and (ii) that it did not indicate an
original mixed intention on the part of the consortium to make a profit in any way which might present
itself, including a sale of shares. Again, in CIR v Middelman (1990) 52 SATC 323 the taxpayer was able to
show that profitable sales of several counters had not been effected as a secondary means of earning from
investment but was merely incidental to his dominant intention to earn a yield from dividends. The court
firmly distinguished the facts from those in the Barnato Holdings case referred to above.
4.6 Fixed and floating capital
Judicial references are sometimes made to “fixed capital”, as indicating accruals on capital account, and to
“floating capital” as indicating accruals on revenue account. A meaning was suggested in CIR v George
Forest Timber Co Ltd (1924) 1 SATC 20 at 23, as: “floating capital is consumed or disappears in the very
process of production, whereas fixed capital does not; though it produces fresh wealth, it remains intact.”;
and: “Ordinary merchandise in the hands of a trader would be floating capital ...”. So also would, for
example, consumable stores; any disposal would be on revenue account despite their not having been
acquired for resale. There is a review of the topic in ITC 1525 (1992) 54 SATC 209, at 213. The concept
is regarded in South Africa as entrenched in the law: C: SARS v Knuth and Industrial Mouldings (Pty) Ltd
(2000) 62 SATC 65 at 72.
It is submitted, nevertheless, that for the purpose of solving most problems of capital and revenue accruals
on sales of assets the test of intention, considered above, will usually suffice.
4.7 Damages and compensation
The question of whether compensation received is capital or revenue has given rise to a large number of
appeals. These have generally been decided on the basis of whether, on the facts, the amount was in respect
of a “loss of a capital asset” (capital) or was to “fill a hole in the income potential” (revenue).

25
In Verrinder Ltd v CIR (1948) 16 SATC 48 it was stated that “so far as the payment was compensation for
the loss of future profits, it is clear law that it must be regarded as income”. Thus in Burmah Steam Ship Co
Ltd v CIR (UK) (1931) 16 TC 67, where compensation received was in recognition of the fact that delays
in repairs had caused a loss of cargo contracts, such compensation was held to be taxable, being to
reimburse profits which might otherwise have been earned. A clear distinction may be seen between these
facts and those in Glenboig Union Fireclay Co Ltd v IRC (1921) 12 TC 427, where compensation was
received for undertaking not to work certain deposits of fireclay, resulting in the sterilisation of a capital
asset. The receipt was held to be of a capital nature.
The distinction is illustrated by the facts in CIR v Illovo Sugar Estates Ltd (1951) 17 SATC 387. When the
company’s land was expropriated for military purposes, compensation for the loss of a crop of sugar-cane
was adjudged to constitute income and to be taxable, while compensation for cancellation of a lease of
cane-growing land was a receipt of a capital nature. This approach to the crop was followed in the more
recent case of Estate A G Bourke v CIR (1991) 53 SATC 86, where the taxpayer was held liable on
compensation received after a pine tree plantation was destroyed by fire.
A test similar to the identification of the income-producing structure was adopted in KBI v Transvaalse
Suikerkorporasie Bpk (1987) 49 SATC 11. The company was a sugar grower and member of a sugar
industry association. A dispute arose as to whether the company was entitled to retain railage charges
which it had recovered from customers or was obliged to pay them to the association. In settlement the
company accepted a substantial sum in return for agreeing not to retain future charges. The receipt was
found to be of a capital nature, as the company had permanently relinquished its ability, which had existed
previously, to generate additional income.
Similarly in ITC 1557 (1993) 55 SATC 218, where a transport operator received compensation from
Government as a result of the closure of one of the operator’s bus routes upon introduction of a railway
service, the amount so received was held to be of a capital nature. The judgment cites a number of
authorities to the effect that in circumstances of this kind it is immaterial whether the amount of
compensation is calculated by reference to loss of profit.
4.8 Restraint of trade
While the Zimbabwe Act contains no general provisions in this respect the South African courts, in relation
to a similar situation at the time, have recognised the principle that a person’s right to trade freely is an
incorporeal asset and that an amount received for a restriction on that right is compensation for its
sterilization. Receipts in respect of restraint of trade are thus generally of a capital nature.
(a) Decisions of the courts regarding business operations include the following.
In Taeuber and Corssen (Pty) Ltd v SIR (1975) 37 SATC 129 an agency agreement included a clause
restraining the company from selling competitors’ products for a period of two years after the termination
of the agency and providing for substantial compensation to be received in consideration of such a restraint.
It was held that, in the light of the agreement as a whole, the restraint which came into effect on termination
of the agency caused a temporary loss of part of the company’s “income-producing structure”; the
compensation was therefore of a capital nature, even though it was calculated as a percentage of the
commission previously earned.
The same result arose in ITC 1341 (1981) 43 SATC 215. The company conducted a share-transfer
business, clients being contractually bound to use its services for specified periods. When some clients
insisted on immediate withdrawal it received negotiated compensation. The court found that the withdrawal
impaired the company’s income-earning structure by 20%, that this was a material part and that the
compensation was accordingly of a capital nature.
See, however, ITC 1549 (1993) 55 SATC 31, where a purported restraint clause in a lease agreement was
dismissed as a “smoke screen”.
(b) Compensation in recognition of restraint of trade may arise in the case of an employee.
In ITC 1338 (1981) 43 SATC 171 the Commissioner had taken the view that an amount, constituting the
equivalent of almost a year’s gross earnings, had been received by a company executive (who was
continuing in the company’s employ) by way of additional remuneration, and was therefore taxable. The
recipient was able to establish that the contract was genuinely in restraint of trade. The accrual was found to
be of a capital nature. (Deductibility of the payment, in the employer’s hands, was not in issue. It is
submitted, on the tests set out at 7.6.1(b)(iv), that a claim would fail.)
In Tuck v CIR (1988) 50 SATC 98 a receipt in the hands of a retired employee was found, on the particular
facts, to have been partly in recognition of services rendered and partly in recognition of future restraint of
trade. The taxpayer was thus held to be liable on 50% of the amount.

26
The decision against the taxpayer in ITC 1512 (1992) 54 SATC 45, however, illustrates the need for any
receipt to be clearly in restraint, as opposed to being in lieu of notice of termination of service.
4.9 Insurance proceeds
Claims under insurance policies fall into two categories, namely (i) recoupment of losses or expenses
incurred (such as through damage to a fixed asset or loss of stock by fire) and (ii) lump-sum payments on
the happening of an event whether or not a financial loss has been incurred (such as on the death of an
insured person).
Most of the receipts in the first category, if they are connected with trade, fall within the “recoupment”
provisions of sections 8(1)(i) and (j) of the Income Tax Act (see chapter 5). A type which would not fall
within those sections, but a claim under which is also taxable, is a policy covering loss of profits: see ITC
597 (1945) 14 SATC 264.
The tax treatment of accident, term and life policies, under which the insured, or life assured, is an
employee (including a director of a company), and the premiums are paid by the employer, is dealt with in
chapter 18.
4.10 Gambling and hobbies
(a) Gambling
Income derived from betting transactions undertaken solely for recreational purposes is not taxable. A
person may, however, conduct his activities in such a systematic way that he becomes regarded as a
professional punter, in which event he will become taxable on winnings: Morrison v CIR (1949) 16 SATC
377. The case is authority also for the proposition that income derived from betting activities as part of a
person’s established business of racing is taxable regardless of whether he bets on his own horses or on
those of others. In ITC 1612 (1997) 59 SATC 180, however, the taxpayer (in seeking to claim a deduction
in respect of losses and expenditure incurred in gambling) failed to convince the court that his activity
constituted a trade.
(b) Hobbies
The test of whether activity is being conducted for recreational purposes similarly applies. (See also
11.10.5.)
In ITC 1510 (1992) 54 SATC 30 a game warden who derived income from the sale of game trophies
acquired on hunting trips was able to show that his activity constituted a pastime which had not developed
into a scheme of profit making.
It is submitted that the same principles would apply to most instances of, for example, stamp collecting.
See also, however, at 4.3.4, ITC 1608 (1997) 59 SATC 63, where the taxpayer’s claim to have purchased
diamonds as a hobby was rejected.
4.11 Isolated transactions
It has already been indicated that the fact that a transaction is entered into only once does not automatically
mean that the profit earned is not taxable.
In Stephan v CIR (1919) 32 SATC 54 it was held that the profit derived from the salvaging of a ship was
taxable as the motive of profit-making was present.
Quite apart from operations in the nature of trade, there are instances in which the isolated nature of the
transaction has not saved the taxpayer from liability, though the decisions are not always consistent.
In ITC 1271 (1977) 40 SATC 123, for example, a husband and wife had transferred a property (which they
had hired, improved and subsequently purchased) to their company, and built a house for themselves on
other land nearby. When the company sold the property, it was held to be taxable on the profit because it
failed to prove that its dominant intention had been to hold the property for the future enjoyment of the
husband and wife. In ITC 1508 (1991) 53 SATC 442 a Zimbabwe civil servant who purchased a
Mercedes Benz while on an overseas posting and, shortly after returning, sold the vehicle in Zimbabwe,
was held to be taxable on the profit.
In another Zimbabwe case, X v COT, Y v COT, Z v COT (1977) 40 SATC 21, employees of a group of
public companies took up options on shares available to them under the companies’ staff share-option
schemes, and re-sold the shares at a profit. They were held to be taxable on the profits as they had sold in
pursuance of a scheme of profit making. (They would not have been liable on any profit had they, instead,
sold the options.)
This decision must be contrasted with that in a subsequent South African case, ITC 1427 (1988) 50 SATC
25 (in which the X, Y, Z case was not cited). The taxpayer, a banker, had entered into an agreement with a
company and its shareholder under which he obtained a five-year lease and occupancy of a farm, and an
option to purchase either the farm or the shares. When he received an unsolicited and highly favourable

27
offer for the shares he exercised his option over them and sold them on the same day. The court was
satisfied that when the taxpayer acquired the option over the shares his intention was to obtain and retain
the rights as capital assets and that his action in exercising his option and selling the shares was motivated
only by the price offered. There was no scheme of profit making and the profit on the sale of the shares was
of a capital nature. (See also ITC 1208 (1974) 36 SATC 80 and references in C: SARS v Knuth and
Industrial Mouldings (Pty) Ltd (2000) 62 SATC 65 at 73.)
4.12 Inheritances, gifts and prizes
Such receipts, whether in cash or otherwise, are almost invariably, as indicated at 4.2, of a capital nature.
Where such receipts are in the form of assets other than cash the nature of subsequent transactions may
vary: see, for example, chapter 11 as regards farmers’ livestock. As has been observed, the proceeds of
realisation of an inherited asset will again almost invariably be of a capital nature, as in ITC 1451 (1989) 51
SATC 93, though as illustrated in CIR v Strathmore Exploration and Management Ltd (1956) 20 SATC
375, it is possible for them to constitute an accrual in the ordinary course of (in that case the inheriting
company’s) trade and thus be on revenue account. In the Strathmore case the finding against the company
was reached on the basis that, in the circumstances, the acquisition of the inheritance and its disposal were
suggestive of a carefully-arranged plan. In ITC 1607 (1996) 58 SATC 340: where disposals were effected
by a company formed to take over shareholdings which had been inherited by family members, the
taxpayer easily established the capital nature of the accruals.
An inheritance or a gift could, however, take the form of an annuity, considered at 5.1.
The taxability of a gift or prize which is linked to a service rendered depends on the circumstances: see 5.2.
In ITC 976 (1961) 24 SATC 812 a journalist (who had also earned royalties from published literary works)
was found to be taxable on monetary prizes which he had won in poetry and play-writing competitions.
4.13 Sale of goodwill
Where, as is generally the position, goodwill constitutes a capital asset of the business, any amount
receivable for its outright sale will, it seems clear, be of a capital nature. This applies even if the fixed sum
is receivable in instalments.
The meaning of the term was considered in KBI v Snyman en Webster (1994) 56 SATC 149. Two former
employees of motor companies, who were highly regarded in the trade, incorporated their own company
and received from it amounts in recognition of their “drawing-power”. The court considered that such
amounts constituted goodwill (as opposed to being for “know-how” or for services to be rendered).
Once the consideration for goodwill is to be measured by reference to a proportion of future profits,
however, the position may well change, with the amount then constituting income: see Jones v CIR (UK)
(1920) 7 TC 310 and Deary v Deputy CIR (1920) 32 SATC 92.
4.14 Proceeds of “know-how”
(a) Where a taxpayer is in possession of, for example, technical information and expertise, which is
passed to another in return for a consideration, difficult questions may arise as to the character of the
receipt. The principle then to be applied is whether the recipient of the amount has, in imparting the
information, either substantially disposed of his property or turned it to account by using it. This is borne
out by decisions of the courts, largely in the United Kingdom.
In Evans Medical Supplies Ltd v Moriarty, HL (1957) 37 TC 540 the company, which manufactured
pharmaceuticals worldwide, carried on business in Burma through an agency. In return for assisting the
Burmese Government to set up an industry it received, among other things, a lump sum for the disclosure
of secret processes. The court found that the amount had arisen from an isolated transaction which had
resulted in the company giving up its Burmese business and that it was therefore of a capital nature.
A different result arose, however, in Jeffrey v Rolls-Royce Ltd, HL (1962) 40 TC 443, as regards sums
received from companies in various countries, over a number of years, for information in connection with
the manufacture of aircraft engines. The taxpayer company was found to have suffered no real diminution
to its property which remained available to be turned to future account. The amounts were held to be
trading receipts. The same result arose, on similar facts, in Musker v English Electric Co Ltd, HL (1964) 41
TC 556.
The Rolls-Royce approach has been followed in South Africa in ITC 1190 (1973) 35 SATC 188, where the
court indicated that it felt that the Evans Medical Supplies decision had turned on the special facts present
in that case.
(b) In any event, if the amount received constitutes a “premium or like consideration”, it may form
gross income by virtue of the deeming provisions of section 8(1)(d): see 5.4.
4.15 Exchange rates: profits and losses

28
The question of whether a profit or loss, resulting from differences in exchange rates, is on revenue or
capital account, turns on the nature of the underlying transaction.
In ITC 808 (1955) 20 SATC 343 the taxpayer, who conducted an agency business in South Africa, and who
earned commission on orders from American manufacturers, derived a profit on exchange due to a
favourable movement in the exchange rate in the time between the crediting of the commission and its
remittance to South Africa. The court held that because the original debt, being in respect of commission,
related to income, so the enhancement in value, or profit on exchange, was of a revenue nature.
The decision in CIR v Brown Bros Ltd (1955) 20 SATC 55, however, illustrates how a further development
in otherwise similar facts can bring about a different result. The taxpayer, a South African company, had
similarly earned commission from America, the customer being its subsidiary company. Amounts of
commission due to be remitted to the taxpayer were left by it with its subsidiary in order to increase the
latter’s working capital. When remittances were eventually made the question of the capital or revenue
nature of their enhanced value arose. The profit was held to be of a capital nature on the grounds that the
receipt was no longer by way of commission but in the nature of repayment of a loan.
Further circumstances in which a profit was held to be on revenue account arose in SA Marine Corporation
Ltd v CIR (1955) 20 SATC 15. The company was a South African shipowner which carried on business
also, through an agent, in America. The nature of the business necessitated the constant transmission, and
conversion, of funds in both directions between the two countries. In reaching its decision the court had
regard to the fact that, while it was acknowledged that the company was not trading in currencies, nor
designedly operating to make profits on conversion, such transmissions and conversions were an integral
part of the company’s scheme of profit making.
The same principles apply to losses.
As to the year in which exchange profits on revenue account are taxable see 5.18.
4.16 Options
An amount received or accrued in consideration for the granting of an option to buy an asset held by the
grantor as capital is a receipt or accrual of a capital nature. Further, in terms of the decisions in ITC 321
(1935) 8 SATC 236 and SIR v Struben Minerals (Pty) Ltd (1966) 28 SATC 248, both concerned with
mining options, when one indivisible amount is received for an option to buy mining rights (capital) and for
the grant of subsidiary rights such as the right to prospect, the whole amount will be treated as capital.
These cases were distinguished in COT v Rezende Gold and Silver Mines (Pvt) Ltd (1975) 37 SATC 39, in
which a separately-identifiable amount received for the right to work a mine during the period of the option
was held to be taxable.
If the grantor of an option is not thereby disposing of a capital asset, any amount received or accrued under
the option contract will be taxable in his hands, as in ITC 721 (1951) 17 SATC 485. There an amount was
received by a landlord in consideration of the right to take over a tenancy if the existing tenant vacated.
The above circumstances must be distinguished from those where an option to acquire an asset is exercised
and the asset sold: see 4.11.
4.17 Illegal trade
Profits from trading operations are taxable even if the trade is illegal: CIR v Delagoa Bay Cigarette Co Ltd
(1918) 32 SATC 47.
In ITC 1545 (1992) 54 SATC 464 (discussed at 2.3(a)) concerning profits from the purchase and sale of
stolen diamonds, the illegality of the taxpayer’s trade did not avail him.
Chapter 5
Items Specifically Included in Gross Income
Synopsis
5.1 Annuities
5.2 Remuneration in respect of services rendered
5.3 Lump sums from benefit and pension funds
5.4 Premiums: leases, patents, “know-how”, etc.
5.5 Improvements under a lease agreement
5.6 Benefits in respect of services rendered
5.7 Growing crops: sales and other disposals
5.8 Trading stock
5.9 Recoupment from capital expenditure (mining)

29
5.10 Recoupments (general)
5.11 Concessions by creditors
5.12 Recoupments: rentals, etc., payable, applied against purchase price
5.13 Grants and subsidies
5.14 Commutation of a retirement annuity
5.15 Designated area grants
5.16 Petroleum operations and special mining leases
5.17 Commutation of a pension
5.18 Exchange rate variations: relevant year
As indicated in chapter 2, the definition of “gross income” in section 8(1) of the Act specifically includes
certain accruals. The amounts set out in that section are, therefore, subject to taxation whether or not an
element of capital is present. Liability will, of course, arise only if they are from a source within, or deemed
to be within, Zimbabwe, as discussed in chapter 3.
5.1 Annuities (para (a))
(a) Any amount received or accrued by way of an annuity constitutes gross income.
(b) The nature of an annuity was considered in ITC 826 (1956) 21 SATC 189 at 190, as follows:
“An annuity may be described as an annual payment in perpetuity for the life of the grantee or for a limited
period ...”.
The following are the necessary characteristics of an annuity (ITC 761 (1952) 19 SATC 103):
– it must be claimable from another person or body. Ex gratia payments, even though
determined annually, for example, do not constitute annuities: CIR v Watermeyer (1965) 27 SATC 117;
– it must be a fixed annual payment (which may be divided into, for example, monthly
instalments);
– it must be repetitive for a period.
(See also KBI v Hogan (1993) 55 SATC 329.)
(c) An annuity may arise in various ways, such as by way of: (i) purchase from an insurance
company; (ii) a gift or a legacy; (iii) a pension for services rendered; (iv) an “annuity on retirement”; or (v)
a sale of assets.
The position as regards each of the above is as follows.
(i) Where an annuity is purchased by the annuitant or his spouse the annuitant is taxable
only on what may be termed the “interest content” of the annuity, i.e. after exclusion of such portion as, in
the Commissioner’s opinion, represents a return of any purchase price. The formula used by the
Commissioner to determine the “interest” and “capital” contents, respectively, of a purchased annuity, is
not set out in the Act but is contained in departmental practice 34.

The effect is illustrated in the following example.


Purchase price of annuity say $1 000 000
Annual receipts $210 000
Expected number of years payable 10
Annual interest content therefore =
$110 000
The number of years may, depending on the terms of the contract, be fixed, or, in the case of a life annuity,
the figure used by the insurance company when establishing the price at the time of purchase. In the latter
case, if the annuitant survives beyond the expectation of life, subsequent receipts are taxable in full as the
capital cost will by then have been allowed.

(ii) An annuity by way of gift or legacy is taxable in full. This applies regardless of whether
the annuity is paid partly out of capital (CIR v Milstein (1942) 11 SATC 279) (for example by trustees
administering a bequest) or out of exempt income (section 14 (3) of the Act and COT v R (1966) 28 SATC
115).
(iii) Where the annuity is in the form of a pension for services rendered it is taxable in full if
either there is no “cost”, as in the case of a pension from a non-contributory fund, or if the cost by way of
the employee’s contributions has been allowed in full.
If his contributions have not been allowed in full, probably, as commonly occurs, by
reason of their having exceeded the maximum amount deductible in any year, the aggregate of the
disallowances is deductible, under the same formula as in (i), from the time the pension starts to run.

30
Example
An employee who is about to retire at 31 December is entitled to a pension of $72 000 a year from his
employer’s fund. The employee’s contributions to such fund have in recent years exceeded the deductible
limit in his hands. The aggregate of the disallowances is established at $30 000. His life expectancy is
12 years.

Solution (annually)
Pension 72 000
Less (1) and (2) 2 500
Taxable (3) $69 500
Notes:
(1) The instalments of $2 500 are deductible in the hands of the pensioner/annuitant or, in the event of
his intervening death, his surviving spouse receiving a related pension or annuity.
(2) The deductible amount ($30 000) would be reduced proportionately in the event of commutation.
(3) This may be subject to partial exemption: see 6.3(v).

(iv) An annuity constituting an “annuity on retirement” (i.e. purchased out of a lump sum
payment from a pension fund: see 16.7.2) is taxable in full.
(v) Where a sale of assets arises one must distinguish between an annuity and a sale price
receivable in instalments. If, for example, X sells a farm to Y for $500 000 and the contract requires Y to
pay the $500 000 in ten annual instalments of $50 000 it is submitted that the instalments do not constitute
an annuity. If, on the other hand, X sells the farm, worth $500 000 with the stipulation that Y pays X $50
000 annually for life, it is submitted that the annual payments will constitute an annuity. In the latter
instance, on the authority of L v COT (1951) 18 SATC 137, the $500 000 may qualify for acceptance as the
purchase price for the purposes of the application of the formula referred to in (i) above.
5.2 Remuneration in respect of services rendered (para (b))
Any amounts received or accrued in respect of services rendered or to be rendered, including amounts
arising by reason of the cessation of employment and amounts received in commutation of amounts due
under a service contract, are included in gross income. An amount received in respect of services
constitutes “gross income” regardless of whether it is contractual. (See, however, 6.3 in relation to
exemptions.)
The following are included.
(a) In general amounts which are connected with employment or services rendered
An award other than cash is nevertheless taxable if it has the nature of a receipt for services rendered, the
“amount” to be taxed being the ascertainable money value. In terms of the decision in Wilkins v Rogerson
(1960) 39 TC 344, if an employer incurs say $5 000 on the purchase of, for example, a suit of clothes for an
employee, the second-hand value being established at $1500, the employee is liable only on the latter sum.
(The wording of the United Kingdom statute under which that case was decided is similar to, though not
identical with, that in paragraph (b) of the definition.) The result would, according to the judgment, have
been different had the employer made a cash award for the employee to effect his own purchase.
The value of any asset, such as shares or a motor vehicle, awarded in respect of services, thus constitutes
“gross income”. This aspect of the definition is concerned with outright grants of assets, as opposed to their
use, which is considered at 5.6.1.
By concession, the Commissioner refrains from taxing wedding presents and other gifts to employees, if
the employer does not claim their cost as a deduction or if the cost is less than $50.
A receipt for services rendered is taxable despite its being entirely gratuitous and unexpected: ITC 938
(1960) 24 SATC 375. This is so even if it comes from someone other than the recipient’s employer, such as
from the employer’s customer or client: ITC 1359 (1982) 44 SATC 165.
There have, however, been exceptions to the latter rule. Where an amount was received by a trainer from
the successful backer of a winning horse, and there was no indication that the trainer had supplied any
information, it was held that such amount was not in respect of services rendered: ITC 701 (1950) 17
SATC 108. (He was held to be liable on an amount received, from the owner of the horse, pursuant to a
promise made prior to the race). In Murray v Goodhews (1977) 52 TC 86 the taxpayer company ran public
houses on tied tenancy agreements with the brewers, Watney. The latter took back two of the tenancies and
made two ex gratia payments to the taxpayer. These were held to be capital receipts, having regard not
only to their being voluntary, unexpected and unsolicited, but also to the fact that the sole underlying

31
reasons were the long and friendly relationship between the parties and Watney’s desire to stand well with
the trade as a whole. In Stander v CIR (1997) 59 SATC 212 the taxpayer had been adjudged one of the top
five bookkeeper/accountants of franchise dealers, marketing vehicles from a major manufacturer. The
manufacturer awarded him, as a prize, an overseas trip for himself and his wife. He was taxed on the cost
borne by the manufacturer. The court held that the prize could not be considered to be in respect of services
rendered having regard to the fact, among other things, that the taxpayer had rendered no services to the
manufacturer and had not expected to receive anything from the manufacturer for the services which he had
rendered to his employer. It held also that the value of the trip fell outside the term “amount”, considered at
2.2; this took into account various factors including the taxpayer not receiving any tickets or vouchers and
not being permitted to take cash instead of going on the trip.
(See also, for example, ITC 1590 (1995) 57 SATC 160 regarding the term “in respect of”.)
An amount received for services still to be rendered constitutes gross income in the year of assessment in
which it is received, and not when the services are actually rendered: CIR v Cowley (1960) 23 SATC 276.
An ex gratia receipt in the hands of a deceased employee’s widow is of a capital nature since the services
were not rendered by her: ITC 1386 (1984) 46 SATC 116.
(b) Cash in lieu of leave
The lump sum is gross income in the year of accrual. (The longstanding provision whereby the employee
could elect that it be treated as accruing in three equal annual instalments was repealed with first effect for
the year of assessment ending 31 December 1999.)
Leave pay received in advance by an employee proceeding on leave accrues on a monthly, etc., basis
during the leave period.
(c) Compensation for loss of employment
Compensation, or a “golden handshake”, arising from the cessation of a contract of employment or of any
right to employment constitutes gross income.
The employee may elect that the “first $20 000” of a lump sum accruing by reason of the termination of
employment be treated as accruing in three equal annual instalments in the year of receipt and the two
following years respectively. Any excess is taxable in the year of accrual.
With effect from 1 January 2003 the ability to elect, in relation to three annual instalments, is withdrawn.
With effect from 1 January, 2001 the limit was increased to the greater of $ 100 000 or one-third of “the
first” $ 500 000.
With effect from 1 January, 2002 the limit was increased to the greater of $ 150 000 or one-third of “the
first” $ 750 000.
With effect from 1 January, 2003 the limit was increased to the greater of $ 300 000 or one-third of “the
first” $ 1 500 000.
More significantly, where the amount arises due to retrenchment under a Government-approved scheme,
the whole or a portion is exempt: see 6.3(iii).
It is submitted that in the case of such retrenchment accruals both provisions apply, as follows:

Retrenchment accrual say 75 000


Less exempt 25 000
50 000
Less: Elected to carry forward 2/3 x $20 000 13 333
Taxable in year of accrual $36 667

(d) Commutation of amounts due under a contract of employment or service


The employee may elect that such lump sum be treated as accruing in the three equal annual instalments in
the year of receipt and the following years respectively.
With effect from 1 January 2003 the ability to elect, in relation to three annual instalments, is withdrawn.
(e) Share options
If a company grants an employee an option to take up its shares it is submitted that his tax position is, in the
absence of specific provisions in the Act, as follows.
Firstly on the basis of a majority decision in the English case of Abbott v Philbin, HL (1960) 39 TC 82,
where an employee is granted an option which is exercisable at any time and unconditionally:
(i) no liability arises at the time the option is granted if the employee is required to pay at
least the market value of the shares prevailing at the date of the grant;

32
(ii) no liability arises upon subsequent exercise by the employee of the option if, during the
delay, there has been an increase in the value of the shares;
(iii) liability does arise at the time the option is granted if the price then is below the market
value. There is an accrual of the difference, on which the employee is taxable regardless of whether he
exercises the option.
On the authority of the Appellate Division case of Mooi v SIR (1971) 34 SATC 1, the position may
change if conditions are attached to the ability to exercise the option. There the employee was granted an
option to subscribe for shares in the employer company, the price being the value at the date of the grant.
He was, however, effectively required to serve the company for a further three years before the option
became first exercisable. He was held to be liable, at the expiry of the period, on the increase in the value of
the shares which had occurred in the meantime.
Also of relevance in the context of share options granted by an employer are the Zimbabwe cases of X v
COT, Y v COT, Z v COT (1977) 40 SATC 21 considered at 4.11. Those cases, however, concerned the
taxability of profits on resale of shares acquired under option schemes and therefore clearly stood on a
different footing from those above which related to amounts from the rendering of services.
5.3 Lump sums from benefit and pension funds (para (c))
See chapter 16.
5.4 Premiums: leases, patents, “know-how”, etc. (para (d))
A landlord and prospective tenant, for example, may provide in a lease agreement that, over and above the
normal rent, a lump sum is to be paid to the lessor for the right to occupy premises or for the use of an asset
so leased.
(a) Subject to what is said below these and similar sums constitute premiums and thus form gross
income of the lessor if receivable:
(i) for the right of use or occupation of land or buildings;
(ii) for the right of use of plant or machinery;
(iii) for the right of use of any patent, design, trade mark, copyright, model, plan, formula or
process or any other property or right of a similar nature;
(iv) for the right of use of any motion picture film or television film, sound recording or
advertising matter connected with such film; or
(v) for “know-how” i.e. the imparting of knowledge connected with the use of any of the
items in (ii), (iii) or (iv).
(b) Two phrases merit further investigation, namely “a premium or like consideration” and “the right
of use of”.
“A premium or like consideration” means consideration having an ascertainable money value,
passing from a lessee to a lessor, whether in cash or otherwise, distinct from and in addition to or in lieu of
rent: CIR v Butcher Bros (1945) 13 SATC 21.
Thus a premium for the use of buildings, etc., cannot arise unless there is a situation involving a
lessor and a lessee. If X hires premises from Y at an annual rental of say $200 000 and in addition must pay
Y a lump sum of $500 000, the $500 000 is a premium and constitutes gross income in Y’s hands. But an
amount received by a predecessor lessee for the cession of a lease to a substituted lessee is not a “premium
or like consideration” referred to in the section: CIR v Myerson (1947) 14 SATC 300. A sub-lessee and a
sub-lessor, however, stand in the same relationship to one another as do a lessee and a lessor. If a sub-
lessee pays a sum in addition to rental to a sub-lessor, for the right of occupation or use of property, such
sum constitutes a premium.
“The right of use of ” requires that the premium relate to the right of use or occupation. It does not
otherwise constitute gross income. Because of these words in the section, a premium or like consideration
is referred to in practice as a “lease premium”. A payment purportedly for goodwill is sometimes nothing
but a lease premium, depending upon the particular trade one is carrying on.
In ITC 1372 (1983) 45 SATC 180 the taxpayer company, the owner of a country hotel, received an amount
termed goodwill, in addition to rental, from the owner of a neighbouring hotel. The company contended
that the rental was adequate, requiring no augmentation. The court found, having regard to the importance
of the locality of the taxpayer’s hotel, and the form and nature of the agreements, that the amount
constituted a premium; even if some element of goodwill was present it was not quantifiable.
(c) The question of know-how, referred to above, raises further aspects. For a receipt to form gross
income, it must not only constitute a premium but must be for knowledge connected with patents, etc. A
receipt for the imparting of knowledge which is not so connected will not, it is submitted, be caught by the

33
provision. Nevertheless, it will remain taxable if it fails the test as to its capital or revenue nature. This is
considered at 4.14.
(d) A premium is taxable in full in the year of its accrual to the lessor. This contrasts with the lessee’s
delayed deductibility: see 8.4.
5.5 Improvements under a lease agreement (para (e))
A common instance of the operation of this provision is where the owner of, for example, a piece of vacant
land enters into a lease agreement for, say, 20 years in terms of which, in addition to rent, it is agreed that
the lessee is obliged to erect a building which will become the property of the landlord. Alternatively the
potential occupant may find a building which is near to his requirements but which needs substantial
additions and, again, he may undertake an obligation that he will carry these out at his own expense.
In either case the value of the improvements is gross income in the hands of the landlord. The value is the
amount stipulated in the lease. If no amount is stipulated, the Commissioner may determine a value.
There must be an enforceable obligation to effect the improvements (ITC 767 (1953) 19 SATC 206).
Where this is not specifically expressed it is likely that the agreement will fall outside the scope of the
section and that no gross income will arise. Depending on the terms, however, it is possible that an
obligation may be found to be implied: Rex Tearoom Cinema (Pty) Ltd v CIR (1946) 14 SATC 76.
A term which was expressed merely as a condition precedent has, on the other hand, been found not to
establish an obligation. Thus, in the circumstances of the particular case, despite the lease agreement
providing that the prospective tenant company would not acquire the right to occupy the premises
concerned if it failed to carry out alterations, this still did not mean that it was obliged to carry them out
(ITC 1464 (1989) 51 SATC 205). See also, however, CIR v Manganese Metal Company (Pty) Ltd (1996)
58 SATC 1.
In ITC 1615 (1997) 59 SATC 264 the taxpayer’s business had always been the provision of horse-racing
facilities and social functions on property which it leased from the local City Council (an exempt body). At
the taxpayer’s request the terms of the existing lease, under which the Council had no power to compel the
taxpayer to improve the premises, were amended, specifically to enable the taxpayer to take advantage of
the equivalent “obligatory improvements” provision, by the insertion of a clause obliging the taxpayer to
effect such improvements, etc., “as may be reasonably required to meet the convenience …. of its patrons
…. ”. The agreement was found to fall within the terms of the Act.
As to the identity of the party who may be said to have the right to enforce an obligation see the comments
at 17.4 on Erf 3183/1 Ladysmith and Another v CIR (1996) 58 SATC 229 and Relier (Pty) Ltd v CIR
(1998) 60 SATC 1.
Where an agreement is amended to increase the obligatory amount such additional amount will fall within
the scope of the paragraph if the revision was made prior to completion of the original improvements (COT
v Ridgeway Hotel Ltd (1961) 24 SATC 616) but not if it was made after completion (Professional Suites
Ltd v COT (1960) 24 SATC 573).
There are also provisions for spreading the gross income over a period of years (ten years or the unexpired
period of the lease, whichever is lesser) and for taxing the previously untaxed balance on a cancellation of
the lease, a sale of the buildings, or the death or insolvency of the lessor.
See 5.12(j) for an example of an agreement containing further clauses.
5.6 Benefits in respect of services rendered (para (f))
5.6.1 General
Employers may remunerate their employees for services rendered either in cash or in any other way, but
any advantage or benefit which can be connected with an employee’s employment forms part of his gross
income. “Advantage or benefit” is defined as: board; the occupation of quarters or of a residence; the use of
furniture or of a motor vehicle; the use or enjoyment of any other property, including a loan; or an
allowance. (As to the meaning of “in respect of” see, for example, ITC 1590 (1995) 57 SATC 160 and
Stander v CIR (1997) 59 SATC 212.)
For the most part, paragraph (f) imposes liability on the employee’s private use of the employer’s assets.
This is because many receipts by employees from employers, in cash or in kind, will already have fallen
within the provisions of paragraph (b), considered earlier. The bases on which the various benefits are to be
valued are laid down as follows:
(i) in the case of the occupation of quarters or of a residence, or the use of furniture: by
reference to the value to the employee;
(ii) in the case of all other benefits: by reference to the cost to the employer (or deemed cost
in the case of motor vehicles and loans: see below.)

34
In all instances save that of the use of a motor vehicle the benefit is reduced, or may be extinguished, if
either the asset is occupied or used to some extent for the purposes of the employer’s business transactions
or if the employee pays the employer for the enjoyment of the benefit. The same principle applies to
allowances expended on the employer’s business. In the case of a motor vehicle it is submitted that no
benefit would arise if the employee were to pay for that portion of its use as was for his private purposes.
5.6.2 Housing
The question of the “value to the employee” is frequently open to dispute. The Commissioner’s yardstick
tends to be, in the case of a rent-free house in a Municipal area, the open-market rental. The approach is
supported by ITC 1374 (1983) 45 SATC 196. This may, however, clearly be contested if relevant factors
arise, such as the house not being of the employee’s choice or being larger than necessary for his domestic
circumstances, or if he is required to undertake entertainment of business-related guests at the house. In
these circumstances the valuation becomes subjective in accordance with the principles laid down in ITC
210 (1934) 6 SATC 59. Where the house is outside Municipal areas the Commissioner’s approach is to
seek the adoption of amounts based on a percentage of salary (or possibly the cost of construction, or of the
value, of the property concerned.) When furniture is provided by the employer the Commissioner’s
(annualised) yardstick has been 8% of the cost of the items.
Some guidance as to the limitations which may be encountered, where an employee argues that a housing
benefit should be reduced, was afforded in the judgment in the abovementioned ITC 1374, at 201, as
follows:
“... I do not think the words ‘... occupied ... for the purpose of the business transactions of the employer ...’
can mean merely that occupying the quarters or residence is more convenient for the employee to carry out
the employer’s business. Such a construction would, for example, open the door to every employee who
was given employer accommodation either on the site of his daily work or sufficiently near to represent a
saving in transport, to claim he was excluded from the operation of s.8 (1) (f)...”. The judgment continued:
“I think the words ... may be paraphrased as ‘... in carrying on the activities of...’ the employer’s business”.
The question of whether there should be any liability at all arose in ITC 1590 (1995) 57 SATC 160.
While the individual concerned, being a director of a company, was by definition an employee, he did not,
other than by enabling it to comply with formalities, render any services (though he had done so in earlier
years). The court found that his rent-free occupation of a flat owned by the company stemmed from his
capacity as a (in fact the majority) shareholder. No taxable benefit arose since such occupation was thus not
“in respect of” services.
5.6.3 Motor vehicle
The paragraph stipulates specific amounts of benefits for the year of assessment, on which the employee is
taxable. These are on a sliding scale, in accordance with the vehicle’s engine capacity, as follows with first
effect from the year of assessment which commenced on 1 January 1999:
[the amounts in {brackets} apply with effect from 1 January,2001. The amounts in [brackets] apply with
effect from 1 January, 2002. The amounts in bold apply with effect from 1 January, 2003. – Editor.]

Up to 1500 c.c. $25 000 {$40 000} [$ 50 000] $ 240 000


Over 1500 c.c. and up to 2000 $40 000 {$ 60 000} [$ 150 000] $ 420 000
Over 2000 c.c. and up to 3000 c.c. $70 000 {$ 100 000} [$ 250 000] $ 540 000
Over 3000 c.c. $100 000 {$150 000} [$ 400 000] $ 900 000

These amounts are mandatory in the sense that they are not subject to variation in relation to running costs
or the vehicle’s value. They are reduced proportionally, however, if the employee uses the vehicle for only
part of the year.
(A system based on running costs had been introduced, on a staggered basis, with first effect for the period
1 April to 31 December 1997 when 50% of the cost of private usage was taxable, followed by 75% for the
year ended 31 December 1998. Thereafter it was overtaken by the introduction of the above stipulations.)
5.6.4 Loan
Where an employer makes a loan to an employee the deemed benefit is a specific percentage, depending on
the amount of the loan, as follows (for current and recent years):
[the amounts in {brackets} apply with effect from 1 January, 2001. - Editor.]
Up to $35 000 10,5% {12,5%}
Over $35 000 14% {16,0%}

35
These percentages relate to interest-free loans. Thus the benefit from an interest-free loan of $100 000 for a
full year would be $14 000. Account is, of course, taken of any interest charged by the employer. No
benefit would arise if the charge was 14% or more. If the charge was at say 4% the benefit would be at
10%, namely, in the instance cited, $10 000.
There is no reduction (or increase) by reference to the actual cost to the employer. The benefit arises even if
the employer has made the loan out of interest-free funds. Conversely it remains at the statutory percentage
despite the employer having incurred a higher rate on his borrowings.
The benefit provisions embrace not only direct loans from employer to employee but use broader
terminology such as “directly or indirectly” and “on behalf of”, and cover also loans to the employee’s
spouse or child and loans by, for example, associated companies and the employer’s pension fund. Loans
for the purpose of the education or technical training or medical treatment of the employee, his spouse or
child are, however, excluded.
It was held in ITC 1626 (1998) 60 SATC 17, where a director took out an insurance policy on his life, in
support of his guarantee of a company’s debts, that the interest-free loan from the company in respect of
premiums on the policy did not give rise to a taxable advantage or benefit in his hands.
In an unreported judgment the Special Court held that, in the case of an interest-free loan to a director, who
was unpaid, no taxable benefit arose since he was not in “gainful” occupation, etc., as required by the
paragraph.
5.6.5 Passage benefit
The cost of any journey undertaken by an employee, his spouse, or child, as is borne by his employer, is
potentially a “passage benefit” and thus taxable in the employee’s hands. There are, however, exceptions to
this rule. A journey undertaken to take up employment is excluded if it represents the first such benefit
granted by that employer to that employee. The same applies to a journey in similar circumstances on
termination of employment. All other journeys for the business purposes of the employer are also, as would
be expected, excluded. Dual-purpose trips, as where the employee, who is required to travel on business,
uses the opportunity for a period of leave, are generally treated by the Commissioner as apportionable, the
employee being taxable in relation to the period spent on leave.
5.6.6 Allowance
An allowance is gross income in the employee’s hands to the extent that he fails to spend it for the purpose
of the business transactions of the employer. The area in which this question arises with probably the
greatest frequency is that of entertainment allowances, where the employee remains liable on any unspent
amount (resulting in double taxation, the entire allowance having been disallowed as a deduction in the
employer’s hands). It rests with the employee, in the event of enquiry, to prove the amount expended or at
least to submit alternative evidence in the absence of strict proof: ITC 679 (1949) 16 SATC 349, quoted
with approval in a subsequent unreported Zimbabwe case.
In The Endeavour Foundation and UDC Ltd v COT (1995) 57 SATC 297 a bursary or scholarship granted
by the foundation, for the education of a child of an employee of a company participating in the funding of
such foundation, was held to constitute an allowance in the hands of such employee. (The foundation in
making such grant was guided by the wishes of the employer.)
Exceptions to the above rules arise in the case of, for example, members of the Government, judges and
senior civil servants, in respect of whom the Third Schedule to the Act provides for specific exemptions:
see chapter 6.
5.7 Growing crops: sales and other disposals (para (g))
See chapter 11.
5.8 Trading stock (para (h))
(a) In line with accounting practice, the value of trading stock on hand at the end of the financial year
is treated as gross income. Similarly, if purchased stock has been taken by a trader for his own use (i.e.
consumed), or has been given away, or has been attached to settle a debt, or in any other way has been
disposed of without resulting in income by way of a sale, the value constitutes gross income.
(b) “Trading stock” is defined in section 2 of the Act as including goods and other property which are
in any way acquired or made or partially made for the purpose of disposal in the ordinary course of trade,
including incidental packing and advertising materials. Fixed assets are not trading stock, nor are inherited
goods which the recipient intends not to trade in but merely to dispose of to the best advantage.
The definition embraces also “goods and other property ... which are acquired in the ordinary
course of trade for the purposes of or in connection with the manufacture, production, breeding,
construction or improvement of goods or other property ...”. This brings in for example, manufacturers’

36
input items, spare parts, etc.. Whether similar items of a taxpayer in a service operation are included
remains undecided. The definition is, however, merely inclusive and not exhaustive.
(c) Ignoring special features relating to farmers (see chapter 11) a taxpayer has an election as to the
method of valuing each item of trading stock which is on hand or which he has donated or consumed. He
may choose, at the date of valuation:
(i) the cost price to him; or
(ii) replacement cost; or
(iii) market value.
The method of determining cost item by item is, of course, often impracticable. The
Commissioner may accept some other basis if he is satisfied that it is impossible or impracticable to
proceed as above. Other methods such as FIFO, average cost, etc., are frequently used, but the
Commissioner does not accept LIFO.
There is no compulsion on a trader to apply the same method in every year of assessment.
Throughout the Second Schedule, “cost price” includes freight, insurance, duty and other expenses
incurred in “bringing the trading stock to hand”.
(d) With regard to the writing down of the value of stock, while the Commissioner will not accept the
application of overall depreciation, the application of realistic percentages to stock which is for example
found to be slow-moving, or no longer fashionable, is acceptable if it determines market value. In T v COT
(1978) 40 SATC 179 a reduction was allowed in respect of stock, in the hands of a share-dealing company,
consisting of English shares and debentures affected by the devaluation of sterling.
(e) In view of the definition of “trading stock” in section 2 of the Act, and the views expressed by the
court in Land Dealing Company v COT (1959) 22 SATC 310, at 322-3, it is accepted by the Commissioner
that fixed property held for sale by a person who is taxed as a property dealer constitutes trading stock.
(f) Work in progress of professional persons is by definition not trading stock for tax purposes,
although it may be regarded as such for accounting purposes. It is therefore generally not taxable until
completion.
In a case involving a publishing company the court examined the categorisation of editorial expenses and
the costs of testing market viability, which were incurred by the taxpayer in producing, prior to a decision
whether to proceed, a blueprint or design document for a prospective book. It was held that the expenditure
did not constitute trading stock as defined in the South African Act: ITC 1662 (1999) 61 SATC 357. It is
submitted that the position is the same under the Zimbabwe definition.
(g) In Richards Bay Iron & Titanium (Pty) Ltd and Another v CIR (1996) 58 SATC 55, the taxpayers’
business was the extraction, from coastal dunes, and beneficiation of minerals. The operations resulted in
the intermediate creation of stockpiles of extracted minerals at various stages in the processing sequence.
The Appellate Division held that such stockpiles constituted trading stock, as defined in the South African
Act notwithstanding that the materials were not capable of sale in their current state. It is submitted, having
regard to the definition in the Zimbabwe Act, that the same decision would have been reached locally.
5.9 Recoupment from capital expenditure (mining) (para (i))
This topic is dealt with in chapter 12.
5.10 Recoupments (general) (para (j))
If a taxpayer has had expenditure or a loss allowed under section 15(2) (which is the principal section
authorising deductions) and subsequently recovers or recoups the expense or loss or any part of it, the
recoupment is gross income. This specifically includes special initial and wear and tear allowances
commonly recovered on a sale of assets. In view of the relationship of this topic to the granting of
allowances its practical application is dealt with in particular in chapters 9 and 11. (A further common
example is the recovery of bad debts.)
The meaning of the term was considered, in relation to somewhat unusual circumstances, in ITC 1634
(1998) 60 SATC 235; see 2.4(b).
The paragraph includes in gross income also any (highly uncommon) recoveries by an employer on the
closing down of a pension or benefit fund. Such recoveries, and those by employees, are considered in
chapter 16.
5.11 Concessions by creditors (para (k))
The circumstances covered are those where:
(a) a deduction has been allowed under section 15(2) in respect of expenditure incurred by the
taxpayer (including expenditure which has ranked for capital allowances in the hands of industrialists,
traders, farmers, miners, etc.); but

37
(b) such expenditure has not actually been paid; and
(c) subsequently it is agreed with the creditor that some smaller sum (or nothing) is payable.
The reduction in liability becomes part of the gross income of the debtor in the same way as a recoupment
above. The application of the paragraph may, however, turn on the precise terms of any arrangement with
creditors, as occurred (though under the provisions of the South African Act) in CIR v Datakor Engineering
(Pty) Ltd (1998) 60 SATC 503, where a substitution of redeemable preference shares for debt was held to
fall within a comparable section.
The inclusion in gross income does not occur, however, if the compromise arises by reason of, for example,
the debtor’s insolvency.
It is submitted that this effective recoupment provision applies only where the creditor stands in respect of
revenue expenditure incurred by the debtor, such as a supplier of the debtor’s trading stock, or is the
supplier of capital assets on which the debtor has obtained capital allowances. The provision appears not to
apply where the concession is granted by, for example, a loan account creditor.
5.12 Recoupments: rentals, etc., payable, applied against purchase price (para (l))
(a) Another instance of recoupments arises where a person who has paid rent for, for example, a
business property, subsequently purchases it for a price which is diminished as a result of taking into
account the rent already paid. Since the rent has been allowed as a deduction for tax purposes the amount
which is then applied in reduction of the purchase price is treated as forming part of the gross income of the
purchaser.
(b) The same principle applies if the purchase price is reduced by a lease premium previously
paid.
(c) Expenditure incurred by a tenant in terms of an agreed obligation to make improvements is treated
as “rent” for the purposes of the operation of the above.
(d) Where a purchase price appears to the Commissioner to be less than the fair market price, the
difference is deemed to have been applied in reduction of the purchase price to the extent that any form of
rent has been paid in the past.
(e) Where a recoupment arises in terms of this paragraph the Commissioner recognises that the
amount recouped has effectively been re-spent by the taxpayer and accordingly in practice grants wear and
tear allowance on the amount attributable to the asset.

(f) Amounts included in gross income under this paragraph may, upon election, be spread over six
years unless in the meantime the purchaser re-sells the property, when the balance must be brought in.
(g) It is not necessary for the rent to have been paid by the same person as purchases the property.
Thus A may let a property to B for $100 000 per annum with an option in favour of B to purchase the
property within say, five years, at a price of $1 000  000, less the rental paid ($500 000). If B cedes the
option to C, who exercises it, the amount of $500 000, (of which B has had the benefit for income tax
purposes) is taxable in C’s hands in the year in which the option is exercised by him.
(h) The provisions appear to apply regardless of whether tax avoidance is involved.
(i) Example: lease-hire
An industrialist enters into an agreement whereby he undertakes to hire an item of plant from a financier
for $200 000 a year for 4 years, with an option thereafter to find a purchaser, at $50 000 (who will supply
the plant to the industrialist at that price). The cost of the plant to the financier is $500 000.
The industrialist’s deductions and recoupments would be as follows.

(i) The rental is deductible at $200 000 a year.


(ii) A recoupment of $278 050 arises, being:
Fair market value established as not exceeding, in terms of the Commissioner’s practice*, the financier’s
cost of $500 000 written down at 10% for four years on the diminishing balance basis
$328 050.00
Less the price paid $ 50 000.00
$278 050.00

*May be subject to revision; open, in any event, to being contested.


The industrialist may elect to spread such recoupment at $46 341 a year over years 5 to 10. Otherwise the
entire $278 050 is recouped in year 5.
(iii) Capital allowances (see chapter 9):

38
$50 000 ranks, upon election, for 25% SIA in year 5 and for “accelerated” wear and tear allowance in years
6, 7 and 8. $278 050 ranks for “long term” wear and tear allowance with effect from year 5.

(j) Example: lease improvements

Mr Tenant is obliged, in terms of a five-year lease agreement, to effect improvements at a cost of $600 000
to a commercial property where he carries on business. He has the right, and elects, to purchase the
property for $700 000 at the end of the lease, at which stage the fair market price is $1 000 000.
The tax effect in Mr Tenant’s hands would be:
Years 1 to 5: $600 000 deductible at $120 000 a year.
Years 6 to 11: $300 000 (being $1 000 000 – $700 000) to be recouped at $50 000 a year (if so elected;
otherwise $300 000 to be recouped in year 6).
Years 6 onwards: wear and tear allowance deductible at 2,5% annually on both the actual cost of $700 000
and the $300 000 recoupable.

5.13 Grants and subsidies (para (m))


Gross income includes any amount received or accrued by way of grant or subsidy in respect of any
expenditure allowed as a deduction. An example would be any subsidy paid by Government after, say, the
construction of a farm dam. The subsidy is subject to tax in the year in which it accrues, even though the
project in respect of which it arises may have been completed during a previous year.
5.14 Commutation of a retirement annuity (para (n))
See chapter 16.
5.15 Designated area grants (para (o))
Any amount accruing in terms of the State scheme (introduced in 1975) is included in gross income.
5.16 Petroleum operations (para (p), s 2(1), s 35 and Sch 20) and special mining leases (para (r), s 2(1),
s 36 and Sch 22 and 23)
Particular provisions relate to exploration for, and production and disposal of, crude oil and natural gas and
to operations under special mining leases. These provisions separate the taxable income from that from
other operations but are of limited interest in view of the very restricted number of operators concerned.
5.17 Commutation of a pension (para (r))
See chapter 16.
5.18 Exchange rate variations: relevant year (s 8(2))
In the case of foreign transactions the amount of income is the amount expressed in Zimbabwe currency.
If, due to a fluctuation in rates, the amount received differs from the amount due, the amount received,
expressed in Zimbabwe currency, constitutes the recipient’s gross income. If the receipt and the accrual
occur in different years of assessment effect must be given to the increase or decrease in the year in which
the amount is received.
Only those variations as are on revenue account refer; see 4.15.
Chapter 6
Exemptions
Synopsis
6.1 Introduction
6.2 Exemption based on the identity of the recipient
6.3 Exemption based on the nature of the income
6.4 Examples
6.1 Introduction (s 14 and Sch 3)
Initially the definition of “gross income” includes all the income received by or accrued to a taxpayer. Only
a certain balance is, however, ultimately made subject to tax. In the reduction process, some items are
eliminated from total income, i.e. granted immunity from income tax in the sense that they are specifically
declared to be exempt.
The relevant exemption provisions effectively create two categories, as below.
6.2 Exemption based on the identity of the recipient (paras 1 to 4)
The amounts which enjoy exemption from income tax by reason of the identity or character of their
recipient are as follows.

39
(a) Public utilities, etc. (para 1)
The income of local authorities, the Reserve Bank of Zimbabwe, the Zambezi River Authority, the Natural
Resources Board and the Post Office Savings Bank is exempt.
(b) Non-profit-sharing bodies (para 2)
Bodies which for the most part fall within such categorisation, and which are exempt, include:
(i) agricultural, mining and commercial societies not operating for the pecuniary benefit of
the members. This would include the various show societies, etc.;
(ii) approved benefit funds, pension funds (see 10.22), building societies, friendly societies
and medical aid societies;
(iii) clubs, societies, institutes and associations which:
(a) are organised and operated solely for social welfare, civic improvement, pleasure,
recreation, or advancement or control of any profession or trade; and
(b) cannot, under their constitutions, distribute their profits to their members, except as bona
fide remuneration for services;
(iv) religious, charitable and educational institutions of a public character and (see The
Endeavour Foundation and UDC Ltd v COT (1995) 57 SATC 297) trusts of a public character.
The exemption in relation to charitable and educational institutions of a public character
may apply not only to their investment income but also to profits from the rendering of services and from
sales of goods: ITC 1565 (1994) 56 SATC 18 and Chancellor, Masters and Scholars of the University of
Oxford v CIR (1996) 58 SATC 45;
(v) trade unions;
(vi) employees’ saving schemes;
(vii) certain funds established by the Treasury in terms of the Audit and Exchequer Act;
(viii) any statutory corporation which is declared by the Minister by notice in the Gazette to be
(subject to any limitations) exempt.
(c) Quasi-Government and other bodies (para 3)
Exemption is conferred on approved Government agencies and on specified international and financial
organisations.
Note:
In all the above cases (a) to (c), only the named recipient is exempt. If that recipient pays amounts to other
persons, such as employees, those recipients do not fall under the exemption provision. (Further, the
exempt body may still have PAYE and other obligations.)
(d) Other persons, including companies and individuals
Receipts and accruals are exempt (para 3) if they are covered by any agreement entered into by the
Government of Zimbabwe with any other Government, or with certain organisations. It is thus possible, if
the terms of the agreement are sufficiently wide, even for trading profits to be exempt. The exemption
extends (para 4) to the emoluments of employees covered by such agreements. Approval of the person by
the Minister by notice in the Gazette is, however, essential.
The receipts and accruals of any company, which has as its principal object the provision of venture capital
for development purposes and which is approved by the Minister by statutory instrument, are exempt (para
3).
(e) Non-residents: loans to the Reserve Bank of Zimbabwe (para 3)
Certain interest is exempt.
(f) The President of Zimbabwe, members of Parliament and other officials (paras 4 and 5)
The President, a Vice-President, and their spouses, and various members of the Cabinet and of Parliament,
the Chief Justice, judges, diplomats, chiefs, certain members of the defence and police forces, senior State
officials, civil servants on duty abroad and urban councillors, are exempt in respect of some of their
income. In the case of the President, his entire Presidential emoluments are exempt. In most other cases the
exemption extends only to special allowances and gratuities but not to salaries.
Pensions and allowances paid to, and facilities provided for, any retired President are exempt.
Allowance payable to spouse of former President
With effect from 1 November 2001 any such allowance payable by the State is exempt.-Editor
6.3 Exemption based on the nature of the income (paras 4(l) to (q) and paras 6 to 16)
The remainder of the Third Schedule stipulates exemptions which hinge more on the type of income than
on the identity of the recipient, though certain conditions may have to be fulfilled. Exempt accruals include
the following:

40
(i) scholarships, bursaries, school fees, etc., provided that these are not remuneration for
services rendered by the student concerned or his near relative. Where the fees, etc., are paid by an
employer in respect of a child of an employee such amount may remain, in terms of the decision in The
Endeavour Foundation and UDC Ltd v COT (1985) 57 SATC 297, taxable in the hands of the employee;
[with effect from 1 November 2000 the exemption in respect of a bonus was increased to
$ 5 000; with effect from 1 November 2001 the exemption in respect of a bonus was increased to $10
000,and with effect from 1 November 2002 the exemption in respect of a bonus or performance–related
award is increased to $20 000. - Editor]
(ii) with effect from 1 November 1999 the “first $3 000” of any bonus accruing to an
employee or agent (or, if this gives a lower exemption, 10% of his other “remuneration” as defined for
PAYE purposes). Where the recipient has more than one bonus the $3 000, or 10%, applies to the
aggregate;
(iii) with effect from 1 January 2000 the “first $20 000”, or one-third, whichever is the greater
(subject to a maximum exemption of $33 333) of any amount received by an employee on the cessation of
his employment, due to retrenchment under a Government-approved scheme;
with effect from 1 January 2001 the limit is increased to the greater of $100 000 or one-
third of “ the first $500 000;
with effect from 1 January 2002 the limit is increased to the greater of $150 000 or one-
third of “ the first $750 000;
with effect from 1 January 2003 the exemption is increased to the greater of $300 000 or
one-third of the first $1 500 000.
(iv) war disability pensions; pensions and gratuities payable in terms of the War Veterans Act
[Chapter 11:15] and regulations thereunder; old age pensions from Government; and pensions and other
amounts granted under a law (such as workmen’s compensation awards) for injury, disease, disablement or
death “suffered in” (see ITC 947 (1961) 24 SATC 463) employment;
With effect from 1 January 2001 this exemption has been repealed, owing to the increase
in the elderly persons credit;
(v) with effect from 1 January 2000 the “first $12 000” a year of a pension from any pension
fund (including a retirement annuity fund) or from the Consolidated Revenue Fund, where the pensioner is
at least 59 years of age prior to the commencement of the year of assessment. Where the recipient has more
than one pension the $12 000 applies to the aggregate;
(vi) benefits in respect of death, injury or sickness, received from a trade union, benefit fund
or medical aid society or in terms of an insurance policy. As regards life insurance policies, in the case of
endowment policies it has long been accepted that the proceeds, despite being received upon maturity
during the insured’s lifetime, are of a capital nature. See the comments in ITC 1608 (1997) 59 SATC 63
at 68;
(vii) the value of medical treatment (including related travelling expenses) and medical aid
society subscriptions, paid by an employer for an employee;
In a South African case, ITC 1663 (1999) 61 SATC 363, an employer claimed to have
entered into a “salary sacrifice” scheme whereby employees’ salaries were reduced, the reduction being
made up by the employer’s payment of the employees’ contributions as members of such a society. The
court could find no proof that the taxpayer, as employer (with PAYE responsibilities) had entered into an
enforceable agreement such as would introduce a valid system, but found, at 369, that such schemes
otherwise are quite lawful in maximising tax effectiveness;
(viii) dividends from companies incorporated in Zimbabwe provided that the paying company
is chargeable to tax. (It is irrelevant whether or not it has taxable income on which tax is in fact charged,
provided it is a company which would pay tax on its profits.) In particular, therefore, dividends received
from building societies are not exempt, since their income is exempt as above. Those dividends which are
exempt from income tax may nevertheless suffer withholding tax; see chapter 19. See also below regarding
class “C” permanent shares;
(ix) interest on Zimbabwe Government savings certificates and on Zimbabwe Post Office
Savings Bank deposits (but not those of other countries);
(x) interest on tax reserve certificates (regardless of whether they are surrendered for cash);
(xi) interest on such loans to the State the yield on which has been declared to be free of tax;
(xii) interest on class “C” permanent shares issued by building societies. The exemption is
restricted to the interest earned on maximum deposits of (currently) $1 400 000 in the case of an individual

41
and $1 000 000 in the case of a company. Where deposits are held in more than one society these limits
apply to the aggregate. The interest on any excess is taxable;
(xiii) interest on any foreign currency denominated account. This exemption does not apply to
recipients which are companies or trusts. Their yield on such accounts will (see 19.8) already have had
residents’ tax on interest withheld resulting in no further liability;
(xiv) interest on any “agricultural bond” issued by the Agricultural Bank of Zimbabwe Ltd and
a consortium of commercial banks;
With effect from 12 November 2002 interest on any ‘agricultural bond’, issued by a
consortium of commercial banks led by Syfrets Corporate and Commercial Bank for the purpose of
advancing funds to support beneficiaries of the land resettlement programme, is exempt;
(xv) with effect from 23 June 2000 interest accruing on any bond issued by the Reserve Bank
of Zimbabwe on behalf of National Fuel Investments Company of Zimbabwe (Pvt) Ltd is exempt;
(xvi) interest from which residents’ tax on interest is required to be withheld (in terms of
Schedule 21: see 19.8). See also 10.25;
(xvii) interest received by persons who are not ordinarily resident in Zimbabwe and do not
carry on business in Zimbabwe at the time the interest accrues, on certain loans including those made for
the purpose of carrying on mining operations or prospecting in Zimbabwe.
The lender loses the exemption, however, in specified circumstances, namely where
either:
(a) the interest is received by companies which are controlled by persons either ordinarily
resident or carrying on business in Zimbabwe; or
(b) no real benefit arises due to the effect of another country’s tax laws;
(xviii) alimony;
(xix) amounts paid by Government to exporters under a scheme to encourage the export trade
(but excluding any duty refunded);
(xx) entertainment allowances received by employees and expended on the employer’s
business. The comments at 5.6.6, in relation to allowances, refer;
(xxi) certain receipts and accruals of “licensed investors” and their employees in an export
processing zone (see chapter 10);
(xxii) certain receipts and accruals of an industrial park developer (see chapter 10).
Duty-free certificate exemption
With effect from 1 January 2001 any amount received by way of sale, etc., of any duty-free certificate,
issued by the Reserve Bank of Zimbabwe to qualifying exporters, is exempt.
Approved employee share ownership schemes or trusts
With effect from 1 January 2002 where an employee participates in such a scheme (as defined), any
amount accruing to him from the sale to or other redemption by the scheme or trust of any stock, shares,
debentures, units or other interest of the employee in the scheme, etc., is exempt from income tax.
6.4 Examples
For examples of the operation of some of the items in 6.3 see chapter 10.
Chapter 7
Deductions: The General Formula
Synopsis
7.1 Introduction
7.2 To the extent to which
7.3 They are incurred
7.4 For the purposes of trade
7.5 Or in the production of the income
7.6 Not of a capital nature
7.7 Prohibited deductions
7.1 Introduction
(a) The steps which have been described in earlier chapters have been (i) to determine the taxpayer’s
“gross income” by ascertaining his total amounts (from specified sources) and eliminating those of a capital
nature, and (ii) to determine his “income” by eliminating exempt amounts. The final operation in order to
determine “taxable income” is to effect those deductions from “income” permitted under sections 15, 17

42
and 18 of the Act. Expenditure and losses in respect of which no deduction is permitted are set out in
section 16.
Only those items stipulated in the Act may be deducted. As was observed in chapter 1, in the case
of Sub-Nigel Ltd v CIR (1948) 15 SATC 381 it was emphasised that the court was not concerned with a
deduction which might be considered proper from an accountant’s point of view or from the point of view
of a prudent trader, but merely with the deductions which were permissible according to the language of the
Act.
The taxpayer is not permitted to claim a deduction under more than one provision but has, in terms
of section 15(4), a choice as to the section, paragraph, etc., under which he wishes to lodge his claim. Each
deduction provision stands alone and, if a taxpayer’s claim fails under one test but succeeds under another,
his expenditure or loss is deductible: L Feldman Ltd v SIR (1969) 31 SATC 121.
(b) Most of the permitted deductions are set out in section 15. Subsection (2)(a) of that section
contains what has become known as the “general deduction formula”. It reads as follows:
“(2) The deductions allowed shall be:
(a) expenditure and losses to the extent to which they are incurred for the purposes of trade
or in the production of the income except to the extent to which they are expenditure or losses of a capital
nature;”.
Comment on each element of the “formula” follows.
7.2 To the extent to which
Expenditure is deductible only to the extent that it is incurred for the permitted purposes.
Where expenditure or losses are incurred partly for one purpose and partly for another (one purpose being
trade and the other not) apportionment arises. No basis is laid down in the Act but, in the case of expenses
of a general character, which cannot accurately be appropriated to taxable and non-taxable income
respectively, it was held in Local Investment Co v COT (1958) 22 SATC 4 that apportionment must be
“fair and reasonable having regard to all the circumstances of the case”.
Where, for example, interest-bearing borrowings have been invested partly on loan account yielding
taxable interest and partly in shares yielding exempt dividends, it is usual for the interest paid attributable
to the dividends, and thus disallowable, to be apportioned by reference to the amount of capital invested in
the shares. Management expenses in such a case would, it is submitted, be more properly apportionable by
reference to the time spent on administration of the respective types of investment but this would, in the
light of ITC 1017 (1963) 25 SATC 337, require positive evidence. In this respect a taxpayer was successful
in ITC 1589 (1995) 57 SATC 153. The principle was recognised also in ITC 1641 (1998) 60 SATC 493.
Apportionment again arises where expenditure is incurred partly for capital and partly for revenue
purposes, as in CIR v VRD Investments (Pty) Ltd (1993) 55 SATC 368.
7.3 They are incurred
(a) Since expenditure is generally deductible only if it has been “incurred” the meaning of the word is
obviously important. It does not mean paid. If legal liability for the payment of an admissible deduction has
arisen, such expenditure is deductible from income even if payment occurs only at some later date: Port
Elizabeth Electric Tramway Co. Ltd v CIR (1936) 8 SATC 13, ITC 1117 (1968) 30 SATC 130 and ITC
1121 (1968) 30 SATC 171.
(The corresponding South African provision uses the term “actually incurred”, the meaning of
which has come before the courts. In the Edgars Stores case, for example, to which further reference is
made below, it was considered, at 94, that “actually” did not add anything to the plain and ordinary
meaning of “incurred”. It is submitted that this approach is to be preferred to contrary views.)
The date at which liability arises has been determined in a number of cases including the
following.
KBI v Nasionale Pers Bpk (1986) 48 SATC 55. The company provided in its annual accounts for bonuses
which would become payable to employees, in terms of their conditions of service, if they were in its
employ on a specified future date.
The court held that the amounts had not been incurred by the company in the years to which the accounts
related.
Edgars Stores Ltd v CIR (1988) 50 SATC 81. The company carried on retail business in various leased
premises. In terms of the lease agreements it was required to pay not only a basic rental (the deductibility of
which was not in issue) but also a “turnover rental” based on the turnover for each year of the lease. Some
of the “lease years” ended on a date subsequent to the company’s year of assessment. It was found that the

43
turnover rental was incurred only at the end of the lease year. The company was accordingly not entitled to
deduct its estimate of the amount attributable to its assessment year.
ITC 1495 (1991) 53 SATC 216. The taxpayer company claimed a deduction in respect of a liability which
it contended had arisen at the year-end in respect of employees’ leave pay for leave not yet taken. The
conditions of service provided, however, for a leave cycle, whereafter any leave not taken within the period
was forfeited. The court found that, since the relevant expenditure might or might not arise in the future, the
company had not incurred an absolute and unqualified liability. The claim was accordingly disallowed. The
result must be contrasted with that in ITC 1516 (1992) 54 SATC 101. There a deduction was allowed on
the grounds that the employer concerned had an absolute liability for leave pay, the payment of which was
governed by statute.
In the context of the supply of materials it is likely that where a taxpayer enters into a binding
contract he will have incurred the relevant expenditure at that point. This may not be so, however, if
conditions have first to be satisfied.
Such was the case in ITC 1444 (1989) 51 SATC 35. There a contract had been concluded during the year of
assessment for the later supply of materials to the taxpayer, the price being either fixed or determinable
under an agreed procedure. Payment of the price was, however, stipulated as being by way of “cash against
documents”. It was held that, in respect of later deliveries of supplies, the taxpayer had not incurred the
expenditure during the year of assessment in issue; his liability was conditional upon performance, or the
tender of performance, by the supplier, and was not absolute and unqualified. This followed the reasoning
in the Nasionale Pers Bpk judgment.
(b) The date when interest, and finance or discount charges, on borrowings may be said to be incurred
has been the subject of, among others, four cases which have in recent years come before the South African
Special Courts, namely ITC 1485 (1990) 52 SATC 337, ITC 1496 (1991) 53 SATC 229, ITC 1587 (1995)
57 SATC 97 and ITC 1588 (1995) 57 SATC 148. All concerned interest or similar charges on negotiable
certificates of deposit, promissory notes and a loan, where the taxpayers’ indebtedness arose in the year of
assessment but continued into a subsequent year or years. In all the cases the taxpayers claimed a deduction
of the entire interest etc., notwithstanding that part, or even the whole, of the expense had to be met only at
a date in a future year.
In the first case the court restricted the deduction in the year of assessment to the portion of
interest applicable to the period falling into such year. In the second, where the liability had to be met some
years hence, no deduction was granted.
In the last two cases, however, a deduction was allowed in full, the previous decisions being
considered to have been wrong. Emphasis was placed on the fact that, in the year of assessment concerned,
there was an unconditional legal liability to pay, even though the actual payments would fall due only in a
later year. (In the last case the taxpayer had undertaken to pay the finance charges in full even if the
agreement were terminated for whatever reason prior to its expiry date.)
In no instance was the case taken further. The position in South Africa has been dealt with by
legislation. In the absence of such action in Zimbabwe it is submitted that taxpayers with unconditional
obligations as above would have valid claims for deductions. It must be borne in mind, however, that the
authorities here may be quick to resort to retrospective legislation to counter an adverse judgment, as
occurred (in different circumstances) following the decision of the High Court in BAT v COT (1995) 57
SATC 271.
(c) Where a taxpayer himself genuinely disputes his liability under an undertaking to pay an amount
of expenditure, such expenditure is, on the authority of CIR v Golden Dumps (Pty) Ltd (1993) 55 SATC
198, incurred only in the year in which the dispute is settled (in that instance by litigation) and not in the
year in which he enters into the undertaking. Nevertheless the taxpayer in ITC 1545 (1992) 54 SATC 464
(see 2.3(a)) who purchased stolen diamonds, and who would have been liable to their owner for their return
or for their value, was held not to have “incurred” expenditure on their purchase in the year of assessment,
since the owner had not at that stage instituted proceedings.
(d) “Incurred” does not mean “necessarily incurred”: COT v Rendle (1965) 26 SATC 326, referred to
below. Excessive expenditure resulting from, for example, poor administration may not have been
necessarily incurred. It nevertheless qualifies as a deduction from income as long as it was incurred in the
production of income or for the purposes of trade. But see below for the Commissioner’s power to
disallow, for example, excessive directors’ fees.
(e) Once the test of “incurred” is satisfied, expenditure is deductible in the year of assessment despite
it not producing income in that particular year, provided that it was incurred for the purpose of producing

44
income in either the current or a future year: Sub-Nigel Ltd v CIR (1948) 15 SATC 381. Thus, bearing in
mind the test in the Zimbabwe legislation considered at 7.4, a full year’s insurance premiums, for example,
falling due in the last month of the taxpayer’s financial year and payable in advance, will be deductible
notwithstanding that virtually the entire amount relates to the following year. This may well result in an
earlier deduction than that arising from the accounting treatment, where the portion of the premium
applicable to the following year may have been carried forward as a prepayment in accordance with the
“matching” concept.
(f) In converse circumstances caution must be exercised as regards expenditure purportedly relating
to previous years. In ITC 1094 (1966) 28 SATC 275 an interest-free loan had been made, but the borrower
accepted in a subsequent year a charge for interest in relation to earlier years in respect of which no liability
had existed. Such interest was disallowed in the hands of the payer. (This rule would apply despite the
taxability of the interest in the hands of the recipient.)
(g) There is no authority for the deduction of reserves or provisions made for anticipated expenditure
or losses; they will not at that stage have been “incurred”. There is in any event a prohibition against such
deduction in section 16(1)(e) (see 7.7).
(h) Section 15(2)(cc) introduces a very limited degree of permitted deduction for future expenses.
This is considered in the next chapter.
7.4 For the purposes of trade
(Under the deductibility test which is about to be considered, a number of decisions of the South African
courts have been cited as illustrating results which would be anticipated also in Zimbabwe. This is an area,
however, where other such cases, resulting in disallowance of expenditure, have been ignored since, in the
courts’ application of the South African provisions at the time, more stringent tests applied.)
(a) “Trade” is defined in section 2 as including “any profession, trade, business, activity, calling,
occupation or venture, including the letting of any property, carried on, engaged in or followed for the
purpose of producing income as defined in subsection (1) of section eight and anything done for the
purpose of producing such income”.
The expression “for the purposes of trade” has been held to mean “for the purpose of enabling a
person to carry on and earn profits in the trade”: Strong & Co of Romsey Ltd v Woodifield (Surveyor of
Taxes) HL (1906) 5 TC 215. Clearly the ordinary recurrent expenses of a business, such as trading licence
fees, rates, audit fees, secretarial fees, insurance premiums, business subscriptions, legal expenses in the
recovery of trade debts, removal expenses relating to trading stock and most advertising costs will usually
pass the test.
Also allowable, however, on the basis of that test, is expenditure, again provided it is not of a
capital nature, incurred for the purpose of avoiding or reducing future costs. In the context of the opening
comments above, an example of a broader application of the test in South Africa, which would represent
the position in Zimbabwe, arose in ITC 1600 (1996) 58 SATC 131. In that case expenditure incurred by the
taxpayer on securing early termination of onerous rental agreements for the use of mainframe computers, in
order for it to switch to the use of personal computers, was held to be deductible.
As to the meaning of “venture” see Burgess v CIR (1993) 55 SATC 185 at 196, where it was held
to include participation in a commercial speculation through risking money to obtain a bank guarantee in
the hope of making a profit.
(b) Expenditure for the purposes of trade may be categorised in two ways, as in COT v Rendle (1965)
26 SATC 326:
“First, money voluntarily and designedly spent by the taxpayer for the purpose of his trade; and second,
money which is what I might call involuntarily spent because of some mischance or misfortune which has
overtaken the taxpayer. For the sake of convenience, I will refer to the first type of expenditure as
‘designed expenditure’, and to the second as ‘fortuitous expenditure’.
The deduction of designed expenditure (so far as the law is concerned) presents little difficulty. Provided it
is designedly and bona fide incurred ... for the purpose of trade, it is deductible, no matter how rash or
unnecessary the expenditure might be. It is not for the Commissioner to direct how a taxpayer should run
his business. The deduction of fortuitous expenditure, however, presents a very difficult legal problem....
Before fortuitous expenditure can be deducted, the taxpayer must show that the risk of the mishap which
gives rise to the expenditure happening, must be inseparable from or a necessary incident of the carrying on
of the particular business.” .
The latter approach to fortuitous expenditure was adopted in ITC 1221 (1974) 36 SATC 233, in
which a loss, suffered by a departmental store, of money which had been embezzled by its

45
secretary/accountant was held to be deductible. The court indicated that the same would not apply to theft
by a managing director, a director or a manager in the position of a proprietor. The court in ITC 1661
(1999) 61 SATC 353, applying the “concomitant” test to which reference is made below, held that losses
incurred in a dental practice arising out of fraudulent conduct by an auditor, being an independent
contractor, and his staff, were not deductible
In A S Joffe & Co (Pty) Ltd v CIR (1946) 13 SATC 354 the court refused to allow a deduction, in the
hands of a firm of reinforced concrete engineers, of damages paid to the dependants of a workman whose
death had been caused by the negligence of the company in carrying out one of its contracts. It was held
that negligent construction was not a necessary concomitant of the trading operations of a reinforced
concrete engineer. The position is different, however, and damages are deductible, if they are paid in terms
of a guarantee given in order to obtain the contract in question: COT v Cathcart (1964) 27 SATC 1.
Another case illustrative of this “concomitant” approach is Port Elizabeth Electric Tramway Co Ltd v CIR
(1936) 8 SATC 13. A driver of one of the company’s vehicles lost control of it and, in the resulting
collision, suffered fatal injuries. Prior to his death the driver claimed compensation under the Workmen’s
Compensation Act. The company was compelled to pay compensation to the widow. It had also incurred
legal costs in trying to resist the claim. The compensation was found to be deductible but not the legal
costs. Had the case been decided under the Zimbabwe Act both amounts would probably have been
allowable.
In Morgan v Tate & Lyle Ltd (1954) HL 35 TC 367, the company had incurred substantial expenditure on a
publicity campaign resisting the threat of nationalisation of its assets. Such expenditure was held to have
been incurred for the purposes of the company’s trade. (It appears that the result would have been different
had expenditure been incurred by shareholders in defending ownership of their shares.)
(c) The fact that a business is conducted in such a way as to contravene the law does not prevent
operating expenses being allowed as deductions: CIR v Delagoa Bay Cigarette Co Ltd (1918) 32 SATC 47.
(d) Deductibility does not, however, extend to expenditure which, though arising out of the manner in
which a taxpayer conducts his trade, falls upon him in his capacity as a law-breaker rather than as a
businessman. Traffic fines are a common instance. These have been disallowed in the case of, for example,
a transport company, including those incurred where its employees had little choice but to break the law in
carrying out their duties: ITC 1199 (1974) 36 SATC 16. The same result has arisen: (i) where the taxpayer
company, which was a dealer in caravans, paid, in the interests of goodwill, a fine imposed on a potential
customer during a test drive: ITC 1212 (1974) 36 SATC 108; and (ii) where the taxpayer company, again a
transport contractor, committed offences on the instructions or at the request of its customers: ITC 1490
(1991) 53 SATC 108.
(e) Interest payable on borrowings used for business purposes is generally allowable.
Interest payable by a company is not deductible where the funds have been borrowed in order to
pay a dividend: ITC 678 (1949) 16 SATC 348. This decision has frequently been reached also where a
company, having declared a dividend and credited the shareholders’ loan accounts, no money changing
hands, incurs interest on such loans: COT v Avenue Buildings (Pvt) Ltd (1963) 25 SATC 366, ITC 1517
(1992) 54 SATC 109 and CIR v G Brollo Properties (Pty) Ltd (1994) 56 SATC 47. It has been held to
apply also where a South African “close corporation” made a distribution to members on the footing that
the amount would immediately be lent back, at interest: CIR v Elma Investments CC (1996) 58 SATC 295.
In ITC 1643 (1999) 61 SATC 12, however, a deduction of interest payable on a dividend credited to a
(family) shareholder’s loan account was allowed. Such indebtedness had effectively enabled the company
to retain an equivalent amount on call account which not only earned a higher rate of interest but also
served a business function in assisting the company to obtain contract guarantees from financial
institutions. (The company did not lack the funds to have paid the dividend, a factor on which the court
distinguished previous cases.)
(Although outside the context of interest payable it is observed here that other costs incurred by a
company in distributing dividends are accepted by the Commissioner as deductible. Departmental practice
No. 16 refers.)
Expenses incurred by an individual in order to secure an office or employment may transpire to be
of a capital nature (ITC 1139 (1970) 32 SATC 83) or, as in CIR v Shapiro (1928) 4 SATC 29, may be
thought to lack a sufficiently close connection to be deductible.
In other circumstances, however, taxpayers have been successful. Where, for example as a condition for
becoming entitled to increased emoluments, an employee was required to purchase shares in the company
which employed him, interest payable by him on funds borrowed to purchase the shares was held to be

46
deductible: ITC 1428 (1988) 50 SATC 34. The taxpayer was successful also in ITC 1504 (1991) 53 SATC
349. He was a consulting civil engineer who, when he was a partner in a practice, had used an interest-
bearing loan to contribute to working capital. When the practice was sold to a company each partner was
obliged to purchase shares. The taxpayer did so out of capital repaid to him from the winding up of the
partnership, thus re-using the borrowed funds. Interest subsequently payable was held to be deductible
against his salary as a director. (See also ITC 1620 (1997) 59 SATC 316.)
In CIR v D G Smith (1998) 60 SATC 397, the court (upholding the lower court’s decision in ITC 1603
(1996) 58 SATC 212 and distinguishing ITC 1583 (1995) 57 SATC 58) approved the deductibility of
interest payable by a partner on funds borrowed to replenish his capital account which he had depleted for
private purposes.
In ITC 1604 (1996) 58 SATC 263 the taxpayer, who had borrowed funds in order to acquire an interest in
a South African close corporation, was also successful, the basis of his claim having been that such
acquisition was closely connected with his earning of an increased salary and a bonus.
In earlier cases where management fees, etc., have been earned from a company in which shares have been
acquired it has sometimes been possible to show (i) that interest payable on loans used to acquire the shares
is partially deductible: ITC 1521 (1992) 54 SATC 175; though the claim failed in ITC 1553 (1993) 55
SATC 105; or (ii) that expenditure is almost entirely connected with the earning of fees and that only a
nominal sum should be disallowed: ITC 1589 (1995) 57 SATC 153.
Where a company incurs expenditure which benefits also, for example, an associated company,
there must, to obtain a deduction, be a close connection between the expenditure and the taxpayer
company’s own activities.
This was emphasised in ITC 1124 (1969) 31 SATC 53. The taxpayer was a plantation company in a group
which included a saw-milling company. To ensure constant through-put to the latter the taxpayer company,
using borrowed funds, purchased shares in other plantation companies. Interest payable on the borrowings
was held not to be deductible. (See also ITC 1566 (1994) 56 SATC 34.)
In CIR v Sunnyside Centre (1996) 58 SATC 319, the company paid interest in excess of the interest
recoverable from its holding company. Although the lower courts had accepted that the company was not
merely incurring the excess for the benefit of its holding company but that its financial well-being and
future were intimately bound up with the success of the group, this finding was overruled in the Appellate
Division, which upheld the Commissioner’s disallowance.
In other cases (involving South African “close corporations”) the disallowance of interest payable by the
corporation itself has been upheld where the court has considered that the corporation’s own welfare has
not been furthered by the indebtedness. This has occurred in Natal Laeveld Boerdery BK v KBI (1998) 60
SATC 81, where a loan was used to buy out the interests of one of the members, and in Ticktin Timbers CC
v CIR (1999) 61 SATC 399 where it was found that the crediting of a distribution to the account of the sole
member, and the incurring of interest on the resultant debt, was intended to increase the income of the
member and not that of the taxpayer.
In ITC 1625 (1997) 59 SATC 383, however, a company’s claim to deduct interest payable against property
rentals was upheld by the court despite the acquisition of the property having also been in furtherance of
the interests of family shareholders of the company: see 17.2.6.
(f) Further areas of dispute are those where a private or personal element is present, such as where the
taxpayer claims to use privately-owned assets, including a portion of his home, for the purposes of his
trade, particularly where he is in employment.
(i) The following cases were decided under the South African legislation, which contains
important differences in wording as compared with the Zimbabwe Act. It is submitted, however, that they
represent likely results in similar circumstances in this country.
ITC 1381 (1984) 46 SATC 75. A medical practitioner in Durban claimed the cost of white safari suits
which for climatic reasons he wore during practice hours and which, he averred, were associated with the
profession by patients. His claim was disallowed.
KBI v Van der Walt (1986) 48 SATC 104. A University lecturer maintained at his home a study where he
spent more than 50% of his working time. The study was, apart from some very small incidental purposes,
entirely used for the furtherance of his lecturing duties for which, the court found, it was essential. The
maintenance costs were held to be allowable.
(This result is important since the taxpayer concerned won his case despite the fact that
he was an employee and was not contractually obliged to undertake the expenditure in dispute.)

47
ITC 1434 (1988) 50 SATC 77. A salaried school-teacher, teaching computer studies and, in the process,
developing computer-based educational programmes, purchased a micro-computer and related software.
These were essential to a proper discharge of his duties though he was not obliged to acquire them. The
court held that he was entitled to capital allowances.
(ii) The deductibility of expenditure (not being of a capital nature) was considered in L v
COT (1992) 54 SATC 91. A legal practitioner had undergone an eye operation which was essential to her
ability to continue in practice. It was held nevertheless that the costs were too remote from her income-
producing operations.
(g) Where over-all losses are incurred disallowances, on the ground that a trade has not been carried
on, have given rise to disputes on many occasions.
In the context of losses on the letting of property, taxpayers’ appeals failed in ITC 1292
(1979) 41 SATC 163 and ITC 1385 (1984) 46 SATC 111. The former concerned the intermittent letting of
a seaside house, at which the taxpayer himself spent vacations, and the latter a letting of a house pending
the taxpayer taking up residence. It is considered that under the Zimbabwe legislation disallowances would,
on similar facts, be sustained. In a Zimbabwe case, ITC 1653 (1999) 61 SATC 120, a loss on the letting of
property, incurred prior to the taxpayer herself taking up residence there, similarly was disallowed. This
decision was reached in the context of the taxpayer, a chartered accountant, having failed to establish in
advance the viability of the proposition and having foreseen the possibility of making the property her
home. In an instance, however, where the taxpayer company let a property to best advantage after its
original intention of demolishing existing buildings and replacing them with a block of flats was frustrated,
losses were held to be allowable: ITC 1342 (1981) 43 SATC 224. In ITC 1592 (1995) 57 SATC 247
similarly the taxpayer succeeded despite having set, for the first five years, only a “break-even” target.
Outside the property field, in J v COT (1993) 55 SATC 62, concerning the breeding and training
of horses for trotting and racing, the court had regard to the diversity and scale of the taxpayer’s operations
and concluded that he had the necessary profit-making intention (towards what had once been a hobby) and
that his loss was therefore allowable.
A number of instances have arisen in the context of farming and are accordingly dealt with in
chapter 11. The matter is considered also at 7.6.2.
(h) Amounts payable by an employer to his employees for services rendered are generally allowable,
being for the purposes of the employer’s trade, regardless of whether they are contractual, where the
employee is continuing in service. Contractual payments on termination of service, or “golden
handshakes”, are also allowable (ITC 414 (1938) 10 SATC 249) but voluntary awards in these
circumstances are allowable only: (i) if they are made in accordance with the general practice of the
particular employer: W F Johnstone & Co Ltd v CIR (1951) 17 SATC 235; or (ii) it is submitted, applying
the test in the Anglo-Persian Oil case, (see 7.6.1(d)) if the employee’s dismissal was calculated to benefit
the employer’s operations.
Certain payments in relation to former employees, which might fail to qualify for deduction under
section 15(2)(a), may qualify under section 15(2)(q): see 8.12.
(i) Where fees are paid in relation to income tax matters, only those applicable to the preparation of
computations and returns are allowable, but see also 8.21.
(j) Although, as remarked earlier, excessive expenditure resulting from bad business administration
or supervision is deductible, the Commissioner is entitled to disallow excessive expenditure on the ground
that it is not properly classifiable as expenditure incurred for the purposes of trade or in the production of
the income, commonly because it is based on some non-business, or even ulterior, motive.
Instances are the paying of remuneration to members of a family or to a family trust or of
excessive directors’ fees from small private companies (ITC 1214 (1974) 36 SATC 155 and ITC 1518
(1992) 54 SATC 113. But in Tobacco Father v COT (1950) 17 SATC 395 remuneration paid to a son
proved to be reasonable in the circumstances of the particular trade.
Interest falls within the same principle. A deduction of the amount payable to a related company
was restricted in ITC 1530 (1992) 54 SATC 261.
(In many instances donations made from charitable motives similarly fail simply by reason of
being made for non-business purposes. Some, however, are nevertheless deductible: see 8.13.)
(k) Expenditure incurred prior to cessation, not for the purposes of carrying on business but to go out
of business (such as pay in lieu of notice) is disallowable, as in Godden v A Wilson’s Stores (Holdings) Ltd
(1962) 40 TC 161.

48
Expenditure incurred after the cessation of business is also usually disallowable, as in
Amalgamated Zinc (de Bavay’s) Ltd v FC of T (1935) 54 CLR 295. The cases are, however, not always
consistent. In 4 CTBR (NS) Case 127 such expenditure was held to be allowable. The occasion of the
outgoing was found to lie in the earlier trading operations; the expenditure in issue had been in settlement
of a trading dispute which had arisen during such operations.
The factual question of whether trade has ceased or commenced, resulting in it being impossible for
expenditure to be incurred “for the purposes of trade” is also clearly important.
In this connection the court found in Timberfellers (Pty) Ltd v CIR (1997) 59 SATC 153 that in the
circumstances of that case the collection of debts after all income-generating activity had ceased did not
constitute trading. From the comments at 164, however, it appears that trade could be considered to have
continued in a case of temporary discontinuance with the intention of a resumption of operations.
Similarly: In Robin Consolidated Industries Ltd v CIR (1997) 59 SATC 199 and C: SARS v Contour
Engineering (Pty) Ltd (1999) 61 SATC 447 the court ruled against the taxpayers. In the former case the
court considered that the realisation, through two transactions by liquidators, of stock held in bond, did not
constitute trading. In the latter the court’s similar ruling related to the mere laying of plans for further trade
i.e. without some actual preparatory activity (such as, it appears, staff recruitment.) The taxpayer was again
unsuccessful in ITC 1672 (2000) 62 SATC 47, where interest which continued to be incurred after the
cessation of trading was, in the circumstances, disallowed.
In relation to the commencement of trade, problems such as the above are frequently overcome
through the operation of the “18-month rule” in section 15(2)(t) of the Act: see 8.15
(l) Where a tradesman purchases his own tools or protective clothing for use in his employment the
Commissioner accepts that the cost is deductible. Departmental practice 11 refers.
7.5 Or in the production of the income
While it might appear that the ability to obtain a deduction under the wording “for the purposes of trade”
has made it superfluous to retain the phrase “in the production of the income” , there are still instances
where the latter is significant. An example is the incurring, by a small investor, of agent’s fees in the
collection of interest (of a taxable nature).
7.6 Not of a capital nature
In the same way as accruals of a capital nature are generally not subject to income tax, so expenditure and
losses, to the extent that they are of a capital nature, are not deductible from income unless the Act
specifically so provides (see chapter 8).
7.6.1 Expenditure
(a) It is often difficult to determine when an item of expenditure is of a capital nature. Despite the
many decisions of the courts on this subject it is not possible to lay down any hard and fast rules. In order
to make a determination it is necessary to investigate the purpose for which, as well as the circumstances in
which, each item of expenditure is incurred.
A general test was laid down in CIR v George Forest Timber Co Ltd (1924) 1 SATC 20:
“Money spent in creating or acquiring an income-producing concern must be capital expenditure. It is
invested to yield future profit and, while the outlay did not recur, the income does. There is a great
difference between money spent in creating or acquiring a source of profit and money spent in working it.
The one is capital expenditure, the other not. The reason is plain; in the one case it is spent to enable the
concern to yield profits in the future, in the other it is spent in working the concern for the present
production of profit”.
A judgment in New State Areas Ltd v CIR (1946) 14 SATC 155 provides a further test:
“The problem which arises when deductions are claimed is therefore usually whether the expenditure in
question should properly be regarded as part of the cost of performing the income-earning operations or as
part of the cost of establishing or improving or adding to the income-earning plant or machinery. The
conclusion to be drawn from all these cases seems to be that the true nature of each transaction must be
inquired into in order to determine whether the expenditure attached to it is capital or revenue expenditure.
Its true nature is a matter of fact and the purpose of the expenditure is an important factor: if it is incurred
for the purpose of acquiring a capital asset for the business it is capital expenditure even if it is paid in
annual instalments; if, on the other hand, it is in truth no more than part of the cost incidental to the
performance of the income-producing machine, then it is revenue expenditure even if it is paid in a lump
sum”.
Applying these principles it is clear that capital expenditure includes that on for example: (i) the
acquisition of fixed assets (including travelling to obtain them); (ii) improvements and alterations to fixed

49
assets; (iii) the acquisition of intangibles such as trade marks and goodwill; (iv) company formation and
preliminary expenses (though in the latter case section 15(2)(t) may apply: see 8.15); and (v) the obtaining
of its share capital by a company and the cost of alterations to its capital or (subject to the Carron principle,
referred to below) to its memorandum or articles of association.
By contrast operational expenses such as those listed at 7.4(a) are allowable as being of a revenue
character.
The approaches adopted in the George Forest and New State Areas cases have been cited with
approval on many subsequent occasions and are regarded as standard, as is the dictum in a judgment in
Atherton v British Insulated and Helsby Cables Ltd, HL (1925) 10 TC 155, i.e. that expenditure is normally
capital if “made, not only once and for all, but with a view to bringing into existence an asset or an
advantage for the enduring benefit of the trade” .
As was observed at 4.2 the question whether expenditure is of a capital or revenue nature in the
hands of the payer cannot be determined by reference to the nature of the amount correspondingly
accruing to the recipient.
(b) The question of the revenue or capital nature of expenditure has reached the courts in a wide
variety of circumstances, some of which are now considered.
(i) As a result of the decisions in CIR v Genn and Co (Pty) Ltd (1955) 20 SATC 113 and
ITC 1019 (1962) 25 SATC 411 it is accepted that, where a loan is utilised for the purposes of revenue
expenditure (including the acquisition of working, also called “floating”, capital) raising fees and guarantee
commissions are allowable deductions. Raising fees, etc., in connection with a loan used for the acquisition
of fixed assets, however, are disallowed, in accordance with ITC 1019. Departmental practice 13 authorises
a proportionate allocation of these costs where the loan has been used for mixed purposes.
Interest on a loan used in a taxpayer’s trade is also deductible. This generally applies
even if the loan has been used in the acquisition of fixed assets: ITC 1124 (1969) 31 SATC 53. See also
ITC 1604 (1996) 58 SATC 263 at 270-1 as to whether interest may be of a capital nature.
(ii) The New State Areas case, referred to above, concerned the costs of connecting the
taxpayer’s business premises with the electrical and sewerage services supplied by the local authority. Such
costs were held to be deductible. The case is authority for the deduction of connection fees generally, but
not including the cost of installations, wiring, etc., which belong to the taxpayer.
(iii) The “acquisition of assets” test and the distinction between “operational” and “structural”
expenditure featured strongly in the following decisions.
In CIR v Carron Co, HL (1968) 45 TC 18 the company had been incorporated, over two hundred years
earlier, by charter which hampered it by restricting its borrowing powers and by preventing the introduction
of a modern management structure. The company petitioned for, and obtained, a supplementary charter to
modernise the existing one. Substantial legal and other expenses were incurred in removing these
impediments and in overcoming the opposition of two shareholders. The expenditure was held to be of a
revenue nature.
Significantly the court had regard to the fact that the expenditure had not resulted in the acquisition of a
new tangible or intangible asset and had not created a new branch of existing trading activities. Nor had
additional fixed capital been obtained. The removal of a disability did not equate to the acquisition of an
additional advantage.
In Palabora Mining Company Ltd v SIR (1973) 35 SATC 159 the company, which urgently required water
supplies for the purposes of the production of copper, arranged a contract whereby it would assist in the
construction of a barrage across a major river, to serve the area in which its mine was situated. The
company paid special inducements to its subcontractors in order to ensure early completion, and thereby
avoided having to wait for the following season’s rainfall. It incurred a substantial loss on the contract.
The lower court had found that the company’s expenditure (or loss: see below) was of a capital nature,
having been directed towards equipping the structure of the mine by giving it a permanent supply of water.
This finding was, however, reversed in the Appellate Division, having regard to the fact that: (i) the barrage
was erected on land which did not belong to the company; (ii) the company had acquired no proprietary
interest in the water rights, etc.; and (iii) the barrage would in any event have been completed by the local
Water Board.
In CIR v Manganese Metal Company (Pty) Ltd (1996) 58 SATC 1, where recurrent expenditure had been
incurred on the construction of a dam to facilitate the disposal of toxic waste products, such features were
absent. It was this, among other things, which caused the expenditure to be classified as being of a capital
nature (the judgment containing a substantial review of case law).

50
In ITC 1528 (1992) 54 SATC 243 the taxpayer-company’s operations had infringed another party’s
copyright. The company incurred legal expenses in opposing the copyright-holder’s application for an
interdict. The expenses were held not to be of a capital nature; they were not designed to create an asset but
to try to remove an obstacle. The judgment cited, among other cases, that of Anglo-Persian Oil Co. Ltd v
Dale (1931) 16 TC 253; expenditure incurred by the company in order to terminate an agency through
which it had been trading, and thereafter to manage its own affairs, was held to be deductible.
Taxpayers were successful also in:
ITC 1452 (1989) 51 SATC 97. A large proportion of the business comprised imported items, for which
import permits were essential. A fee, paid by the taxpayer to a “consultant” for obtaining unutilised
portions of other importers’ permits, was held not to be expenditure of a capital nature, and therefore to be
deductible, being incurred in the acquisition of trading stock;
1TC 1594 (1995) 57 SATC 259. Fees paid to a consultant, for rationalisation of the taxpayer-company’s
procedures and activities in a post-merger situation, were held to be of a revenue nature.
A claim failed however, in Seeff Properties CC v CIR (1998) 60 SATC 407. The taxpayer corporation,
having some years earlier granted another party a franchise over an area, cancelled it by effecting a
payment, thereby becoming able to operate in that area. It was held to have acquired an income-producing
asset (in contrast to the facts in the Anglo Pension Oil case) by reason of re-acquiring the right to trade in
the area concerned.
(iv) Expenditure may relate to the elimination of competition and to restraint of trade (where
the question of the creation of an asset or “enduring advantage” or the “income-earning structure” may
again arise).
Where legal expenses had been incurred in an (unsuccessful) attempt to secure a protracted, if not
permanent, restraint on particular operations of a potential competitor, such expenses were held to be of a
capital nature: SIR v Cadac Engineering Works (Pty) Ltd (1965) 27 SATC 61. An amount paid by
agreement in consideration of the recipient ceasing production for one year, however, was held to be
deductible: COT v N’Changa Consolidated Copper Mines Ltd (1964) 26 SATC 37.
In CIR v VRD Investments (Pty) Ltd (1993) 55 SATC 368 the taxpayer was a member of a consortium (of
milk distributors) whose market had been penetrated by a competitor. The taxpayer incurred net
expenditure on acquisition of the latter’s business and resale of its assets. The expenditure was held to be
apportionable on the basis of 75% as revenue expenditure and 25% as capital. A crucial factor was that the
competitor had been expected to collapse and that its elimination had been merely accelerated.
(v) The acquisition of contractual rights is an area where the cases clearly indicate that
caution on the part of the payer is needed.
In ITC 1063 (1965) 27 SATC 57 a distributor of gramophone records paid a manufacturer a fixed sum for
sole distribution rights for three years. The expenditure was held to be of a capital nature (and not to
constitute a royalty).
Again, in ITC 1365 (1983) 45 SATC 27 a mining company claimed a deduction in respect of a payment
described as a royalty. The company had acquired, by cession from the original lessee, the rights to a
mining lease and the payment was found to be part of the payment for the acquisition of such lease. The
company’s claim for a deduction accordingly failed. (The result of the case was financially disastrous for
the company which was, in the face of non-deductibility, unable to make a profit on the operation, whereas
had actual royalty payments been made they would surely have been deductible.)
A similar result arose in Rand Mines (Mining & Services) Ltd v CIR (1997) 59 SATC 85 where
expenditure by a mine management company on acquiring a contract to manage a mine was held to be of a
capital nature.
(vi) Expenditure by an individual in the acquisition of further knowledge with which to carry
on his trade is frequently of a capital nature.
This arises under the general tests in the George Forest Timber and New State Areas cases referred to
above. A distinction was, however, drawn in ITC 1433 (1988) 50 SATC 73. A computer analyst, practising
as a consultant, incurred expenditure in travelling overseas periodically to acquaint himself with recent
developments and techniques in the computer industry. Such expenditure was held to be deductible. The
court had regard to, among other things, the fact that the knowledge which the taxpayer acquired was not,
in the circumstances, of an enduring nature; the developments with which he had become acquainted were
of use to him only until such time as they were replaced by new developments.
(vii) A further test which has been adopted by the courts in identifying expenditure as being of
a capital nature is that it has been incurred in the protection of capital assets.

51
In ITC 1598 (1996) 58 SATC 35 the taxpayer, a farmer, had earlier instituted proceedings in a Water Court
for an order in a dispute over whether he possessed rights to an allocation of water from a river. His legal
costs in such action were held to constitute expenditure of a capital nature, having been incurred in the
protection of a capital asset.
In ITC 1648 (1999) 61 SATC 58 the taxpayer, again a farmer, incurred legal expenses in trying to resist an
action to eliminate boreholes which he had sunk on his farm. It was held that the expenditure, having been
incurred to preserve or protect capital assets, was of a capital nature; it related more to such factors than to
the farming operations themselves and conferred an enduring benefit.
Where, however, expenditure relates not to the protection of (i) a species of property, including a
contractual right, or (ii) of goodwill, but to, for example, the protection of an individual’s ability to
continue in his profession, it may be found not to be of a capital nature, as in Smith v CIR (1968) 30 SATC
35 (where the Appellate Division’s conclusion ran counter to that reached by a lower court in the same
taxpayer’s earlier appeal, on the identical point in relation to the previous year of assessment: ITC 992
(1963) 25 SATC 129). Further, there are cases from English courts, such as Southern v Borax Consolidated
Ltd (1940) 23 TC 597, where it was been held that expenditure incurred to protect and preserve business
assets (without altering or enhancing them, or creating a new asset) is on revenue account. Such a view is,
it is submitted, preferable.
(viii) A landlord’s costs, such as compensation, in evicting one tenant in order to obtain a
higher rental from another, are allowable in terms of the decision in ITC 1267 (1977) 39 SATC 146 but not
if incurred in order for the landlord either to alter the use of the property by, for example, carrying on
trading operations himself (ITC 819 (1955) 21 SATC 71) or, it seems clear, to sell a building held as a
capital asset.
(ix) Legal expenses, as has been indicated, constitute a major area where the revenue or
capital nature turns on the subject matter in issue.
Where, for example, as commonly arises, such costs are incurred in the preparation of a
lease agreement over a business property, the landlord’s expenditure is deductible as part of the cost of
operating his asset, whereas the tenant’s cost is of a capital nature since it relates to his acquisition of rights
under the lease. Legal expenses in relation to, for example, the formation of a company, or the conveyance
of fixed property constituting a capital asset, are also of a capital nature. Further examples have been seen
in cases such as SIR v Cadac Engineering Works (Pty) Ltd (1965) 27 SATC 61 and Smith v CIR (1968) 30
SATC 35, referred to above.
7.6.2 Losses
As in the case of expenditure, disputes have arisen over the revenue or capital nature of a loss. (The
alternative of whether expenditure or loss was incurred “for the purposes of trade” has already been
examined. The result goes against the taxpayer, of course, if he fails on either ground.) It will be observed
that the concept falls into two distinct categories, namely (i) whether a loss was so incurred in the sense of
having deliberately been sustained in an attempt otherwise to make a profit, and (ii) where it has arisen
rather as the involuntary result of his operations.
(a) Losses relating to the ownership and use of property have frequently given rise to dispute.
Where the building is vacant, expenses such as rates and bond interest are normally disallowable,
though they may prove to be deductible if active steps have been taken, for example, to find a tenant.
In Borstlap v SBI (1981) 43 SATC 195 the taxpayer was unsuccessful. He had purchased a
property with the intention of demolishing the existing buildings which were let, and replacing them with a
block of flats. Bond interest payable in the meantime exceeded the rentals being earned. The deduction was
restricted to the amount of the rentals.
(b) The matter has arisen on a number of occasions, in the context of farming operations, particularly
in relation to individuals: see chapter 11.
(c) The danger attaching to a claim for losses incurred in the developmental stage of operations was
illustrated in ITC 1403 (1985) 47 SATC 226. The taxpayer, who was in full-time employment, earned fees
also as a freelance journalist. Such fees were exceeded each year by expenditure incurred both in earning
them and in order to build up a market for his future work after his retirement. The excess expenditure,
constituting his loss, was held to be of a capital nature.
(d) The term “dividend-stripping” generally describes operations (with variations) whereby shares are
purchased for their short-term dividend distribution potential, the dividend being received and the shares
then sold at a loss. Losses on such schemes have consistently been disallowed in recent cases such as CIR v
Nemojim (Pty) Ltd (1983) 45 SATC 241, De Beers Holdings (Pty) Ltd v CIR (1985) 47 SATC 229, Gerber

52
v CIR (1989) 51 SATC 183, and CIR v SA Mutual Unit Trust Management Company Ltd (1990) 52 SATC
205. (Apportionments and formulae have been adopted in recognition of the partially taxable nature of the
dividends under South African law at the time.)
(e) The tests for whether a loss is of a capital nature assume great importance when a taxpayer has
made a loan which proves to be irrecoverable. Section 15(2)(g) does not apply since it refers only to bad
debts in respect of amounts which have been included in the taxpayer’s taxable income for any year. The
question is, therefore, whether the loss constitutes a permissible deduction under section 15(2)(a). If it is
clear that the taxpayer is carrying on a business of moneylending, his cash resources, as in the case of a
bank, represent his goods or (see 4.6) floating capital. He is “dealing” in money, and losses will therefore
fall under the general deduction formula. The difficulty arises mainly in distinguishing between the
carrying on of a business of moneylending and the mere investing of money.
Whether or not a taxpayer is a moneylender or carries on the business of a moneylender depends
on the facts of the case. The criteria were summed up in ITC 1270 (1977) 40 SATC 65, at 69:
“(a) that the lending occurred according to a system or plan disclosing a degree of continuity
in the turnover of capital;
(b) that the money was lent to make a profit;
(c) that security was obtained (this is a usual but not an essential feature);
(d) the proportion of income from loans to ordinary income (but the smallness of the
proportion is not decisive);
(e) that a reasonably high rate of interest applied.”
On the facts, although a substantial number of the taxpayer’s transactions fulfilled criteria (b) to (e), there
was no coherent system of lending and a significant number of transactions were influenced by personal
considerations or coupled with an equity investment. The taxpayer was therefore held not to be a
moneylender.
In the same case the Special Court relied also on a dictum in Stone v SIR (1974) 36 SATC 117
that irrecoverable loans are deductible:
“... provided that the business is purely that of moneylender and the loans are not made in order to acquire
an asset or advantage calculated to promote the interests and profits of some other business conducted by
the taxpayer”.
In ITC 1363 (1983) 45 SATC 17 the taxpayer was an investment-holding and management company
which, in the furtherance of the operation of its subsidiary company, advanced monies to a third party. The
loss of such monies was held to be of a capital nature as the loan was not part of the holding company’s
ordinary business and had been made with a view to securing an enduring advantage through the
enhancement of the subsidiary.
A loss incurred under agreements of suretyship or guarantee was disallowed in CIR v Hilewitz (1998) 60
SATC 86. The taxpayer had given the suretyships to creditors of a company of which he was a director and
shareholder, thus enabling the company to continue trading. The company had in return become obliged to
pay him certain emoluments. His loss was incurred when the company, despite the arrangements, was
liquidated. It was held that the loss was of a capital nature by reason of the suretyships having been granted
to try to ensure the continued existence of the company which was the structure enabling the taxpayer to
earn his emoluments.
It is possible also for moneylending loans to become converted to capital account.
This was found to have occurred in ITC 1419 (1987) 49 SATC 45. The taxpayer was a partner in an
accountancy practice which had engaged in moneylending operations to further the interests of the firm.
The business was transferred into a company, the consideration being left on loan account. When the
company subsequently failed the taxpayer’s loss was held to be of a capital nature.
Where, in a group of companies, one acts in a capacity akin to that of group treasurer, conducting
its operations for its own benefit, on an arm’s length basis with its associated companies and in accordance
with the tests laid down in ITC 1270 above, it is submitted that losses on such operations are allowable.
Any variation, however, particularly if the taxpayer company’s actions are for the benefit of the group, is
likely to lead to the failure of a claim, as occurred in Plate Glass and Shatterprufe Industries (Finance
Company) (Pty) Ltd v SIR (1979) 41 SATC 103 and in Solaglass Finance Co (Pty) Ltd v CIR (1991) 53
SATC 1.
In ITC 1650 (1999) 61 SATC 72 the court found, despite the possibility that other loans by the taxpayer
had been made as money-lending transactions, that the loan in issue was not. Particular regard was had to
the fact that this was a long-term loan and that it had no repayment stipulations; it thus lacked the character

53
of floating capital and would, if it were irrecoverable, constitute a loss of a capital nature. Other issues
were: that since the taxpayer had participated in a scheme of arrangement, which it had instigated, in terms
of the South African Companies Act, there had not been the necessary “involuntary deprivation”; and that
the taxpayer’s loss, if any, had not arisen in the year of assessment concerned.
Clearly, therefore, difficulties face the taxpayer, who is not a moneylender or banker, in claiming a
loss on an irrecoverable loan. Nevertheless, in the light of comment in Sentra-Oes Koöperatief Bpk v KBI
(1995) 57 SATC 109 at 120, it does not necessarily follow that such loss will in all other circumstances be
on capital account.
Indeed in ITC 807 (1955) 20 SATC 338 the taxpayer company, carrying on the business of wholesale
merchants, was successful. It had, in order to overcome difficulties in obtaining supplies, made cash
advances to suppliers including a manufacturing company. One of the advances to the latter proved to be
irrecoverable. The court held the loss to be deductible on the grounds that it was integrated in and an
adjunct to the taxpayer’s ordinary trading.
Applying, among other things, the Genn test the court found in CIR v Creative Productions (Pty) Ltd
(1999) 61 SATC 106 that where a manufacturer, having received an amount by way of a promissory note,
discounted the note at a loss in order to raise further working capital, such loss was deductible.
Quite apart from the question of carrying on a moneylending trade, cases of speculative schemes
involving a combination of a loan and a shareholding have reached the courts, with varying results.
The taxpayer in Burman v CIR (1991) 53 SATC 63 made an advance to, and took up shares in, property-
development companies. The overall objective was a profit-making scheme although the loan per se could
not have produced a profit. The loss which eventuated on the loan was disallowed. (See also ITC 1621
(1997) 59 SATC 324 in which at 332 an earlier decision in ITC 1344 (1982) 44 SATC 19 was considered
no longer to be good law.)
In the case of loans to employees, if it is the custom of the trade to make such loans to ensure the
retention of a satisfactory labour force, those which prove irrecoverable are deductible: ITC 870 (1959) 23
SATC 79.
(f) Exchange rates
As in the case of profits on exchange rates, considered at 4.15, the question of whether a loss,
resulting from differences in exchange rates, is on revenue or capital account turns on the nature of the
transaction underlying such loss.
In CIR v General Motors SA (Pty) Ltd (1981) 43 SATC 249, a manufacturer had sustained exchange losses
in repaying foreign loans which had been used to finance the purchase abroad of stocks for its business.
Such losses were held to be of a revenue nature.

As to the year in which exchange losses on revenue account are deductible see 8.30.
7.7 Prohibited deductions
As has been seen, the “general deduction formula” contains its own restrictions. Further restrictions on
deductibility arise under section 16, parts of which forbid a deduction despite the expense passing the tests
of purposes of trade, non-capital nature, etc.. Other parts merely ensure a disallowance of certain items of
expenditure, the deductibility of which might be in doubt.
Since section 16(l) imposes prohibitions unless a deduction is “expressly provided” elsewhere in the Act,
the prohibitions apply only to claims under the “general deduction formula” in section 15(2)(a), all other
deductions in the Act being expressly provided. Prohibitions under section 16(1) relate to the following.
(a) The cost incurred in the maintenance of the taxpayer, his family or establishment.
(b) Domestic expenses, including travelling between home and place of business or between two
entirely distinct trades.
(i) A taxpayer who carries on trade at home and travels to other places where he carries on
the same trade may avoid both of these prohibitions. The point is one of some difficulty, as illustrated by
the following.
In ITC 1206 (1974) 36 SATC 56 a prospector operating from an office in his home was allowed to deduct
his expenses in travelling from that office to prospecting sites. But in ITC 1253 (1976) 38 SATC 181 a
taxpayer who carried on the trade of letting houses failed to establish that he maintained a place of business
in his home. His travelling expenses from his home to the houses let were caught by the section.
A similar result arose in relation to certain expenses incurred by the appellant in ITC 1437 (1988) 50 SATC
154. He was a jockey, based in Harare, who raced from time to time in Bulawayo. His claim to deduct
expenses such as meals and car hire failed because he too was unable to show that his home was the place

54
from which his trade was conducted. (The major item, being the cost of travelling between Harare and
Bulawayo appears, however, to have been allowed and not to have been in issue. Further, the
Commissioner usually accepts as deductible all expenses incurred, away from “base” , on travelling and
subsistence.)
The taxpayer failed also in ITC 1410 (1986) 48 SATC 32. He was employed as a lecturer but in addition
gave freelance lectures. These involved travelling from his home where he used a room, partially at least,
for the preparation of related notes and study material. The court concluded, applying a similar provision,
that his travelling remained that between home and place of business.
In the case of dual purpose expenditure it is considered likely that a claim on an
apportioned basis would be acceptable.
(ii) Domestic expenses (which would include alimony) have been held also to include:
Wages paid to a domestic servant who was recruited to enable the taxpayer’s wife to take up employment:
ITC 833 (1956) 21 SATC 324;
The cost of an eye operation undergone by a legal practitioner: L v COT (1992) 54 SATC 91.
(c) Any loss or expense which is recoverable under any contract of insurance or indemnity.
The precise circumstances to which this provision relates are obscure. Where, for example, an
insurance recovery arises in respect of a tobacco crop which has suffered hail damage, the Commissioner
certainly does not attempt to disallow the earlier planting and cultivation costs.
Possibly the prohibition could thwart deductibility of expenditure which is technically “incurred”
in a year of assessment, by participants in schemes, but which, by reason of another party’s guarantee, will
never have to be met by the taxpayers themselves.
(d) Tax levied on income and interest on overdue tax.
There are nevertheless instances where tax may rank as a credit against other tax payable: see
chapters 10, 19 and 20.
(e) Income carried to any reserve fund or capitalised in any way.
The Commissioner accepts, however, that specific provisions for directors’ fees and staff bonuses
are deductible subject to two conditions, i.e., (i) that they are voted by the date of the relevant accounts or
annual general meeting and (ii) that they accrue for tax purposes in, at the latest, the year of assessment
after that in which they are claimed as a deduction.
Deductions in respect of liabilities for staff leave pay are allowable if they satisfy the test that they
have been “incurred” (see 7.3) and despite their being termed “provisions” .
(f) Expenditure incurred in respect of any amounts received or accrued which are not included in the
term “income” , as defined.
A common example of such disallowance is that of interest payable on a loan used to purchase
Zimbabwe shares yielding exempt dividends. Administration expenses may also be affected. (Comment on
apportionment of expenditure has already been made at 7.2). It is accepted, however, that audit and
accountancy fees remain fully deductible.
Although there are frequent disallowances of interest which is directly applicable to the accrual of
dividends, circumstances have arisen of a deduction being permitted where the accrual of dividends has
been incidental, and not the real objective.
An example arose in CIR v Drakensberg Garden Hotel (Pty) Ltd (1960) 23 SATC 251. The taxpayer
company was the lessee of hotel premises. In order to ensure its unrestricted control of the premises it
purchased the shares in the lessor company. Interest payable was held to be deductible.
Where funds are borrowed for general business purposes the fact that certain investments are
unproductive will not, on the authority of Producer v COT (1948) 15 SATC 405 and Financier v COT
(1950) 17 SATC 34, preclude a deduction. The same goes for a situation in which a financial institution
places a small proportion of borrowed funds into shares producing exempt dividends: CIR v Standard
Bank of SA Ltd (1985) 47 SATC 179. These are, however, exceptions. In most instances a disallowance on
an apportioned basis is likely to be difficult for a taxpayer to resist.
Where funds are borrowed for a purpose productive of income which is taxable, but are then
switched to produce exempt amounts, interest payable would remain deductible under the tests in the
Producer and Financier cases. It is understood, however, that the Commissioner relies on remarks in
ITC 1116 (1968) 30 SATC 98, at 106, as authorising subsequent disallowance. It appears that the
converse should similarly apply.

55
In instances where interest is payable on, for example, a fluctuating current account, such as a
bank overdraft, on the authority of Volkskas Bpk v Meyer 1966 (2) SA 379 (T) any disallowance ceases
once the account moves even temporarily into credit.
(g) Contributions by employers to pension/annuity/sickness/etc. funds for employees, except to the
extent permitted in the Sixth Schedule.
The effect is that only contributions to funds approved or registered in accordance with laid-down
procedures are deductible, subject then to the limits imposed in that schedule (see chapter 16).
(h) Notional interest which is lost as a result of investing capital in a trade.
(i) The rent of, or cost of repairs to, or expenses incurred on, any premises not occupied for trade, or
of any dwelling or domestic premises except in respect of such part as may be occupied for the purposes of
trade.
(See, however, the submission at 8.2 that expenditure on repairs resulting from the letting of
property remains allowable.)
(j) The cost of securing sole selling rights.
An example is the cost as might be incurred by a petrol company in payments to a service station
which then sells only that company’s brand of petrol.
(k) Amounts, in excess of (from 1 January 1999) $200 000 paid for leasing a “passenger motor
vehicle” (“PMV”) as defined in the Fourth Schedule (see chapter 9).
This is a cumbersome cross-reference since not all of the terminology used in the definition is
appropriate to a lessee. The Commissioner’s interpretation is, however, that (apart from special cases such
as hotels) a lessee of, for example, a sedan or stationwagon (used for business, including mining) will
encounter this restriction on the deduction of vehicle rentals, including lease-hire contracts.
The restriction is cumulative on each PMV from year to year, so that if, say, $90 000 was paid and
deducted in respect of a vehicle first leased by the taxpayer in the 1999 year, only $110 000 would be
deductible thereafter; any further rental in respect of that vehicle would be disallowed.
The limit for immediately-preceeding years was $75 000.
No corresponding increase of capital allowances in relation to rentals for the hiring of PMVs was
made in the year beginning January 2001. [see para 9.2.5.]
(l) The cost of any shares awarded by the company to an employee or director.
This prohibition would counter any claim for a deduction by a company in respect of either (i) an
issue of its own shares or (ii) an award of shares in another company.
(m) Expenditure incurred on entertainment, whether directly or by the provision of an allowance to
any employee, including a director.
“Entertainment” is defined as including “hospitality in any form”. A deduction is therefore clearly
precluded in respect of the cost of, for example, a lunch for business associates, despite the host’s purpose
being the furtherance of trade relationships.
The approach in the judgment in ITC 1394 (1985) 47 SATC 119 was that this would extend to the
provision of meals for employees. That case however concerned a financial institution whose employees
were provided with lunch because the nature of the business demanded that they work throughout banking
hours, which included lunchtime. The court found that “the element of entertainment or hospitality
disappears where the employer requires the employees, during the meal, to continue with their normal
duties”.
In the context of staff the Commissioner accepts deductions in respect of Christmas parties,
canteen meals and office teas.
(n) Expenditure incurred in the production of any dividends from foreign companies. These are liable
to income tax in the hands of a taxpayer ordinarily resident in Zimbabwe, and are thus taxable (at a flat
rate: see chapter 10) without any deductions for related expenditure. The latter could be a minor matter
such as bank charges, or more substantial such as interest payable on monies borrowed to purchase the
shares.
(o) Expenditure incurred in the production of interest on any loan or deposit with certain types of
financial institution. (It appears that, in recognition of the taxability of such interest earned by other
financial institutions, this prohibition should have been repealed at the same time (1 April 1996) as
alterations were made to the regime under which such interest is treated in the hands of other investors).
(p) Any tobacco levy (see 11.10.3).
Note regarding section 16(2): This prohibits any deduction “which is not a deduction in respect of
expenditure or loss” unless the matter is “directly related to the trade carried on by the taxpayer in

56
Zimbabwe”. This somewhat tortuous wording appears to prohibit capital allowances on assets used for a
non-Zimbabwe trade.
Chapter 8
Deductions: Specific
Synopsis
SECTION 15(2)
8.1 Introduction
8.2 Repairs
8.3 Capital allowances
8.4 Premiums: leases, patents, “know-how”, etc.
8.5 Improvements under a lease agreement
8.6 Miners
8.7 Bad and doubtful debts
8.8 Contributions to benefit and pension funds and to medical aid societies
8.9 Crops and timber
8.10 Experiments and research
8.11 Educational grants
8.12 Pensions payable to former employees and their dependants
8.13 Donations
8.14 Subscriptions
8.15 Expenditure prior to commencement of business
8.16 Trading stock on hand
8.17 Trading stock acquired or brought to hand otherwise than in the ordinary course of trade
8.18 Conventions and trade missions
8.19 Co-operatives
8.20 Farmers
8.21 Legal costs of income tax appeals
8.22 Expenditure not yet incurred
8.23 Growth point area deductions
8.24 Petroleum operations and special mining leases
8.25 Export-market development expenditure
8.26 Employment creation incentive: manufacturers
SECTION 15(3),(5),(6), (7) AND (8)
8.27 Assessed losses
SECTION 15(4)
8.28 Deductions under more than one section
ALLOWANCES, ETC. UNDER OTHER SECTIONS
8.29 Hire-purchase transactions and instalment-credit sales
8.30 Exchange rates: losses
8.31 Deduction of tax from dividends
SECTION 15(2)
8.1 Introduction
Some types of expenditure do not fall within the general deduction formula in paragraph (a) of section
15(2) because, for example, they are of a capital nature, or are not incurred for the purposes of trade.
Further paragraphs specifically provide for a deduction in many such cases, as below. As is observed
elsewhere these are not caught by the provisions of section 16 prohibiting deductibility.
8.2 Repairs (para (b))
Regardless of any possible deductibility under paragraph (a) of section 15(2), paragraph (b) constitutes a
positive avenue for a claim and provides for the deduction of (i) repairs to articles, implements, machinery
and utensils used, and to property occupied, for the purposes of trade and (ii) repairs resulting from the
letting of property.
(a) There is no definition of “repairs” in the Act but the following principles (abbreviated) which were
laid down in ITC 617 (1946) 14 SATC 474, at 476, have been observed in many subsequent cases.
(1) Repair is restoration by renewal or replacement of subsidiary parts of the whole.

57
(2) It is not necessary, however, that the materials used should be identical with the materials
replaced.
(3) Repairs are to be distinguished from improvements.
Further principles which emerge from the case and which have received approval are:
(4) that reconstruction of the entirety (meaning not necessarily the whole but substantially
the whole subject matter under discussion) would not constitute a repair; and
(5) that the test for improvements is whether a new asset has been created resulting in an
increase in the income-earning capacity, or whether the work undertaken merely represents the cost of
restoring the asset to a state in which it will continue to earn income as before.
Further precedents are as follows. As will be observed the dispute is virtually always over
whether the work which has been done constitutes an improvement.
Rhodesia Railways v CIT Bechuanaland (1933) 6 SATC 225
Part of a railway track had become worn and dangerous. The re-laying of 74 miles of track, the total length
of which was 588 miles, was held to be a repair.
Rhodesia Railways Ltd and Others v COT (1925) 1 SATC 133
The original construction of a railway line had been faulty and had caused heavy operational costs.
Subsequent expenditure on deviations and improved gradients was held not to constitute repairs.
ITC 915 (1960) 24 SATC 219
Where a lean-to roof, which had deteriorated, was replaced by a higher-pitched roof the work was held to
constitute an improvement and not a repair.
CIR v African Products Manufacturing Co. Ltd (1944) 13 SATC 164
The roof of a kiln, comprising mainly Baltic deal, was in need of repair. As such material was unavailable
the work was executed using reinforced concrete. It was held that the use of different material did not
create an improvement and that the costs were accordingly deductible as repairs.
(Admittedly the new materials used were in fact not as suitable as the old but it is submitted that, provided
that the intention is restoration of a deteriorated part, the use of different materials, even at greater cost,
does not take the work out of the category of a repair.)
ITC 1264 (1977) 39 SATC 133
This decision represents an extension of the concept that the use of different materials is not a bar to
deductibility.
A hotel which had previously utilised a septic tank drainage and sewerage system became obliged to make
use of the municipal system. The court held (accepting that the existing system required replacing by
reason of deterioration) that the cost of conversion to the municipal system constituted a repair.
ITC 1408 (1986) 48 SATC 21
This case is a further and perhaps ultimate extension of the “different materials” concept.
The appellant’s commercial building had outer walls comprising for the most part two sections, the inner
being hollow concrete blocks and the outer a facade of face-bricks. Within a relatively short time after
completion of construction the facade had deteriorated by reason of the method used. It was accordingly
replaced by pre-cast concrete panels attached to the inner concrete blocks.
The work, despite being of some magnitude, was held to constitute repairs. This decision was reached
despite the finding also that the building had been rendered more structurally sound than it had been
originally; notwithstanding its inherent faults it had been safe and functional.
The requirement that the work be necessitated by reason of deterioration was decisive also in ITC
1537 (1992) 54 SATC 383. A deduction was denied where replacement of old partitioning stemmed from it
having become redundant in relation to the requirements of a new tenant, rather than from it having become
worn out. The same approach was taken in Flemming v KBI (1995) 57 SATC 73 regarding replacement of
a borehole, where there arose also the element of improvement of the property as a whole.
(b) Where an asset has deteriorated, expenditure on work which constitutes improvement is not an
allowable deduction, even though repairs, which would otherwise have been necessary, have been avoided:
B v COT (1955) 19 SATC 353. Thus the notional repair content of improvements is not deductible.
Where however repairs, which are undertaken along with improvements, are separately
identifiable, their cost is allowable: ITC 1038 (1963) 26 SATC 123 and ITC 1457 (1989) 51 SATC 131.
(c) In the case of property the cost of repairs effected by a taxpayer is deductible if either he occupies
the property for the purposes of trade or if he lets or sublets. In the latter case it is submitted that the repairs
remain deductible even if they are incurred after termination of the lease and regardless of the purpose for
which the taxpayer uses the property at that stage. Even subsequent private occupation by the taxpayer

58
would therefore not deprive him of a deduction if he could establish that the repairs resulted from his
earlier letting.
8.3 Capital allowances (para (c))
In dealing with the contents of section 15(2) of the Act the present chapter follows, for the most part, the
sequence of topics in that section. The subject of capital allowances is, however, sufficiently large to
warrant separate treatment: see chapter 9.
8.4 Premiums: leases, patents, “know-how”, etc. (para (d))
Provision is made for the deduction of an allowance in respect of a premium, or consideration in the nature
of a premium, which is paid by a lessee to a lessor for the right of use of an asset used by the lessee for the
purposes of trade or in the production of income. The transactions concerned, listed above, and the meaning
to be attached to the word “premium”, are considered at 5.4.
Any deduction is subject to conditions, namely:
(i) in the case of a lease of property it may not, in respect of any single year, exceed an
amount determined by dividing the total premium by the number of years representing the duration of the
agreement;
(ii) if the lease is for a period longer than ten years or for an indefinite period, the amount of
the annual deduction is determined by dividing the total premium by ten;
(iii) if the lessee has an option to renew the lease such option is ignored in determining the
amount of the annual deduction;
(iv) the property in respect of which the premium is paid must be used or occupied for the
purposes of the lessee’s trade; if there is, say, partly private use an apportionment is made;
(v) if the lessee subsequently purchases the property any unclaimed balance falls away.
Property leases, being those most commonly encountered, are used for the purposes of the above
description but premiums in respect of the other assets are subject to similar principles.
8.5 Improvements under a lease agreement (para (e))
(a) If a taxpayer incurs expenditure, under an obligation contained in a lease agreement to effect
improvements to land or buildings, which are used or occupied by him for business purposes, he is entitled
to a deduction in respect of the cost of the improvements.
The allowance takes the form of an annual deduction based on the amount stipulated in the
agreement as the value of the improvements or the amount to be spent on them. In the absence of either
stipulation in the agreement the amount is whatever in the opinion of the Commissioner represents the fair
and reasonable value of the obligatory improvements.
The annual allowance is determined by dividing the amount representing the agreed value of the
improvements by the number of years (calculated from the day on which the improvements are completed)
during which the taxpayer will be entitled to the use or occupation of the land or property. Alternatively the
value is divided by ten, depending upon which method will yield the faster deduction. If the taxpayer is
entitled to such use or occupation for an indefinite period, the period is deemed to be ten years.
The other conditions in relation to a premium, which were considered above, are also applicable,
i.e.: if there is “mixed” use (for trade and other) an apportionment is made; optional extensions are ignored;
if the property is purchased the unclaimed allowances fall away.
(b) For the allowance to apply, the lessee must be under an obligation to effect the improvements. If
they are effected voluntarily, the lessee is not entitled to the deduction: A v COT (1954) 19 SATC 29.
Voluntary improvements would still, however, rank in his hands for the appropriate capital allowances: see
chapter 9.
The question of whether, or to what extent, improvements are obligatory, has given rise to dispute
on a number of occasions. If, therefore, expenditure is contemplated in expectation of a deduction in terms
of this paragraph, it is essential that the obligation be clearly imposed in the relevant agreement. Certain
decisions of the courts have already been considered in relation to section 8(1)(e) of the Act which deals
with the position of improvements in the hands of the lessor and which is the counterpart of section 15(2)
(e): see 5.5.
(c) Example
In order to procure the right of occupation of leased premises Mr A undertook, in terms of a lease
agreement entered into on 1 September 1998 to effect improvements to the property to the value of $5
000 000. The improvements were completed on 30 November 1999 at a cost of $6 000 000, but were not
put into use for the purposes of Mr A’s trade until 1 April 2000. The lease agreement expires on 31 March
2015. Allowances would be calculated as follows.

59
1999: No deduction, because the improvements had not been used by 31 December 1999.
2000: The improvements were used for 9 months of the year. The lease period calculated from the date of
use of the improvements until the date of cessation of the lease exceeds 10 years and is therefore limited to
10 years for tax purposes. The amount is limited to that stipulated in the lease, i.e. $5 000 000. Therefore
the deduction is:

$375 000
$500 000

and so on.
(d) A claim, by a lessee, for an allowance in this form should clearly be made only if, having regard to
the nature of the building, the most favourable result is achieved. In the case of an industrial building a
much faster deduction would be obtained, on a long lease, if the alternative route of special initial
allowance and accelerated wear and tear allowance (see chapter 9) were chosen. In the case of a
commercial building the obligatory improvement claim is always the more favourable. (The lessor is
unaffected and remains liable on the value of the obligatory improvements.)
(e) Where the lessee spends more than he is obliged to, as in the example above, the excess ranks for
capital allowances in his hands regardless of which option he has chosen for the obligatory amount.
Assuming that the improvements there constituted construction of, or additions to, a “commercial building”
as defined (again see chapter 9) the voluntary excess expenditure of $1 000 000 would have ranked for
2,5% wear and tear allowance in 2000 and subsequent years, in addition to the allowance in terms of
section 15(2)(e) on $5 000 000.
8.6 Miners (para (f))
See chapter 12.
8.7 Bad and doubtful debts (para (g))
(a) In terms of sub-paragraph (i) a deduction may be claimed in respect of debts which are
irrecoverable. Before the deduction can be obtained three requirements must be satisfied.
(i) The debts must be due and payable to the taxpayer. If he has sold them they are no longer
due to him and he cannot claim any allowance: Cooper v COT (1952) 18 SATC 259. If he repossesses the
debts, he can then claim them as bad: SIR v Kempton Furnishers (Pty) Ltd (1974) 36 SATC 67.
(ii) It must be proved to the satisfaction of the Commissioner that the debts are irrecoverable
as at the end of the taxpayer’s financial year, i.e., by implication, at that stage, the debtor must have been
unable to meet the debt. This fact may become known to the creditor only at a later stage, such as when he
prepares his financial statements for that year, but it is submitted that all circumstances as had arisen by the
end of that year may be taken into account.
Conversely a catastrophe overtaking the debtor after the end of the year does not make
the debt bad in the year in question.
In terms of the decision in COT v “A” Company (1979) 41 SATC 59 if the debtor has gone insolvent or
into liquidation during the year the whole debt may be claimed as bad, and any amount subsequently
recovered will be treated as a recoupment under section 8(1)(j). Further, the taxpayer’s decision to write the
debt off as bad need not have been taken before the end of the tax year.
(iii) The debts must have been included in the taxable income of the taxpayer, either in the
current or in a previous year of assessment. This stipulation rules out any deduction for debts which have
been purchased by the taxpayer (though debts purchased in the operation of a factoring business would,
obviously, stand on a different footing). In the same way it is not possible for an incoming partner, or a
person who acquires a business concern by inheritance, to claim an allowance for bad debts in respect of
debts which then prove to be irrecoverable, but which were included in the income of the earlier party.
Irrecoverable amounts which have been advanced to debtors on loan account similarly cannot rank
under section 15(2)(g). Deductibility of such amounts in terms of section 15(2)(a), in the case of, for
example, a moneylender, is considered in chapter 7.
(b) Under sub-paragraph (ii) an allowance may be claimed annually in respect of debts which have
been included in the taxpayer’s income, and which are legally due to him, but the recovery of which is a
matter of doubt. This allowance is one of the very few in the nature of a provision for contingencies which
can be deducted for tax purposes. As is the case with all such provisions, the allowance made in one year of
assessment is included in income in the following year of assessment, when a new allowance of a similar
nature may be claimed.

60
Whenever a business is discontinued as a result of the death of the owner, the allowance may still
be granted in respect of the period of assessment ending upon the date of death. In such cases the relative
allowance is not included for taxation in the subsequent period in the hands of the deceased estate since the
latter does not constitute the same “person”.
(c) The Commissioner will not allow deductions for either bad or doubtful debts based simply on a
percentage of debtors. Every application for an allowance under section 15(2)(g) should be accompanied by
a schedule giving the following particulars in relation to each of the two categories: (i) name of the debtor;
(ii) date on which the debt was incurred; (iii) amount of the debt; (iv) particulars regarding the debt; and (v)
reasons for regarding the recoverability of the debt as bad or doubtful.
8.8 Contributions to benefit and pension funds and to medical aid societies (paras (h),(i) and (j))
See chapter 16.
8.9 Crops and timber (paras (k) and (l))
See chapter 11.
8.10 Experiments and research (paras (m), (n) and (o))
A taxpayer may deduct expenditure incurred during the year in carrying out experiments and research
relating to his trade, other than expenditure of a capital nature incurred on plant, machinery, land or
premises or on the acquisition of rights (such as patents).
The principle is extended to sums which the taxpayer contributes to other persons carrying out such
experiments and research relating to the taxpayer’s trade (or a proportion of such contribution if the other
person’s expenditure is not wholly of this nature).
A deduction is permitted also of sums which are contributed to approved scientific or educational bodies
with the condition that they be used for industrial research or scientific experimental work connected with
the taxpayer’s trade.
With effect from 1 January,2002, a deduction is allowed of payments of up to $ 10 million per annum made
to a research institution approved by the Minister responsible for higher or tertiary education.
With effect from 1 January 2003:
(i) the deduction, which is allowed in respect of contributions to public scientific or
educational institutions for research and development in connection with the taxpayer’s trade, is increased
to an amount equal to double the sum contributed; and
(ii) the deduction, which is allowed in respect of payments to a research institution approved
by the Minister responsible for higher or tertiary education, is increased to $20 million.
8.11 Educational grants (para (p))
A deduction is allowed of grants, bursaries or scholarships paid in respect of a person undergoing technical
education, provided that: (i) the course is related to the taxpayer’s trade; (ii) the course is at an educational
institution; (iii) the person is not the taxpayer himself or his spouse or a near relative of either of them.
(“Near relative” is extensively defined in section 2); and (iv) if the taxpayer is a company, the person is
neither an individual controlling the company nor his spouse or near relative, nor a director of the company
or his spouse or near relative (unless the director works full time for the company and controls not more
than 5% of the share votes).
8.12 Pensions payable to former employees and their dependants (para (q))
Where, for example, employees have little or no right to a pension from a fund, the employer may decide to
make ex gratia payments by way of an annuity, allowance or pension. Such payments are allowed as a
deduction subject to the following:
(i) the employee must have retired because of ill-health, infirmity or old age;
(ii) the amount allowed is restricted to the “first $3 000” per annum for each ex-employee;
(iii) in the case of payments to dependants of the employee (or to persons who were
dependent on a deceased former employee) the annual restriction is to $2 000 in respect of all dependants
of each such ex-employee.
In the case, therefore, of each surviving ex-employee who has dependants, payments as above totalling
$5 000 are deductible.
In all cases the amount allowed is reduced by any pension or annuity received during the year, by the ex-
employee or dependant, from any fund of the former employer.
Further factors are:
(iv) the Commissioner accepts that an undertaking by the employer to make payments year
after year is not an essential factor, and that an “allowance” can be determined annually;

61
(v) “employees”, for the purpose of this paragraph, include, in the case of a partnership,
former partners who have retired from a partnership on the grounds of ill-health, infirmity or old age;
(vi) a lump sum paid by an employer to an insurance company in order to purchase such a
pension, being its actuarial equivalent, is deductible, applying the principle in Hancock v General
Reversionary & Investment Co Ltd (1918) 7 TC 358;
(vii) persons whose employment was of a domestic or private nature are excluded in all
contexts.
It must not be overlooked that if the payment was part of the employee’s service contract (or is one
customarily made to former staff or their dependants) the limitation in paragraph (q) is irrelevant as the
expenditure can instead be claimed under paragraph (a): see 7.1.
8.13 Donations (paras (r) and (ri))
Payments to the following, made without any consideration whatsoever, are deductible, despite their being
made from non-business motives:
(i) the National Scholarship Fund;
(ii) the National Bursary Fund;
(iii) charitable trusts administered by the Minister responsible for social welfare or the
Minister responsible for health. In this connection, while there should be no difficulty over the deductibility
of donations to those organisations as themselves perform, for example, welfare functions, it appears to be
advisable that donors should, in relation to donations to service organisations, stipulate the “operational”
organisation to which the monies are to be channelled;
(iv) the State or any fund, of an amount of up to $10 000 000 in a year of assessment for
various specified purposes approved by the Minister responsible for health;
(v) approved employee share ownership scheme or trust, of any shares, etc donated by an
employer in pursuance of such a trust, with effect from 1 January,2002. [See Chap 6.3];
(vi) With effect from 1 January 2003 up to $10 million annually is allowed as a deduction in
respect of payments for specified purposes approved by the Minister responsible for education.
8.14 Subscriptions (para (s))
A deduction is allowed of subscriptions paid by an individual in respect of his continued membership of
any business, trade, technical or professional association. The use of the word “continued” results in the
exclusion of any entrance fee. (Nor can the entrance fee be claimed under section 15(2)(a) since it is of a
capital nature.)
8.15 Expenditure prior to commencement of business (para (t))
The deduction is permitted from business income, in the year in which a business commences, of
expenditure incurred during the 18 months before the business began, provided that:
(i) it was incurred by the taxpayer (and not, therefore, on behalf of a company yet to be
formed);
(ii) it would have been allowed as a deduction had it been incurred after beginning the
business.
In COT v Parkwell House (Pvt) Ltd (1972) 34 SATC 105 the court found that interest incurred during
building operations, on a loan used for building purposes, was expenditure of a capital nature and was not,
therefore, allowable. In such circumstances the interest may be added to the cost of the building for the
purposes of capital allowances (departmental practice 18). It is submitted that the same position would
obtain in relation to other costs, such as rates, incurred during that period.
8.16 Trading stock on hand (para (u))
Accounting principles are effectively recognised and the taxpayer is permitted to deduct, as opening stock,
the value of trading stock which was on hand at the end of the preceding year. (The value of stock which,
exceptionally, had been attached in pursuance of an order of court, also qualifies.)
8.17 Trading stock acquired or brought to hand otherwise than in the ordinary course of trade (para (v))
The commonest example of the above is probably that of trading stock received by way of donation or
inheritance. Resale of such trading stock in the course of trade gives rise, of course, to income against
which, were it not for this paragraph, no deduction would be allowable since no expenditure has been
incurred. The provision covers also items which, having previously formed the subject of, for example, the
taxpayer’s hobby, are then introduced by him into his trade.
The amount of the deduction is whatever the Commissioner considers was the fair and reasonable value of
the stock when it was acquired or brought to hand, subject to an important restriction: the deduction in
respect of most such stock may not exceed the deduction available to the person from whom it is acquired.

62
Thus trader B, receiving donated stock from trader A, is entitled to a deduction, not of the fair market value
at the time, but only of (probably) whatever the stock cost trader A. The restriction does not apply in the
case of inheritance; the estate valuation then constitutes the fair and reasonable value for deduction in the
hands of the beneficiary.
These questions are considered further, in relation to farming operations, in chapter 11.
8.18 Conventions and trade missions (para (w))
The cost of attending conventions and trade missions, which are connected with the taxpayer’s trade, is
allowed as a deduction subject to the following:
(i) the allowance is restricted to (for, for example, the year ended 31 December 2000) the
“first $3 600”, increased with effect from 1 January, 2003 to the “first $100 000”, of the amount spent per
annum and must relate to not more than one convention or one trade mission (not both);
(ii) if the convention or mission takes place over the taxpayer’s financial year-end, the
deduction is allowed in the year in which the event ends;
(iii) if the person attending is a member of a partnership and the partnership bears the
expense, each partner is allowed to deduct an amount in proportion to his share of profits. In such a case the
limit of $3 600 is applicable to one visit by each partner.
A ruling regarding deductibility may be obtained in advance from the Commissioner: departmental practice
3.
(In many instances such cost would in any event be fully deductible in terms of section 15(2)(a).)
8.19 Co-operatives (para (y))
Bodies registered under the Companies Act [Chapter 24:03] as co-operative companies or under the Co-
operative Societies Act [Chapter 24:05] may deduct the amounts which they distribute, commonly known
as rebates, which are directly proportionate to the value of the purchases made by the recipient from the co-
operative. In effect, these are discounts although they are usually distributed only once a year, based on the
annual total of the member’s purchases.
Additionally, these bodies are granted a special deduction of $1 for each dollar by which their taxable
income is less than $15 000. Thus a co-operative with a potential taxable income of, say $8 000 would
obtain a special deduction of $7 000, resulting in an actual taxable income of only $1 000. This allowance
is apportioned in the ratio of taxable incomes in the case of a group of co-operatives under common
control.
The amount of the special deduction is restricted to the taxable income otherwise arising i.e. this deduction
may not create or be added to an assessed loss.
8.20 Farmers (para (z))
See chapter 11.
8.21 Legal costs of income tax appeals (paras (aa) and (bb))
Taxpayers who appeal against any decision by the Commissioner, to which they had objected, (see chapter
21) and whose appeals are allowed in full in the Special Court or the High Court, may deduct their “taxed
costs”. “Taxed” in relation to legal costs, has no connection with the usual meaning of tax. It means
“allowed by the registrar of the court as being in accordance with the proper scale for such costs”. The
deduction arises in the year of assessment in which the costs are so “taxed”.
If the appeal is allowed to a substantial degree but not in full, the court may direct that the costs be
deductible.
Should an appeal be taken further (by either party) to the Supreme Court, and the taxpayer’s case be upheld
wholly or partially, again the court may at its discretion permit the costs to be deducted.
8.22 Expenditure not yet incurred (para (cc))
Expenditure may normally be claimed as a deduction only in the year in which it is incurred. An exception
is provided in cases where income accrues in one year of assessment, in respect of services to be rendered
or goods to be delivered in a subsequent year, and it is known that expenditure related to such income will
be incurred in subsequent years. A typical example is the accrual, to a bookseller, of a subscription to a
periodical. The subscription is part of gross income, while related expenses may be incurred in subsequent
years. An allowance for such expected costs may be claimed in the year of accrual of the income, but
subject to four provisos:
(i) the amount of the allowance will be at the discretion of the Commissioner (not subject to
objection or appeal);
(ii) expenditure of a capital nature is ignored;

63
(iii) current expenditure which relates directly to future years’ income, and which would have
been claimable in the current year, is set off against the allowance;
(iv) any allowance granted must be brought back into income in the following year.
In practical terms the impact of proviso (iii) is frequently so onerous as to nullify the attraction of claims
under this paragraph.
A comparable provision in the South African Act contains different wording. The decisions in ITC 1601
(1996) 58 SATC 172, ITC 1660 (1999) 61 SATC 249 and ITC 1667 (1999) 61 SATC 439 provide,
however, some guidance as to the likely exercise of the Commissioner’s discretion under the Zimbabwe
provision.
8.23 Growth point area deductions (para (dd))
See chapter 9.
8.24 Petroleum operations (para (ee) and Sch 20) and special mining leases (para (ff) and Sch 22)
Particular provisions relate to exploration for, and production and disposal of, crude oil and natural gas and
to expenditure in terms of such leases. These are of limited interest in view of the very restricted number of
operators concerned.
8.25 Export-market development expenditure (para (gg))
(a) An effective double deduction is granted in respect of such expenditure. The expenditure itself
would probably be deductible under the general formula in any event. The following comments are
therefore made in relation to the prospects of a further 100% deduction.
The term is extensively defined, initially as expenditure, other than of a capital nature, incurred
wholly and exclusively for the purpose of seeking opportunities for the export of goods from Zimbabwe or
of creating or increasing the demand for such exports. Certain specified purposes are then listed, such as
research into foreign markets, foreign advertising, bringing in prospective buyers, etc. .
The goods for export have to have been manufactured, produced, grown, packed, etc., in
Zimbabwe.
(b) (i) The case of C v COT (1973) 35 SATC 241 is relevant in that, although the earlier local
provision to which it related contained the “to the extent that” test, it required, as does the present
provision, that the expenditure be incurred for the purpose of seeking opportunities for the export of goods
or of creating or increasing the demand for such export. In the latter respect the judgment included the
following points:
– that not all expenditure incurred in exporting goods to other countries falls within the
definition;
– the definition does not create an “export allowance”;
– that expenditure would not qualify merely because it was connected with or incurred for
the purpose of export; this would preclude expenditure incurred for the purpose of satisfying an existing
demand or for the purpose of fulfilling existing orders;
– that expenditure would not qualify if it was incurred for the purpose of fulfilling a
contractual obligation and not for the purpose of obtaining the contract;
– that expenses incurred for the purpose of obtaining an order or a contract in a foreign
country would qualify.
(ii) It appears, therefore, that expenditure on supplying goods to a customer, such as railage,
will fail, unless there are special circumstances. In Metal Pressing and Enamelling Co (Pvt) Ltd v COT
(1970) 32 SATC 229, for example, the taxpayer was able to show that it was only by incurring such
expenditure that it was able to ward off competition and expand its markets. The cost of labels, and of
information pamphlets supplied with the goods, was held not to qualify under the South African legislation
in ITC 1524 (1992) 54 SATC 201 and, despite differences in wording, the same position appears likely to
obtain in Zimbabwe.
(iii) It is submitted that expenditure such as agent’s commission, on export sales, could
qualify in terms of the general test above. Since expenditure would qualify if it were incurred on an
abortive attempt to achieve such sales it would be absurd if expenditure resulting in success should not do
so. The position with regard to pre-existing agency contracts, however, remains to be decided.
(iv) Where a seller’s product has to be blended with that of another prior to export such seller
may still be found to be exporting, provided, it appears, that such product has not been intrinsically altered:
CIR v Wandrag Asbestos (Pty) Ltd (1995) 57 SATC 123.

64
(v) Expenditure on entertainment, provided that it satisfied the tests in (a) above, would
qualify for the effective double deduction; the comment at 8.1 refers. No doubt this would be rapidly
negated by legislation.
With effect from the 1 January, 2001, the Finance Act, 2000 repealed the above double
deduction.
Export incentives
With effect from 1 January 2002:
• the rate of tax on a manufacturer’s taxable income from manufacturing, if exports are at least 60%
(by quantity or volume, not value) of goods manufactured, is reduced from 30,9% to 20%. There is a
reference also to “processing” but it is unclear whether there is an intention thus to widen the benefit of the
reduced tax rate ;
• the double deduction in respect of export-market development expenditure, which had been
repealed with effect from 1 January 2001, is reintroduced (see 8.25).
There is thus only a one-year “gap”.
Tourism operations incentives
A new concept of “approved tourist development zone”, or ATDZ, (i.e. declared under the Tourism Act
and requiring the Commissioner’s approval) has been introduced with effect from 1 January 2002.
The intended result of amendments appears to be, though this is unclear, that the following rates of tax will
apply to taxable income from operations of a tourist facility, as defined in the Tourism Act, in an ATDZ :
• in the case of new operations:
The first five years of the operations 0%
The second five years of the operations 15%
The third five years of the operations 20%
• in the case of existing operations, provided that at least 60% of the turnover consists of receipts in
foreign currencies 20%
8.26 Employment creation incentive: manufacturers (para (hh))
Any manufacturer who, during the year of assessment:
(a) engages 50 additional employees; or
(b) increases the number of permanent staff by 25%;
is entitled to an effective double deduction in respect of the additional employees’ remuneration. This
additional deduction falls away, however, if the amount would exceed the taxpayer’s taxable income for the
year of assessment concerned.
Various conditions regarding additional employees have to be satisfied for their numbers to qualify for the
purposes of the employer’s claim. “Managerial” employees (as defined) for example, do not qualify; nor do
persons previously employed by the employer.
SECTION 15(3),(5),(6), (7) AND (8)
8.27 Assessed losses
(a) As has already been observed “income” is reduced to “taxable income”, both terms as defined in
section 8(1), by the permitted deductions. The latter arise mainly in terms of section 15 but there are
deductions also under other sections, which will be dealt with below. If the deductions exceed the income
the excess is defined, in section 2, as the “assessed loss”. A deduction is made also of any assessed loss
determined for the previous year.
Where a taxpayer has income from one (business, etc.) activity, but sustains a loss on another, the
latter may be set off and only the balance will be taxable.
(b) In certain instances there are restrictions against the above set-off rule.
(i) The most wide-reaching of these is that a loss attributable to business operations may not
be set off against income accruing under a contract of employment. The latter income remains taxable in
the year of assessment in which it accrues and the business loss is carried forward to the following year,
and so on.
Example
An individual’s return for the year ended 31 December 2000 reflects the following (see
chapter 10 for terminology and tax calculation):

Taxable income from employment 900 000


Taxable income from trade or investment 600 000
Assessed loss incurred in year ended 31 December 1999 (700 000)

65
Potential net $800 000
Assessable as follows:
(1) Taxable income from employment $900 000
Tax payable thereon (see chapter 10) $360 990
(2) Taxable income from trade or investment
Assessed loss brought forward 600 000
(700 000)
Assessed loss carried forward to year ended 31 December 2001 ($100 000)

(ii) An assessed loss is not deductible against income earned on loans or deposits with
financial institutions generally. (It appears that, in recognition of the taxability of such interest earned by
other financial institutions, this prohibition should have been repealed at the same time (1 April 1996) as
alterations were made to the regime under which such interest is treated in the hands of other investors.)
(iii) Losses in the more rare instances of petroleum operations and special mining leases are
also “ring-fenced” and may not be set off against income from any other operations.
(c) There is a time limit on the set-off process. Each year’s result, i.e. loss or taxable income, is
identified separately and any loss which is effectively more than six assessment years old is cancelled, as in
the following illustration.

Example 1 Example 2
$ $
Assessed loss incurred
in March 1995 (2 350 000) (1 000 000)
Assessed loss March 1996
(Example 1) (500 000)
Or taxable income March 1996
(Example 2) 650 000
Assessed loss March 1997 (700 000) (700 000)
Assessed loss December 1997 (550 000) (550 000)
Assessed loss December 1998 (250 000) (250 000)
Taxable income December 1999 600 000 600 000
Taxable income December 2000 1 350 000 1 350 000
Potential accumulated
assessed loss December 2000 (Ex.1) ($2 400 000)
Or taxable income December 2000 (Ex.2) $100 000
Forfeit 6-year old content:
March 1995 (2 350 000)
Set off December 1999 taxable income 600 000
Set off December 2000 taxable income 1 350 000
Therefore forfeited $400 000
Balance carried forward *($2 000 000)
*$500 000 of this, relating to March 1996, will be subject to forfeiture at 31 December 2001, depending on
the income for the year ending on that date.

The effect is that the year ended 31 December 2001 represents the taxpayer’s last chance to “use
up” an assessed loss in respect of the year ended 31 March 1996; December 2002 for March 1997; and so
on.
(d) The six-year limit on the carry-forward of an assessed loss does not apply to such loss arising from
mining operations. Such right is retained even if the mining operations which gave rise to the loss have
ceased and been replaced by profitable non-mining operations. There is so far no recognition, however, of
long-standing assessed losses in non-mining activity arising from, for example, capital allowances on huge
investment in factories or plant, or in long-term operations such as timber, orchards, etc.
Where both mining and other operations are conducted some form of allocation may be needed.
(e) Subject to the above, an assessed loss is not cancelled by, for example, the cessation of trade for a
period, a complete change of trade, or any other factor, with two exceptions as follows.

66
(i) Where a taxpayer has become insolvent or has assigned his estate for the benefit of his
creditors (but not a company in liquidation), any assessed loss incurred before the date of insolvency or
assignment is eliminated.
(ii) If any shares are transferred in a company with an assessed loss (or in its controlling
company), and the Commissioner is satisfied that the sole or main reason for the transfer was to take
advantage of the assessed loss, the loss will be cancelled.

(f) Transfers of shares have led to dispute on a number of occasions, with varying results.
Where a company is a mere shell it will obviously be difficult for the person acquiring its shares to prove
that its assessed loss was not his sole or main target: see ITC 1123 (1969) 31 SATC 48, where the court
upheld the disallowance of the loss. Exceptionally, however, in these circumstances, the appeal succeeded
in ITC 1209 (1973) 36 SATC 84 where the taxpayer convinced the court that she had no knowledge of the
existence of the assessed loss. Similarly in S (Pvt) Ltd v COT (1969) 31 SATC 77, concerning the shares in
a dormant (formerly furniture) company, the purchaser (who had then transferred his own profitable
furniture business into the company) proved that his other reasons for acquisition had predominated; the
company had belonged to a late relative, it had his own name in its title and it had a well-established
reputation. Again in ITC 1514 (1992) 54 SATC 65 the court, in allowing the appeal, accepted that the
purchaser of the shares was unaware of, or at least unconcerned about, the existence of an assessed loss.
Where the company is operational at the time there may still be difficulties in the way of the person who
has acquired its shares. Appeals were lost in, for example, H Ltd v COT (1972) 34 SATC 57 and Glen Anil
Development Corporation Ltd v SIR (1975) 37 SATC 319. In ITC 1347 (1982) 44 SATC 33, however, the
appeal succeeded. The company with an assessed loss, whose shares were acquired, was a manufacturer of
men’s clothing. The acquiring company conducted a similar business but in another city and was able to
establish that the potential business advantages of the acquisition outweighed those of the assessed loss.
It is submitted that in most disputes it is the latter aspect of business considerations which is likely
to prevail in cases of a continuing business. The acquisition of shares in a company is, after all, such a
common occurrence and, further, as in ITC 1514 above, the seller may well find a share transaction
preferable.
(g) Generally a loss attaches to the taxpayer on whom it was assessed and cannot pass to another
taxpayer. There are exceptions to this rule, including the following.
Proviso (iii) to section 15(3) relates to circumstances which are rare. It permits the transfer of an
assessed loss from one taxpayer to another in the case of a foreign-incorporated company, having carried
on its principal business in Zimbabwe, being voluntarily wound up and transferring all its business to a
Zimbabwean company, the members’ shareholdings being exchanged for shares in the new company.
Proviso (v) permits the transfer of an assessed loss upon conversion of a company into a private
business corporation, or vice versa. This surely will generally be automatic but the Commissioner has to be
satisfied that the sole or main reason for the conversion was not to take advantage of the loss.
SECTION 15(4)
8.28 Deductions under more than one section
See 7.1
ALLOWANCES, ETC. UNDER OTHER SECTIONS
8.29 Hire-purchase transactions and instalment-credit sales (ss 17 and 18)
(a) In the case of:
(i) hire-purchase and suspensive sales (where ownership or transfer of property is effected
only after certain amounts have been paid); and
(ii) instalment-credit sales (where transfer of ownership is immediate but payment is by
instalments);
the whole of the amount payable is deemed to have accrued to the vendor at the date on which the
agreement was entered into.
To prevent hardship to the vendor, however, an allowance may be deducted of a “reasonable”
amount, in addition to the normal deduction for bad and doubtful debts, in respect of payments not yet
received. The allowance is determined by the Commissioner:
(i) in the case of immovable property: in accordance with a strictly proportionate formula
given in proviso (ii) to section 17; and
(ii) in the case of movable property: in accordance with the special circumstances of the
particular type of trade. The method of calculation is set out in departmental practice 33, as follows:

67
(a) the full sale price including the finance charge, must be returned as income in the year in
which the sale takes place;
(b) a deduction for bad and doubtful debts under section 15(2)(g) is made, subject to
production of evidence;
(c) the average gross profit percentage is calculated and applied to the aggregate instalments
(after elimination of the section 15(2)(g) allowance) not yet receivable at the end of the year.
The allowance granted must be brought in as income in the following year and, at that year end, a
new allowance calculated. If the vendor disposes of an agreement (for example by discounting it with a
finance house) no allowance can be claimed.
The allowance does not apply to any other credit sales.
(b) Examples

(i) Suspensive sale of immovable property


ZA Ltd in February 1999 acquired, for $6 000 000, 40 hectares of land which it subdivided for
development and resale. 60 plots of 0,5 ha. each thus became available, with the balance of the land being
used for roads and other amenities. Development costs were:
$
Survey fees 1 000 000
Levelling, etc. 2 000 000
Roads 7 000 000
Water reticulation 8 000 000
Sales of 40 plots were effected on 30 September 1999, at $1 100 000 each, half of which was
payable immediately and half on 30 June 2000 at which stage ownership would pass. Interest at 40% was
receivable on amounts outstanding.
The remaining plots were sold for cash on 1 May 2000 at $1 300 000.
Assuming that all the terms were met, the taxable income of ZA Ltd for the years ended 31
December 1999 and 2000 would be calculated as follows.
Overall costs:
Purchase price 6 000 000
Development 18 000 000
$24 000 000
Cost per stand therefore $400 000
1999
Sales of 40 plots at $1 100 000 44 000 000
Less cost at $400 000 16 000 000
Profit 28 000 000
Less s 17 allowance for instalments not due at 31 December 1999:
22 000 000 × 28 000 000
44 000 000
14 000 000
14 000 000
Interest on $22 000 000 at 40% for 3 months2 200 000
Taxable income $16 200 000
2000
Allowance granted in 1999 14 000 000
Interest on $22 000 000 at 40% for
6 months 4 400 000
Sales of 20 plots at $1 300 000 26 000 000
Less cost at $400 000 8 000 000 18 000 000
Taxable income $36 400 000
(ii) Instalment-credit sale of movables
Mr Z retails goods on credit, with instalments being receivable over a six-monthly period.
Relevant information for two years of assessment is:
1999 2000
Instalment sale debtors not due at 31 December (gross) 800 000 900 000
Applicable specific bad debts (s 15(2)(g)(i)) 40 000 35 000

68
Year-end specific provision for applicable doubtful debts (s 15(2)(g)(ii))
50 000
45 000
Gross profit percentage on sales 55 60
The calculations of the allowances would be as follows:
1999
s 18 allowance:
($800 000 – $50 000 – $40 000) i.e. $710 000 at 55% 390 500
s 15(2)(g)(i) allowance 40 000
s 15(2)(g)(ii) allowance 50 000
Total allowances $480 500
2000
s 18 allowance:
($900 000 – $35 000 – $45 000) i.e. $820 000 at 60% 492 000
s 15(2)(g)(i) allowance 35 000
s 15(2)(g)(ii) allowance 45 000
572 000
Less previous year’s allowances reversed:
s 18 390 500
s 15(2)(g)(ii) 50 000 440 500
Total net allowances $131 500

8.30 Exchange rates: losses (s 15(1))


If, between the date when deductible foreign expenditure is incurred and the date when it is paid, there is an
alteration in the rate of exchange between Zimbabwe and the other country, the amount to be claimed as a
deduction is the amount actually paid in Zimbabwean currency.
If the incurring of the liability and the payment occur in different years of assessment the adjustment is to
be made in the later year.
8.31 Deduction of tax from dividends (s 25)
Zimbabwean companies may, if they so wish, reflect on dividend certificates not merely the dividend
declared, i.e. from the company’s after-tax profits, but the pre-tax amount and the income tax borne by the
company; the net figure then constitutes the dividend. This provision is of relevance only to those foreign
shareholders who may be required to produce such evidence to their own tax authorities. It is essentially an
administrative matter and should not be confused with the much more important withholding tax provisions
considered in chapter 19.

Chapter 9
Deductions: Specific: Capital Allowances
Synopsis
9.1 Introduction
9.2 The assets
9.3 The allowances and recoupments
9.4 Allocation of purchase price
9.5 Reconstructions, mergers, etc: elections
9.6 Other election circumstances
9.7 Growth point areas
9.1 Introduction
(a) Section 15(2)(c) authorises the deduction of allowances, set out in the Fourth Schedule, in respect
of the cost of various fixed assets. The schedule’s provisions relate not only to, for example, traders and
industrialists, but also to farmers. The latter, however, enjoy other specific allowances, in terms of the
Seventh Schedule, which will be described when the topic of farmers as a whole is considered in chapter
11.
There are a number of tests which have to be satisfied if an asset is to qualify for a particular
allowance. Some of the tests are achieved through the medium of restrictive definitions. Since, for example,

69
an industrial building enjoys much more favourable tax treatment than a commercial building, it is clearly
advisable that a taxpayer should be aware of the dividing line, particularly at the construction or expansion
stage.
(b) Mostly the allowances are calculated on the “cost” (either original, or diminishing balance) of the
asset. In this respect charges relating to the obtaining, importation, installation or removal of, for example,
machinery, should generally be added to the basic capital cost for the purpose of calculating any allowance.
So also should finance charges which, in a hire-purchase transaction, are included in the total purchase
price: ITC 774 (1953) 19 SATC 311. (Any difficulty which might have arisen out of the fact that ownership
does not immediately pass under a hire-purchase transaction is overcome by its being deemed, in paragraph
10 of the Fourth Schedule, to constitute a sale on credit. Similarly a sale under a suspensive
condition is deemed to have effected a change of ownership as from the date of such sale.)
There are provisions regarding the determination of a value, for capital allowance purposes, in the
case of, for example, buildings which have previously been used for a non-qualifying purpose and articles,
etc., which have been similarly used or which have been acquired for no consideration.
In the case of a building which is constructed by the taxpayer, out of borrowed funds, interest
incurred during the construction period is expenditure of a capital nature (COT v Parkwell House (Pvt) Ltd
(1972) 34 SATC 105). The Commissioner accepts, despite the decision in SIR v Eaton Hall (Pty) Ltd
(1975) 37 SATC 343, that such interest forms part of the cost of the building for the purposes of capital
allowances. The same applies, in terms of departmental practice 13, to any applicable loan-raising fee. The
term “building”, as would be expected, does not include the land on which a building is constructed: ITC
1619 (1997) 59 SATC 309 (decided under specific South African provisions but likely to apply in
Zimbabwe.)
The question of the treatment of foreign exchange losses, in the establishment of “cost”, may also
arise. In ITC 1333 (1981) 43 SATC 90 the taxpayer company had arranged a foreign loan, repayable in
instalments, to finance the importation of machinery. Additional expenditure arising out of adverse
movement in the exchange rate was held to constitute additional cost, for the purpose of allowances on the
machinery, in each of the subsequent years in which it was incurred.
(c) Claims for allowances almost invariably depend on the asset having been “used” or “put into use”,
etc.. The point can be important in determining whether an allowance is available in an earlier or later year
or, indeed, at all.
In ITC 1669 (1999) 61 SATC 479 additional items of machinery and equipment had been installed by the
taxpayer company. They were ready for use but at that stage the company had closed down its operations
for the year in order to carry out maintenance. The Special Court held that the items had not been used in
that year because they had not been actually used. This applied particularly to an item which had not yet
been assized in accordance with legal requirements. (An earlier case which had come before the South
African Special Court, namely ITC 1538 (1992) 54 SATC 387, was considered to have been wrongly
decided. There, a helicopter, which had been delivered to an architect for his professional activities, had
crashed the following day during a conversion training course undertaken by a pilot in his employ. The
architect was held to have used it for the purposes of his trade.)
(d) The paragraph references below, unless otherwise stated, are to the Fourth Schedule.
9.2 The assets
9.2.1 “Commercial building” (def’n in para 1(1))
The term is restricted to a building, the erection of which was commenced on or after 1 April 1975, and at
least 90% of the floor area of which is used for the purposes of trade. The following do not qualify:
(a) buildings (other than flats and those hotels as are registered under the Tourism Act [Chapter
14:20]) which are residential to the extent of 10% or more. (This disqualifies, for example, (i) a house used
as a residence by either the taxpayer or a tenant; and (ii) a shop where the trader lives on the premises and
occupies privately 10% or more of the floor area);
(b) flats or apartments owned by companies, partnerships or associations, the shareholders, partners or
members of which have the right of occupation (such as condominiums);
(c) buildings covered by other definitions, namely farm improvements, industrial buildings (including
licensed hotels), staff housing and tobacco barns.
Subject to the 1975 limit on construction those buildings which most commonly qualify, therefore, are:
shops, showrooms, offices (though a manufacturer’s office may well constitute an industrial building),
trading stores, unlicensed hotels (though not boarding houses) and commercially-let blocks of flats.

70
Additions to such buildings qualify also, but not, in terms of ITC 1286 (1979) 41 SATC 98, where the
original building does not itself satisfy the 1975 limit.
It may be difficult also in terms of ITC 1511 (1992) 54 SATC 39, for a beerhall, forming part of a farm
compound complex, to qualify.
9.2.2 “Farm improvement” (def’n in para 1(1))
This means in general “any building, structure or work of a permanent nature (including a water furrow)
used in farming operations”. It includes any building used for the purposes of a school, hospital, nursing
home or clinic, in connection with the taxpayer’s farming operations. It excludes any dwelling used by
the taxpayer as a homestead for himself and his family, and farm assets covered by other specific
provisions, namely staff housing, tobacco barns (though see below) and, naturally, items such as
boreholes, etc., falling under the Seventh Schedule.
Items qualifying under the general test include sheds, canals, permanent roads, bridges and cattle dips
though not, in terms of ITC 1511 (1992) 54 SATC 39, a beerhall forming part of a farm compound
complex.
With regard to schools, etc. where the cost of construction exceeds specified limits the excess is, for the
purposes of calculating allowances, ignored. Recent and current limits, depending on the date the
expenditure was incurred, are as follows:
(i) aggregate expenditure on e.g. the entire school:
Between 1 April 1995 and 31 December 1998 $500 000
Subsequently $1 500 000
The limit was increased to “the first“ $3 500 000
effective from 1 January, 2002.
The limit is increased to “the first“ $10 000 000
effective from 1 January, 2003.
(ii) in respect of any building used wholly or mainly for the housing of staff employed at the
school, etc.:
Expenditure incurred on each such building:
Between 1 April 1995 and 31 December 1998 $50 000
Subsequently $100 000
The limit was increased to “the first“ $ 250 000
effective from 1 January, 2002.
The limit is increased to “the first“ $ 1 000 000
effective from 1 January, 2003.
(It will be observed that this latter provision in relation to farm schools, etc., staff differs
lightly from that relating to staff housing generally, referred to below.)
Although the term “tobacco barn” is separately defined as meaning any building used for the curing of
tobacco, and there are other references to tobacco barns in the schedule, such buildings are subject to the
same tests as farm improvements and are treated accordingly in this book.
9.2.3 “Industrial building” (def’n in para 1(1))
(a) The meaning is precisely defined but briefly includes any building:
(i) used mainly for the operation of machinery worked by mechanical, etc., power;
(ii) on the same premises as (i) and suffering depreciation by reason of the operation of the
machinery;
(iii) suffering depreciation through the use of chemicals etc.;
(iv) used mainly for industrial research or for experiments in manufacturing;
(v) used mainly for licensed hotel or casino operations. Ancillary structures are included,
enabling a swimming pool, tennis court, golf course, etc., to qualify. Further, in terms of a Zimbabwe court
decision (unreported in SATC) housing for hotel staff may similarly fall within this definition and is not
subject to the same restrictions on cost as apply to “staff housing” for other operations;.
(vi) used mainly for the purposes of a trade which consists in the manufacture of goods or
materials (including staff welfare buildings such as canteens but excluding dwelling houses, retail shops
and showrooms). Storage buildings both for raw materials and for finished products qualify, save for those
occupied by a tenant. The latter, however, would probably constitute “commercial buildings” by reason of
passing the tests under the definition: see 9.2.1.
The definition covers also:

71
(vii) buildings used mainly for the purposes of a trade which consists in the distribution of
hydro-carbon oils by pipeline;
(viii) any works for the prevention of pollution;
(ix) certain buildings related to computer operations;
(x) any toll-road or toll-bridge declared in terms of the Toll-roads Act;
(xi) fencing surrounding any building qualifying as industrial. It is accepted that this includes
walls;
(xii) any permanent sealing of the surrounding ground area. This would embrace car-parks,
courtyards, driveways and the like.
Apart from storage buildings under (vi) landlords’ buildings, etc., qualify if they are used for the
above purposes by a tenant, i.e. as if they had been so used by the owner.
(b) In the case of industrialists’ buildings the test most commonly applicable is that under either item
(i) or item (vi) above.
Each of these requires that the building be used “mainly” for the stated purpose. It is submitted
that, having regard to the court’s approach in SBI v Lourens Erasmus (Edms) Bpk (1966) 28 SATC 233,
admittedly in a different context, a building is used mainly for a particular purpose if more than 50% of its
floor area is so used. It is submitted further however that, having regard to comments in the judgment in
ITC 1049 (1964) 26 SATC 229 at 234, the mere use of floor space is not the only test and that, having
regard to the operations in a building, the relative importance of, for example, the use of power-driven
machinery would be the alternative determining factor.
Where a building fails to qualify under item (i) above, and the taxpayer has to rely on item (vi),
the meaning of “manufacturing” becomes the decisive factor. Save for a reference to the processing of
tobacco there is no definition. In practical terms it will in many instances be obvious whether a taxpayer is
operating a “manufacturing” business. This and similar expressions such as “process of manufacture” have,
however, had to be considered by the courts on a number of occasions. The point of probably the greatest
interest, and on which many cases have turned, has been the question of the required degree of change in
the materials being subjected to a process. It is submitted that a liberal approach to this question is
appropriate, having regard to the decisions in cases such as SIR v Hersamer (Pty) Ltd (1967) 29 SATC 53,
and COT v Processing Enterprises (Pvt) Ltd (1975) 37 SATC 109.
In the Hersamer case the facts were that the company carried on business as a metal merchant, buying scrap
metal in various forms for resale, as steel scrap, to steel manufacturers. The manufacturers’ requirements,
dictated by the design and capability of their furnaces, were that the steel scrap supplied to them should be
of particular size and density. The taxpayer company, in order to meet these requirements, cut and pressed
the scrap metal which it had purchased, and thereby supplied the manufacturers with “briquettes” and
blocks suitable to the latter’s needs. The company’s operation was held to be a “process of manufacture.”
With regard to references in previous cases to the need for there to have been a change in the character of
the raw materials out of which something is made, or for the difference to be substantial or essential, it was
observed at p.64 of the report:
“As a result of being processed a change may take place in regard to the nature or form or shape or utility,
etc., of the previous article or material or substance. There can be no fixed criteria as to when any such
change can be said to have effected an essential difference. It is a matter to be decided on the particular
facts of the case under consideration.”
The court found in favour of the company.
In the Processing Enterprises case it was held that cotton ginning, i.e. the dividing up of seed cotton into
lint and cotton seed and the cleansing of the lint, constituted manufacturing. The view was expressed, at
115, that:
“... there need be no change in the actual substance of the raw material before the process of dealing with it
may be regarded as a manufacturing process provided skill has been applied in some way to that raw
material and to such dealing, and its actual character as opposed to its mere substance, has changed ...”
Another operation which may be carried on in a building, and which may cause its qualification to be called
into question, is that of repairs. This arises particularly as, in the absence of extensive use of machinery
within a building, the taxpayer will again be likely to have to rely on the alternative “manufacturing” test.
In ITC 1246 (1975) 38 SATC 21 the taxpayer’s business was that of the reconditioning of fuel pumps, etc..
It was found that, although the taxpayer’s work brought about a radical change in the usefulness of the
units, it did not essentially change their character. The operations did not constitute manufacturing but
remained as repairs.

72
A contrast must be drawn between this and ITC 1315 (1980) 42 SATC 219. The taxpayer company’s main
business was that of replacing coils in damaged electrical motors and generators. The company itself,
however, made the replacement coils, the latter representing most of the company’s operational costs and
of its use of the building. It was accordingly held that the building was used mainly for a trade which
consisted in the manufacture of goods or materials.
Building operations themselves, comprising the attachment of materials to immovable property, do not
constitute manufacturing: ITC 1247 (1976) 38 SATC 27 but, on the authority of that case and Formscaff
Investments (Pty) Ltd v CIR (1993) 55 SATC 251, prior operations such as the production of beams, either
in a factory or on site, would do so.
Other recent cases which have turned on the meaning of the expressions include the following.
SIR v Safranmark (Pty) Ltd (1981) 43 SATC 235
The company held the franchise for the preparation and sale of Kentucky Fried Chicken. The court held
that the plant used in the production of the food sold to customers was used in a process of manufacture.
Regard was had to the fact that, on completion of the process, a different compound substance had been
produced having a special quality.
It is submitted that the Safranmark case is not to be taken as indicating that every operation constituting the
cooking and preparation of food is one of manufacture.
Automated Business Systems (Pty) Ltd v CIR (1986) 48 SATC 41
Data-capture machines used by a computer firm in the processing of written information supplied by clients
were held not to be used in a process of manufacture. (Such operations might now enable a building to
qualify under item (ix) in 9.2.3 above.)
Ovation Recording Studios (Pty) Ltd v CIR (1990) 52 SATC 163
The production of master recording tapes (i.e. the recording of popular music on to master tapes which
were then fit for use in the mass-production of records and cassette tapes) was held to constitute a process
of manufacture.
(So, similarly, was the scouring, bleaching and dyeing of raw yarn, in ITC 1465 (1990) 52 SATC 1;
abattoir operations, in ITC 1591 (1995) 57 SATC 212; but not film production, where the nature of the
process was essentially photographic, in ITC 1559 (1993) 55 SATC 286.
The question of the point at which manufacturing operations begin was considered in SIR v Cape Lime
Company Ltd (1967) 29 SATC 131 and National Co-operative Dairies Ltd v CIR (1992) 54 SATC 1.
9.2.4 Articles, implements, machinery and utensils
(a) The terms are not defined. In many instances it is obvious whether or not an item constitutes, for
example, an article. In others it becomes a matter of dispute particularly where items are attached to
buildings or to other immovable property. The success or failure of a claim can mean the difference
between, on the one hand, obtaining SIA and the “accelerated” wear and tear allowance (see below),
and, on the other, obtaining either much less favourable allowances or none at all.
Cases in which the courts have adjudicated on the meaning of “article” include the following.
ITC 1248 (1976) 38 SATC 35
A storage platform, or mezzanine floor, installed by the taxpayer company in a leased building, did not
qualify since it lacked independent identity.
SIR v Charkay Properties (Pty) Ltd (1976) 38 SATC 159
Demountable partitions qualified, having been installed in lieu of internal walls in an office building which
was owned and let by the taxpayer company. The same result arose in COT v C (1981) 43 SATC 9, where
the taxpayer company occupied part of the building, which it owned.
ITC 1313 (1980) 42 SATC 197
Carports were erected at an airport, by taxpayers engaged in the travel industry, and were intended to be
dismantled and reassembled elsewhere. They qualified despite not being removed over a long period.
(b) A further aspect concerns articles, etc., which are not owned by the taxpayer but which are, for
example, leased by him, and used for the purposes of his trade. Any expenditure which he incurs on
additions or alterations to such assets is recognised, in paragraph 11 of the Schedule, as ranking for capital
allowances.
9.2.5 “Passenger motor vehicle” (def’n in para 14(2))
In the case of any vehicle constituting a passenger motor vehicle, if the cost exceeds specified limits the
excess is ignored for the purpose of calculating capital allowances. Recent and current limits, depending on
the date of acquisition by the taxpayer are:
Between 1 April 1992 and 31 March 1995 $50 000

73
Between 1 April 1995 and 31 December 1998 $75 000
Subsequently $200 000
The limit was increased to “the first“ $ 500 000
effective from 1 January, 2002.
The limit is increased to “the first“ $ 1 000 000
effective from 1 January, 2003.
The term “passenger motor vehicle” currently excludes the following which are thus unrestricted: taxis,
hotel courtesy cars ( such as sedans); vehicles seating fifteen or more passengers; and vehicles purchased
for the purpose of leasing to a third party with an option to buy or dispose of them. (In the latter connection
the Commissioner accepts that any vehicle purchased for leasing purposes is unrestricted.)
Apart from the exclusions the definition embraces most vehicles intended for use mainly for the
conveyance of passengers. In ITC 1596 (1995) 57 SATC 341, a “4x4 double cab” type of vehicle was held
to fall within such description and would thus suffer the restriction.
9.2.6 “Railway lines” (def’n in para 1(1))
Rails, sleepers and equipment qualify, though not ballast, embankments, bridges (but see toll-bridges),
culverts, etc..
9.2.7 “Staff housing” (def’n in para 1(1) and para. 15(2))
(a) This means any permanent building used by the taxpayer for the purposes of his trade wholly or
mainly for the housing of his employees. It does not, however, include any building comprising or
incorporating any residential unit the cost of which exceeds specified limits. Recent and current limits,
depending on the date of commencement of construction of the building, are as follows:
Between 1 April 1995 and 31 December 1998 $100 000 **
Subsequently $200 000 **
** in which event “the first” $50 000 ranks for allowances
** in which event “the first” $100 000 ranks for allowances.
The cost limitation in respect of expenditure incurred on or after 1 January 2003 in each residential unit is
increased, as is the amount ranking for allowances.
The resulting limits, depending on the date of commencement of construction of the building are, for
example:
• year commencing 1 January 2002 $500 000 *
• year commencing 1 January 2003 and subsequent years $3 000 000**
* in which event ‘the first’ $250 000 ranks for allowances;
** in which event ‘the first’ $1 000 000 ranks for allowances
The term “housing” is, in terms of ITC 1511 (1992) 54 SATC 39, to be strictly applied and does
not cover, for example, a beerhall forming part of a farm compound complex.
(b) Example

Industrialist A constructed and brought into use in the period ended 31 December 1997 a block of six
identical units of housing costing a total of $500 000 (occupied by factory workers) and a supervisor’s
house costing $120 000.
The property was sold in the year ended 31 December 2000 to industrialist B, the consideration being
allocated as $650 000 for the block and $90 000 for the house.B constructed and brought into use a second
supervisor’s house in that year at a cost of $150 000. The treatment for purposes of capital allowances
would be as follows.
Mr A
In 1997 the block, at $500 000, would rank (to the extent of 6 x $50 000) but the first supervisor’s house
would not. In 2000 there would be a recoupment (see 9.3.4) and capital gains tax considerations (see
chapter 22).
Mr B
The block would rank in the same way but the supervisor’s house again would not, both results being
determined by the limit stipulated for the year of construction. The first $100 000 of the second house
would rank.

9.2.8 “Tobacco barn”


See 9.2.2.
9.2.9 “Training building”: “Training equipment”

74
See 9.3.5.
9.3 The allowances and recoupments
The four types of allowance which, as indicated earlier, are available to a wide range of taxpayers, namely
the special initial, wear and tear, scrapping and training investment allowances, are considered at this point.
With regard to a fifth, investment allowance, which is restricted to operations in growth point areas, see
9.7.
9.3.1 Special initial allowance (paras 2 and 9)
The taxpayer may elect to deduct special initial allowance (“SIA”) on the capital expenditure incurred by
him on:
(a) the construction of (i.e. not the acquisition of existing) farm improvements, industrial buildings,
railway lines, staff housing and tobacco barns but not commercial buildings;
(b) additions or alterations to existing farm improvements, industrial buildings, railway lines, staff
housing and tobacco barns;
(c) the purchase of articles, implements, machinery or utensils, whether new or second-hand
(provided that the taxpayer has not purchased them for the purpose of leasing to a third party with the latter
having an option to buy or dispose of them. Finance houses’ operations constitute probably the commonest
examples of such contracts.)
In all cases, the asset must be: (i) used by the taxpayer, (ii) during the year, (iii) for the purposes of his
trade. In the case of expenditure incurred in one year, relating to an asset brought into use in a following
year, the allowance is granted in the year in which the asset is first used.
In the case of articles, etc., there is a requirement that, in order to qualify, they be purchased wholly or
almost wholly for the purposes of the taxpayer’s trade. This requirement is interpreted by the
Commissioner as meaning 90% or more of the asset’s total use. In determining whether the asset has been
so purchased he is empowered to have regard to the use to which it is put either in the year in which it is
first used or in the next following year. A switch of use from business to private purposes could throw
doubt on the purported original purpose; once the Commissioner is satisfied, however, the allowance still
arises in the year of first use.
The rate of SIA in respect of expenditure for which the allowance is granted is 25%, increased to 50% with
effect from 1 January, 2001. (In respect of the years of assessment which ended on 31 March 1992 and
1993 it was 50% and for many years prior to 1992 it was 100%.)
There is never any apportionment of SIA. An asset ranks either totally or not at all. Thus, if a building
constructed by a taxpayer is used to the extent of, say, 60% for the operation of power-driven machinery,
the whole building ranks as industrial notwithstanding that the remainder is used for merely commercial
purposes. Similarly if an individual trader uses, say, his delivery van almost wholly for business purposes,
the vehicle ranks, whereas further private use would disqualify it entirely. In the hands of an employer,
however, an asset such as a vehicle almost invariably ranks for SIA notwithstanding that he permits an
employee to use it partly, or even wholly, for the latter’s private purposes; the provision of the use of a
vehicle to an employee would generally still constitute use for the purposes of the employer’s trade.
There is also no apportionment on a time basis. If an asset satisfies the definition, or test, the allowance is
granted in full even if the asset has been used for only part of the year.
Assets acquired without any cost being incurred, such as by way of donation or inheritance, do not rank for
SIA since no capital expenditure will have been incurred by the taxpayer.
The allowance is granted only if the taxpayer so elects. Such election is virtually automatic in order to
obtain the advantage of the early deduction.
The taxpayer may choose to exercise the election against either all, or only selected, qualifying assets.
9.3.2 Wear and tear allowance (paras 3, 6 and 7)
The allowance (“W&T”) is available on:
(i) commercial buildings, farm improvements, industrial buildings, railway lines, staff
housing and tobacco barns acquired or constructed and in both cases used by the taxpayer for the purposes
of his trade;
(ii) articles, implements, machinery and utensils belonging to and used by the taxpayer for
the purposes of his trade;
the value of which has been diminished by reason of wear and tear during the year of assessment. (Despite
the reference to “value”, the allowance is based on cost, thus restricting claims accordingly where assets are
acquired at bargain prices: ITC 1546 (1992) 54 SATC 477).
The rate and general application of W&T is approached in different contexts as follows.

75
(a) “Accelerated” W&T
Assuming that SIA is granted on a qualifying asset the result in terms of SIA and W&T is as
follows.

$
Year 1: Cost say 100 000
Less SIA: 25% of cost (No W&T in year 1) 25 000
Income tax value (“ITV”) 75 000
Year 2: Less accelerated W&T: 25% SL 25 000
ITV 50 000
Year 3: Less accelerated W&T: 25% SL 25 000
ITV 25 000
Year 4: Less accelerated W&T: 25% SL 25 000
ITV –

(b) “Long-term” W&T in other instances.


Clearly no question of W&T arises where an asset has qualified for, and the taxpayer has already
been granted, 100% allowances by way of SIA and accelerated W&T.
An asset which did not so qualify, and which has a residual ITV, continues to qualify for “long-
term” W&T. The circumstances are where:
(i) an existing industrial building, etc., was purchased; or
(ii) an asset was acquired without payment of valuable consideration, such as by way of
donation or inheritance; or
(iii) the asset constitutes a “commercial building”; or
(iv) (exceptionally) the taxpayer elected not to claim SIA despite the asset qualifying.
(c) The rates of W&T in the latter circumstances are:
In the case of immovables: straight-line basis:
Statutory rates:
On farm improvements, industrial buildings, railway lines,
staff housing and tobacco barns 5%
On commercial buildings 2,5%
In the case of movables i.e. articles, etc.:
Diminishing balance basis:
Varying rates at the Commissioner’s discretion, published in
departmental practice 40, the commonest being:
On most furniture, plant and machinery 10%
On most motor vehicles 20%
(d) Example: Year ended 31 December, 2000
Existing assets at 1 January 2000: ITVs
Industrial building (purchased for $8 000 000) $4 800 000
Computer received by way of donation (at value of $40 000:
ranking at say 10%) $32 400
Commercial building (cost $6 000 000) $5 250 000

Calculation of W&T and ITVs


Ind. Comm. Total
Bldg. Computer Bldg. Allowance
ITV
1 January 2000 4 800 000 32 400 5 250 000
W&T 400 000 3 240 150 000 $553 240
ITV
31 December 2000 $4 400 000 $29 160 $5 100 000

(e) Any asset which first ranks for allowances in a recent or the current year of assessment but which
did not or does not qualify for SIA (i.e., in abbreviated terms, purchased industrial building, etc., donation,

76
etc., commercial building or non-election, as set out in (b) above) qualifies only for long term rates of
W&T as set out in (c) and (d).
(f) Unlike the position with SIA which, as has been seen, is never apportionable, apportionment of
“long-term” (not accelerated) W&T may well arise, but only in the case of movables. Probably the
commonest instance is where an individual trader ab initio uses an asset, such as a motor vehicle, partly for
business and partly for private purposes. The allowance is calculated at the appropriate percentage, but only
that portion as is attributable to business use, on a proportionate basis, is allowed. Apportionment of the
allowance arises also, on a time basis, where business commences or ceases part of the way through a year.
A full year’s allowance is granted, however, on movables acquired at any time during a full year’s trading.
An example of the apportionment arising from partly-private use is given below in the context of
scrapping allowance.
There is no apportionment of W&T on immovables, either on a time basis or in relation to partly-
private use. The percentage is mandatory and the asset either qualifies or fails under the above tests.
(g) The Commissioner accepts (i) that expenditure on a major removal of existing plant or machinery
ranks for long-term W&T, though not for SIA and accelerated W&T, and (ii) that the cost of minor
removals, such as within the same factory, may be written off.
9.3.3 Scrapping allowance (para 4)
(a) If an asset referred to above is scrapped during the year any remaining balance, as has not been
allowed by way of SIA and W&T, and taking into account any proceeds of sale, is deductible. Such claims
arise, obviously, only where there is such a residual value or ITV. (If the sale proceeds exceed the ITV a
recoupment is, instead, bought into gross income: see 9.3.4.)
(b) The word “scrapped” is not defined. It has, in the light of numerous court decisions, a restricted
meaning.
An asset is accepted as having been scrapped if it has become worn out for the needs of a
taxpayer’s continuing business. This is often signalled by disposal of the asset but it is not necessary that
disposal should have occurred during the year of assessment provided that there has been a decision to
scrap and a cessation of use: ITC 631 (1946) 15 SATC 100 and ITC 1538 (1992) 54 SATC 387.
An asset will, however, generally not be regarded as having been scrapped, and therefore no
allowance will be forthcoming, if it is disposed of upon cessation of the taxpayer’s trade. In the latter
circumstances it will not have become useless or worn-out but will have ceased to serve its purpose simply
by reason of the closure of the business. As was indicated in ITC 955 (1961) 24 SATC 631 at 632: “... it is
implicit that the scrapping referred to is a scrapping in the course of the taxpayer’s trade or income-
producing activity ...”.
An even narrower approach was adopted in ITC 1456 (1989) 51 SATC 125, where the allowance was
refused in respect of refrigeration machinery which was sold due to the market for its products having
fallen away. Earlier refinements to this concept, however, resulted in taxpayers’ claims succeeding: ITC
795 (1954) 20 SATC 107, where the taxpayer, having operated two hotels, discontinued one and sold off
such assets as were not transferred to the other; and ITC 631, above (closure of a branch).
In view of the fine distinctions which have been drawn by the courts the question of whether a
scrapping allowance is available should clearly be approached with caution whenever any element of
business closure or contraction is present.
(c) Further aspects with regard to the scrapping allowance are as follows.
Where, upon the cessation of trade, the taxpayer incurs losses on some assets but suffers
recoupment on others the Commissioner accepts a set-off, thus taxing only a net recoupment. In the light of
what was said earlier a net loss would not be allowed.
In the year in which an asset is scrapped it is common for taxpayers to rely entirely on the
scrapping allowance and to claim no wear and tear allowance in respect of such asset. If the above
difficulties prevent a claim for scrapping allowance wear and tear allowance should be claimed for the final
period of use of the asset.
Where wear and tear allowance on an asset has been apportioned in recognition of partially private
use any scrapping allowance is similarly reduced proportionately.
(d) Examples

(i) Sales, while business continues, at 1 January 2000


Truck (cost $600 000) Plant Scrapping Allowance
ITV 31 December 1999 – 287 000

77
Sale price 100 000 127 000
Recoupment (scrapping allowance) $100 000 $160 000)
($60 000)
(ii) Sales as a result of cessation of business
On the facts in (i) above no scrapping allowance would be granted. There would however be no
recoupment as the Commissioner would accept that the $100 000 be set off against the capital loss of
$160 000.
(iii) Sale of passenger motor vehicle while business continues
Cost $600 000 in the year ended 31 December 1998; SIA and accelerated W&T granted on
$75 000; sold in the year ended 31 December 2000 for $250 000.
Restricted cost 75 000
SIA in the year ended 31 December 1998 (18 750)
W&T in the year ended 31 December 1999 (18 750)
ITV at 31 December 1999 37 500
Less sale proceeds applicable to restricted cost: 75 000 x 250 000
600 000 = 31 250
Scrapping allowance $6 250
(iv) Partly-private use
Motor-cycle purchased, on 1 January 1998 for $200 000, by an individual and used by him to the
extent of: (i) in the year ended 31 December 1998, 75% for his farming operations and 25% for private
purposes; and (ii) in the year ended 31 December 1999, 50% for each. Sold on 1 January 2000 for $100
000.
Disallowed
Asset W&T Allowed as private
Cost 200 000
W&T at 20% 40 000 40 000 30 000 10 000
ITV 31 December 1998 160 000
W&T 32 000 32 000 16 000 16 000
ITV 31 December 1999: 128 000 $72 000 $46 000 $26 000
Proceeds 100 000
Loss 28 000
Disallowed 10 111 being 26 000 x 28 000
72 000
Scrapping allowance $17 889

9.3.4 Recoupments (s 8(1)(j))


(a) Although a sale of assets is the commonest cause, recoupment arises also where there is a recovery
of proceeds under an insurance policy. There are, however, exceptions to the latter namely that the
proceeds of insurance policies covering the assets against damage or loss are not treated as
recoupments, provided that two conditions are satisfied.
Firstly, the amounts received under the policies must be used to replace the assets within 18
months and the replacement must be brought into use within a further 18 months. (If only, say, four-fifths
of the proceeds were so expended there would be a recoupment of the remaining one-fifth.)
Secondly the assets either:
(i) must rank for investment allowance under the current, very restricted, provisions
applicable to growth point areas (see 9.7); or
(ii) would have ranked for the former widely-available allowance, had the provisions in force
up to 31 March 1981 not been repealed.
The latter point, though it may appear obscure, is important. The assets which ranked for
investment allowance prior to the 1981 repeal were many of those used by manufacturers, farmers and
licensed hoteliers. (Most motor vehicles did not qualify and other assets had to satisfy various tests.) The
effect of the provision is that in the case even of a factory or farm improvement, etc., constructed, or
machinery acquired, in the years since 1981, there is no recoupment of insurance proceeds derived on
destruction of such an asset if they are expended on a replacement and the time limits stated are satisfied.
Since buildings are frequently the subject of such proceeds, giving rise to capital gains tax
considerations, the matter is considered further in chapter 22.

78
(b) The meaning of recovered or recouped has to a limited extent come before the courts.
In ITC 1435 (1988) 50 SATC 117 the taxpayer, two years after incurring expenditure on an asset on which
W&T allowance had been granted, received a grant-in-aid. It was held that such amount did not constitute a
“recovery or recoupment”. See however, 5.13.

In ITC 1467 (1990) 52 SATC 28 the taxpayer company had sold its hotel property to a developer who had
thereafter demolished the building to prepare for alternative development. The hotel building had ranked
for capital allowances in the taxpayer’s hands. The sale price of the entire property greatly exceeded its
book value but, because of the developer’s intentions, there was no allocation of part of the price to the
building. It was held that the allowances had nevertheless been recouped. The judgment indicates,
surprisingly, it is submitted, that a recoupment would have arisen even if the building had not been in
existence at the date of sale; this was not a matter in issue in the case and remains to be decided.
(c) Examples

(i) Sales of passenger motor vehicles


(a) Cost $400 000 in year ended 31 March 1997; sold in year ended 31 December 2000
Restricted cost 75 000
SIA in year ended 31 March 1997 (18 750)
W&T in year ended 31 December 1997 (18 750)
W&T in year ended 31 December 1998 (18 750)
W&T in year ended 31 December 1999 (18 750)
If sold for $500 000: recoupment $75 000
If sold for $150 000: recoupment 75 000 x 150 000 = $28 125
400 000

(b) Cost $320 000 in year ended 31 December 1997; sold in year ended 31 December 2000
Restricted cost 75 000
SIA in year ended 31 December 1997 (18 750)
W&T in year ended 31 December 1998 (18 750)
W&T in year ended 31 December 1999 (18 750)
ITV $18 750
If sold for $350 000: recoupment $56 250
If sold for $120 000: recoupment
75 000 x 120 000 – ITV $18 750 = $9 375
320 000
(ii) Partly-private use
See the example on scrapping allowance at 9.3.3.
Had the motor-cycle realised $178 000 the recoupment, applying the same principle,
would have been:
ITV 128 000
Proceeds 178 000
Profit 50 000
Non-taxable being 26 000 x 50 000
72 000 18 056
Recoupment $31 944

(d) A further aspect of recoupments concerns the position where the proceeds are receivable in
instalments running into years of assessment subsequent to the year of sale. It is submitted that amounts are
then “recovered or recouped” only in the years in which payment is receivable by the seller, as in the
following example.
A taxpayer sold, in the year ended 31 December 2000, plant which had originally cost $200 000,
which had an ITV, at 31 December 1999, of $162 000, and which realised $240 000. The proceeds
were receivable equally over the year of sale and the year ending 31 December 2001. The position would
be:

Proceeds 240 000

79
Less: Capital profit ($240 000 – $200 000) 40 000
$200 000

Proceeds applicable to 2000 100 000


ITV 162 000
Balance of ITV carried forward (therefore no recoupment yet) 62 000
Proceeds applicable to 2001 100 000
Recoupment in 2001, equal to allowances granted $38 000

9.3.5 Training investment allowance (paras 1(1) (def’ns), 1(2) and 5)


This was discontinued with effect from 1 January, 2001.
(a) Any taxpayer carrying on a trade is entitled to a deduction of 50% of the cost to him of certain
buildings and equipment. This deduction is particularly valuable as it is over and above the capital
allowances, such as SIA and W&T, for which the item may qualify, and any recovery on subsequent
disposal, etc., is not subject to tax as a recoupment.
The qualifying items are:
(a) any training building; or
(b) any addition or alteration to any training building; or
(c) any training equipment;
which is bought into use in any year of assessment.
A “training building” is one which is constructed by the taxpayer and which is used exclusively to
provide trade training for the taxpayer’s employees or intended employees or for employees of any
associated company (namely any company which controls or is controlled by, or is under common control
with, the taxpayer company). Subsequent additions to such building by that taxpayer also qualify. A
building purchased by the taxpayer does not qualify, nor does one erected prior to the date quoted.
“Training equipment” means new or unused articles, implements, machinery or utensils purchased
and used by the taxpayer exclusively for the purpose of providing trade training for the taxpayer’s
employees or intended employees, etc., as above. (In forming an opinion the Commissioner may have
regard to the use to which the articles, etc., are put in their first year of use and in the next following
assessment year.)
“Trade training” means any education or training (other than education or training which is
provided as part of the general school education of a pupil) which is intended to train persons to perform
work in connection with the trade of the taxpayer or to improve their performance of such work.
(b) Example

In the year ended 31 March 1997 an industrialist expended $2 000 000 on the construction of a “training
building” and $300 000 on “training equipment”, SIA and accelerated W&T being granted. The facility
was sold in the year ended 31 December 2000, the building realising $3 500 000 and the equipment
$350 000.
The tax consequences would be as follows

1997 to 1999 Building Equipment Total Allowances


Cost 2 000 000 300 000
SIA and W&T 2 000 000 300 000 2 300 000
ITV $        –    $      –   
Training investment
allowance (March 1997) $1 000 000 $150 000 1 150 000
$3 450 000
2000
Total Building Equipment Recoupment
Recoupment $2 000 000 $300 000 $2 300 000

80
The capital profits of $1 500 000 on the building and $50 000 on the equipment are not subject to income
tax but the former would give rise to capital gains tax consequences (see chapter 22).
The above, which ignores any possible alternative treatment, such as a prior allocation of the proceeds to
the training investment allowance, or their apportionment, reflects what is understood to be the
Commissioner’s approach.

9.4 Allocation of purchase price (para 8(1) and (2))


Whenever there is a transfer of immovable property, such as commercial or industrial buildings, the parties
are required to furnish the Commissioner with a jointly-signed statement as to the cost to the transferee.
Whenever assets as above, whether immovables or movables, are transferred for a lump sum, a similar
statement is required allocating the price to the various classes of assets.
The Commissioner has various powers to determine the price if he is dissatisfied with the statement,
particularly with regard to fair market price, or if no statement is submitted.
9.5 Reconstructions, mergers, etc: elections (para 8(3))
9.5.1 General
The most significant aspect of the provision is that, where assets, on which SIA and/or W&T has been
granted, are transferred between companies under the same control, an election is available to transfer
effectively at “tax value” for tax purposes regardless of the actual price. This applies only if transfer takes
place in the course of a scheme of reconstruction of a group of companies or a merger (or other business
operation which, in the opinion of the Commissioner, is of a similar nature). The effect is one of deferment
in that no taxable recoupment arises at that stage and possible cash-flow problems are thus avoided. Full
liability is swept up later, in the hands of the transferee company, upon its reselling any such assets outside
the group.
The topic is important to any exercise in planning group affairs.
9.5.2 Example
In the course of a group reconstruction scheme company A, which was already in a taxable position on its
trading operations, sold to its wholly-owned subsidiary company B, in the year ended 31 December 2000,
for $1 000 000, plant which some years ago had cost it $1 200 000 and on which 100% SIA and W&T had
been granted.
If the election is exercised, company A will suffer no recoupment and company B will obtain no
allowances. (In the event of a subsequent resale, outside the group, for say $1 300 000, company B will
suffer a recoupment of the $1 200 000 allowed to company A.)If the election is not exercised, company A
will suffer a recoupment of $1 000 000 in the year ended 31 December 2000. Company B will be entitled to
claim, at best, SIA of $250 000 in the year ended 31 December 2000 and $250 000 accelerated W&T in
each of the years ending 31 December 2001 to 2003.
9.5.3 Further points on reconstructions, mergers, etc.
(a) For companies to be under the same control it is not necessary that there be 100% shareholding.
Section 2(2) of the Act provides that a company is deemed to be under the control of a person if the
majority of the voting rights attaching to all classes of shares in the company is controlled, directly or
indirectly, by the person (including an individual).
(b) In the example above, the exercise of the election would clearly be even more worthwhile if
company B were already in the position of having an assessed loss or if such a loss would be created by the
SIA flowing from non-election.
(c) The exercise of the election is not always appropriate.
In the example above, the election would have been pointless had company A instead had a
substantial assessed loss to absorb the recoupment arising from non-election.
Similarly if, due to (substantiated) increases in values, the inter-company price of the plant had,
instead, been $2 000 000, the exercise of the election would require careful consideration, depending on
respective taxable income and assessed loss positions. Such price would rank for SIA and accelerated
W&T in the hands of company B but company A’s recoupment would be restricted to $1 200 000.
(d) Where immovables are included in the scheme further factors arise.
Firstly, in the context of non-election, it must be borne in mind that such assets do not rank for
SIA and accelerated (but only long-term) W&T in the hands of the purchaser.
Secondly, the impact of capital gains tax, for which a deferment election is also available, must be
considered: chapter 22 refers. The elections under the two statutes are independent; one may be
chosen without the other.

81
(e) Where a number of group companies are involved the election may be exercised by some but not
by others.

Example (holding company with three wholly-owned subsidiary companies)


Holding
company A B C
Assessed loss brought
forward – – $400 000 –
Industrial building
Cost – $1 000 000 – –
ITV 1 January – – – –
Market value – $1 200 000 – –
Plant
Cost – – $500 000 $500 000
ITV 1 January – – – –
Market value – – $700 000 $450 000
Assuming:
(i) that the group is to be divisionalised and reconstructed by, for example, transferring the assets (at
full value) and operations to the holding company; and
(ii) that the scheme is accepted by the Commissioner as falling within the terms of paragraph 8(3);
the following decisions would be likely to achieve the best results.
Company A:
The election should be exercised as that company would otherwise suffer a recoupment of $1 000 000,
while the holding company would obtain W&T allowance of merely 5% per annum, though on $1 200 000.
(There would also be capital gains tax considerations: see chapter 22.)
Company B:
No election should be made since the recoupment would be restricted to $500 000, giving rise to a taxable
income of only $100 000, while the holding company would obtain SIA of $175 000 in year 1 and
accelerated W&T of $175 000 in each of years 2, 3 and 4.
Company C:
The election should be exercised since there is otherwise a recoupment of $450 000 in year 1, whereas the
holding company obtains SIA and W&T over only years 1 to 4.

(f) Where an asset is transferred subject to an election, and control of the recipient company is
thereafter taken over by another group, the election is again available in the event of a subsequent transfer
of the asset in a scheme of reconstruction, etc., within that new group.
9.6 Other election circumstances (para 8(3) and (4))
The election is available also:
(i) where assets are transferred between husband and wife;
(ii) where (a rare occurrence) a foreign-incorporated company, having carried on its principal
business in Zimbabwe, is being voluntarily wound up and transferring all its business to a Zimbabwe
company, the members’ shareholding being exchanged for shares in the new company; or
(iii) upon conversion of a company into a private business corporation or vice versa.
9.7 Growth point areas (s 15(2)(dd) and Sch 14)
9.7.1 General
To encourage commercial and industrial development in selected parts of the country the Minister may
prescribe an area as a growth point area (referred to below as a “GPA”). Where a taxpayer carries on or
launches a commercial or industrial operation in the prescribed area: (i) the assets used there for such
operation may qualify for further capital allowances; and (ii) the taxpayer may qualify for a favourable rate
of tax. Both aspects of the topic are dealt with here.
Attention must, however, be paid to the terms of the particular prescribing notice. In some instances
industrial operations only are favoured, whereas in others recognition is extended to commercial operations
also.
Geographically various rural areas, including mining locations, predominate, but recognition has been
afforded to certain urban areas including Chitungwiza (industrial operations only) and parts of Ruwa.
9.7.2 The allowances

82
(a) Buildings
Where the area has been recognised for commercial purposes the taxpayer may elect to claim, on the
construction of commercial buildings there, the 25% SIA and (it appears to be accepted) the accelerated
W&T allowances normally available only on industrial and farm buildings. Subsequent additions and
alterations rank similarly.
(b) Assets generally
For all areas there is a 15% investment allowance, over and above the normal capital allowances, (resulting
in deductions eventually totalling 115%) on the cost of business assets used in the area, namely:
(i) on the construction of, or additions or alterations to, commercial or industrial buildings or
staff housing; and
(ii) on new or unused articles, machinery, etc. (but excluding most motor vehicles).
This 15% allowance is not recoupable on any subsequent disposal.
These benefits extend also to movables and immovable property owned by an “outside” taxpayer but leased
to a company which carries on the operations in the area and in which the lessor owns at least 25% of the
shares.
9.7.3 Example
In the year ended 31 December 2000 a trader constructed a commercial building at a cost of $500 000, and
commenced trade, in an area which had been prescribed for all purposes as a GPA.
Allowances arising during the first four years are contrasted with those elsewhere, as follows.

GPA Non-GPA Total allowances


Asset Asset GPA Non-GPA
Cost 500 000 500 000
SIA 125 000 – 125 000 –
W&T – 12 500 – 12 500
Investment allowance 75 000 – 75 000 –
ITV 31 December 2000 375 000 487 500
W&T 125 000 12 500 125 000 12 500
ITV 31 December 2001 250 000 475 000
W&T 125 000 12 500 125 000 12 500
ITV 31 December 2002 125 000 462 500
W&T 125 000 12 500 125 000 12 500
ITV 31 December 2003 $ – $450 000 $575 000 $50 000

9.7.4 The rates of tax (s 14(3) of the FA)


(i) Where a person commences specified operations on a “new project” (approved by the
Commissioner) in a growth point area the taxable income from such operations is taxable, for the year of
commencement and the following four years, at the following rates :
Manufacturing 10%
Provision of infrastructure 15%
Normal rates of tax apply thereafter.
The term “infrastructure” relates to roads, bridges, sanitation or water reticulation.
There is no requirement that the above should constitute the taxpayer’s sole activity. Branch operations in a
growth point area therefore also qualify.
Paradoxically, if heavy capital expenditure is incurred on assets brought into use on the project in its early
years, the effective value of capital allowances is diminished through their being “used up” against taxable
income which attracts the low rate of tax.
(ii) Operations other than the above, which are conducted in a growth point area attract the
same rates of tax as those in other areas.

Chapter 10
Determination of Income Tax Payable
Synopsis
10.1 Introduction

83
A.  INDIVIDUALS : GENERAL
10.2 Rates of income tax generally
10.3 Credits (personal)
10.4 Examples
10.5 Rate of income tax: foreign dividends
10.6 Double taxation relief generally
10.7 Rate of income tax: lump sum payment from pension or benefit fund
10.8 Employees’ tax (“PAYE”) credits
B.  COMPANIES : GENERAL
10.9 Rate of income tax (s 14(2)(b) of the FA)
10.10 Branch profits tax
C.  GROWTH POINT AREAS
D.  EXPORT PROCESSING ZONES
10.11 General
10.12 Income tax
10.13 Withholding taxes
10.14 Capital gains tax
E.  INDUSTRIAL PARK DEVELOPERS
10.15 General
10.16 Income tax
10.17 Withholding taxes
10.18 Capital gains tax
F.  OTHER EXPORTERS
10.19 Manufacturing companies
G.  “BOOT” ARRANGEMENTS
10.20 General (FA s 14(1))
10.21 Income tax (FA s 14(2)(g))
H.  PENSION FUNDS
10.22 General
I.  DECEASED ESTATES
10.23 Rate of income tax
J.  TRUSTS
10.24 Rate of income tax (s 14 (2)(b) of the FA)
K.  TREASURY BILLS
10.25 Financial institutions and short-term insurers
L.  TOURISM
10.26 Tourism operations incentives
10.1 Introduction
Once the taxable income of a taxpayer for a year of assessment has been determined in terms of the Income
Tax Act there remain the calculations of income tax and, in the case of individuals, personal credits as
provided in the Finance Act. This chapter is largely devoted to such topic though certain additional
comments are, for ease of reference, included here.
A. INDIVIDUALS : GENERAL
Features in relation to individuals are that :
(i) the tax on their taxable income from employment is imposed by reference to a scale,
shown below;
(ii) the tax on their taxable income from trade or investment is at a fixed rate;
(iii) a husband and wife are assessed separately on their respective taxable incomes.
The term “taxable income from employment” is defined (s 14(1) of the FA) as “any part of an individual’s
taxable income which consists of remuneration as defined in the Thirteenth (i.e. PAYE) Schedule” (to the
Income Tax Act.).
The term “taxable income from trade or investment” is also defined there and is effectively an individual’s
taxable income other than from employment.
10.2 Rates of income tax generally (s 14 of, and Sch to, the FA)
The rates of tax for an individual for the years of assessment ended 31 December 1999 and 2000 are as
follows, subject to exceptions referred to below.

84
10.2.1 Taxable income from employment

Taxable
Income Bands Amount
Within
Band Rate
of Tax Amount
of Tax Cumulative
Total Tax
$ $ % $ $

1999
1 to 24 000
24 001 to 48 000
48 001 to 72 000
72 001 to 96 000
96 001 to 150 000
150 001 to 720 000 720 001 and over 24 000
24 000
24 000
24 000
54 000
570 000 -
20
25
30
35
40
50* -
4 800
6 000
7 200
18 900
228 000 -
4 800
10 800
18 000
36 900
264 900
* This is the effective rate, being 40% plus a 25% surcharge*
2000 (Tax payable to be increased by 3% AIDS levy)
1 to 30 000
30 001 to 60 000
60 001 to 90 000
90 001 to 120 000
120 001 to 150 000
150 001 to 720 000
720 001 and over 30 000
30 000
30 000
30 000
30 000
570 000 0
20

85
25
30
35
40
50* 0
6 000
7 500
9 000
10 500
228 000 0
6 000
13 500
22 500
33 000
261 000

* This is the effective rate, being 40% plus 25% surcharge


The AIDS levy does not, however, apply to the surcharge, which slightly complicates the tax calculation in
respect of higher levels of taxable income from employment. Thus, ignoring, for the purpose of this
illustration, personal credits which are dealt with below, the tax payable for the year ending 31
December 2000 by an individual with $1 000 000 of such taxable income would be :

Tax

720 000 sliding scale as above 261 000,00


280 000 at 40% 112 000,00
$1 000 000 at basic rates 373 000,00

Surcharge on $280 000 at 25% of 40% 28 000,00


AIDS levy : 3% of $373 000 11 190,00
$412 190,00

The sliding scale has been revised, resulting in the following for the years beginning 1 January. The tables
include higher-income surcharge for 2001, which is thereafter abolished and, for both years, the 3% AIDS
levy.

Taxable
Income Bands Amount
Within
Band Effective Rate
of Tax Amount
of Tax Cumulative
Total Tax
$ $ % $ $

2001 (AIDS levy 3%)


1 to 60 000
60 001 to 90 000
90 001 to 120 000
120 001 to 150 000
150 001 to 180 000
180 001 to 840 000 840 001 and over 60 000
30 000
30 000

86
30 000
30 000
660 000 0
20.60
25.75
30.90
36.05
41.20
53.20(1) 0
6 180
7 725
9 270
10 815
271 920 0
6 180
13 905
23 175
33 990
305 910
2002  (AIDS levy 3%)
1 to 90 000
90 001 to 120 000
120 001 to 150 000
150 001 to 180 000
180 001 to 480 000
480 001 to 840 000
840 001 and over 90 000
30 000
30 000
30 000
300 000
360 000 0
20.60
25.75
30.90
36.05
41.20
46.35 (2) 0
6 180
7 725
9 270
108 150
148 320 0
6 180
13 905
23 175
131 325
279 645
(1) This rate is the effective rate, being 40% plus a 30% surcharge, plus 3% AIDS levy on 40%.
(2) This rate is the effective rate, being 45% plus 3% AIDS levy.

The sliding scale has been revised, resulting in the following for the years beginning 1 January. The
effective rates of tax include the 3% AIDS levy.

Taxable
Income Bands Amount

87
Within
Band Effective
Rate
of Tax Amount
of Tax Cumulative
Total Tax
$ $ % $ $

2002
1 to 90 000
90 001 to 120 000
120 001 to 150 000
150 001 to 180 000
180 001 to 480 000
480 001 to 840 000
840 001 and over 90 000
30 000
30 000
30 000
300 000
360 000 0
20.60
25.75
30.90
36.05
41.20
46.35 (2) 0
6 180
7 725
9 270
108 150
148 320 0
6 180
13 905
23 175
131 325
279 645
2003
1 to 180 000
180 001 to 260 000
260 001 to 340 000
340 001 to 420 000
420 001 to 500 000
500 001 to 1 500 000
1 500 001 and over 180 000
80 000
80 000
80 000
80 000
1 000 000 0
20.60
25.75
30.90
36.05

88
41.20
46.35 0
46 480
20 600
24 720
28 840
412 000 0
16 480
37 080
61 800
90 640
502 640

10.2.2 Taxable income from trade or investment


There is no sliding scale. Such taxable income is taxable at a flat rate of 35% for the year of assessment
ended 31 December 1999 and an effective flat rate of 36,05%, being 35% plus the 3% AIDS levy, for the
year of assessment ended 31 December 2000. The 25% surcharge, which is applicable to the higher levels
of taxable income from employment does not apply to taxable income from trade or investment.
See 9.7.4 regarding growth point areas and below regarding other incentives.
Years beginning 1 January
(“Effective rates” including the AIDS levy. It appears not to apply to taxable income which is liable at the
various reduced rates.)

2001 2002
% %
Taxable income of:
Individuals from trade or investment 30,9 30,9
Companies and trusts (general) 30,9 30,9
Licensed investor in export-processing zone : 1st 5 years Exempt Exempt
Thereafter 15 15
Special mining leases 25 25
Companies or trusts : mining operations 25 25
Individuals : mining operations 30,9 30,9
BOOT and BOT(1) arrangements : 1st 5 years 0 0
2nd 5 years 15 15
3rd 5 years 20 20
Industrial park developer : 1st 5 years Exempt Exempt
2nd 5 years 10 10
New manufacturing project in growth point area : 1st 5 years 10 10
New tourism operations in an ATDZ(2) : 1st 5 years 30,9 0
2nd 5 years 30,9 15
3rd 5 years 30,9 20
Existing tourism operations in an ATDZ(2) 30,9 20
Export manufacturing companies(3) 30,9 20
Financial institutions and short-term insurers : including income from Treasury bills, bankers’ acceptances
and other discountable instruments 30,9 30,9
With effect from the year of assessment beginning 1 January 2002 :
(1) BOT (build, operate, transfer) agreements are accorded the same favourable rates as
BOOT arrangements. This provision constitutes a replacement of that relating to growth point area
infrastructure (see 9.7.4) which has been repealed. It is not clear whether such repeal was intended.
(2) See paragraph 2.3 above.
(3) See paragraph 2.2 above.

(These are the ‘effective’ rates, including, where considered applicable, the AIDS levy)

Years beginning 1 January 2002 2003

89
Taxable income of:
Individual from trade or investment 30.9 30.9
Company or trust (general) 30.9 30.9
Licensed investor in export-processing zone:First 5 years Exempt Exempt
Thereafter 15 15
Special mining lease 25 25
Company or trust: mining operations 25 25
Individual: mining operations 30.9 30.9
BOOT or BOT arrangement: First 5 years 0 0
Second 5 years 15 15
Third 5 years 20 20
Thereafter 30 30
Industrial park developer: First 5 years Exempt Exempt
Thereafter 10 10
New project in growth point area:
Manufacturing First 5 years 10 10
Thereafter 15 15
Provision of infrastructure (1) First 5 years 10 10
Thereafter 15 15
Tourism operator in approved tourist development zone (2) 15 15
Other tourism operators (3) 20 20
Export manufacturing company (4) 20 20
Financial institutions and short-term insurers: including income from Treasury bills, bankers’ acceptances
and other discountable instruments 30.9 30.9
(1) The favourable rate for growth point infrastructure, which had been repealed in error by the
Finance Act 2001, is being restored retrospectively by the General Laws Amendment (No. 2) Act
(2) See paragraph 2.4 above.
(3) 20% rate is available only if the operator’s turnover is not less than 60% in foreign currency.
(4) See 2.2 above.

10.2.3
Totally anomalous results arise from different rates of tax having been enacted for the two categories of
taxable income. Where, for example, the annual amount of such income (for 2000) is below $540 000 the
taxpayer whose earnings are entirely from employment pays less tax than his counterpart on the same
amount. Once their comparative taxable incomes exceed this amount it is the taxpayer whose taxable
income is entirely from trade or investment who pays the lesser. Conversely, where a taxpayer’s annual
taxable income is say, $30 000, no liability arises if this is from employment whereas if it is from trade or
investment the tax payable (being wholly at 36,05%) is $10 815.
10.3 Credits (personal)
10.3.1 General
The personal credits for the year of assessment ended 31 December 2000 are listed below, the range being
very limited. There are no longer any credits in recognition of, for example, marital status or, in general,
children and other dependants of the taxpayer.
10.3.2 Types of credit (ss 10 to 13 of, and Sch to, the FA)
* - increased with effect from 1 January, 2002
# - increased with effect from 1 January, 2003

** Elderly persons credit (s 10)


Taxpayer aged 59 years or more $3 000
* $ 12 000
# $ 20 000
Blind persons credit (s 11) $3 000
* $7500
# $20 000
Medical expenses credit (s 12)
*Treatment, etc., and invalid appliances

90
Contributions to approval medical aid society 50% of expenditure
50% of expenditure
* Disabled persons credits (s 13)
Disabled taxpayer $3 000
*$7500
#$20 000
Taxpayer with disabled child (including step-child and adopted child)
$3 000
*$7500
#$20 000
*

**
Credit not available if the taxpayer is ordinarily resident outside Zimbabwe throughout the year.
Reduced proportionately if the period of assessment is less than a year. This is done on a time-apportioned
basis where the taxpayer exists for less than the year of assessment. Virtually the sole instances are where
an individual dies or becomes insolvent.
There is an extensive definition of “medical expenses” which covers the net cost of, for example:
(a) any invalid appliance or fitting for the taxpayer, his spouse or child;
(b) medical or dental practitioner’s fees, prescribed drugs and medicines; accommodation,
treatment, etc., at a hospital etc; and conveyance by ambulance (all in respect of the taxpayer, his spouse
and minor children);
(c) contributions paid to a medical aid society in respect of the taxpayer, his spouse and
minor children.
If a taxpayer dies leaving medical expenses unpaid, any payment made out of the deceased estate
is treated as having been made before his death and thus ranks for credit accordingly.

With regard to the higher-incomes surcharge for the years beginning 1 January 1999, 2000 and 2001,
credits were, as the law stood, deductible before its application. (See the example at 10.4). A retrospective
amendment has resulted in this sequence being reversed, resulting in additional liability.
Example : Individual
Example showing the calculation of the income tax payable for the year ending 31 December 2002.

Salary } from own 850 000


Bonus } company 60 000 910 000
Director’s fees from other company25 000
Interest from own company 15 000
Interest from bank (after deduction of withholding tax) 8 400
Dividend from own company (after deduction of withholding tax) 6 000

Total $964 400
Net medical expenses paid $24 000

Example : individuals
Example showing the income tax payable for the year ending 31 December 2003. This illustrates total
liability. On the assumption that the employer concerned was on the ‘FDS’ system (see 21.5.3 (b)), PAYE
withheld will have already met the liability in respect of the employee’s taxable income from employment,
leaving only the amount due from him on his taxable income from trade or investment. Such amount
would be payable on his annual payment dates (see 21.5.2).

Salary } from own 1 700 000


Bonus } company 300 000
2 000 000
Director’s fees from other company50 000
Interest from own company 30 000

91
Interest from bank (after deduction of withholding tax) 10 000
Dividend from own company (after deduction of withholding tax) 12 000
Total $2 102 000
Net medical expenses paid $28 000

Calculation of taxable income:


At ‘employment’ rates:
Salary 1 700 000
Bonus 300 000
Less exempt (20 000) 280 000
$1 980 000

At ‘trade or investment’ flat rate:


Director’s fees 50 000
Interest 30 000
$80 000

(Bank interest and company dividend exempt)


Calculation of tax payable:
Tax on $1 980 000 at ‘employment’ rates:
1 500 000 at sliding scale 488 000
480 000 at 45% 216 000
$1 980 000 704 000

Less: Credit for medical expenses (available against either ‘employment’ or ‘trade or investment’ tax):

$28 000 at 50% (14 000)


690 000
Add: AIDS levy: $690 000 at 3% 20 700
(a) $710 700
Tax on $80 000 at ‘trade or investment’ flat rate:
$80 000 at 30% 24 000
Add: AIDS levy: $24 000 at 3% 720
(b) $24 720
Total tax payable: (a)+(b) $735 420
(Less credit for PAYE)

10.3.3 Transferable credits


In the case of certain credits of married persons, where the income tax payable by the entitled spouse is
insufficient to absorb them the excess may be carried over against the income tax payable by the other
spouse. Thus:
(i) where either spouse is blind the blind persons credit may pass to the other;
(ii) certain credits are always granted firstly to the husband but may pass to the wife, namely
the disabled child credit, and any credit in respect of the husband’s own disability;
(iii) where the wife is disabled, her disabled persons credit may pass to her husband.

92
The elderly persons and medical expenses credits are non-transferable and, if unused against the tax
payable by the entitled spouse, fall away.
10.4 Examples
Example 1
The following is a basic example showing the calculation of the amounts payable by a husband and wife,
who were ordinarily resident in Zimbabwe, for the year ended 31 December 2000.

Husband
Salary 840 000
Bonus 70 000
Directors’ fees from employer 50 000
Interest from Zimbabwe company 20 000
Dividend from same company 10 000
Zimbabwe bank interest, net after 30% residents’ tax on interest 5 600
Total $995 600
He paid $3 000 by way of medical expenses.
PAYE totalling $380 000 was withheld from his remuneration by his employer.
Wife
Profit (= taxable income) from own business $1 000 000
Husband’s assessment
Employment
Salary 840 000
Bonus: $3 000 exempt 67 000
Director’s fees 50 000
Taxable income from employment $957 000

Tax payable thereon


On 720 000 (at sliding scale) 261 000,00
On 237 000 (at 40%) 94 800,00
$957 000

Less: credit for medical expenses: 50% of $3 000 1 500,00 93 300,00


354 300,00
AIDS levy: 3% of $354 300 10 629,00
Surcharge on $93 300 at 25% 23 325,00
$388 254,00

Trade or investment
Interest from Zimbabwe company 20 000
Dividend (exempt) -
Zimbabwe bank interest (exempt) -
Taxable income from trade or investment $20 000

Tax payable thereon


At 35% 7 000,00
AIDS levy: 3% of $7 000 210,00
$7 210,00

Total tax payable as above


On taxable income from employment 388 254,00
On taxable income from trade or investment 7 210,00
395 464,00
Less: PAYE withheld 380 000,00
Tax payable $15 464,00

Wife’s assessment

93
Taxable income from trade or investment $1 000 000
Tax payable thereon
At 35% 350 000,00
AIDS levy: 3% of $350 000 10 500,00
$360 500,00

Example 2
Mr E and his wife, having been ordinarily resident in Zimbabwe, emigrated to South Africa on 31 July
2000. They were aged 60 and 58 respectively. Their accruals for the year ended 31 December 2000 were
as follows.
1 January 1 August
to 31 July to 31 December
$ $
Mr E
Salary from Zimbabwe employer 193 600 –
Salary from South African employer (1) – 50 000
Pension, from United Kingdom fund, for services rendered in Zimbabwe (2)

30 000
Sickness insurance policy proceeds (3) 16 800 –
Travel costs, for medical treatment, paid by employer (4) 25 000 –
South African POSB interest (5) 2 000 1 800
Interest on $200 000 Zimbabwe
building societies class “C” shares (6) 30 000 –
Interest from United Kingdom investment (gross) (7) 15 000 8 000
Less tax at source say (8) (1 500) (800)
Mortgage bond interest: Zimbabwe 14 000 –
Bank interest: Zimbabwe (9) 700 600
Less residents’ tax on interest (210) –
Less non-residents’ tax on interest (NRTI) (10) – 60
Mrs E
Salary from Zimbabwe employer 60 000 –
Bonus from Zimbabwe employer (11) 5 000 –
One-third commutation from Zimbabwe
retirement annuity fund (12) 16 000 –
Annuity from same fund (13) 10 500 7 500
Alimony (son’s maintenance) from
former husband (14) 7 000 3 800

Further information

Mrs E contributed $100 per month to an approved Zimbabwe medical aid society throughout the year (15)
and in September 2000 paid a $450 shortfall on hospital fees (15).
P.A.Y.E withheld in Zimbabwe was : Mr E $60 000,00
: Mrs E $6 496,00
The tax payable or refundable would be calculated as follows.
Mr E
Employment
Salary 193 600
Pension $30 000 – $12 000 18 000
Taxable income from employment $211 600

Tax payable thereon (at sliding scale) 57 640,00


AIDS levy: 3% of $57 640,00 1 729,20
$59 369,20
Trade or investment

94
South African interest 2 600
United Kingdom interest 15 000
Mortgage bond interest : Zimbabwe 14 000
Bank interest : Zimbabwe 600
$32 200

Tax payable thereon


At 35% 11 270,00
Less: Relief for United Kingdom tax (1 500,00)
Relief for Zimbabwe NRTI (60,00)
9 710,00
AIDS levy: 3% of $9 710 291,30
$10 001,30

Total tax payable as above


On taxable income from employment 59 369,20
On taxable income from trade or investment 10 001,30
69 370,50
Less: PAYE withheld 60 000,00
Tax payable $9 370,50

Mrs E
Employment
Salary 60 000
Bonus $5 000 - $3 000 2 000
Retirement annuity 18 000
Taxable income from employment $80 000

Tax payable thereon (at sliding scale) 11 000,00


Less: Credit for medical expenses: 50% of $1650 825,00
10 175,00
AIDS levy: 3% of $10 175 305,25
10 480,25
Less: PAYE withheld 6 496,00
Tax payable $3 984,25
Notes
(1) See 3.2.4: but the taxpayer was no longer ordinarily resident.
(2) See 3.2.6 and 6.3(v).
(3) See 6.3(vi).
(4) See 6.3(vii).
(5) See 3.2.7 and 6.3(ix).
(6) 6.3(xii), re maximum deposits qualifying for exemption, refers.
(7) See 3.2.7 and 10.6.
(8) See 20.2.3.
(9) See 6.3(xv).
(10) See 19.4(a)
(11) See 6.3(ii).
(12) See 16.6.2.
(13) See 3.1.2.
(14) See 6.3(xvii).
(15) See 10.3.2: re the shortfall the taxpayer was ordinarily resident for part of the year.

10.5 Rate of income tax: foreign dividends (para (f) of the definition of “securities” in s 2(1) of the
ITA: s 14(5) of the FA)
Dividends from foreign companies other than building societies, accruing to an individual ordinarily
resident in Zimbabwe, are liable to tax at a fixed rate of 20%. They are, accordingly, effectively separated

95
from income liable at “normal” rates. The tax is calculated on the dollar equivalent of the gross amount of
the dividend. No collection charges or other expenses in earning or relating to the dividend are deductible.
The Zimbabwe tax on the gross dividend may be reduced by any foreign income tax and withholding tax
(see 20.1).
Example
For the year ended 31 December 2000 the taxable income from employment of a locally-resident individual
comprised $105 000. He also earned $10 000 from foreign dividends (after incurring $200 expenses and
$1 800 foreign tax).
His tax payable would be calculated as follows.

Employment
Taxable income $105 000

Tax payable thereon at sliding (scale) 18 000


AIDS levy: 3% of $18 000 540
$18 540
Foreign dividends
Taxable income $12 000

Tax payable thereon at 20% 2 400


Less: Relief for foreign tax 1 800
$600

Total tax payable as above


On taxable income from employment 18 540,00
On foreign dividends 600,00
$19 140,00

10.6 Double taxation relief generally (ss 92 and 93 of the ITA)


While the principles are considered in chapter 20, the practicalities are illustrated at this point.
Example
A Zimbabwe resident had the following income for the year ended 31 December 2000.

Zimbabwe POSB interest 21 000


Other interest (from a company in Zimbabwe) 50 000
Foreign interest:
Switzerland (gross $30 000 less $12 000 NRT) 18 000
Ireland * (gross $30 000 less $9 000 NRT) 21 000
Foreign company dividends
Switzerland (gross $15 000 less $5 250 NRST) 9 750
Ireland * (gross $12 000 less $1 800 NRST) 10 200
$129 950
His medical expenses rank for a credit of $4 000.
The tax payable, taking into account relief (“DTR”) would be calculated as follows.
Zimbabwe POSB interest –
Other Zimbabwe interest 50 000
Foreign interest : Switzerland 30 000
: Ireland *30 000 60 000
$110 000
Tax thereon at 36,05% 39 655
Less DTR (see below) 19 815
19 840
Less medical expenses credit 4 000
15 840

Dividends: $27 000 at 20% 5 400

96
Less DTR (see below) 4 800 600
$16 440
DTR on interest
Tax payable prior to DTR 39 655
Taxable income excluding foreign interest: $50 000
Tax thereon at 36,05% 18 025
Zimbabwe tax attributable to foreign interest $21 630
Relief is the lesser of the Zimbabwe or the foreign tax in each case:
Switzerland * : 30 000 x 21 630
60 000 = $10 815 or $12 000 i.e. 10 815
Ireland* : 30 000 x 21 630
60 000 = $10 815 or $9 000 i.e. 9 000
Relief $19 815
DTR on dividends
Applying the same principle :
Switzerland * : 20% of $15 000 = $3 000 or $5 250 3 000
Ireland *: 20% of $12 000 = $2 400 or $1 800 1 800
Relief $4 800
* Zimbabwe does not have double taxation agreements. But see 20.7.

10.7 Rate of income tax: lump sum payment from pension or benefit fund (s 14(4) of the FA)
The principles are referred to at 16.7.2.
The amount being taxable income from employment, would, had there been no specific provision, been
taxable in accordance with the sliding scale as above. It is, however, subject instead to what is commonly
termed a “special rate” of tax. Thus an individual with taxable income from employment, for the year
ended 31 December 2000 of $110 000, plus a lump sum payment of $25 000 on withdrawal from a
pension fund, would generally be liable to the following (before credits):

Tax on $110 000 at sliding scale rates 19 500


Tax on $25 000 at 30% i.e. not proceeding into the 35% “band”
though the total taxable income from employment exceeds $120 000 7 500
$27 000
(Any credits are to be deducted and the AIDS levy added.)

10.8 Employees’ tax (“PAYE”) credits


As will have been seen from the above example any PAYE withheld by an employer from an
employee’s remuneration is credited against the tax payable by him, upon assessment, after the end of the
year. (See also 21.5.3.)
B. COMPANIES : GENERAL
10.9 Rate of income tax (s 14(2)(b) of the FA)
Subject to the exceptions referred to below the general effective rates of tax for a company (including a
private business corporation) for the years ended 31 December 1999 and 2000 is 36,05%, being a basic
35% plus an AIDS levy of 3%. There are no personal credits.
If the recipient company is ordinarily resident in Zimbabwe, foreign interest is taxable at the above rates,
and dividends accruing from foreign companies other than building societies are taxable at 20%. As in the
case of individuals relief is granted of the lesser of the foreign tax or the attributable Zimbabwe tax.
See 9.7.4 regarding growth point areas and below regarding other incentives.
10.10 Branch profits tax
This has been discontinued. It was last in force for the year of assessment ended 31 December 1998.
C. GROWTH POINT AREAS
See 9.7
D. EXPORT PROCESSING ZONES (ITA s 2(1) def’n and para 16 of Sch 3: FA s 14(2)(e))
The following is a summary of the income tax and other aspects, to which reference is made in other
chapters.
10.11 General

97
An “export processing zone” means any part of Zimbabwe which is so declared in terms of the Export
Processing Zones Act [Chapter 14:07]. A “licensed investor” in turn is the holder of an investment licence
covering his operations in the zone.
The concessions below are confined to the operations and distributions, etc., and assets, to which the
licence and operations relate.
10.12 Income tax
(a) Rate
A licensed investor is exempt for the year of assessment in which he commences operations and
for the following four years. Liability thereafter is at the rate of 15%.
(b) Employees
Any advantage or benefit, such as housing, “soft” loans, and the use of a motor vehicle or
furniture, is exempt. With effect from the 12 January, 2001, the exemption is restricted to 50% of the
employee’s other taxable income from the investor.
10.13 Withholding taxes
There are exemptions from non-resident and resident shareholders’ taxes, and from non-residents’ taxes on
interest, fees, royalties and remittances.
10.14 Capital gains tax
As indicated at 22.5, sales of “specified assets” are exempt.
E. INDUSTRIAL PARK DEVELOPERS (ITA s 2(1) def’n and para 17 of Sch 3: FA s 14(2)(h))
The following is a summary of the income tax and other aspects, to which reference is made in other
chapters.
10.15 General
An “industrial park developer” means a person who owns and maintains an “industrial park”.
The latter term is defined as any premises or area which is approved by the Minister by statutory instrument
and in which two or more persons, independently of the industrial park developer, carry on the business of
manufacturing or processing either goods for export from Zimbabwe or components of goods which are
intended for export from Zimbabwe.
The concessions below are confined to the operations and distributions to which the licence and operations
relate.
10.16 Income tax
An industrial park developer is exempt, for the year of assessment in which the industrial park is
established or approved (whichever is the earlier) and for the following four years of assessment, on
receipts and accruals directly from the operation of his industrial park. Liability on the taxable income
thereafter is at the rate of 15%.
10.17 Withholding taxes
There are exemptions from non-resident and resident shareholders’ taxes and from non-residents’ taxes on
interest, fees and royalties.
10.18 Capital gains tax
As indicated at 22.5, sales of “specified assets” are exempt.
F. OTHER EXPORTERS
10.19 Manufacturing companies
(a) Current position (FA s 14(3e) and 3(f))
With first effect for the year of assessment which ended on 31 December 2000:
where a company, wherever incorporated, conducts manufacturing operations in Zimbabwe
and in any year of assessment the value of its manufacturing output that it exports is increased by
20% or more as compared with the immediately-proceeding year of assessment,
the income tax with which the company is chargeable
in respect of so much of its taxable income as, in the Commissioner’s opinion, is derived from that
increased level of exports during that year of assessment
is calculated at a rate 10 percentage points less than the normal rate for companies.
With effect from the 1st January, 2001, the Finance Act, 2000 repealed the above reduction.
The value of a company’s exports is to be calculated in United States dollars or in some other convertible
currency approved by the Commissioner.
In the case of a company supplying both domestic and export markets some form of apportionment would
be necessary to arrive at the taxable income from the latter. This may be illustrated as below.

98
It is submitted that the effective tax rate of 25% applies, once the 20% threshold is reached, to the whole of
the increase in the value of exports.
Example
A manufacturing company has a taxable income, for the year ended 31 December 2000, of $8 400 000. The
value of its manufactured exports is Z$10 000 000 and the value of its total sales is Z$14 000 000.

Z$
Taxable income from exports 10 000 000 x 8 400 000
14 000 000 = 6 000 000
Deduct previous year’s taxable income from exports, using the same formula : say
4 000 000
Increase in taxable income from exports 2 000 000

Assume that this increase exceeds that of the previous year by at least 20% in US$ terms.

Tax payable
Z$ Z$
Taxable income liable at 25% 2 000 000 500 000
Balance of taxable income liable at 36,05% 6 400 000 2 307 200
8 400 000
Total tax payable 2 807 200

Had the entire taxable income been liable at 36,05% the tax payable would have been $3 028 200.
(As will probably have been appreciated the provisions benefit those manufacturing/exporting companies
as do not enjoy even more advantageous rates by reason of their operating in export processing zones.
Exporters operating in growth point areas would, however, find the above beneficial once their first four
years of operations had passed.)
(b) 1999 position (FA s 14(3b) to (3d))There was also a provision, which was operative only for the
year of assessment which ended on 31 December 1999, whereby:
A manufacturing company which was established during that year and which exported at least
40% of its output was taxable at the rate of 25% on its taxable income from manufacturing for that year;
and
Any other manufacturing company which exported 50% or more of its output was taxable at an
effective rate of 27% on its taxable income from manufacturing for that year.
“Output” at that time was measured by reference to quantity or volume and not to value.
The emphasis thus shifted from the percentage of output being exported to the percentage increase in
exports.
With effect from 1 January 2002 :
• the rate of tax on a manufacturer’s taxable income from manufacturing, if exports are at least 60%
(by quantity or volume, not value) of goods manufactured, is reduced from 30,9% to 20%. There is a
reference also to “processing” but it is unclear whether there is an intention thus to widen the benefit of the
reduced tax rate ;
• the double deduction in respect of export-market development expenditure, which had been
repealed with effect from 1 January 2001, is reintroduced (see 8.25). There is thus only a one-year “gap”.
With effect from 1 January 2003, the export qualification for the reduced rate of tax is reduced to 50% of
total manufacturing output. The term ‘manufacturing’ includes substantially changing the original form of,
or adding value to, the product.
(The reference to ‘processing’ has been removed retrospectively.)
G. “BOOT” ARRANGEMENTS
The following is a summary of the income tax and other aspects to which reference is made in other
chapters.
10.20 General (FA s 14(1))
A BOOT arrangement, the term being derived from build, (own), operate and transfer, is defined as one
“approved by the Commissioner, under which a person undertakes to construct an item of infrastructure for
the State or a statutory corporation in consideration for the right to operate or control it for a specified

99
period, after which period he will transfer or restore ownership or control of the item to the State or the
statutory corporation concerned”.
10.21 Income tax (FA s 14(2)(g))
The rates of tax for the periods after the commencement of the arrangement are :
For the first five years 0%
For the second five years 15%
Thereafter Normal
H. PENSION FUNDS (ITA para 2(i) of Sch 3: FA: s 14(2)(d))
10.22 General
Pension funds, having previously been exempt, have become categorised as liable to income tax. The rate is
to be 15%. Exemption is, however, sustained until such time as the Minister specifies by notice in the
Gazette. No particular method of ultimately determining taxable income has been enacted.
I. DECEASED ESTATES
10.23 Rate of income tax
See 15.2.1(e).
J. TRUSTS
10.24 Rate of income tax (s 14 (2)(b) of the FA)
See 15.4.1(d).
K. TREASURY BILLS
10.25 Financial institutions and short-term insurers
While, as observed at 6.3(xv), most holders of Treasury bills are exempt from income tax on the yield, this
is not so in the case principally of the above since no residents’ tax on interest is required to be withheld.
The rate of income tax on their taxable income from such bills is 30% (s 14 (2)(i) of the FA).
L. TOURISM
10.26 Tourism operations incentives
A new concept of “approved tourist development zone”, or ATDZ, (i.e. declared under the Tourism Act
and requiring the Commissioner’s approval) has been introduced with effect from 1 January 2002.
The intended result of amendments appears to be, though this is unclear, that the following rates of tax will
apply to taxable income from operations of a tourist facility, as defined in the Tourism Act, in an ATDZ:
• in the case of new operations :
The first five years of the operations 0%
The second five years of the operations 15%
The third five years of the operations 20%
• in the case of existing operations, provided that at least 60% of the turnover consists of receipts in
foreign currencies 20%
The Finance Act, 2002 is silent. The General Laws Amendment (No.2)Act No.14 of 2002 , promulgated
on the 24th January,2003, retrospectively repeals the 0%, 15% and 20% rates which had been introduced,
from 1 January 2002, in relation to tourism operations in an approved tourist development zone. It
provides only for a 15% rate to apply ‘after the fifth year’ of such operations. Clarification by way of
further amendment is awaited.

Chapter 11
Farmers
Synopsis
11.1 Introduction
11.2 Livestock
11.3 Crops and other stocks valuation
11.4 Disposal of stock otherwise than by way of sale
11.5 Growing crops: sales and other disposals
11.6 Drought relief and epidemic disease relief
11.7 Capital allowances
11.8 Timber
11.9 Orchards and vineyards
11.10 Further aspects

100
11.11 Example
11.1 Introduction
The method of income tax assessment of farmers does not differ greatly from that applicable to other
businesses. There are, however, special provisions relating mainly to: (i) the valuation of livestock and
crops for income tax purposes (Part III of the Second Schedule); (ii) the deduction of certain kinds of
expenditure (the Seventh Schedule); and (iii) the treatment of certain sales and purchases.
“Farmer” is defined in section 2(1) of the Act as “any person who derives income from pastoral,
agricultural or other farming activities including any person who derives income from the letting of a farm
used for such purposes”. The phrases “farming operations” and “farming purposes” are to be construed in a
similar vein. The owner of a farm who lets it and carries on no farming operations himself is thus
nevertheless able to claim the tax treatment accorded to farmers.
The question of whether operations constitute “farming” is one of fact and in many instances is self-
evident. It is accepted that it includes for example market-gardening and horticulture. The breeding of
racehorses has been held to constitute “farming”: ITC 1373 (1983) 45 SATC 189. It is sometimes difficult
to establish, particularly when a loss on operations is sustained, whether a part-time farmer is entitled to be
treated as a farmer for the purposes of the Act, or whether his farming activities are no more than a hobby.
The position is discussed at 11.10.5. In ITC 1629 (1998) 60 SATC 53 an agricultural contractor, using
machinery to carry out land preparation and harvesting for a large number of farmers, was held to be using
it for farming purposes though he himself was not, in the ordinary sense, a farmer.
11.2 Livestock
(a) Since much farming activity is concerned with “livestock” the meaning of the term is important,
yet it is not defined for general purposes. The scope of the Commissioner’s approach is, however, apparent
from his acceptance of, for example, chinchillas, crocodiles and ostriches as falling within such meaning. In
the case of game it appears that the animals would need to be under some form of control by the taxpayer
before they could be said to constitute his “trading stock”, the meaning of which is considered below.
(b) Section 8(1)(h) of the Act brings the value of closing stock into gross income. Section 15(2)(u)
then permits its deduction as opening stock in the following year. Part III of the Second Schedule sets out
the rules for determining its value, with particular references in paragraphs 10, 11 and 12(a), as below.
(c) Livestock is divided into “stud livestock”, which is defined as livestock bought by a farmer for
stud purposes, and “ordinary livestock”, the latter including, therefore, all livestock born on the farm and
any purchased for non-stud purposes.
(d) As regards stud livestock, a farmer may elect to adopt either:
(i) the “purchase price value” (defined as the cost price or, if the cost price is $200 or more,
an amount of $200, whichever the farmer chooses); or
(ii) the “fixed standard value” (“FSV”) (defined as the standard value fixed by the farmer
with the approval of the Commissioner, or, if the cost price is $200 or more, an amount of $200, whichever
the farmer chooses).
Regardless of the actual cost, the Commissioner has no power to require a higher value than $200
to be placed on any such animal (although the farmer may if he so wishes) but FSVs below $200 must be
officially approved. The effect of this provision is that a stud animal, being written down to $200, is
allowed to be virtually written off for tax purposes in the year of purchase.
(e) As regards ordinary livestock, the farmer again must make an election, this time between FSVs
(fixed by the farmer with the approval of the Commissioner, irrespective of whether or not the cost was
$200 or more) and “cost and maintenance value”. The latter means the cost of purchase or the cost incurred
in breeding the animals, as nearly as it can be ascertained, plus the cost of maintaining them. The “cost and
maintenance value” method therefore results in the stock value increasing each year until stock is sold, by
the cost of labour, feedstuff, dips, etc., incurred during the year.
The latter method is very rarely chosen in practice. The following comments are therefore
restricted to the FSV method.
(f) Whether the livestock is stud or ordinary, two basic rules apply.
Firstly the elections as to method and the determination of the values are both related to “classes”
of livestock, chosen by the farmer and approved by the Commissioner. The classes might be bulls, cows,
heifers, etc., but it is possible to apply further refinements, for example, according to the ages of
animals within each group.
Secondly, once an election has been made and an FSV for a class has been accepted:
(i) the Commissioner has no power unilaterally to alter it; and

101
(ii) while the farmer may alter it he may do so only with the Commissioner’s approval.
(g) The approval will not be granted unless the change results in a realistic stock valuation.
The “paper profit” resulting from an increase in FSVs can be substantial, particularly in the case of
a herd with low values. Taxation of such profit may, on prior application to the Commissioner, be spread
over more than one tax year. In general such an increase is pointless since it merely accelerates the
incurring of liability without sales proceeds being available. It should be undertaken, therefore, only if there
are good reasons, such as avoidance of forfeiture of an assessed loss under the “six-year rule”.
(h) Losses of livestock as a result of death are automatically allowed, since such livestock do not form
part of the stock on hand at the end of the year of assessment. Natural increases in livestock are
automatically taxed, in the period during which the livestock is in the farmer’s possession, since the value
of such increases is taken into account as each animal moves up, i.e. is “promoted”, from one class to
another with higher values at the end of each year of assessment.
Further, when considering livestock, the customary division of assets into “fixed” and “current” is
not relevant. All farm livestock falls within the definition of “trading stock”. Purchases are allowed as a
deduction and sales constitute “gross income”.

(i) As regards profits for tax purposes, the impact of “promotions”, referred to above, is seen in the
following example, which ignores purchases, disposals and deaths.

Stud Promo-

Bulls Cows Heifers Tollies Calves tions


Value
FSVs $200 $3 000 $1 500 $1 500 $500

Opening

Stock:

Value $600 $600 000 $150 000 $75 000 $70 000
895 600
Head 3 200 100 50 140

Births 120

Promotions:

In 100 90 50 240

Out (100) (140) (240)

Closing

Stock:

Head 3 300 90 100 120

Value $600 $900 000 $135 000 $150 000


$60 000 $1 245 600
Profit
$350 000

(j) A further aspect concerns livestock acquired by a person without payment of consideration, for
example by way of inheritance or donation. The treatment in such person’s hands is as follows.

102
If the heir or donee merely sells off the livestock without conducting farming operations with them
the proceeds are of a capital nature. If he commences farming, or introduces them into existing farming
operations, however, the position varies, as follows.
A deduction is allowable of, in the case of an heir, the fair market value, for which the valuation in
the estate concerned would be used. In the case of a donee the deduction is restricted to the amount which
would have been deductible in the donor’s hands had he sold the livestock. This usually means that the
donee’s deduction is at the donor’s FSVs.
For closing stock purposes an heir or donee who was already farming may use his existing FSVs.
Where the livestock in question have been used to commence farming, however, the farmer will have to
adopt FSVs, subject to approval. The Commissioner in practice accepts FSVs approximating to the estate
valuation with the result that there is little or no “paper profit” in the beneficiary’s hands in this respect.
11.3 Crops and other stocks valuation (para 12(b) of Sch 2)
Crops fall into two categories, namely those which have been reaped and those which remain in the ground
at the year-end. If crops have been reaped but remain unsold they must be taken at what the Commissioner
considers to be their “fair and reasonable” value. This could be taken to mean selling price but it is
understood that the Commissioner accepts cost by analogy with the position of a (non-farming) trader.
Crops which are not yet reaped are, in terms of departmental practice 21, to be valued at their estimated
cost of production. Strictly, however, it is considered that such crops do not fall within the definition of
“trading stock” (examined at 5.8) and that, if the farmer so wishes, no amount need be brought to account
in this respect, despite the planting and cultivation costs having been allowed. The same goes for, for
example, fertilizer which has been ploughed in.
Other stocks, comprising unutilized consumable stores (such as fertilizer, fuel and spare parts) constitute
“trading stock” as defined and must be brought in at an amount which the Commissioner considers to be
the fair and reasonable value, usually the cost price.
11.4 Disposal of stock otherwise than by way of sale (paras 12 and 13 of Sch 2)
If stock is consumed by the farmer or his family a “fair and reasonable value”, in practice accepted as cost
(or, in the case of livestock, fixed standard value) is to be brought into gross income. The same applies to
farm trading stock which is given away by the farmer. If, however, the donation is made in pursuance of a
scheme of tax avoidance, the Commissioner may include, instead, his valuation of the ordinary sale price of
such stock.
11.5 Growing crops: sales and other disposals (s 8(1)(g))
Where farm land is sold, the proceeds do not normally attract income tax. Where reaped crops are sold, the
proceeds are normally taxable. Where land is sold ordonated with crops growing on it, provided the crops
were grown with the intention of sale, their market value at the time of disposal is taxable. Liability does
not arise, however, on crops which were growing on land which has been inherited or received as a gift if
the new owner then disposes of the land, with the standing crops, without farming it.
11.6 Drought relief and epidemic disease relief (Sch 7)
The Act recognises, in two independent ways, the hardship arising where a farmer is forced, through stress
of drought or epidemic disease, to sell livestock (including the return of grazers) from a drought-stricken or
an epidemic area (as defined). The term “livestock” is defined as including in the present context, cattle,
sheep, goats, pigs, crocodiles, ostriches, fowls and any other animals or birds that are raised or possessed
by the farmer as livestock in the course of his farming operations. The two aspects relate to:
(i) the sales themselves, where there is provision for reduction and deferment of the income
tax liability on the (in practice estimated) taxable income from such sales; and
(ii) subsequent restocking, where there is provision for an outright additional deduction.
A.  Drought relief
11.6.1 Sales (para 5)
The operation of the relief provision is illustrated by the following example.
A taxpayer sold, in a drought-proclaimed period, 25 oxen which realised $112 500. His FSV for oxen, all
of which had been on hand at the beginning of the year, was $2 000. The direct herd expenses (opening
stock 160 head and closing stock 140 head) were $150 000. Drought relief on sales could be calculated as
follows.

Drought sales 112 500


Less: 25 head at $2 000 50 000
Expenses: $150 000 x 25 =

103
Less: ½ x (160 + 140) 25 000 75 000
Estimated taxable income from drought sales $37 500

Only one-third of the amount of $37 500, i.e. $12 500, is taxable in the current year, the remaining thirds
being taxable in each of the following two years.
Further points to note with regard to drought relief on sales are as follows.
(a) The rate of tax applicable to a taxpayer’s income from drought sales for a year of assessment was,
prior to the introduction of a flat rate of tax on taxable income from trade or investment (see chapter 10)
restricted to that taxpayer’s marginal rate for that year (commonly referred to as the special rate which, in
another context, is considered at 10.7). These considerations no longer arise in relation to drought sales.
(b) The deferment, or “three-year-spread”, as illustrated above, is at the taxpayer’s election.
The exercise of such election should now be automatic in the case of most taxpayers in view of the
flat rate of tax. In the case of frequent assessed losses, however, a deferment of the effective accrual of
taxable income would be counter-productive if a previous loss were thereby forfeited under the “six year
rule”.
A further consideration for an individual is that if his taxable income is low, a deferred accrual
could be subjected to much higher rates of income tax in the following year if his total income then
increased substantially.
(c) If a farmer makes a loss on his other farming operations his actual taxable income for the year will
be less than the estimated taxable income from drought sales. In these circumstances he may elect instead
that his actual taxable income be spread over three years.
(d) It is understood that the Commissioner accepts that all livestock sales effected during a drought-
proclaimed period are forced through stress of drought and accordingly rank for relief.
(e) Any return of cattle to the Cold Storage Company, under a grazier agreement, ranks in the same
way as sales of the farmer’s own cattle.
(f) Periods of drought, and the precise drought-stricken farms are all proclaimed by Statutory
Instrument through the Gazette.
11.6.2 Restocking (para 6)
The provision applies to the cost of restocking a herd which, in a drought-proclaimed period, has been
depleted by forced sales or by deaths. The cost of purchases is, of course, allowable in any event. The effect
of the provision is to allow as a deduction a further 50% of such cost. Such additional deduction is subject
to a restriction recognising the assessed carrying capacity of the land (“ACCL”) as determined by the
Department of Agricultural, Technical and Extension Services.
The operation of this provision is illustrated by an example.
A farmer’s livestock trading account for the year ended 30 September (assume a drought-proclaimed
period) reflects the following:

Head Head
350 Opening stock at FSV 500 000 100 Sales (April) 600 000
70 Purchases (September) 210 000 320 Closing stock at FSV
450 000
Expenses 150 000
Profit 190 000
420 $1 050 000 420 $1 050 000
The potential restocking allowance, at 50% of $210 000 is $105 000. This is subject to the restriction,
referred to above, in relation to the ACCL. Continuing the above example: Head
Assume ACCL determined at: 300
Compare with number on land immediately prior to the purchase (350 – 100) 250
Difference which should not have been exceeded 50
Restocking allowance restricted to 50/70 x $105 000 = $75 000 which is allowable
against the profit of $190 000.
(Any drought sales relief is ignored in this example.)

B.  Epidemic disease relief


The provisions considered at A. above in relation to drought relief apply equally where a farmer’s forced
sales arise through stress of an epidemic disease in an epidemic area (as defined).

104
11.7 Capital allowances
11.7.1 General
Farmers are entitled to special deductions in terms of the Seventh Schedule which are considered below.
They are entitled also, however, to various allowances, in terms of the Fourth Schedule, which were
considered in chapter 9; in brief the assets covered include farm staff housing, most other farm buildings,
permanent roads, water furrows and most movables such as plant and vehicles.
The distinction between Fourth and Seventh Schedule items is important, particularly because (i) an asset
qualifying under one does not qualify under the other and (ii) each schedule contains different tests and
restrictions. The most important differences are that, unlike assets falling under the Fourth Schedule,
Seventh Schedule items:
(i) have to be constructed, erected, etc. by the taxpayer and do not rank for deduction if they
are purchased with an existing farm;
(ii) rank for deduction in full, and not for SIA and W&T; and
(iii) do not suffer recoupment on disposal; Fourth Schedule allowances, with the exception of
the training investment allowance, are recoupable, as observed at 9.3.5.
This affects not only the income tax position but also, as will be seen in chapter 22, that for capital gains
tax on disposal.
11.7.2 The Seventh Schedule
The schedule contains provisions for the deduction of expenditure on the following.
(a) Water conservation work
This is defined as meaning “any reservoir, weir, dam or embankment constructed for the
impounding of water”. Unlike assets falling under the Fourth Schedule there is no requirement that the
dam, etc., be used during the year of assessment. Expenditure incurred during the year of assessment is
deductible in that year notwithstanding that the work may still be in progress.
(b) Boreholes and wells
Again expenditure on the sinking of the above is deductible in the year of assessment in which it is
incurred even if the work has not been completed.
It is understood that the cost of lining a borehole is treated in the same manner as the sinking
costs. Other items such as piping and pumps fall, however, under the Fourth Schedule.
(c) Fencing
The schedule requires not only that the expenditure be incurred during the year of assessment on
the erection of fencing by the taxpayer but also that the fencing be used in the carrying on of farming
operations.
(d) Expenditure incurred during the year of assessment on:
(i) the stumping and clearing of lands;
(ii) works for the prevention of soil erosion, such as contour ridging; and
(iii) aerial and geophysical surveys.
11.8 Timber
(a) Although there are alternative methods, to which reference is made below, a farmer who
establishes a timber plantation is entitled to be assessed in the same way as one who grows, for example,
tobacco or maize. Timber, notwithstanding its long-term nature, remains a crop and the principles relating
to crops, which have already been considered, apply equally, as do the following.
(i) All working expenditure is allowable as and when it is incurred. This includes ground
preparation, planting costs (including purchased trees, seedlings and seeds), cultivation, maintenance and
the establishment of fire-breaks.
(ii) Fourth and Seventh Schedule allowances on buildings, dams, etc., arise in accordance
with the usual rules.
(iii) The plantation, being effectively a crop in the ground, does not constitute trading stock
and need not be brought to account.
(iv) Sales of cut timber are taxable as and when they arise.
(b) The above method will generally be found to be the most beneficial through the early deductibility
of establishment and running expenses.
Since a claim for current deduction of working expenditure may, in the absence of other income,
contribute to an early build-up of assessed losses, the danger of elimination of the latter under the six-year
rule must again be borne in mind when choosing from the alternative methods.

105
(c) Another method which has been accepted by the Commissioner is that the farmer may elect to
accumulate working expenditure and carry it forward until such time as the cutting of timber for sale is
commenced, and at that stage to claim a proportion of the accumulated expenditure each year in relation to
the volume of timber sold.
(d) A further method, which is an election laid down in the Seventh Schedule, is more complicated.
Planting costs are carried forward until the timber reaches maturity (though maintenance costs and Fourth
and Seventh Schedule capital allowances remain deductible on a current basis). Such planting costs are
escalated annually by 5%, called the “fixed percentage”, and the amount of the fixed percentage is treated
as income for the year. Once timber sales commence a proportionate part of the aggregate of the planting
costs and the accumulated fixed percentage is deductible. The effect is to spread the natural increase in the
timber value over the years of growth instead of taxing it all in the year of sale. This very feature, however,
can make the election unattractive (apart from avoiding the six-year loss elimination referred to above).
(e) A further aspect of the taxation of timber is that of the purchase and/or sale of land with standing
timber, and related matters (ss 8(1)(g) and 15(2)(k) and (l)). The position is as follows.
(i) Where a farmer, who purchased a farm with standing timber, cuts and sells such timber,
or part of it, he is taxable on the proceeds but an appropriate proportion of the purchase price, approved by
the Commissioner, is deductible against such proceeds. The same principle applies to deductibility of an
approved value, if the farmer acquired the farm without payment of consideration, for example by way of
inheritance or donation.
(ii) Only in exceptional circumstances, such as those which obtained in ITC 1494 (1991) 53
SATC 206, considered at 4.3.2, will a farmer who cuts and sells his timber escape liability.
(iii) A farmer selling a farm on which he himself has grown timber for resale is taxable on the
value of the standing timber, less, it appears, any attributable working expenditure accumulated and, by
election, not already claimed (see above).
(iv) A person who sells a farm which he has acquired, for example, by way of inheritance or
donation, but which he has not farmed, is not liable on the value of standing timber even if it was grown for
resale by his predecessor.
(v) A person who incurs expenditure on acquiring the right to fell timber is entitled, once
cutting commences, to a deduction in proportion to the area felled.
11.9 Orchards and vineyards
(The rules referred to below are accepted by the Commissioner as applying in principle also to tea and
coffee plantations.)
As in the case of timber, alternative methods of taxation are available upon election by the farmer. Again
the straightforward approach will probably be found to be the more beneficial in order to obtain early
deductibility of running expenses, as follows (abbreviated from the comments above on timber).
(i) All working expenditure is allowable as and when it is incurred. Unlike the case of
timber, however, the cost of acquisition of purchased trees and seedlings is not allowable (on the principle
that, unlike a timber plantation, trees in the case of an orchard are part of the capital structure). A deduction
of the cost of seeds is, nevertheless, accepted.
(ii) Fourth and Seventh Schedule allowances arise.
(iii) The orchard, etc, including fruit on the trees, does not constitute trading stock and need
not be brought to account.
(iv) Sales of fruit are taxable as and when they arise.
(v) The cost of replacement trees is deductible.
The alternative method which the farmer may choose, and which is laid down in the Seventh Schedule,
provides for the carry-forward of planting costs (excluding the cost of trees) and all maintenance costs,
though Fourth and Seventh Schedule capital allowances arise, as always, in their appropriate years. Once
the orchard, etc., reaches maturity the working expenditure brought forward is allowable proportionately
over the estimated productive life. (There are also rules regarding the treatment of uprooting and
replanting.) Where an orchard, etc., which has been the subject of the election, is sold after it has reached
the productive stage, recoupment may arise of working expenditure allowed.
Again, the six-year limit on losses must not be overlooked when making the election.
11.10 Further aspects
11.10.1 Flower growing
With regard to expenditure on purchases of root stock, the Commissioner accepts that, where the
anticipated productive life is up to three years, such expenditure is deductible in the year in which it is

106
incurred. In instances of longer life the expenditure is viewed as being of a capital nature, as in the case of
trees for orchards.
11.10.2 Insurance premiums
Although premiums payable by a farmer relating to his farming operations or assets will almost invariably
be allowable as a deduction in terms of section 15(2)(a) of the Act, this does not apply to premiums payable
by an individual farmer on a policy on his own life. Such non-deductibility arises even in a situation where
an individual, as a condition for obtaining a loan for farming operations, is required by the lender to
participate in a group life assurance scheme: ITC 1389 (1985) 47 SATC 57.
11.10.3 Tobacco levy
The Draconian prohibition of deduction of the levy, to which reference is made at 7.7(p) and the
unavailability of relief by way of any credit, affects both the farmer and the tobacco trader. Its impact on a
small farmer is broadly illustrated as follows, for the year ending 31 December 2000 (the previous rate
having been 5%):

Sales of tobacco at auction300 000


Less: Tax-deductible expenditure, etc. (260 000)
Less: Levy: 2,5% of sales (7 500)
Financial profit $32 500
Tax payable on $32 500 + $7 500, say at 36% $14 400
Effective tax rate 44%
A farmer who operates at a loss for the year nevertheless suffers the levy. Further, its disallowance as a
deduction may create taxable income for such year.

11.10.4 Departmental practices


Further detail on some of the above topics will be found in the Commissioner’s published notes.
11.10.5 “Farming”: trade or hobby, etc.
Further to the comment at 11.1 it is necessary that the operations be undertaken with a view to making a
profit, and that this test will have been satisfied if such was the taxpayer’s serious intention despite its being
misguided. Where such intention is not established the claim will fail, with the operations falling into the
category of what is colloquially termed “hobby-farming” as opposed to a trade. Factors to which, it is
thought, the Commissioner will have regard include the size of the operation, the degree of development
undertaken, the availability of capital and the general surrounding circumstances. Together (as in the case
of capital and revenue accruals, considered in chapter 4) they will either support or discredit the taxpayer’s
statement as to his intention.
Claims for deductibility of losses arising from small-scale operations are likely to encounter resistance. In
H v COT (1981) 43 SATC 22, for example, the taxpayer’s use of the property in question, where he
resided, indicated merely a preference for country pursuits and way of life, while he was in full-time
employment in a nearby city. By contrast the taxpayer in ITC 1177 (1972) 34 SATC 147 succeeded in
establishing his intention ultimately to make a profit despite his livestock on hand, and sales of produce,
being meagre. He also overcame a further danger of disallowance, referred to elsewhere, namely the
contention that his operations were only preliminary, designed to bring his smallholding to an income-
producing state, in anticipation of his retirement.
An important approach to the test of intention emerged from the judgment in ITC 1424 (1987) 49 SATC
99. The Commissioner had argued that a loss should be disallowed on the grounds that, having regard to
the consistently unprofitable experience of the taxpayer’s enterprise in the past, no probability existed of
any ultimate profit. The court held, however, that a subjective test should be adopted. In this respect the
taxpayer had established the existence of a genuine intention to develop his land as a farming proposition in
the hope, based on reasonable grounds, that an ultimate profit would be derived.
11.11 Example
The following accounts and information relate to an individual, Mr G, in respect of the year ended 31
December 1999.

Accounts for the year ended 30 September 1999


(a) Livestock Account
Head Head
Opening stock:

107
2 Bulls 400 60 Sales 275 000
60 Cows @ $3 000 180 000 10 Deaths –
20 Oxen @ $2 500 50 000 Closing stock:
10 Heifers @ $1 50015 000 5 Bulls @ $200 1 000
40 Tollies @ $1 500 60 000 130 Cows @ $3 000 390 000
50 Calves @ $500 25 000 330 400 30 Oxen @ $2 500 75 000
Purchases 40 Heifers @ $1 50060 000
1 Bull 40 000 40 Tollies @ $1 500 60 000
2 Bulls @ $15 000 30 000 60 Calves @ $500 30 000 616 000
50 Cows 250 000 320 000
50 Inherited –
90 Births –
Profit 240 600
375 $891 000 375 $891 000
(b) Income Statement
Expenditure Income
Livestock expenses 380 000 Livestock profit 240 600
Crop labour 350 000 Tobacco sales (after $112 500 levy)2 137 500
Fertilizer 300 000 Maize sales 650 000
Electricity & telephone 60 000 Medical aid society refund (Mr G) 3 000
Motor vehicle expenses 250 000 Insurance claim 100 000
Repairs to machinery 95 000 Proceeds from sale of truck 325 000
Interest 200 000 Interest 21 500
General expenses 37 500 Private consumption 25 000
Insurance 125 000 Zimbabwean company dividend 7 500
Sundry expenses 102 500
Fencing 100 000
Hire of plant 110 000
Depreciation 200 000
Net profit 1 200 100
$3 510 100 $3 510 100

Notes
1. The taxpayer inherited 50 head of cattle from his late father’s estate and incorporated them into his
own herd. The following is established:
Inheritance F.S.V. of deceased Value for Estate duty
30 Cows$250 $90 000
20 Heifers $150 $40 000
2. The interest paid was in respect of a loan of $1 000 000. Of this, $750 000 was used to build his
house which was completed during the year. The balance was used to finance farming operational costs.
3. On 1 July 1998 a $300 000 lease-hire agreement relating to a maize-shelling machine expired and
the sheller was then purchased at a cost to Mr G of $60 000. The fair market value at that time was $150
000.
4. The insurance claim was in respect of destruction, by fire, of a grading shed.
5. The following schedules were submitted:
(a) General Expenses
Legal fees in respect of acquisition of water rights 25 000
Packing materials12 500
$37 500
(b) Insurance
Valuation fee (buildings: fire insurance) 5 000
Fire 50 000
Hail 60 000
Mr G’s life 10 000

108
$125 000
(c) Sundry Expenses
Aerial survey 25 000
Contour ridging to prevent soil erosion 37 500
Temporary roads 20 000
Attendance at beef seminar 5 500
Donation: church fete 2 500
Loan raising fee (re loan of $1 000 000) 12 000
$102 500

6. Assets
(a) Income tax values at 31 December 1998:– $
Grading shed 60 000
Plant and equipment } purchased in year 450 000
Trailers } ended 31 December 1997 300 000
Truck 125 000
(b) Other additions brought into use during the year:- $
Staff house 180 000
Grading shed replacement 150 000
Homestead 750 000
Pick-up (purchased October 1998 and used 90% for farming purposes)
1 500 000
Weir 180 000

Mr G elects to claim SIA


(c) Sales
The truck was sold in Mozambique. The original cost was $305 175 which had been
written off in earlier years’ accounts.
His taxable income would be calculated as follows.
– +
Profit per accounts 1 200 100
Tobacco levy 112 500
Depreciation 200 000
Inherited livestock { 90 000
{ 40 000
Proceeds from sale of truck 325 000
Dividend 7 500
Private interest payable 750 000 x 200 000 150 000
1 000 000
Maize-sheller recoupment (but see 5.12(i)) 90 000
Grading shed recoupment ($40 000) –
Legal fees 25 000
Life insurance 10 000
Donation 2 500
Private loan-raising fee 750 000 x 12 000 9 000
1 000 000
Insurance claim 100 000
Medical aid society refund 3 000
Recoupment 180 175
Fourth and Seventh Schedule allowances 891 500
$1 457 000 1 979 275
1 457 000
Taxable income $522 275

109
Capital allowances and deductions
Asset ITVs and Recoupment SIA/W&T
ITV
Additions or 7th Sch. 31 Dec
Allowances 1999
ITV
1 Jan 1999 $

Grading shed
(proceeds $100 000) 60 000 –
Plant 450 000 150 000 300 000
Trailers 300 000 100 000 200 000
Truck
(proceeds $325 000) 125 000 180 175 – –
Additions:
Staff house 180 000 25 000 75 000
Grading shed 150 000 37 500 112 500
Homestead 750 000 – –
Pick-up 1 500 000 375 000 1 125 000
Maize-sheller } 60 000 15 000 45 000
Maize-sheller } 90 000 9 000 81 000
Weir 180 000 180 000 –
Total recoupment $180 175
Total allowances $891 500

Chapter 12
Miners
Synopsis
12.1 Introduction
12.2 Capital redemption allowance generally
12.3 Capital redemption allowance: alternative methods
12.4 Capital redemption allowance: further aspects
12.5 Training investment allowance
12.6 “Mineral” and “mining operations”: meanings
12.7 Prospecting
12.8 Sale of claims
12.9 Reconstructions, mergers, etc.
12.10 Transfers between spouses
12.11 Example
12.1 Introduction
A miner is assessed in a manner similar to that applicable to a trader, with certain exceptions, notably the
method for claiming capital allowances. In the case of miners the capital redemption allowance, or “CRA”,
replaces most of those available to other taxpayers, namely the special initial, wear and tear, and scrapping
allowances, the allowances in respect of lease premiums and improvements, and the (section 15(2)(t)) “18-
months deduction”.
The six-year limit on the carry-forward of assessed losses, referred to at 8.27(c), does not apply to those
arising from mining operations.
Mining must be distinguished from prospecting as indicated at 12.7.
12.2 Capital redemption allowance generally

110
The allowance is granted on “capital expenditure” as defined in paragraph 1 of the Fifth Schedule. The
definition is wide and includes not only obvious items, such as mine buildings and plant, but also shaft-
sinking and pre-production administration costs, including interest. It does not include, for example, the
cost of claims and goodwill, or company flotation expenses.
In the case of certain assets, where the cost exceeds specified limits the excess is, for the purpose of
calculating the allowance, ignored. Recent and current limits, dependent on the date the expenditure was
incurred, are as follows:
(a) passenger motor vehicle: see 9.2.5 The limit was increased to “the first $500 000 effective from 1
January, 2002, and further to “the first $1 000 000 effective from 1 January, 2003;
(b) Staff housing generally (para. 1(1) and (3)):
There are, unlike the position for other industries, no restrictions on the allowances on dwellings
for miners’ employees save as regards employees at schools, etc., as below.
(c) Building used for the purposes of a school, hospital, nursing home or clinic in connection with the
taxpayer’s mining operations:
Aggregate expenditure on the entire school etc:
Between 1 April 1995 and 31 December 1998 $500 000
Subsequently $1 500 000
Thus “the first” $1 500 000 currently qualifies. There is no disqualification related to total
expenditure.
The limits on expenditure which is recognised for capital redemption allowances were increased,
effective from 1 January, 2002:
• in the case of aggregate expenditure on the entire school, etc., to “the first” $3 500 000;
• in the case of expenditure on each building used for housing school, etc., employees, to “the first”
$250 000.
And were further increased, effective from 1 Janaury, 2003:
• in the case of aggregate expenditure on the entire school, etc., to ‘the first’ $10 million;
• in the case of expenditure on each building used for housing school, etc., employees, to ‘the first’
$1 million.
There are tests, relating to occupancy, for determining whether a school or hospital is used for the
purposes of a taxpayer’s mining operations.
(d) Dwelling occupied by staff employed at the mine school, etc.:
Expenditure incurred on each dwelling:
Between 1 April 1995 and 31 December 1998 $50 000
Subsequently $100 000
Thus “the first” $100 000 currently qualifies. There is no disqualification related to total
expenditure.
(e) Dwelling occupied by individual shareholder where he is one of not more than four controlling
the company:
As for (d) above save that the restrictions in relation to shareholders’ dwellings are more strict in
that they are calculated by reference to the year in which the building was constructed.
(f) Example
During its financial year ended 31 December 2000 a mining company incurred the following expenditure
on its buildings which were brought into use in that year:

School: Building A 1 200 000


Building B 700 000
$1 900 000
Teacher’s house A 150 000
Teacher’s house B 90 000
$240 000
Clinic: Building $1 600 000
Nurse’s house $120 000
Ranking for allowances
School: Buildings: Maximum 1 500 000
Teacher’s house A: Maximum 100 000
Teacher’s house B 90 000

111
Clinic: Building: Maximum 1 500 000
Nurse’s house: Maximum 100 000
$3 290 000

12.3 Capital redemption allowance: alternative methods


No CRA arises until the year of assessment in which a mine first commences production. There can be an
exception to this rule where a taxpayer has more than one mine; see 12.4.2.
(The Act continues to provide miners with a choice of alternative methods of treatment of capital
expenditure incurred in periods prior to and after commencement of production respectively, such
alternatives affecting the speed at which CRA becomes deductible. For the latter reason the “new mines”
basis has for some time been the most favourable since it offers the fastest method of deductibility and
since there is no danger from the “six-year rule” regarding assessed losses. With the introduction of a flat
rate of income tax for individuals’ taxable income from trade or investment the alternative bases have
ceased to have any degree of importance. Only brief references are therefore made below to the “life of
mine” and “mixed” bases respectively.)
12.3.1 “New mines” basis (paras 4(4) and 4(8))
A person who conducts mining operations in a new mine may elect to deduct, in the year of assessment in
which production commences, both (i) the accumulated capital expenditure incurred prior to
commencement and (ii) that incurred subsequent to commencement, in that year. Capital expenditure in
subsequent years is deductible as and when it is incurred.
The term “new mine” is defined as including not merely one which only now first commences regular
production but also one which, having previously been in production, either has been closed down and
subsequently reopened, or has changed ownership and been reorganised with substantially new
development and new plant.
The basis is available both to individuals and to companies and applies regardless of whether the taxpayer
owns or tributes the mine.
12.3.2 “Life of mine” basis (para 2)
Where a taxpayer adopts this method he submits to the Commissioner an estimate (subject to certain
limitations) of the number of years for which operations are expected to continue, based on certified
estimates of ore reserves. The capital expenditure ranking for CRA (less any recoupments) is divided by the
life of the mine calculated from the commencement of the year of assessment concerned. A new estimate of
the life of the mine is required for each year of assessment.
12.3.3 “Mixed” basis (paras 4(2) and 4(3))
The above term does not appear in the Act but is a convenient means of describing a further method
whereby a taxpayer may choose what is effectively, in most cases, a mixture of the life of mine and new
mines bases.
12.4 Capital redemption allowance: further aspects
12.4.1 Replacements election (para 6)
A taxpayer with a producing mine may elect to deduct in full the cost of replacing any capital asset,
provided that such cost does not exceed $40 000. Clearly it would be only in the case of the life of mine
basis that such election would have any impact on the pace of deductibility. In the event of a subsequent
disposal of the replacement item, however, the choice may have an impact on recoupment of the proceeds;
see 12.4.3.
Since the monetary limit has not been increased for very many years the matter is of somewhat academic
interest.
12.4.2 Non-contiguous mine (para 5)
Where a taxpayer has both a productive mine and (as owner or tributor) one which is non-productive and
which is non-contiguous (taken by the Commissioner as not having a common boundary) he may elect to
set off the year’s capital expenditure incurred on the unproductive mine against that year’s income from the
productive mine, thus reducing his liability or creating or increasing an assessed loss. This applies
regardless of whether different minerals are involved.
Such election is always beneficial. Failure to take advantage of it means that such capital expenditure: (i)
will be allowable only when that other mine becomes productive (and then on whichever basis the taxpayer
chooses); and (ii) will never be deductible if that mine fails to produce.
12.4.3 Recoupment from capital expenditure (s 2(1) def’n and s 8(1)(i))

112
As in the case of, for example, manufacturers, there are usually income tax consequences when a miner
disposes of (or otherwise effects a recovery in respect of) any asset or expenditure on which he has
obtained allowances.
(a) In the case of such other taxpayers the general rule, as has been seen, is that the tax impact is
restricted to allowances granted. Thus a clothing manufacturer disposing of a sewing-machine for $3 000,
on which allowances of $2 000, being 100% of cost, had been granted, would suffer recoupment of only
$2 000; his capital profit of $1 000 would be tax-free. A miner’s recoupment would, however, by reason of
the definition, be $3 000.
(b) There are exceptions to this “full recoupment” rule. These arise, firstly, in the case of: (i) the sale
of an asset which has been the subject of a “replacement election” referred to at 12.4.1; and (ii) a recovery
(usually by way of insurance proceeds) in respect of damage to or destruction of an asset. In these instances
recoupment is restricted to the allowances granted to the taxpayer.
(c) Where the asset which is sold has been subject to a restriction of its cost, it is understood that the
Commissioner accepts an apportionment of proceeds for recoupment purposes. Thus where there is a sale
of, for example, a passenger motor vehicle which had cost $450 000, with CRA having been restricted to
$200 000, the recoupment would be:

Sale price say $450 000 : 200 000 x 450 000


450 000 = $200 000

Sale price say $500 000 : 200 000 x 500 000


450 000 = $222 222

Sale price say $360 000 : 200 000 x 360 000


450 000 = $160 000

(d) Recoupment may in certain circumstances not arise: see 12.9 and 12.10.
12.4.4 Change of ownership of a mine (para 8)
(a) When a mine is sold the parties are required to furnish the Commissioner with a jointly-signed
statement as to the proportion of the price relating to “capital expenditure” referred to at 12.2. The
Commissioner has powers to determine the proportion if he is dissatisfied with the statement or if no
statement is submitted.
The amount so determined constitutes a recoupment from capital expenditure in the hands of the
transferor and ranks for redemption in the hands of the transferee.
(b) Where the ownership of a mine is transferred for no consideration, such as in the case of a
donation, the transferor’s unredeemed balance of capital expenditure, if any, such as in a “life of mine” or
“mixed” basis case, is effectively eliminated in his hands and recognised for redemption in those of the
transferee.
(c) Where (a rare instance) a foreign-incorporated company which:
(i) has carried on its principal business in Zimbabwe; and
(ii) is being voluntarily wound up; and
(iii) is transferring all its business to a Zimbabwean company, with the members’
shareholding being exchanged for shares in the new company;
any unredeemed balance of capital expenditure effectively passes to the new company.
(d) Additional aspects of change of ownership are examined at 12.9 and 12.10.
12.5 Training investment allowance (paras 1(1),1(2) and 7)
In relation to the training of his mine employees a miner is entitled to an additional deduction of 50% of the
cost to him of certain buildings and equipment in the same way as a person carrying on any other trade. The
comments at 9.3.5, to which reference should be made, are relevant.
12.6 “Mineral” and “mining operations”: meanings
12.6.1 Mineral (def’n in s 2(1))
The effect is to include most substances readily recognisable to the layman but to exclude “any clay (other
than fire-clay), gravel, sand, stone (other than limestone) or other like substance ordinarily won by . . .
quarrying”.
The meaning is, of course, fundamental to the application of the entire Fifth Schedule.
12.6.2 Mining operations (def’n in s 2(1))

113
The effect of the definition is to include recognition of the following:
(a) any operations for the purpose of winning a mineral from the earth;
(b) subsequent smelting and refining, by the same taxpayer, of minerals won from the earth; and
(c) any other operations recognised by the Commissioner, such as the re-working of mine-dumps.
12.7 Prospecting (s 15(2)(f)(ii))
This term is not specifically used but, effectively, the section applies to expenditure incurred by a person on
operations either in searching for a potential claim or in searching for minerals after a claim has been
pegged.
Such expenditure is allowable in the year of assessment in which it is so incurred and, in the absence of
income from such operations, may be set off against other income from trade or investment.
Exceptionally the prospector who intends eventually to mine may elect that the expenditure be carried
forward to be allowed against subsequent income from mining operations. Such course could, however, be
disadvantageous since, if mining operations were to fail to materialise, deductibility of the expenditure
could be lost. The election is binding for any year of assessment for which it is made but not for any
subsequent year.
12.8 Sale of claims (s 9)
The tax consequences of the sale of a mining claim depend on whether the proceeds are of a revenue of a
capital nature (see chapter 22 regarding capital gains tax). The usual tests for capital and revenue accruals
apply, as discussed in chapter 4. Probably the commonest example of income tax liability arising on the
sale of a claim is where the taxpayer is unable to establish a genuine intention to mine the claim, giving rise
to an inference of speculative intentions.
In such a case the taxpayer has an election to spread the resultant taxable income over four years
commencing with the year of sale. In the event of death or insolvency in the case of an individual, or a
winding up in the case of a company, any outstanding instalments are brought into taxable income in the
period to date of death, etc.
12.9 Reconstructions, mergers, etc. (para 8(6))
As in the case of, for example, manufacturing companies (falling under the Fourth Schedule) considered at
9.5, an election is available where the ownership of a mine is transferred from one company to another in
the course or furtherance of a scheme of reconstruction of a group of companies or a merger (or other
business operation which the Commissioner agrees is of a similar nature). The effect is to permit such
transfers to be at “tax value” for tax purposes, regardless of the actual consideration. Where therefore the
transferor company has already claimed CRA in full, there will be no recoupment in its hands and no
allowance, in respect of the assets transferred, in the hands of the transferee company. Where the transferor
has an unredeemed balance of capital expenditure such balance is eliminated in the latter’s hands and ranks
for redemption in the hands of the transferee.
Unlike the Fourth Schedule there is no specific requirement that the companies be under the same control.
12.10 Transfers between spouses (para 8(7))
If the ownership of a mine is transferred from a husband to his wife, or vice versa, an election is available
similar to that considered at 12.9. The effect in the hands of the parties is the same.
12.11 Example
Messrs A and B had been in equal partnership in an emerald mine, “M Mine”, for ten years. At close of
business on 30 September 1999 they sold the mine to their newly-formed company for $5 650 000. They
admitted a third equal shareholder, Mr C, who contributed modern plant and equipment which would be
used to open up further gem deposits.
The accounting date of the partnership had been 30 September but the company adopted a 31 December
year end. The following accounts and information refer to the year of assessment ended 31 December 1999.

1. Partnership accounts for the financial year ended 30 September 1999

Administration 1 300 000 Sales of emeralds 4 750 000


Cost of extraction 3 850 000 Loss 467 500
Interest on capital accounts:
Mr A 37 500
Mr B 30 000

114
$5 217 500 $5 217 500

2. Allocation of sale price of fixed assets (accepted)


Partnership’s
Sale price cost price
Mine buildings 1 250 000 200 000
House occupied by Mr A 500 000 80 000
Mine manager’s house 400 000 70 000
Underground development 1 000 000 200 000
Plant and equipment 750 000 175 000
Reservoir 550 000 225 000
Claims 950 000 40 000
Goodwill 250 000 –
$5 650 000 $990 000

The partnership had adopted the “life of mine” basis and the unredeemed balance of capital
expenditure at 1 October 1998 was $60 000.
3. Company accounts and information for the period ended 31 December 1999

(a)  M Mine
Extraction costs 3 700 000 Sale of emeralds: 8 600 000
Gross profit 4 900 000
$8 600 000 $8 600 000
Administration 2 000 000 Gross profit 4 900 000
Salaries: Mr A 360 000 Profit on sale of claim 28 000
Mr B 360 000 Profit from insurance
Mr C 90 000 proceeds: boiler 36 000
Scrapping of plant (at cost) 170 000 Profit on sale of plant 54 000
Net profit 2 038 000
$5 018 000 $5 018 000

Capital expenditure:
Additions during the year
Shaft sinking 250 000
Boiler (replacement) (second hand) 35 000
Plant and equipment (ex Mr C) 500 000
$785 000

Disposals during the year


Asset Cost NBV Proceeds Profit
$ $ $ $
Claim 72 000 72 000 100 000 28 000
Boiler (insurance) 90 000 81 000 117 000 36 000
Plant 60 000 54 000 108 000 54 000

115
(b)  Dev Mine
This is situated 15km from the M Mine. It had not commenced production but the following
preliminary expenditure had been incurred during the year.
Salary: Mr C 337 500
Wages 500 000
Electricity 100 000
Interest (working capital) 50 000
Capital expenditure:
Borehole 175 000
Earth-moving equipment 2 250 000
Plant (bought from Mr C) 1 250 000
Administration block 300 000
House built for Mr C 1 625 000
$6 587 500

Computations, for income tax purposes, of the former partners and the company for the year ended 31
December 1999, using the elections to best advantage.

1. Partnership: Mr A and Mr B
(a) Loss per partnership accounts 467 500
Less: Recoupment (below) 3 983 750
Joint taxable income $3 516 250

Recoupment:
Total proceeds 5 650 000
Deduct: not subject to recoupment since had not ranked for CRA:

Mr A’s house 65 000 x 500 000 406 250


80 000
Claims 950 000
Goodwill 250 000 1 606 250
4 043 750
Less: unredeemed balance of capital expenditure 60 000
$3 983 750
Mr A Mr B
(b) Allocation of joint taxable income (see chapter 13) 1 758 125
1 758 125
Add: Interest on capital accounts 37 500 30 000
(1) $1 795 625 (1) $1 788 125

Salaries from company (2) $360 000 (2) $360 000

(1) Taxable income from trade or investment


(2) Taxable income from employment
(c) Capital gains tax consequences
See 22.22, example 3.

116
2. Company computation
Profit per accounts 2 038 000
Add: Plant scrapped 170 000
2 208 000
Less:
Profit on sale of claim 28 000
Boiler insurance net proceeds 36 000
Profit on sale of plant 54 000
CRA:
M Mine expenditure
New mines basis as below 4 517 000
Boiler replacement: “para 6” basis 35 000
Dev Mine expenditure: “para 5” basis: 6 587 500
Less: Excess on Mr C’s
house ($1 625 000 – $100 000) 1 525 000 5 062 500
9 732 500
Assessed loss $7 524 500

CRA: M mine expenditure: new mines basis


Ex partnership 5 650 000
Less: Claims 950 000
Goodwill 250 000
Excess on Mr A’s house:
($500 000 – $15 000 early limit) 485 000 1 685 000
3 965 000
Additional 785 000
Less: Boiler claimed on more favourable basis 35 000 750 000
4 715 000
Less: Recoupments:
Claims –
Boiler (restricted to cost) 90 000
Plant 108 000 198 000
CRA $4 517 000

Chapter 13
Insurance Companies
Synopsis
13.1 Introduction
13.2 Short-term insurance: general
13.3 Life insurance: general
13.4 Example: both short-term and life insurance business
13.5 Life insurance: further points
13.6 Sales of investments: short-term insurers
13.7 Demutualisation levy
13.1 Introduction
Taxable income derived from insurance business is, in terms of section 20 of the Income Tax Act,
determined in accordance with the provisions of the Eighth Schedule. Any non-insurance income accruing
to the same taxpayer is dealt with under the general provisions of the Act.
The Eighth Schedule deals with short-term insurance business and life insurance business respectively. The
former includes, for example, fire, accident, employers’ liability, marine, fidelity and guarantee policies.
The schedule is restricted to Zimbabwe policies and assets in Zimbabwe, in accordance with the concept of

117
“source” in the general provisions of the Act. Its terms are such that, particularly in the case of life
insurance, it overrides not only the operating results reflected in the insurer’s financial statements but also
the general provisions of the Act relating to the determination of taxable income.
13.2 Short-term insurance: general (paras 4 and 5 of the schedule)
The taxable income derived from short-term insurance business is calculated by establishing items
specified (i) as income and (ii) as deductions. These are respectively:
(i) (para 4):
(a) short-term insurance premiums received;
(b) other amounts received from Zimbabwe short-term insurance operations (such as
reinsurance commissions, even though the reinsurance may have been placed abroad);
(c) the reserve referred to in paragraph 5(e) of the schedule (see below) as at the end of the
previous year;
(d) any recoupment of amounts previously allowed under (see below) paragraph 5(a),(b),(c)
or (d) of the schedule.
(ii) (para 5):
(a) premiums paid on reinsurance;
(b) losses (i.e. claims paid under policies) less recoveries from reinsurers;
(c) other expenditure and losses to the extent to which they are incurred for the purposes of
the short-term insurance business or in the production of the amounts referred to in paragraph 4(b) of the
schedule, except those of a capital nature. (This follows closely the “general deduction formula” in section
15(2)(a));
(d) appropriate deductions provided for in section 15(2), i.e. regarding repairs, capital
allowances, lease premiums, etc., to the extent that these have not already been deducted from other income
and that they relate to short-term insurance. There is no deduction in respect of bad or doubtful debts. This
would in any event have been inappropriate since only receipts are brought in by paragraph 4, as indicated
above;
(e) the reserve for unexpired risks at the percentage adopted by the particular insurer and set
aside by him at the end of the year of assessment (commonly 40%).
(The Commissioner’s approach is understood to be that no deduction is available in
respect of provisions, made by some companies, in respect of accidents incurred but not reported
(“AINR”)).
13.3 Life insurance: general (paras 6 to 10)
Taxable income attributable to life insurance is calculated by using a formula. Its broad effect is to charge
to tax:
(a) the amount determined by multiplying the insurer’s local actuarial liabilities (other than pension
and benefit fund liabilities) i.e. factors A, B, C, and D in the formula, by the average percentage of interest
in excess of 3,5% earned on his local investments (factor E); plus or minus
(b) part of the net profits or losses on the realisation of local investments (other than those with the
State or local authorities) i.e. factors F and G; minus
(c) the income from the “top slice” of investments in Zimbabwe local authority and Government
stocks, namely such investments in excess of 35% of the insurer’s actuarial liabilities.
Factors A to E thus form the cornerstone of the formula. With regard to factor E, “interest” includes rentals
from immovable property (see 13.5.1) but not company dividends on ordinary shares. Factors F and G,
however, include profits and losses on sales of shares.
The principles of insurance taxation are best illustrated by means of an example, indicating the factors in
the life formula.
13.4 Example: both short-term and life insurance business

$
Premiums received during year
Short-term business 100 000 000
Local life business 170 000 000
Reinsurance outwards
One-quarter of all short-term policies,
at a commission of 20%
Claims payable (gross) during year

118
Fire and accident policies 80 000 000
Life policies 40 000 000
Administration expenses
Allocated to short-term 10 000 000
Allocated to life 25 000 000
Reserve for unexpired risks percentage: 40%
Reserve for unexpired risks (previous year) 45 000 000
Actuarial liabilities (previous year-end)
All local life policies 1 100 000 000 A
Policies re pension funds 190 000 000 B
Actuarial liabilities (current year-end)
All local life policies 1 300 000 000 C
Policies re pension funds 210 000 000 D
Total assets in Zimbabwe relating to
life business
At commencement of year 900 000 000 J
At year-end 1 220 000 000 K
Productive assets, related to life
business: average during year 1 050 000 000 }
Interest derived from all investments } E
related to life business 220 500 000 }
This includes $75 600 000 (M) on Government
securities, representing a yield
of 17% (O)
Profit on sale of investments (relating
to life business), of which $60 000 000
(re H) arose from the sale of Government
securities. 90 000 000 F

Solution
The taxable income is calculated as follows using, for illustration, a step-by-step approach in the case of the
life insurance formula.

(i) Short-term insurance business


Premiums received 100 000 000
Commission on reinsurance
(20% × $25 000 000) 5 000 000
Reserve for unexpired risks (previous year) 45 000 000
150 000 000
Less:
Reinsurance outwards (1/4 × $100 000 000) 25 000 000
Claims paid (3/4 × $80 000 000) 60 000 000
Administration expenses 10 000 000
Reserve for unexpired risks
(40% x $100 000 000) 40 000 000 135 000 000
Taxable income attributable to
short-term business $15 000 000
(ii) Local life insurance business
Step 1: Actuarial liabilities

(A – B) + (C – D)
2 : (1 100 000 000 – 190 000 000) + (1 300 000 000 – 210 000 000)
2

119
= 910 000 000 + 1 090 000 000
2 = 2 000 000 000
2
= $1 000 000 000.

Step 2: Rate of interest


E = 21 (per cent)
(E x 2) – 7
200 = 17,5 (per cent).
Step 3: Application of above
17,5% of $1 000 000 000 = $175 000 000.
Note:
If the insurer concerned had made no profits or losses on sales of investments, and disregarding
for a moment the special factor H allowance recognising investment in Government stock, the taxable
income would have been $175 000 000.
Step 4: Factors F and G: profits and losses on sales of investments
Profit $90 000 000 less $60 000 000 on Government stock = $30 000 000.
Apportion by reference to two alternatives i.e. total liabilities or total assets:
$30 000 000 x (1 100 000 000 – 190 000 000) + (1 300 000 000 – 210 000 000)
(1 100 000 000 + 1 300 000 000)
= $25 000 000: or
$30 000 000 x (1 100 000 000 – 190 000 000) + (1 300 000 000 – 210 000 000)
(900 000 000 + 1 220 000 000)
= $28 301 887.
Taxable profit therefore (the lesser): $25 000 000.
(Had a loss on sale arisen factor G would have required similar comparative apportionments.)
Step 5: Factor H: recognition of investment in Government stock
Firstly determine factor N, namely the interest derived from public securities related to life
policies connected with pension fund, etc., business.
N= 75 600 000
2 × (190 000 000
(1 100 000 000 + 210 000 000)
1 300 000 000)
= $12 635 245.
Factor H is therefore (factor O being 17 as indicated in the example):
75 600 000 – 12 635 245 – 17 × 7
2 000 × 1 000 000 000
= $3 464 755.
Step 6: Taxable income attributable to life business
Step 1 to 3 (factors A,B,C,D and E) 175 000 000
Step 4 (factor F) 25 000 000
Step 5 (factor H) (3 464 755)
$196 535 245

Note: In the example the following items relating to life business are, in view of the formula,
irrelevant:

120
Premiums received 170 000 000
Claims paid 40 000 000
Administration expenses 25 000 000

13.5 Life insurance: further points


13.5.1 Factor E: “interest”
Although factor E refers to the rate of “interest” earned, the calculation of the percentage yield must take
into account rentals earned by the insurer from life insurance operations, and thus also the cost of properties
let. This arises from the definitions of “productive assets in Zimbabwe” in paragraph 1(1) of the Eighth
Schedule and “securities” in section 2(1) of the Act. (Rentals comprise the net amount after expenses but
ignoring capital allowances.) Property which is occupied by the insurer in connection with his local life
insurance business is, however, excluded.
Ordinary stocks and shares, and dividends thereon, are similarly excluded, as are any assets as, in the
opinion of the Commissioner, did not produce investment income during the year of assessment.
The Commissioner as a matter of practice requires that factors E and O be calculated to five places of
decimals.
13.5.2 Factor F: reconstructions, mergers, etc.
Where an insurer’s investments in Zimbabwe are transferred to another insurer an election is available to
transfer, for tax purposes, at the transferor’s cost of acquisition, regardless of the actual sale price. This
applies only if the transfer takes place in the course of reconstruction of a group of insurers or a merger (or
other business operation which, in the opinion of the Commissioner, is of a similar nature).
The principle is otherwise the same as that described at 9.5.1.
13.5.3 Factor H: losses
Factor H cannot create an assessed loss. Where it exceeds the insurer’s taxable income from life insurance
for the year of assessment, i.e. the amount after steps 1 to 4 above, it is restricted to such amount. There is
also no carry-forward or carry-back of any excess. This does not, however, affect any loss on other
operations.
13.6 Sales of investments: short-term insurers
Where profits and losses on such sales arise in the course of carrying on insurance business they fall within
the terms of the schedule. If, however, investments are held other than for the purposes of insurance
business, any disposal falls to be tested, as indicated earlier, under the general provisions of the Act. In
either event the principles set out in chapters 4 and 7 regarding accruals, expenditure and losses of a capital
nature apply and again each case turns on its own facts. In ITC 1366 (1983) 45 SATC 33, for example, the
company failed to prove that profits on shares were of a capital nature, whereas in CIR v Guardian
Assurance Company South Africa Ltd (1991) 53 SATC 129, the company was successful. In Sentra-Oes
Koöperatief Bpk v KBI (1995) 57 SATC 109 the taxpayer failed to prove that it was a moneylender and
that a loss of funds placed on deposit was on revenue account.
13.7 Demutualisation levy (ITA s 36D and Sch 27: FA s 22D)
In the (rare) event of a demutualisation of an insurer, that is the conversion of a mutual insurance society
into an insurance company, a levy is charged on a computed part of the free reserves of the society.
Zimbabwe policyholders benefit from the conversion through the allotment of free shares in the new
company. The levy is apportioned to them according to their respective allotments but is payable by the
company on their behalf. Thus no “physical” liability for the levy normally arises in shareholders’ hands.
The rate of levy is 2,5 %.
Chapter 14
Partnerships
Synopsis
14.1 Introduction
14.2 Example
14.3 Accounting years
14.4 Change in membership
14.5 Source of profits
14.1 Introduction

121
A partnership is not a legal persona and this lack of such status is perpetuated by the definition of “person”
in section 2(1) of the Act. Although section 37(15) requires persons carrying on any trade in partnership to
submit a joint return, supported by such accounts as may be necessary to show the results for the year of
assessment, section 51(5) provides nevertheless for separate assessments to be made on each partner. In
practice the Commissioner accepts the provision of accounts by only one, usually the senior partner, which
are used for the purposes of assessing all the members of the partnership. Each partner is, again in terms of
section 37(15), liable to tax only in his individual capacity. A partner’s assessed loss from partnership
activities is allowed to be set off, in his hands, against his taxable income from other trade or investment,
and vice versa.
(A partnership may also suffer withholding taxes on various kinds of Zimbabwe income. See Chapter 19.)
Despite a partnership not constituting a “person” the Commissioner in practice accepts partnership
computations drawn up as reflecting, for example, disallowances of capital expenditure in the hands of the
partnership. (The effect, obviously, is in any event to disallow each partner’s proportionate share in his
hands in accordance with the ratio for sharing profits and losses as laid down in the particular partnership
agreement.) A joint taxable income (or loss) of the partnership is thus established at that stage of the
computation, which is apportioned to each partner, with further adjustments then being made in the hands
of each in recognition of items affecting them personally. This two-stage method of assessing each
individual’s taxable income is illustrated in the following example which reflects also the treatment of
certain other matters arising in the context of the taxation of partnerships. (It follows that a partner’s
drawings against partnership profits are irrelevant to the determination of his taxable income.)
14.2 Example
Mr T and Mr K practise as quantity surveyors in Bulawayo. Their accounts in support of tax returns for
the year ended 31 December 1999 reflect a profit of $4 000 000 (Mr T, 60%, $2 400 000; Mr K, 40%,
$1 600 000). Other details from the accounts are as follows.

$
Income:
Fees accrued 7 250 000
POSB interest (Bulawayo) 4 250
Debenture interest 9 600
Dividend from Zimbabwe-quoted company 5 500
Deductions:
Partners’ salaries:
Mr T 960 000
Mr K 840 000
Insurance premiums:
Loss of profit 40 000
Partnership joint life policy 80 000
Life policies for each partner’s benefit:
Mr T 30 000
Mr K 36 000
Medical aid society contributions for:
Mr T 7 200
Mr K 5 200
Staff 31 000
Annuity to widow of deceased employee 10 000
Interest on capital accounts:
Mr T 25 000
Mr K 22 000
Depreciation 44 750
Further information:
Neither partner has taxable income from any other source.
The partnership during the year purchased and used technical equipment which cost $400 000. SIA is
claimed.
Mr T borrowed funds to purchase his share in the partnership practice. Interest payable by him during the
year amounted to $32 000.

122
Mr T contributed $60 000 to the partnership pension fund and Mr K, not being a member of the latter,
contributed $50 000 to a retirement annuity fund. These amounts have not been charged in the accounts.
The deceased employee did not belong to the partnership pension fund.

Calculation of taxable income of each partner


– +
Profit per accounts 4 000 000
POSB interest exempt 4 250
Dividend exempt 5 500
Insurance premium: joint life (capital) 80 000
Annuity: excess ($10 000 – $2 000) 8 000
Depreciation 44 750
SIA 100 000
“Joint” taxable income 4 023 000
$4 132 750 $4 132 750

Mr T Mr K
Allocated between the partners 60/40 2 413 800 1 609 200
Partners’ salaries (1) 960 000 840 000
Insurance premiums: each partner’s benefit 30 000 36 000
Medical aid society contributions: each partner’s benefit 7 200 5 200
Interest on capital accounts 25 000 22 000
3 436 000 2 512 400

Interest paid by Mr T (32 000)


Pension fund contributions: Mr T (restricted) (2) (15 000)
Retirement annuity fund contributions: Mr K (restricted) (15 000)
Taxable incomes $3 389 000 $2 497 400

Notes
(1) Constituting taxable income from trade or investment, not from employment.
(2) Partnership deemed (para 1 of Sch 6) to be his employer in this respect.

14.3 Accounting years


14.3.1 General (ss 10(2) and 37(15))
Unlike the case of an individual operating as a sole trader, whose profits accrue day by day, each partner’s
share of the joint profit of a partnership accrues only on the “accounting date”, i.e. the date to which
partnership accounts for a year or period are agreed to be drawn (Sacks v CIR (1946) 13 SATC 343, now
embodied in the Act). Thus, in the case of a partnership commencing on 1 October and drawing its first
accounts for the year ending 30 September, each partner’s share in the whole of the first year’s profit will
accrue to him for tax purposes at 30 September only.
If, however, it is agreed that a partner is entitled to a monthly salary, such salary accrues to him monthly
(COT v Newfield (1970) 32 SATC 157).
With regard to the accrual of profits on the accounting date it is not possible for this to be deferred
indefinitely as the joint return and accounts are required to be submitted for each year of assessment.
14.3.2 Death of a partner (s 37(15))
Where a partner dies, if accounts are prepared to show the results of the partnership for the period to his
date of death, there will potentially be an accrual of profits to all the partners because such action gives rise
to an accounting date. Only the estate of the deceased partner is, however, liable at that stage on his share

123
of those profits. Accrual of the surviving partners’ shares is deemed to be delayed until the normal
accounting date.
If accounts are not prepared at the date of death the accrual both to the surviving partners and to the estate
of the deceased partner will await the next accounting date.
14.4 Change in membership
14.4.1 Tangible assets
(a) Where a partner leaves, or a new partner is admitted to, a partnership, this may give rise to: (i)
recoupments, in the hands of both the outgoing and the remaining partners, of capital allowances previously
granted; and (ii) the granting of further allowances to the remaining and incoming partners. This is the
Commissioner’s approach where, for example, the assets change hands at book value but, in relation to the
remaining partners it may, in the light of ITC 124 (1928) 4 SATC 119, be open to question. It would be so
open in any instance where there had clearly been no recoupment, such as, it is submitted, the case where a
deceased partner’s share is inherited either by a surviving partner or by a third party. (There appears in any
event to be some inconsistency in practice in the application of the approach.)
(b) Example
X and Y, who were already in partnership, admitted Z with effect from 1 January 1998. X retired with
effect from 1 January 1999. The values of the movable assets at the above dates, used for the purposes of
the settlement of partners’ interests, and there being no intervening additions, were respectively $450 000
and $540 000. The original cost had been $600 000 which had been allowed in full to the XY partnership.
SIA is claimed by the later partnerships. The accounting date in all cases is 31 December.
It is understood that in terms of the Commissioner’s practice the following allowances and recoupments
would arise in the years ended 31 December 1998 and 1999.

1998 XY partnership: recoupment ($450 000)


XYZ partnership: SIA $112 500
Effective impact on each partner:
X Y Z
Recoupment $450 000 225 000 225 000 -
SIA : 25% of $450 000 (37 500) (37 500) (37 500)
Net recoupment/(allowance) $187 500 $187 500 ($37
500)

1999 XYZ partnership: recoupment ($112 500)


YZ partnership: SIA on $540 000
Effective impact on each partner:
X Y Z
Recoupment 37 500 37 500 37 500
SIA - (67 500) (67 500)
Net recoupment/(allowance) $37 500 ($30 000) ($30 000)
Years ended 31 December 2000 to 2002 : YZ partnership W&T at $135 000 annually being 25% on
$540 000.

14.4.2 Intangibles
(a) Goodwill
Probably the commonest intangible giving rise to tax aspects is goodwill, the purchase and sale of which is
generally on capital account, with the seller thus incurring no liability and the purchaser obtaining no
deduction. Difficulties arise, however, if goodwill is linked with a share of profit, as indicated below.
(b) Share of profits
Caution should be exercised by taxpayers whenever an agreement is contemplated regarding the disposal of
a share of profits, whether by way of goodwill or otherwise. The purchaser, such as a remaining or
incoming partner, is likely to be denied a deduction on the grounds that the expenditure, on the acquisition
of a right to profits, is of a capital nature; yet the profits will be taxable in his hands.

124
Such a disallowance was upheld in ITC 1392 (1985) 47 SATC 75 where, as required in the partnership
agreement, after the death of a partner his estate was paid by way of goodwill 25% of the profits of the
partnership for each of the next five years. Liability on those profits nevertheless continues to arise in the
hands of the seller: ITC 350 (1936) 9 SATC 69. These results thus differ from the concept of a cession of a
right to income which, as was seen at 2.4, escapes liability.
On a related matter, in ITC 1358 (1982) 44 SATC 155, a retiring partner who received an amount termed
remuneration and share of work in progress was found liable as he was unable to establish that the amount
really represented goodwill under another name.

(c) Debtors
An incoming partner in, for example, a professional or retail business partnership will not be allowed a
deduction in respect of his share of debts which become bad, and which existed at the date of his
admission, since no part of such debts will have been included in his income. Such portion of bad debts, if
acquired by him, will also not be claimable by the existing partners since it will no longer belong to them
(see 8.7).
Conversely the incoming partner will not be liable on his portion of a recovery of any bad debt which was
allowed to the partnership prior to his entry. Existing partners will also not be taxable on his portion of a
recovery.
14.5 Source of profits
Reference was made at 3.1.3 to the case of CIR v Epstein (1954) 19 SATC 221. The partner conducting his
activities in one country was held to have derived his share of profits from a source in that country on the
grounds that this income represented the quid pro quo for the services which he rendered in earning
partnership profits.
Circumstances may arise where the above principle will not apply. Where, for example, a United Kingdom
resident, rendering no services in the United Kingdom, is a member of a partnership whose activities are
conducted in Zimbabwe by Zimbabwe partners, the source of his allocated share will, it is considered, be
Zimbabwe.

Chapter 15
Estates and Trusts
Synopsis
15.1 Introduction
15.2 Deceased estates
15.3 Insolvent and assigned estates
15.4 Trusts
15.1 Introduction
(a) As in the case of individuals and companies it is possible also for a (deceased or insolvent, etc.)
estate, or a trust, to constitute a taxpayer and to be liable to tax on its taxable income.
(b) An estate is a legal persona which comes into being by operation of law, as follows.
A deceased estate commences its existence with the death of an individual. It consists of the whole
of the deceased’s property, is administered under the Administration of Estates Act [Chapter 6:01] by the
executor or administrator, and is controlled by the Master of the High Court. It terminates when any
necessary realisation of assets has been effected, the final liquidation and distribution account has been
approved by the Master, the liabilities have been settled and the net assets have been distributed to the
heirs. (Distributions, under an interim account, may have been made in the meantime.)
An insolvent or assigned estate is created by order of the court on presentation of a petition for
surrender, sequestration or statutory assignment of a debtor’s estate. The estate is then administered under
the Insolvency Act [Chapter 6:04] by a trustee or assignee under the control of the Master of the High
Court. An insolvent estate terminates on a successful application for rehabilitation of the debtor.
For tax purposes an estate is in any event a “person” by virtue of the definition in section 2(1) of
the Income Tax Act.
(c) A trust, on the other hand, is not generally a persona though, in the present context, it may be a
“person” for tax purposes, as will be seen below. It is not created under any law and is not controlled by
any official. Trusts are commonly formed either:

125
(i) by an existing person (not necessarily an individual) who, by a written trust deed, names
persons as trustees and makes over to them various assets as the initial capital of the trust. The trustees then
administer this capital and deal with the income thereon in accordance with the conditions set out in the
trust deed; or
(ii) by the will of a deceased individual who does not wish all or part of his estate to be
handed over immediately to his heirs. Instead of keeping the deceased estate open for possibly a long time,
the will may nominate trustees to whom the executors are to hand over the assets in trust, to be dealt with in
accordance with the conditions set out in the will. The estate can then be wound up and the Master’s file
closed.
(d) The provisions of the Income Tax Act in relation to estates and trusts contain important but
sometimes convoluted definitions to which reference may in specific cases be necessary. These include, in
section 2, “beneficiary with a vested right”, and “income the subject of a trust to which no beneficiary is
entitled”, and in section 11(1) in relation to deceased estates only, “ascertained beneficiary”. Abbreviated
references to these and others will be made below in the contexts of estates and trusts respectively.
References to the theory of taxation of estates and trusts respectively have been kept separate
below but there is overlapping in some examples since trusts so frequently arise out of wills.
15.2 Deceased estates
15.2.1 General
(a) On the death of a taxpayer an assessment is raised on the deceased’s taxable income accruing to
that date and the tax is calculated as illustrated in chapter 10. A new taxpayer, the deceased estate, then
comes into being and there arises the question of determining in which party’s hands the income accruing
in the post-death period should be taxed (the bequest of the asset itself never giving rise to liability). Such
parties are the beneficiaries of the deceased, the estate itself or any trust created in terms of the deceased’s
will.
(b) The terms of the will are crucial, the basic rules as to who is taxable on income subsequently
derived from assets in the estate being as follows.
(i) Where a specific asset is left to a specific individual (e.g. “my debentures to my son S”)
the son is taxable on the income earned by that asset from the day after the date of death of the deceased.
The son is in this respect an “ascertained beneficiary” i.e. “a person named or identified in the will . . .
who . . . acquires...an immediate certain right to claim the present or future enjoyment of the income”
arising from a particular asset in the estate.
(ii) Where the will provides for a “residue” in an estate the estate is taxable, on the income
from the assets in residue, from the day after the date of death until the date of distribution by the executor.
(“Residue” arises in terms of a will through a clause such as: “. . . my debentures to my son S and the
residue of my estate to my daughter D”.) The estate is initially taxable on any income produced by the
assets in residue. The daughter is taxable only on income produced by such assets after their distribution to
her. The estate’s liability stems from there being no ascertained beneficiary pending the determination of
the residue. The same principle applies whenever it is not possible to point to the person who is to benefit,
as where the will confers income on, for example, “. . . children as may be born . . .”.
(iii) Where the will does neither of the above but provides for the whole estate to go to a
specific individual it appears to be the Commissioner’s practice to treat the position as being the same as
(ii), i.e. to tax the estate on the income in the period prior to distribution and the beneficiary only thereafter.
(c) Wills are, of course, not necessarily as straightforward as to leave assets outright. A common
variation is a usufruct, giving a beneficiary a right to income but not to inheritance of an asset itself. Thus a
father may leave an asset to his son but grant the widow a life usufruct, raising the question of whether the
widow or the estate is taxable in the post-death period while the estate is being administered by the
executor. The rules for liability on such income are, in comparison with the above:
(i) asset specified: the same as for (b)(i): the “ascertained beneficiary” is liable on the
income immediately;
(ii) residue: the same as for (b)(ii): the estate is liable on the income until the date of
distribution, and the beneficiary only thereafter;
(iii) “whole”: the position may differ from (b)(iii) in that the usufructuary is generally liable
immediately on the post-death income.
(d) In cases of intestacy the treatment is the same as that for a residue under a will.
(e) As regards the rate of tax to which an estate is liable, since the nature of the income will (apart
from exemptions) almost invariably comprise for example rentals and interest, such rate will be that

126
applicable to taxable income from trade or investment. (It is submitted that, even in the very remote
instances of an estate being liable on amounts arising from the employment of a deceased individual, such
amounts will not constitute “taxable income from employment”.)
There are no personal credits.
If the estate is ordinarily resident in Zimbabwe and its income includes foreign interest or
dividends, such income is taxable (dividends at 20%) and in both cases relief arises in respect of foreign tax
(see chapter 10).
In the (rare) event of an estate carrying on certain operations it could qualify for incentive tax rates
; again see chapter 10.
15.2.2 Examples
Example 1
Mr A died on 30 April 1999. His will specifically bequeathed to his sister the sum of $500 000 and to his
only son a block of flats. The remainder of the estate was left to the testator’s son and daughter in equal
shares. Among the assets forming the residue of the estate were interest-bearing debentures. The executor
of the estate distributed these to the son and daughter on 1 September 1999. The executor’s First and Final
Liquidation and Distribution account, for the period to 30 November 1999 was confirmed by the Master.
The income which accrued from 1 May to 31 December 1999 is assessable as follows.

(i) The son is taxable, as an “ascertained beneficiary”, on the rentals from the block of flats
from 1 May 1999, being the day after the death of Mr A.
(ii) The estate is taxable on interest accruing in the period 1 May to 31 August 1999.
(iii) The son and daughter are taxable on interest accruing in the period 1 September to 31
December 1999. 
(iv) The estate is taxable on any income from other assets in residue accruing in the period 1
May to 30 November 1999.
(v) The son and daughter are taxable on any income, from other assets formerly in residue,
accruing in the month of December 1999.
(vi) The sister incurs no liability on the cash bequest.

Example 2
Mr B died on 30 June 1999. In terms of his will he bequeathed: (i) his farm to his son, subject to a life
usufruct in favour of the widow, and (ii) the residue of his estate, comprising interest-bearing debentures, to
his son. The date of ascertainment was 30 November 1999.
The income accruing from 1 July to 31 December 1999 is assessable as follows.

(i) The widow is taxable on the farming income accruing throughout the period.
(ii) The estate is liable on interest accruing in the period 1 July to 30 November 1999.
(iii) The son is liable on interest accruing in the month of December 1999.

Example 3
Mr A (who was married out of community of property: see 15.2.3) died on 3 January 1999. His will
provided as follows:

(i) His farm was bequeathed as a going concern to his son B, on condition that an annuity of
$200 000 p.a. be paid to B’s mother out of the income from the farm.
(ii) A block of flats which he owned was bequeathed to his sons C and D and his daughter E
jointly, on condition that the income therefrom was to be held in trust until they attained the age of 21. (C
was born in 1976, D in 1982 and E in 1984.) If they died before reaching 21 the sons’ accumulations would
be payable immediately to their executors but the daughter’s would fall away and her share would devolve
on her surviving brothers.
(iii) The residue of the estate was to be divided equally among all his children.

The estate was wound up on 17 December 1999. Operations during the administration period were
reflected as follows.

Income: $

127
Sales of crops and farm livestock 600 000
Rents from blocks of flats 800 000
Dividends from share investments 70 000
Interest from company 10 000
Expenditure:
Farm expenses (all allowable) 150 000
Maintenance of flats 260 000
Alterations to flats 240 000
Interest on loan used to purchase shares 12 000
Costs of disposing of various assets
in the estate residue 13 000

The livestock was valued as follows:


At 3 January 1999, Mr A’s farm accounts included livestock at fixed standard values (“FSVs”) of
$800 000; the market value then was $1 000 000.
At 17 December 1999, the estate livestock had a market value of $1 100 000; at A’s FSVs they would be
valued at $600 000, while at B’s FSVs they would be valued at $700 000. (B was already farming and had
established his own FSVs.)
The taxable income of the various parties, arising from the executor’s accounts, would be determined as
follows:

B’s taxable income/(loss)


Sales of crops and livestock 600 000
Closing stock (at B’s FSVs) 700 000
1 300 000
Less:
Inherited livestock 1 000 000
Farm expenses 150 000
Annuity to mother 200 000 1 350 000
Loss $(50 000)

C’s taxable income


Rent from flats 800 000
Less maintenance of flats 260 000
$540 000
One-third thereof $180 000
D’s taxable income
The same as for C, since D is an ascertained beneficiary in respect of his share of the flats, even though the
income is to be accumulated for his benefit until 2003.

$180 000

E’s taxable income


Nil from the estate. She is not 21 and, if she dies before reaching 21, no share of accumulated income from
the flats will fall into her estate; she is not an ascertained beneficiary.
Mother’s taxable income
Annuity out of the farm income $200 000
Trust’s taxable income
Share of rentals from the flats, since E is not an ascertained beneficiary $180 000

Estate’s taxable income


Interest $10 000

128
Notes:

(i) Livestock is valued at the beginning of the period at the estate valuation.
(ii) The annuity, which is taxable in the hands of the mother, is reflected above as a
deduction in B’s hands. See further, however, at 15.2.3.
(iii) Alterations to the flats are an expense of a capital nature and are not allowable.
(iv) Interest on the loan used to purchase shares is (as well as the exempt dividends) removed
from the estate’s computation.
(v) Expenditure on realising estate assets is of a capital nature.

15.2.3 Further points regarding estates


(a) Assessed losses
Any assessed loss incurred by a deceased falls away, upon his death, and cannot be carried forward against
the income of the estate or any other taxpayer. Similarly any loss incurred by the estate in operations during
the post-death period falls away upon the winding up of the estate.
(b) Expenditure incurred against post-death income
As will have been seen in example 3, expenditure (of an allowable nature under the general rules of
deductibility) incurred during the period of administration of the estate, is allowable in the determination of
the taxable income in the hands of the appropriate party, be it the estate itself or a beneficiary. Conversely
inadmissible expenditure is disallowed in the determination of the taxable income of the party concerned.
(c) Marriage out of or in community of property
In the case of a marriage out of community of property the above rules apply only to income from the
assets of the deceased spouse since the assets of the surviving spouse would not fall into the estate. The
survivor would remain taxable on his or her income.
In the case of a marriage in community of property it appears, from the approach in Union Government v
Leask’s Executor 1918 AD 458 that upon the death of, for example, the husband, the wife would, during
the post-death period, be taxed in her own right on one-half of the income accruing from the assets in the
joint estate. Only the other one-half would fall to be dealt with in accordance with either the late husband’s
will or the rules of intestacy. (An exception occurs in the case of “massing” which is not dealt with here.)
(d) Ordinary residence
An estate is deemed by section 2(3) of the Act to be ordinarily resident in Zimbabwe if the deceased was
ordinarily resident in Zimbabwe at the time of his death.
This can be important in the case of, for example, the receipt of foreign interest.
(e) Annuities
In example 3 above the annuity was reflected as a deduction in B’s hands.
More specifically the reason why he was not liable on that portion of the farm income is that, having regard
to the terms of the will, he was a mere conduit pipe for the transmission of the income to the mother. In
these circumstances the will had created a fideicommissum. This principle rests on the decision in the
leading case of Holley v CIR (1947) 14 SATC 407.
The position must be contrasted with that where a will imposes the duty to pay an annuity in such a way
that it constitutes a bequest price; although the income beneficiary is burdened by the condition the annuity
is not linked to the income under the will (and may impliedly have to be paid out of the beneficiary’s other
resources). In these circumstances his income is not reduced for tax purposes, despite the annuitant also
being taxable on the annuity: Lambson v CIR (1946) 14 SATC 57, also a leading case.
In KBI v van Blommestein (1999) 61 SATC 145 the taxpayer had inherited two farms from his late father
subject to the condition that he pay interest to his mother and a bequest price to his two sisters. He paid the
former and paid interest on the latter.
The South African Supreme Court found: (i) that the interest paid to his mother was not deductible in
determining his taxable income from rentals earned from letting one of the farms; that the provisions of the
will did not create a fideicommissum in relation to the income earned from the bequeathed farm; and (ii)
that the interest paid to his sisters was partially deductible i.e. in the ratio of the value of income-producing
assets to the value of the whole inheritance.
(The case is illustrative also of the difficulty attaching to the topic as indicated by the degree of judicial
dissent. The Special Court had found in favour of the Commissioner in all respects; the High Court, in van
Blommestein v KBI (1997) 59 SATC 221, had found that the amounts received by the taxpayer and paid to

129
his mother did not form part of his gross income and had, regarding the payments to the taxpayer’s sisters,
reached the finding which was upheld by the Supreme Court. The latter’s findings were by a majority of 4
to 1.)
(f) Income from employment
An individual’s employment may give rise to accruals which are (i) taxable in either the pre-death or the
post-death period or (ii) not taxable in either.
Any amount which accrues during the pre-death period is taxable in the assessment to date of death. This
includes salary earned, a bonus already voted, and contractual commissions due at that stage.
Amounts accruing after death and which are taxable in the assessment for the post-death period are those to
which the deceased had a right, and which would have been taxable in his hands, had they accrued during
his lifetime. These include leave pay under a contract of employment, and contractual commissions falling
due after date of death.
Amounts accruing after death which are not taxable (in either period) are those to which the deceased had
no right, such as non-contractual leave pay (in the case of, for example, civil servants), a bonus voted after
death and directors’ fees as are not fixed in the company’s articles of association.
15.3 Insolvent and assigned estates
(a) Where an individual becomes insolvent during the year of assessment it is the Commissioner’s
practice to raise two assessments on him for, firstly, the period from 1 January to the date of insolvency
and, secondly, in respect of any income earned by the insolvent in his own right, for the period from the
latter date to 31 December.
In the case of both periods personal credits will (as a minor issue) arise, subject to apportionment:
see chapter 10.
(b) Additionally a new taxpayer, the insolvent estate, comes into being from the date of insolvency to
the date of rehabilitation. It is taxable on, for example, any income earned from the continuation by the
trustee, for the benefit of creditors, of any business previously carried on by the insolvent. (This applies
also to statutory, though not voluntary, assignments.) The estate is not entitled to any personal credits.
(c) In no case is any assessed loss, incurred prior to insolvency, carried forward against subsequent
income, i.e. of the insolvent personally, or of the estate during the insolvency period, or later after his
rehabilitation. (This applies also to both statutory and voluntary assignments.)
(d) The rules for ordinary residence of insolvent estates are the same as those for deceased estates,
referred to above.
15.4 Trusts
15.4.1 General
The comments which follow relate specifically to any trust created in terms of the will of a deceased
taxpayer, commonly termed a will trust. It must not be overlooked, however, that a taxpayer may, as a
donor, create a trust during his lifetime, i.e. an inter vivos trust, to which the provisions of section 10(5) and
(6) may apply to income accruing during his lifetime: see 2.7.5 and 2.7.6. Where such a trust continues
after the death of the donor the subsequent taxability of the parties follows the same rules as those for will
trusts.
The existence of a trust as a person for tax purposes depends on the terms of the will or deed under which it
is created. In terms of the definition in section  2(1) of the Act, a trust is a person only “in relation to
income to which no beneficiary is entitled”. The practical problem, however, is the same as has been seen
in the case of estates, namely that one must determine in which party’s hands the trust income should be
taxed after the death of the testator, such parties being: (i) the beneficiaries of the trust; or (ii) the trust
itself. Bearing in mind the definition referred to earlier, the question turns on whether any beneficiary of a
trust has a vested right to income. If he has he will, to that extent, be taxable on the trust income. Any other
trust income is taxable in the hands of the trust.
In this connection there are three common possibilities, as follows:
(a) a clear vested right, i.e. where the income has to be paid to a beneficiary, the trustee having no
discretion in the matter. Such income accrues to the beneficiary and, subject to its nature, he is taxable on
it;
(b) again a vested right, i.e. where, although the trustee has a discretion over the amount distributed,
any undistributed amount is nevertheless accumulated for the beneficiary. He is again, as in (a), taxable on
the income: CIR v Polonsky (1942) 12 SATC 11;
(c) a delay in the vesting of any right, i.e. where the beneficiary’s enjoyment of the income is entirely
at the discretion of the trustee and the income may otherwise fall into other hands. The trust is taxable on

130
all undistributed income. The beneficiary is taxable only on distributions for his benefit: Estate Munro v
CIR (1925) 1 SATC 163. There is similarly an accrual to the beneficiary once the trustee (or executor)
exercises his discretion by making an award to him such as by crediting an account, and despite distribution
as such being delayed: ITC 1570 (1994) 56 SATC 120.
The general rule is that the trust income retains its identity (e.g. dividends, etc) in the hands of the
beneficiaries. This is known as the “conduit pipe” principle: Armstrong v CIR (1938) 10 SATC 1.
Annuities form the exception to this rule and are taxable regardless of whether any of the trust’s income,
from which they are paid, is exempt: section 14 (3) of the Act.
For further comment on beneficiaries’ rights see 15.4.3(c).
(d) As regards the rate of tax to which a trust is liable, the Finance Act (s 14(2)(b)) provides that the
rate applicable to companies is to apply. (This is now, in fact, the same as that applicable to taxable income
from trade or investment in the hands of any person other than a company.)
There are no personal credits.
If the trust is ordinarily resident in Zimbabwe and its income includes foreign interest or
dividends, such income is taxable (dividends at 20%) and in both cases relief arises in respect of foreign tax
(see chapter 10).
In the (rare) event of a trust carrying on certain operations it could qualify for incentive tax rates;
again see chapter 10.
15.4.2 Examples
Example 1
Mr Z died on 30 June 1999 and the residue of his estate was left upon trust. The residue was ascertained on
31 December 1999. The trust assets comprised property and investments, and the accounts for the year
ended 31 December 2000 reflected the following income, net after deducting relevant expenditure.

Zimbabwe dividends 75 000


Zimbabwe (company) interest 125 000
Foreign rents 150 000
Foreign interest 50 000
$400 000

Mr Z was a Zimbabwe resident and his will was executed in Zimbabwe.


The will provided that:

(i) Mr X be paid $25 000 per annum out of trust income;


(ii) the testator’s son, a Zimbabwe resident, be paid 50% of the trust income remaining after
payment of the annuity;
(iii) after the death of both Mr X and the son the trust capital and accumulated income be paid
to Mr Y or, should he not survive, in equal shares to his surviving children.

The income for the year ended 31 December 2000 is assessable as follows.

(i) Mr X is taxable on the annuity of $25 000.


(ii) The son is liable on his 50% share of the remaining taxable income, and the trust is liable on its
50% , determined as follows:
Total net accruals 400 000
Less: Exempt: Zimbabwe dividends 75 000
Other non-taxable:
foreign rents 150 000 225 000
175 000
Less: Proportion of annuity: 10 938
$164 062
Son 50% $82 031
Trust 50% $82 031

131
(The annuity is not an allowable deduction as such. The effect is, however, the same; it has accrued to Mr
X and must therefore, after appropriate apportionment, be excluded from the taxable incomes of the other
taxpayers.)
Example 2
Mr D died on 30 June 1998 and the residue of his estate was left upon trust. The residue was ascertained on
31 December 1998. The trust created in terms of the will provided that the trustees should apply so much of
the income as in their discretion was necessary for the maintenance and education of the testator’s minor
son, to whom the trust capital and accumulated income was to be paid on his reaching the age of 25. The
will provided for alternative heirs should the son not survive to 25. During the year ended 31 December
1999 the trust assets earned net Zimbabwe rentals of $750 000 after charging $300 000 for structural
alterations to the property let. $65 000 of the $750 000 was paid for the maintenance and education of the
minor son.
The income for the year ended 31 December 1999 is assessable as follows.
The son is taxable on the distribution, for his maintenance and education, of $65 000.
The trust is taxable as follows:

Total net accruals 750 000


Add: Alterations (capital) 300 000
1 050 000
Less: Distributed to son 65 000
Taxable income $985 000

15.4.3 Further points regarding trusts


(a) Identity of the trust income
By way of expansion on what was said earlier regarding the retention of the identity of trust income in the
hands of beneficiaries (Armstrong’s case) this may apply only to distributions made in the same year as the
receipt by the trust. On the basis of remarks in SIR v Rosen (1971) 32 SATC 249, at 270, sums which are
held in the trust over the end of a year of assessment, i.e. 31 December, and distributed only later, may lose
their original identity. In the hands of the beneficiaries under a will trust they would become capital
receipts.
It appears that the same applies in a “post-Munro situation”. As indicated above, in relation to income
accruing during a year of assessment to a trust in a “Munro situation”, the beneficiaries are liable only on
the distributed income and the trust is liable on the amount of the balance. If the latter amount (after tax) is
distributed in a subsequent year such distributions will, on the basis of those remarks in Rosen’s case,
constitute capital receipts in the beneficiaries’ hands.
(b) Expenditure incurred against trust income
As in the case of a deceased estate, expenditure incurred by the trustee is allowable (subject to the general
rules) in the determination of the taxable income in the hands of the appropriate party, be it the trust itself
or a beneficiary. Conversely, inadmissible expenditure is disallowed in the determination of the respective
taxable incomes. Importantly, where a beneficiary has a vested right to capital any disallowance of capital
expenditure occurs in his hands: ITC 636 (1947) 15 SATC 120. It appears therefore that, in the converse
circumstances, the disallowance must occur in the hands of the trust, as illustrated in example 2 above.
(c) Beneficiary with a vested right
Some expansion is necessary on the remarks above regarding the meaning of this term. Quite apart from
the Polonsky and Munro cases, which form cornerstones, the point has arisen in a number of other
instances, including the following.
In ITC 239 (1932) 6 SATC 358 the facts were that the trustees were to determine a number of “deserving
institutions” and to distribute the income and, eventually, the assets to those selected. The institutions
would undoubtedly have been exempt from income tax under the equivalent of paragraph 2 of the Third
Schedule. It was held however that, except in respect of any income actually distributed in the year of its
receipt, no beneficiary had a vested right and tax was payable by the trust on all of its remaining income. In
Hilda Holt Will Trust v CIR (1993) 55 SATC 1, despite beneficiaries’ rights to the remainder of the income
being subject to an annuitant’s rights, such remainder was held to have accrued to such identified
beneficiaries under vested rights.
Where trust income is payable to a guardian for the benefit of a child the identification of the party in
whose hands such income is taxable appears to rest on whether the guardian is accountable to the trustees

132
for any underspending. Where such accountability does not exist the guardian is taxable: ITC 417 (1938)
10 SATC 264. Where it does the guardian is merely an intermediary for the child’s income: ITC 919 (1959)
24 SATC 236.
(d) Ordinary residence
A trust is deemed by section 2(3) of the Act to be ordinarily resident in Zimbabwe if :
(i) part of its income is from a source in Zimbabwe or the trustee is ordinarily resident in
Zimbabwe; and
(ii) the person who created the trust was ordinarily resident in Zimbabwe at the time he made
the trust instrument (i.e. the will, deed of donation, etc.).
This can affect liability in the case of, for example, the receipt of foreign interest (to
which, in terms of the will or deed, no beneficiary is entitled).
It could raise the question whether the appointment of a Zimbabwe-resident trustee is
undesirable, and whether separate trusts should be created for Zimbabwe and foreign securities.
(e) Employee share trusts
In some instances employers have created trusts for the benefit of their employees in order to encourage
loyalty and to provide an inducement to productivity. Such trusts take various forms, with the tax
consequences varying accordingly. The contribution, by the employer, of the trust capital normally
constitutes expenditure of a capital nature, though in the particular circumstances of the case, in Heather v
PE Consulting Group Ltd (1973) 48 TC 293, the taxpayer company succeeded in its claim for a deduction.
Employees generally remain (i) taxable on the value of any awards to them from the trust by way of, for
example, shares in their employing company, but (ii) exempt on dividends distributed via the trust by such
company: see Armstrong’s and Rosen’s cases above.
Where the trust purchases and sells shares it may find itself held to be taxable on any profits, as in ITC
1413 (1986) 48 SATC 167 (though subject to criticism in ITC 1450 (1989) 51 SATC 70) or may succeed in
a contention that they are of a capital nature, as in CIR v Pick ‘n Pay Employee Share Purchase Trust
(1992) 54 SATC 271.
With effect from 1 January 2002 where an employee participates in such a scheme (as defined), any
amount accruing to him from the sale to or other redemption by the scheme or trust of any stock, shares,
debentures, units or other interest of the employee in the scheme, etc., is exempt from income tax.
The definition of the term has, retroactively, been expanded to the effect that the trust (not ‘scheme’)
arrangements must be embodied in a notarized deed.
With effect from 1 January 2003 an amount received or accrued on an employee’s sale or disposal to the
trust of his shares or interest in such trust is exempt from the tax.

Chapter 16
Pension and Benefit Funds:
Contributions to and Amounts from
Synopsis
16.1 Introduction
16.2 Definition of a fund
16.3 Medical aid societies
16.4 Contributions to funds
16.5 Arrear contributions
16.6 Benefits from funds
16.7 Lump sum payments from funds
16.1 Introduction
(a) A taxpayer is entitled to the deductions set out in the Sixth Schedule to the Income Tax Act in
respect of contributions to a pension fund, or to a retirement annuity fund and, in the case of employers to a
benefit fund, each as defined in section 2. The position is considered below.
(A retirement annuity fund is basically a contract between an insurance company and an
individual, whether or not an employee, whereby the latter contributes in order in due course to draw an
annuity to supplement, or in lieu of, any pension from his employer’s pension fund. A retirement annuity
fund is technically a pension fund in terms of the definition but, since different tax consequences ensue, the
terms are distinguished throughout this book.)

133
(b) Amounts received in the ordinary course of events from the above funds comprise the benefits
envisaged by the fund, i.e. pensions in the case of a pension fund; sickness, accident or unemployment
benefits in the case of a benefit fund; and annuities in the case of a retirement annuity fund. Quite
frequently, however, an employee’s membership of, for example, a pension fund is terminated before the
envisaged benefits are due to commence, such as where an employee resigns or a fund is wound up,
resulting in his receiving a lump sum payment from the fund. This type of receipt (to which the First
Schedule to the Act relates) attracts different tax treatment and is also considered below.
16.2 Definition of a fund (s 2)
For contributions, etc., to qualify for the treatment provided in the Act a pension fund must either be
established by law in Zimbabwe, such as under the National Social Security Authority Act [Chapter 17:04]
or be registered under the Pension and Provident Funds Act [Chapter 24:09]. A retirement annuity fund
must also be so registered.
A benefit fund must be registered under the latter Act if it is of the (now uncommon) “provident fund”
type, i.e. providing for a lump-sum benefit on retirement. If, however, a benefit fund is of the type which
provides group life assurance cover, it must be approved by the Commissioner under section 13 (1) of the
Income Tax Act. (For conditions of approval see departmental practice 41.)
A fund which is established by an employer to fulfil any of the functions of a pension fund or benefit fund,
but which is neither registered as above nor approved by the Commissioner, is defined in the First Schedule
to the Income Tax Act as an “unapproved fund”. Contributions are not allowed as deductions.
16.3 Medical aid societies (s 2(1) def’n and s 13(2))
The rules relating to payments to and receipts from societies approved by the Commissioner are as follows.
(a) Contributions paid by employers are deductible: section 15(2)(j). Their value is exempt in the
hands of the employee: Third Schedule, paragraph 8(2).
(b) Contributions paid by an employee qualify for a credit in his hands (see chapter 10);
(c) Benefits received by participants are exempt: Third Schedule, paragraph 7(d).
16.4 Contributions to funds
16.4.1 General (s 15(2)(h) and Sch 6)
The Act authorises, and places limitations on, the deduction of the amount of contributions:
(i) by employers to benefit and pension funds;
(ii) by employees to pension funds;
(iii) by, for example, civil servants and by members of the Zimbabwe Republic Police and the
Armed Forces, to the Consolidated Revenue Fund; and
(iv) by individuals generally to retirement annuity funds.
Where restricted amounts are quoted below in relation to deductibility of contributions these reflect the
current position. Lower restrictions may have applied to earlier years’ contributions.
16.4.2 Employers’ contributions to benefit funds
(a) In respect of each member who joined the fund on or after 1 April 1958 the annual deduction is
restricted to $600 per member. Contributions in respect of the latter persons who are members of more
than one benefit fund are aggregated for the purposes of the limit.
(b) There is no restriction on the contributions allowable in respect of (the very rare instances of)
earlier members.
16.4.3 Employers’ contributions to pension funds
(a) In the case of any member who joined the fund on or after 1 July 1960, the deduction is restricted
to $15 000 per member; increased to $45 000 with effect from 1 January, 2002, and further increased to
$90 000 with effect from 1 January, 2003.
(b) As with benefit funds, if an employee is a member of more than one fund, the restriction on
deductibility of the employer’s contributions is applied to the employer’s aggregate contributions.
(c) The allowable annual deduction is, in respect of members who joined the fund before 1 July 1960,
restricted to the amount which would have been allowed under the fund’s rules as they were prior to that
date. If, therefore, the rules have not been changed since then, there is no restriction in the case of those
members.
16.4.4 Employer’s lump-sum contribution to a pension fund
Such payment is usually made in order to make up a shortfall disclosed by an actuarial valuation and thus
differs in principle from the employer’s “ordinary contributions” which are dealt with above. It is allowed
in full but the Commissioner may direct that the deduction be spread over such years as he determines.
16.4.5 Members’ contributions to pension funds

134
(a) If a member joined the fund on or after 1 July 1960 his deduction is restricted to $15 000 per
annum. If a person is a member of more than one pension fund the restriction is applied to the aggregate
contributions.
(b) For persons who contribute to retirement annuity funds only, the maximum deduction is similarly
$15 000. (No deduction is likely to arise in the case of a non-resident in view of the conditions imposed by
section 15(2)(h)).
(c) If a person is a member of his employer’s pension fund and contributes also to a retirement
annuity fund the aggregate $15 000 limit applies (i.e. despite a retirement annuity fund contribution
technically not satisfying the definition of “ordinary contribution” in paragraph 1 of the Schedule: ITC
1269 (1977) 40 SATC 60).
(d) If the member joined the fund before 1 July 1960, his then existing rights are continued, but
restrictions arise if the rules were changed on or after that date to increase contributions in order to confer
greater benefits.
With effect from 1 January 2001 the maximum deductible contribution to an approved pension
fund and/or retirement annuity fund was increased to $30 000.
With effect from 1 January 2002 the maximum deductible contribution to an approved pension
fund or retirement annuity fund was increased to $45 000.
With effect from 1 January 2003 the maximum deductible contribution to an approved pension
fund or retirement annuity fund is increased to $90 000.
16.4.6 Members’ contributions to benefit funds
The provisions under which these ranked for personal credits have been repealed. They last applied to the
year ended 31 March 1995. (Such contributions were never deductible in establishing taxable income.)
16.5 Arrear contributions (s 15(2)(i))
Arrear contributions by employees to pension funds, i.e. payments in respect of past service, may be
deducted in the year of payment, subject to restrictions reflecting the ceilings applicable to deductibility in
each such past year. Instances of such contributions are rare.
With effect from the 12 January, 2001 amounts allowed have been further aligned with the amounts
deductible for ordinary contributions.
16.6 Benefits from funds
16.6.1 Sums received from pension and benefit funds
(a) A retired employee’s pension from a source or deemed source in Zimbabwe is subject to tax
(section 8(1)(a)).
(For an example of the taxation of an employee, where contributions have been partly disallowed,
see 5.1(c).)
(b) Where the retiree exercises a commutation election the commuted amount is almost invariably not
taxable. This stems from the fact that :
(i) it is understood that the Registrar of Pension and Provident Funds does not approve fund
rules which would permit commutations in excess of one-third of a pension (save where the monthly
pension is less than $400); and
(ii) section 8(1)(q) of the Act taxes either :
– any commutation in excess of one-third; or
– $20 000;
whichever is the greater.
With effect from 1 January, 2001 the non-taxable portion of a pension commutation is increased to
the greater of one-third of the amount due or $60 000.
With effect from 1 January, 2003 the non-taxable portion of a pension commutation is increased to
the greater of one-third of the amount due or $250 000.
(c) Benefits paid by a benefit fund in respect of injury, sickness or death are exempt: Third Schedule,
paragraph 7(b).
16.6.2 Annuity from a retirement annuity fund
The position regarding a commutation depends on whether the taxpayer’s contributions commenced either
(i) prior to, or (ii) on or after, 1 August 1970. In the case of the former the commutation, even if it relates to
the entire annuity, is excluded from gross income and is thus not taxable. In the case of the latter the
exclusion applies only to a maximum commutation of one-third, with any excess being taxable.
Section 8(1)(n) refers.
(The difference in treatment reflects the date from which the Act was amended to cure the deficiency.)

135
16.7 Lump sum payments from funds
16.7.1 General (s 8(1)(c) and Sch 1)
As indicated earlier, an employee may receive an amount, on termination of his membership of a pension
fund, either (i) because he resigns (before retirement age) or for some reason ceases to be a member of the
fund, or (ii) when a fund is wound up because an employer goes out of business or decides to discontinue
the fund. Such amount is referred to as a “terminal benefit”. It thus differs from a commutation, which is
referred to above, and which arises upon retirement and the commencement of pension accruals.
For tax purposes the terminal benefit is firstly apportioned in the ratio of the employee’s periods of
employment inside and outside Zimbabwe respectively in respect of which contributions were originally
made, thus eliminating the computed non-Zimbabwe portion. The remaining portion which applies to the
Zimbabwe employment is referred to as a “lump sum payment”.
A payment to a member of a benefit fund constitutes a terminal benefit, and thus a lump sum payment, if he
receives it either on his premature withdrawal from or the winding up of the fund or upon his retirement.
16.7.2 Lump sum payments from pension funds (s 8(1)(c)(iii) and paras 1 and 6 to 9 of Sch 1)
(a) Members who joined on or after 1 July 1960
(These are the cases which are most frequently encountered.)
Where the payment does not exceed $300 it is exempt.
Where it exceeds $300 there is an exemption of any amount as:
(i) is used to purchase an “annuity on retirement” or is transferred to another pension fund;
or
(ii) represents a return of a member’s contributions which were disallowed as a deduction at
the time they were paid.
(b) Members who joined before 1 July 1960
If the fund has not changed its rules payments are entirely exempt.
If the fund has changed its rules , where the payment does not exceed $300 it is exempt; otherwise
there is an exemption of:
(i) the lump sum which the member would have received had the rules not been changed;
plus
(ii) so much of the balance as is used to purchase an “annuity on retirement” or is transferred
to another pension fund.
With effect from 1 January, 2001, the exemption was increased to $ 2000.
16.7.3 Lump sum payments from benefit funds (paras 2 to 5 of Sch 1)
As indicated earlier such finds are now rarely encountered.
(a) In the case of members who joined on or after 1 July 1960 there is an exemption of $4 000 plus so
much of the balance as (i) is used to acquire an “annuity on retirement” or (ii) is transferred to another
benefit fund or to a pension fund.
(b) In the case of members who joined before 1 July 1960, if the fund has not changed its rules since
then, payments to those members are entirely exempt. If the fund has changed its rules since that date
members are exempt on the greater of:
(i) $4 000; or
(ii) the amount they would have received had the rules not been changed;
plus any portion of the payment as they use for the purchase of an “annuity on
retirement” or for transfer to another fund.
With effect from 1 January, 2001, the exemption was increased to $ 8400.
16.7.4 Lump sum payments from unapproved funds (para 10 of Sch 1)
A refund of the employee’s contributions is exempt but any excess is taxable.
16.7.5 Recovery by an employer (s 8(1)(j), proviso (ii))
Where an employer recovers or recoups an amount (i) on the winding up of a benefit or pension fund, or
(ii) on his ceasing to be an employer because of insolvency or liquidation, or (iii) on the withdrawal of all
his employees from membership of the fund, such amount constitutes “gross income” in his hands and is
taxable.
Chapter 17
Tax Avoidance
Synopsis

136
17.1 Introduction
17.2 Section 98
17.3 Other provisions
17.4 “Tax avoidance” test unnecessary
17.1 Introduction
Tax avoidance, subject to what is said below, may be broadly described as the legitimate ordering of one’s
affairs in such a way as to minimise the tax which is chargeable. The distinction between this and evasion
must at the outset be clearly understood. Evasion is, for example, the deliberate submission of false returns,
the omission of sales from the trading account, the inflation of claims for deduction of expenditure, etc.. All
of these latter may attract penalties upon assessment and are offences which are open to prosecution.
The ordering of one’s affairs, referred to above, is permissible unless it falls within the terms of the specific
anti-avoidance sections in the Act. It is stressed, however, that this principle has still to be considered by
the Zimbabwe courts. The approach of the courts in some countries whose legislation does not specifically
deal with the matter has been, in general terms, to nullify taxpayers’ arrangements which include steps
taken for no commercial purpose, i.e. taken purely for tax reasons. In the United Kingdom, such wide
approach has been adopted in leading cases, notably WT Ramsay v IRC (1981) 1 All ER 865 (HL) and
Furniss v Dawson (1984) 1 All ER 530 (HL). In South African courts, in the context of a specific
provision, there has been a mixed reception of the Ramsay and Furniss v Dawson doctrine; in ITC 1606
(1996) 58 SATC 328 the court at 332 and 338-9, favoured the principle but in ITC 1611 (1997) 59 SATC
126 at 142 it did not. Even if the Zimbabwe courts accept that such broad approach is inappropriate, it is
likely that schemes incorporating such steps will similarly fall foul of the Zimbabwe provisions. The effect
of each of those provisions is considered below.
It must be borne in mind also that the courts may, as will be seen, set aside a scheme, as constituting a
sham, without deeming it necessary to rely on specific provisions. (Reverting briefly to the term “tax
avoidance” itself, this has in common parlance come to be used as signifying schemes which have fallen
foul of the anti-avoidance provisions or are considered likely to do so.)
17.2 Section 98
17.2.1 General
This is the principal section dealing with tax avoidance. Its targets are transactions, operations or schemes
which have the effect of avoiding, reducing or postponing payment of tax. The term “avoidance” is used in
all of these senses below.
“Tax” is defined in section 2(1) of the Income Tax Act as any tax leviable in terms of the Act. Schemes for
the avoidance of, for example, any of the withholding taxes are therefore just as open to attack by the
Commissioner as are schemes for the avoidance of income tax.
For the section to be invoked, however, such avoidance must have been, in the Commissioner’s opinion,
the sole or one of the main purposes of the scheme. If these requirements are fulfilled, the Commissioner
effectively has a further hurdle, i.e. that the scheme must be tainted by “abnormality”. Here he is given two
alternative lines of attack and, if he succeeds on either, he may invoke the section. They are, having regard
to the circumstances under which the scheme was entered into or carried out :
(i) that it was entered into or carried out by means or in a manner which would not normally
be employed for such a scheme; or
(ii) that it has created rights or obligations which would not normally be created between
persons dealing at arm’s length under such a scheme.
Section 98 may therefore be viewed as a combination of three tests, the first two being the avoidance effect
and purpose respectively and the third, abnormality comprising two alternative halves as above.
Its interpretation, and that of the equivalent South African section 103(1) (or their predecessors, all such
provisions being referred to in this paragraph 17.2 as “the section”) has come before the courts of both
countries on a number of occasions. The tax authorities, naturally, invoke the section only where a tax
avoidance effect has been perceived. In some instances the courts then find that avoidance has not even
occurred, though most cases turn on the questions of purpose or abnormality (or both). Some of these are
referred to below, although it is stressed that what follows is not an exhaustive review but a selection of
what are considered to be illustrative precedents.
The Commissioner’s remedies are also important. He is empowered, as he thinks fit, to assess either as if
the scheme had not taken place or in such a manner as he considers appropriate in order to prevent or
diminish the avoidance.
17.2.2 Basics

137
The first case selected is ITC 963 (1961) 24 SATC 705. It illustrates the kind of basic transaction where the
Commissioner failed in an attempt to invoke the section.
The taxpayer had shifted funds from a taxable interest-yielding investment to one which yielded the same
rate of interest but which was exempt. The court held that the transaction was not caught by the tax
avoidance provisions, despite the exempt character of the new investment having instigated the taxpayer’s
action.
The judgment applied the principles laid down in CIR v King (1947) 14 SATC 184. The Commissioner had
failed, in that case also, despite his path having been potentially easier by reason of the absence from the
section, at that time, of any abnormality requirement.
17.2.3 Purpose
The case of SIR v Gallagher (1978) 40 SATC 39 is illustrative of the “purpose” test referred to above. The
taxpayer in that case conceded both that his scheme had the effect of avoiding tax and that it was caught by
the abnormality test. He succeeded in his appeal, however, by convincing the court that the purpose of his
scheme was to save estate duty and not to avoid income tax. (The South African Act was subsequently
changed to embrace such schemes also. The Zimbabwe law remains silent in this respect.) The court
approved also the adoption of a subjective rather than an objective test as to purpose; the question was what
the taxpayer had in mind.
It must be borne in mind that the Zimbabwe Commissioner’s powers are somewhat wider in that he may
invoke the section where avoidance is, as stated above, only one of the main purposes.
In R Ltd and K Ltd v COT (1983) 45 SATC 148, however, to which further reference is made below, the
court, citing United Kingdom and Australian precedents, observed at 165 :
“... when a genuine commercial transaction is considered and there are two ways of carrying it out, one that
involves paying more tax than the other, it is quite wrong to draw the inference, as a necessary
consequence, that in adopting the course which involves paying less tax, one of the main objects is to avoid
tax.”
17.2.4 Abnormality
In Hicklin v SIR, (1980) 41 SATC 179, the taxpayer succeeded in relation to the abnormality test despite
failing as regards tax avoidance effect and purpose.
The taxpayer was a shareholder in a company which had disposed of its business and become dormant. The
company had substantial distributable reserves, having never declared a dividend, and even larger non-
distributable reserves from the sale of the business. Its funds had over a period been lent to the taxpayer and
his fellow-shareholders on unsecured, interest-free loans. No further action was contemplated.
The shareholders then received an unexpected offer for their shares at net asset value less 10% of the
distributable reserves. This was accepted, the loans being repaid by an exchange of cheques.
The Commissioner attempted to tax the taxpayer on the basis that his share of the distributable reserves
could be regarded as a dividend (which would have been taxable under the law as it stood), these profits
having been taken out by way of loans and the loans liquidated by the proceeds from the sale of shares.
The court rejected this. It considered that the “scheme” which had to be examined was the agreement under
which the shares were alienated and not the conduct of the shareholders ever since the sale by the company
of its business. It then examined the agreement of sale of the shares and found nothing abnormal about
either the means or manner in which it had been entered into or the rights and obligations which it had
created.
Certain factors should be taken into account in evaluating this decision. Firstly the sale agreement was with
an independent third party in any event. Admittedly this is not conclusive, but a transaction with an
associated third party would be more suspect. Secondly, the courts frequently adopt a wider approach to
where a “scheme” begins and ends.
17.2.5 Purpose and abnormality
Other cases have dealt at length with both of these factors. Three such cases are considered here.
(a) SIR v Geustyn, Forsyth and Joubert (1971) 33 SATC 113
A company with unlimited liability had been formed to take over the business of consulting engineers
formerly carried on in partnership by its shareholders/directors. The company undertook to employ the
three former partners at a salary and to pay the partnership an amount for goodwill equal to three years’
partnership profits. There were no service contracts and there was no guarantee for payment of the
goodwill. The latter was credited to interest-earning loan accounts of the former partners.
The Commissioner failed in his attempt to invoke the section (by treating the company’s income as if it
were that of the three former partners).

138
Although this is a leading case it is not considered further here since substantially the same facts
were present subsequently in Louw’s case which, in view of its demonstrative value, is now examined in
some detail.
(b) CIR v Louw (1983) 45 SATC 113
The background
This case, as was that of Guestyn, was concerned with the incorporation, into an unlimited company, of a
consulting engineers’ practice formerly carried on in partnership.
The consideration was equal to net asset value reflected by audited accounts, plus goodwill valued on the
basis of 18 months’ profits. This consideration was satisfied partly by the allotment of shares and partly by
crediting interest-free loan accounts.
Although incorporation took place in 1966 it was not until some years later that the tax avoidance
provisions were invoked, i.e. in respect of the 1974 to 1976 tax years. In the meantime the volume of work
had increased dramatically. Meanwhile also the directors’ loan accounts had moved very substantially into
debit, a factor which assumed an importance of its own and which is dealt with later.
The Commissioner’s method
The Commissioner chose to remedy the situation by calculating additional tax payable by Louw for each of
the three years as follows: (i) to apportion the net profits of the company to him in proportion to his
shareholding; (ii) to deduct, from the amount so apportioned, the dividend received by him from the
company; and (iii) to reduce his resultant additional tax by an amount representing a proportionate share of
the tax payable by the company. The objective appears to have been to revise the taxpayer’s liability to an
amount calculated as if the company did not exist and the taxpayer had remained in partnership.
The court’s approach
Having rejected a contention by the Commissioner that the subsequent lending of money to the directors
was an integral part of the original scheme to incorporate the practice, the court proceeded to examine the
incorporation of the practice as a separate scheme in itself and to consider whether the Commissioner was
justified in invoking the section.
The “purpose” test
In examining the incorporation of the practice in the context of whether avoidance was the sole or main
purpose of the scheme the court had regard to the business and professional advantages which were cited in
evidence (along somewhat similar lines to those in Guestyn’s case) and came to the conclusion that,
although tax was a factor, it was not a major factor in influencing incorporation. The Commissioner thus
failed on the purpose aspect.
The “abnormality” test
On the two alternative abnormality tests the Commissioner conceded that abnormality did not exist in the
means or manner employed. He thus recognised:
(i) the normality of transferring a professional practice into a company with unlimited
liability, the widespread incidence of such transactions, and some of the attendant advantages;
(ii) that the goodwill price was reasonable; and
(iii) the preponderance of incorporation of consulting engineers’ practices.
He attacked, however, the rights and obligations created under the scheme, specifically:
(a) the sales of the assets of the partnership, including its goodwill, to the company on credit
without requiring the payment of interest;
(b) the provision in the vendors’ agreement that the balance of the purchase price would be
payable as the financial circumstances of the purchaser permitted;
(c) the lending of large sums of money to the shareholders free of interest and without any
definite conditions for repayment; and
(d) the conclusion of service contracts between the company and its shareholders in terms of
which (i) no set remuneration was stipulated and (ii) Louw himself received a salary which was much
smaller than the income which had accrued to him for performing the same services during the existence of
the partnership.
Abnormality: arm’s length
Item (c) above related solely to the directors’ loans and, as said earlier, was considered separately by the
court, as indicated below.
The remaining features raised the same problem as had arisen, but not been resolved, in Guestyn’s case, as
to how in such cases one should apply the criterion as to whether the scheme “has created rights or
obligations which would not normally be created between persons dealing at arm’s length under a ...

139
scheme of the nature of the ... scheme in question”. The problem was that the court was “concerned with
partners who have ... made over their practice, not to an independent third party with whom they would
ordinarily deal at arm’s length, but to an unlimited company of which they are the sole shareholders and
directors and whereof they have full and complete control” (137). The question for the court was whether it
should, in applying the “normality” yardstick, take account of the special relationship between the erstwhile
partners and the company which they had formed, or ignore it and apply the yardstick as though the
company were a stranger.
The court’s decision
The Appellate Division, in a unanimous judgment, did not see how the court could ignore this special
relationship. It was of the very nature of the incorporation scheme that the company to which the practice
was sold by the partners would have as its shareholders and directors those self-same partners, and would
be controlled by them. These were the realities of the situation.
Thus, as to the arrangement that the payment of the purchase price was to be made only as and when the
company was in a financial position to do so, there was little else that the parties could have done. Initially
the company had very limited capital and the idea was that it would pay off the purchase price out of
profits, which it proceeded to do. Since the sellers were the persons mainly instrumental in earning those
profits and were in complete control over the company, it was a perfectly sound and business-like
arrangement. “It was not an arrangement which would not normally have been created by persons dealing
at arm’s length in this type of transaction” (138).
The same went for non-payment of interest on the purchase price. This increased the profits to the
company, which in turn directly benefited the erstwhile partners as shareholders, for it enabled the
company to pay off the purchase price more rapidly.
Similarly there was no abnormality about the non-stipulation of salaries. As controllers of the company the
erstwhile partners were able from year to year to determine in their own interests what their salaries were to
be.
The loans to the directors
Finally the court considered as a separate issue whether the loans to the directors were caught by the
section and concluded that they were.
The basic facts were that, after the loans went into debit in 1972, the amounts advanced to the directors, out
of the company’s distributable reserves, were very substantial. Further, post-incorporation salary and
dividends in the case of Louw himself were substantially less than the income which had accrued to him
from the partnership practice.
Further facts which were established or could be inferred were:
(i) that each director received by way of salary or dividend sufficient to meet his normal
living costs and that the portion of his income which (under, for example, a partnership system) he would
normally have saved and invested, was advanced to him on loan by the company;
(ii) the bulk of the company’s surplus funds as was not required for its business purposes was
loaned to the directors;
(iii) the directors were never called upon to repay any portion of the loans; and
(iv) the loans were not granted in terms of specific board resolutions. Periodic drawings were
made, part of which were subsequently cleared by credits to loan account in respect of salary or dividends
due.
Against this background the court considered that, had the loans not been made, the directors would,
instead, have received substantially equivalent amounts by way of salary or dividend.
It then found that the inference that avoidance had been the sole or main purpose of the granting of the
loans was inescapable.
As regards whether the transactions created rights or obligations which would not normally be created
between persons dealing at arm’s length under a transaction of the nature of the transaction in question,
whereas the lower court had held that they did not, the Appellate Division held that they did. Regard was
had to the factors above, namely: the size of the loans in comparison with the size of the salary and
dividends; that the salary and dividends met living costs while the loan was effectively in lieu of surplus
income; and the draw-down method of making the loans.
In these circumstances the court concluded that the loans were abnormal both as to the means and manner
employed in granting them and as to the rights and obligations created thereby.The outcome

140
The result was thus that, although the taxpayer succeeded in so far as the court still found in his favour as
regards the scheme of incorporation, he lost in so far as the section was found to be applicable to the loans.
(The case was remitted to the Commissioner for him to re-assess on this basis.)
(c) ITC 1496 (1991) 53 SATC 229
The taxpayer had subscribed a sum by way of a five-year promissory note to a venture, which comprised
what were styled as a series of partnerships, in a long-term plantation-farming scheme. The amount
subscribed comprised two elements, being a contribution towards working capital to meet management
fees, plus interest for the five-year period until the note fell due. The entire amount, which was substantial,
was claimed as a deduction in the year of assessment in which the note was issued. Had the claim
succeeded, the set-off against the taxpayer’s income from other activities would have resulted in no tax
being payable.
The court found that the scheme was caught by the section.
With regard to the purpose test, regard was had to the haste with which the scheme was concluded in the
final days of the tax year of its inception, and the thirteen-year deferment of liability which was sought.
As regards abnormality the court examined the arrangements which purported to create partnerships
between the taxpayer and 19 other persons and, by extension, de facto between the taxpayer and 71 others.
Regard was had to the fact that the taxpayer knew only a few of the other parties, the lack of likelihood of
strangers entering into joint and several liability, and the failure of most parties to examine the geographical
area where operations were to be conducted. The court concluded that what had been created was an
artificial structure designed to camouflage, as partners, persons who really were investors. Numerous other
features were cited as being abnormal to a partnership relationship.
(The court considered the Commissioner to have been justified in invoking the section not only in relation
to the claim for a deduction of management fees, which rested on the existence of a partnership, but also in
relation to the claim for a deduction of interest. It had found as a separate issue that such interest had, in
any event, not been “incurred” in terms of the general deduction formula: the discussion at 7.3(b) refers).
17.2.6 Further decisions
(a) Other important cases are as follows.
COT v Ferera (1976) 38 SATC 66
The taxpayer had formed a company to take over his existing shareholdings, the sole purpose of the
formation being, it was accepted, the avoidance of death duties.
Subsequently, the taxpayer, acting in terms of other provisions of the Act, fixed the respective dividend
distributions at precisely the proportions needed to avoid undistributed profits tax (then in force on
companies) and to minimise the then supertax in his own hands. He was held to be caught by the section.
The decision has been criticised in various quarters (including the judgment in R Ltd and K Ltd v COT
(1983) 45 SATC 148: see 17.3.3) as going too far. The case serves, however, as a reminder that the courts
may have regard not only to the circumstances surrounding the inception of a scheme but also to its
operation.
ITC 1173 (1972) 34 SATC 113
The taxpayer had previously been a bookmaker operating through a company in which he was the sole
shareholder. When losses were incurred he ceased operations and sought employment with an estate agent
“S”. On the advice of the taxpayer’s accountant, however, in order to take advantage of the assessed loss,
his company was instead appointed as a subagent. The taxpayer personally became a director of the S
agency but received no remuneration.
When the S agency arrangement came to an end the taxpayer entered into an agreement with another estate
agent, “G”, under which his company was employed on a salary basis, plus commission on sales and on
management fees earned. It was understood tacitly that, although the company was the employee, the
taxpayer was to render the required service.
The Commissioner having attacked both of these arrangements the court found that in both instances the
main purpose of the scheme had been the avoidance of tax. With regard to abnormality, however, the court
found in the taxpayer’s favour in relation to the S arrangement but against him in relation to the G
arrangement.
In reaching these conclusions regard was had to various factors, including the following. In relation to the S
arrangement: (i) there was nothing abnormal in the taxpayer causing his company to be a subagent of S:
and (ii) the scheme could not be said to have created rights and obligations, either between his company
and S or between his company and him, which would not normally be created between persons at arm’s
length. The G arrangements had to be considered in a different light. They did not create a sub-agency but a

141
relationship which purported to be that of master and servant; a salary was stipulated and continuous
service was expected from the company. A master/servant relationship implies the rendering of personal
services by the servant, which the company obviously was unable to do. In all the circumstances the court
did not consider the relationship established as normal. The income received by the company from G had
therefore been correctly taxed by the Commissioner in the taxpayer’s hands.
It is submitted that the decision underlines the need for care when an individual seeks to interpose a
company between himself and his employer.
Interestingly, although in ITC 1173 the Commissioner contended, as an alternative to his attack under the
section, that the contracts with S and G were “simulated”, the court did not accept this.
ITC 1513 (1992) 54 SATC 56
The taxpayer, who was employed by his father as farm manager, claimed capital allowances on equipment
leased to his father for use on the farm. He established that the transaction fell outside both the “purpose”
and “abnormality’ requirements.
ITC 1518 (1992) 54 SATC 113
Two brothers were the sole directors of two companies and were trustees of two trusts which received
managerial remuneration from the companies. It was found that there was abnormality in payment to the
trusts (and that liability fell instead on the brothers). See also Meyerowitz v CIR (1963) 25 SATC 287.
ITC 1625 (1997) 59 SATC 383
The taxpayer company had been formed by a family as part of a scheme to unlock funds to enable certain
of their members to emigrate. They had been members of a property-owning close corporation. The
property was sold at market value to the company, the proceeds enabling the corporation to pay a dividend
which achieved the family’s objective. The point in issue was the disallowance by the Commissioner of
interest payable by the company on borrowings used to purchase the property.
The court found (i.e. notwithstanding the motives, which were themselves “tax-innocent”, and which had
given rise to the taxpayer’s acquisition of the property and the related incurring of the interest in question):
that the parties’ intentions regarding the transfer of the property were honest and that the agreement of sale
was a genuine reflection of their intention and beliefs; that no tax had been avoided by the company
through the payment of interest in order to earn income which was taxable; that avoidance was not the sole
or one of the main purposes of the scheme; and that the Commissioner was therefore not correct in having
invoked the section.
CIR v Conhage (Pty) Ltd (1999) 61 SATC 391
A manufacturing company had entered into agreements, with a bank, comprising the sale and leaseback of
some of its manufacturing plant. The court recognised the validity of the agreements, and thus approved the
deductibility of rentals payable by the company, on the basis that, notwithstanding that the company had
been aware of the potential tax benefits, the reason for the transaction had been the need to raise finance.
ITC 1669 (1999) 61 SATC 479
See 2.4(b); an inter-company scheme incorporating the purported granting of rebates was held, in the
alternative, to be caught by the section.
(b) Further points arising out of court decisions are :
(i) that the section is not a penalty section (King’s case and Ferera’s case);
(ii) that, while the powers which are accorded to the Commissioner are wide, they must not
be exercised in such a manner as to subject any income to double income tax (H v COT (1972) 34 SATC
39). (The term “double taxation” has been held, in another context, to mean the taxation of the same
income more than once in the hands of the same person: ITC 1364 (1983) 45 SATC 23); and
(iii) that where a taxpayer leaves his financial affairs in the hands of, for example, his
accountants, under a general mandate to act in his interests, his subsequent conduct may amount to
ratification and their intentions thus may be imputed to him (L v COT (1975) 37 SATC 116);
(iv) that an interest-free loan to a shareholder may be perfectly normal (unreported case
referred to at 5.6.4);
(v) that the Commissioner, in applying the section, and where non-disclosure is established,
is (see ITC 1582 (1995) 57 SATC 27) not subject to the then three-year, now six-year, restriction,
considered at 21.2.6 (a). (There appears to be a conflict here with the rule, referred to above, that the
section is not a penalty section.)
17.3 Other provisions
17.3.1 Fair market price (s 23(1))

142
This section, while not specifically termed as such, would serve to counter tax avoidance. It provides that,
where a person carrying on a trade in Zimbabwe either (a) purchases property (movable or immovable) at a
price in excess of the fair market price or (b) sells at less than the fair market price, the Commissioner may
determine a fair market price for the purposes of that person’s assessment.
The Commissioner’s power in this respect is, however, not entirely unfettered.
In Elite Wholesale (Rhodesia) (Pvt) Ltd v COT (1955) 20 SATC 33 the facts were that three brothers, who
controlled the company, used it as an organisation for buying goods at wholesale prices for supply to
various retail businesses carried on by themselves and members of their family. The company’s low mark-
up was attacked by the Commissioner who increased it to 15% on cost. The court found in favour of the
taxpayer on the following basis:
“If the transaction is a perfectly innocent one, if it is not fictitious or colourable, the mere fact that a
reduction in income has resulted is not a sufficient justification for the exercise of the power. An occasion
for its exercise arises when there is something about the transaction which indicates an intention . . . to
evade assessment or tax, something which shows a lack of good faith or the presence of (quoting from
precedents) ‘moral dishonesty in the taxpayer’s mind’”.
See also ITC 1554 (1993) 55 SATC 115, considered at 22.9.
In any instance where the Commissioner succeeds in invoking the section the effect is double taxation in
the hands of the respective parties. A seller selling at too low a price will have the higher fair price imposed
upon him but the purchaser’s cost will remain at the price which he has paid. Conversely a purchaser who
overpays will be allowed to deduct only the lower fair price but the seller will remain liable on the actual
proceeds.
Section 23(1) does not apply to transactions other than sales. An excessively high or low rate of interest, for
example, or a charge for management services, or a rate of royalty, etc., cannot be upset under this section.
They may well, however, be caught under section 98, considered above. In the case of expenditure there is
always, of course, the Commissioner’s general power to disallow excessive amounts under the “purposes of
trade” test considered in chapter 7.
17.3.2 Fair market price (cross-border transactions) (s 24)
This section, as in the case of section 23(1), also is essentially aimed at countering tax avoidance, despite
not being specifically so termed.
It deals with the above transactions (of all kinds, not merely sales) where there is a foreign element of
management, control or capital, and the Commissioner considers that the conditions between the connected
parties differ from those which would arise between parties at arm’s length. He is then empowered to
determine, on an arm’s length basis, the taxable income of any party carrying on business in Zimbabwe.
Transactions containing an element of what is commonly known as “transfer pricing” are those most likely
to be attacked under this provision, though they might similarly be vulnerable to section 98.
17.3.3 Disposals of trading stock (Sch 2, provisos to paras 4 and 12)
The provisions deal with the situation where a taxpayer has disposed of non-farm trading stock and farm
trading stock respectively, in various ways, such as donation, and the Commissioner is of the opinion that
tax avoidance was the sole or a main purpose of the disposal. He is empowered to determine the amount
which the stock would have realised had it been sold in the ordinary course of trade and to include such
amount in the taxpayer’s gross income.
Unlike section 98 there is no “abnormality” requirement. Fewer conditions have therefore to be satisfied for
the Commissioner to launch an attack.
The scope of the proviso to paragraph 12 was considered in R Ltd and K Ltd v COT (1983) 45 SATC 148.
The facts were briefly that K Ltd, which had been conducting farming operations with a large number of
livestock, leased the herd for 15 years to R Ltd, which was an associated company and which had an
assessed loss of $1,2 million. The agreement was one of mutuum, or “sheep-lease” under which R Ltd
undertook at the end of the lease to return not the same animals but an equivalent number of the same
breed, type and standard.
The Commissioner attacked the arrangement on the grounds that K Ltd had effected a disposal as
contemplated by the proviso and that this had been done with tax avoidance as its sole or main object.
Among other actions he included nearly $1,5 million, being the market value of the livestock, in the taxable
income of K Ltd.
The court found that the contract did not constitute a disposal, thus dismissing the Commissioner’s attempt
to invoke the proviso.

143
In case the matter was taken further, however (which it was not) the court examined the avoidance
requirement. It was found, on the basis of testimony by a witness who was an agricultural expert, that there
were genuine commercial reasons for the transaction, independently of the tax considerations. The court
concluded with regard to the contract that: “While the form which it eventually took obviously had regard
to the preservation and use of R’s assessed loss, that was not one of the main objects of the scheme ...”. It is
a matter of particular interest that the judgment continued:
“... and even if it was I do not think ... that to use an assessed loss which a taxpayer has already incurred
amounts to tax avoidance or reduction in terms of this paragraph of the Second Schedule”. Among the
reasons given by the court for this view was that “... the Commissioner’s conclusion, if here upheld, would
mean that R would have to go on trading in a way that would mean it could never avail itself of its assessed
loss ... For it could only improve its profitability by improving its cattle selling potential .... The accruing of
an assessed loss entitles a taxpayer to relief from tax while that loss is recouped.”
Since the case had already been decided on the first ground the above comments were obiter and were
made in the specific context of the proviso in issue. It is submitted, nevertheless, that they may well be of
assistance to taxpayers in a wider sphere.
17.3.4 Other more limited tax avoidance provisions (s 10(4),(5) and (6))
Reference has been made elsewhere: see 2.7.4 to 2.7.6.
17.4 “Tax avoidance” test unnecessary
Although this chapter has otherwise been concerned with the tax avoidance provisions of the Income Tax
Act it is convenient to refer to cases where, at one end of the scale, the court in finding against the taxpayer
did not even find it necessary to have recourse to such provisions and, at the other, the facts may be
perceived as coming close to avoidance though the provisions were not cited.
Erf 3183/1 Ladysmith and Another v CIR (1996) 58 SATC 229
It is suggested that the court adopted what may be styled as a “hidden agenda” approach.
The taxpayers concerned had contractually leased stands to a (tax-exempt) pension fund which in turn had
subleased the stands to a manufacturing company. Under the sublease the fund was to receive a premium
from the company and was obliged to erect a factory for occupation by the latter.
The dispute centred on whether, although the taxpayers’ lease agreement with the fund imposed no
obligation on the fund to effect improvements, the taxpayers nevertheless had the right to enforce such
obligation against either the fund or the company and whether they were thus taxable under the equivalent
“obligatory improvements” provisions of the legislation (see 5.5.)
The court held: that between themselves the parties did not intend that the agreements should have the
effect which was expressed; that there was an intention which had not been divulged; that it was likely that
the main lease was simulated; and that the taxpayers had failed to show that they had no right to have the
improvements effected.
The Commissioner’s assessments, including penalties, were thus upheld.
The court applied similar principles in Relier (Pty) Ltd v CIR (1998) 60 SATC 1.
The taxpayer company, which was wholly owned by a (tax-exempt) provident fund, purchased a property,
which it leased to the fund. The fund subleased the property to a third party, the sublease, which ran for a
period contemporaneous with the lease, containing a clause which was interpreted as obliging the third
party to effect improvements to the property. The court found the taxpayer to be liable on the amount of the
obligatory improvements on the grounds that it had the right to compel the fund to procure their
construction.
ITC 1618 (1997) 59 SATC 290
An individual in South Africa, having previously performed professional services for an enterprise in his
individual capacity as a draughtsman, interposed a close corporation to render such services, with the
individual continuing physically to carry out the work. The court upheld the taxpayer’s contention that the
corporation and not the individual was taxable on the income from the enterprise. Although the agreement
in question contained features indicating that the enterprise’s relationship was with the individual, such as
stipulated hours of work, and fee rate per hour, the court was satisfied that the parties intended such
agreement to have the effect of creating a contractual relationship with the corporation.
ITC 1670 (2000) 62 SATC 34
The taxpayer was a close corporation whose only member rendered professional services in its name to an
engineering company. The contract between the corporation and the company related to various
assignments as they materialised which were administered on an ad hoc basis at a fixed tariff. The work
was to be undertaken by the corporation’s employees, with the company having the right to refuse, for good

144
reason, to accept a specific employee. Apart from the engineer the corporation had only administrative
staff.
The court found, again in favour of the taxpayer, that the services to the company had been rendered by the
corporation and not by the engineer as an employee of the company.
Chapter 18
Sundry Technical Matters
Synopsis
A.  ACCOUNTING YEARS
18.1 Introduction
18.2 Individuals
18.3 Companies
B.  ORDINARILY RESIDENT
18.4 Introduction
18.5 Individuals
18.6 Companies
18.7 Estates and trusts
C.  CARRYING ON BUSINESS
18.8 Introduction
18.9 Meaning
D.  INSURANCE PREMIUMS AND PROCEEDS
18.10 General
A. ACCOUNTING YEARS
18.1 Introduction (s 37(13))
The tax year in Zimbabwe formerly ended on 31 March. In 1997 this was changed to 31 December
(resulting in a nine-month “year” at that point from 1 April to 31 December 1997). Most taxpayers
conducting business or other operations prepare their annual accounts to coincide with the tax year, i.e. now
ending 31 December. The Commissioner has, however, a discretion to accept accounts drawn up by a
taxpayer to a date other than 31 December, an action commonly referred to as the adoption of an
accounting year.
The accounts are submitted in fact to the taxpayer’s Assistant Commissioner of Taxes. Once an accounting
year has been accepted it is usual for this to remain undisturbed. (When such acceptance was
retrospectively set aside by the Commissioner his action was upheld in ITC 1441 (1989) 51 SATC 10 but
there were other features in that case and it is not anticipated that such action will prove to be common.)
18.2 Individuals
18.2.1  In the case of an individual there is a requirement that the Commissioner be satisfied that the
taxpayer usually draws up his accounts to such other date. A tobacco farmer might adopt a year ending say
30 September to coincide with the crop year and submit accounts for, for example, the year ended 30
September 1999, in support of his return for the year ended 31 December 1999, i.e. on a “prior” basis.
Adoption of an accounting year quite commonly occurs in the case also of, for example, traders and
manufacturers. The adoption of a “prior” accounting year is far more common and generally advantageous
compared with that of a “subsequent” year, the latter being, for example, the submission of accounts for the
year ended 31 January 2000, in support of the return for the year ended 31 December 1999. The
Commissioner is in any event reluctant to accept “subsequent” accounting years since they may delay
submission of returns. He is reluctant also to accept “prior” accounting years ending earlier than
September.
18.2.2  All cases are subject to the imposition of terms by the Commissioner. The commonest instance of
such terms is the adoption of a procedure known as “doubling up”, as follows.
Example
Mr A, having previously been in employment, commenced business as a general trader on 1 October 1998,
and proposed to submit accounts for the year ended 30 September 1999, in support of his return for the year
ended 31 December 1999.
The Commissioner’s steps would usually be:
(i) to include in the assessment for the year ended 31 December 1998 (in addition clearly to,
and separate from, Mr A’s employment income for the nine months from January to September 1998) three

145
months trading profit for the period 1 October to 31 December 1998. This might have to be estimated in the
first instance pending the production of accounts from which it could be apportioned (on either a time or
turnover basis);
(ii) to base the assessment for the year ended 31 December 1999 on the accounts for the year
ended 30 September 1999, such action constituting the “acceptance” of the accounts on the “prior year
basis”.
The outcome would be that the profits for the period of three months from October to December 1998
would have been taxed twice but that Mr A, having traded for 15 months by 31 December 1999, would
similarly have been assessed on 15 months’ profits.
Thereafter accounts for the year ended 30 September 2000, would support the return for the year ended 31
December 2000, and would form the basis of the assessment for that year, and so on. Any change of
accounting year would be subject to the Commissioner’s agreement and conditions.
In due course on the cessation of trading by, or in the event of the death of, Mr A, the Commissioner
would, as a matter of practice, “undouble” or “gross down”. If, for example, trading ceased on 31 July
2002, only the profit for the seven months from January 2002 would be assessed for the year ending 31
December 2002, thus “undoubling” the three months from October to December 2001, the exclusion of
which would in principle balance the initial period of doubling up.
18.2.3  Capital allowances are calculated on a 31 December basis, but in relation to the assets reflected in
the accounts for the accounting year.
Example, reverting to Mr A
Taxable income established from the accounts, prior to capital allowances:

Year ended 30 September 1999 300 000


Year ended 30 October 2000 400 000
Furniture and fittings purchased and used:
Period 1 October to 31 December 1998 48 000
Period 1 January to 30 September 1999 24 000
Year ended 30 September 2000 80 000
Calculation of taxable income for the years ended 31 December 1998 to 2000
1998
Year ended 30 September 1999 : say 3/12= 75 000
Less SIA on $48 000 (25%) (12 000)
$63 000

1999
Year ended 30 September 1999 300 000
Less: W&T on $48 000 (25%) (12 000)
SIA on $24 000 (25%) (6 000)
$282 000

2000
Year ended 30 September 2000 400 000
Less: SIA on $80 000 (25%) (20 000)
W&T on $48 000 + $24 000 (25%) (18 000)
$362 000

18.2.4  If a trader incurred assessed losses in the initial trading years the Commissioner would merely
apportion these on a December basis and would delay the doubling up process, and acceptance of the
accounting year, until taxable income from trade or investment for a year (ignoring the loss brought
forward) arose.
18.2.5  As was pointed out in ITC 1441 (1989) 51 SATC 10, the doubling up procedures are merely a
matter of practice applied by the Commissioner. They are not statutory, nor are they published in a form

146
which is available to the public. It was also held in ITC 1352 (1982) 44 SATC 64 that there was no
obligation on the Commissioner to “undouble” a trader’s taxable income for the final period even where
doubling up had taken place initially. It is understood, however, that the general approach described earlier
will continue to be applied.
18.2.6  As regards an individual who is a member of a partnership see Chapter 14.
18.3 Companies
(a) Acceptance of an accounting year other than 31 December is again at the Commissioner’s
discretion, but no “doubling up” procedures are applied. A change of accounting year is similarly subject
to his agreement and conditions. On cessation of trading no undoubling arises, and income for the final
period is simply reflected in the final return.
The comments at 18.2.1 regarding “prior” and “subsequent” accounting years again apply.
(b) Capital allowances for the tax year ended 31 December are calculated on the assets reflected in the
supporting accounts for the accounting year.
Example
A manufacturing company commenced operations in October 1998 and submits accounts for the year
ended 30 September 1999 in support of its return for the year ended 31 December 1999. Assets reflected in
the accounts and brought into use during the year ended 30 September 1999 included the following:

$
Industrial building constructed 10 000 000
Truck purchased November 1998 600 000
Sedan No. 1 purchased November 1998 300 000
Sedan No. 2 purchased May 1999 500 000

Maximum allowances for the year ended 30 September 1999, for the purposes of calculating the taxable
income for the year ended 31 December 1999, would be as follows.

Industrial Sedan Sedan Total


building Truck No 1 No 2 Allowances
Cost 10 000 000 600 000 75 000 200 000
SIA 2 500 000 150 000 18 750 50 000 $2 718
750
ITV
30.9.99 $7 500 000 $450 000 $56 250 $150 000

B. ORDINARILY RESIDENT
18.4 Introduction
Although the Zimbabwe tax system rests so largely on the concept of source, that of ordinary residence,
which is not defined in the Act, is nevertheless of importance in relation to, in particular:
(i) income which is taxable only if the recipient is ordinarily resident in Zimbabwe (such as
certain remuneration and foreign interest and dividends): see, for example, 3.2.4 and 3.2.7;
(ii) where the Zimbabwean payer is required to withhold tax from income accruing to a
recipient who is not ordinarily resident in this country: see chapter 19; and
(iii) the operation of double taxation agreements: see chapter 20.
18.5 Individuals
In most instances it will be obvious whether an individual is ordinarily resident in Zimbabwe, such as when
his only residential property and his place of business or employment are here.
On the basis of United Kingdom decisions an individual could be ordinarily resident in a country in other
circumstances. While the current approach in Zimbabwe remains to be established, that of the South
African courts emerged in CIR v Kuttel (1992) 54 SATC 298. The taxpayer had emigrated to the United
States but frequently visited South Africa where a company house was always available to him and where
he (compulsorily) retained substantial investments. He was held no longer to be ordinarily resident in South
Africa. It was not considered helpful to discuss the United Kingdom precedents, including the concept of
dual ordinary residence.
18.6 Companies

147
A company is resident where its “central management and control actually abides”: De Beers Consolidated
Mines Ltd v Howe (Surveyor of Taxes) (1906) 5 TC 198. That term has been held to mean the country
“where the controlling power and authority which ... is vested in its board of directors, and the exercise of
that power and authority, is to some substantial degree to be found”: Union Corporation Ltd and Others v
CIR (UK) (1953) 34 TC 207. It is submitted accordingly that a company is resident in the country where its
head office is situated since that is where the central management and control is usually centred. Further it
appears that, in the case of a company, no distinction can arise between its residence and its ordinary
residence and, strictly, that a company can only be “resident”.
18.7 Estates and trusts
See chapter 15.
C. CARRYING ON BUSINESS
18.8 Introduction
The relevance of the term in the Zimbabwe tax context is restricted to:
(i) the operation of double taxation agreements: see chapter 20;
(ii) the withholding of non-residents’ tax on interest: see 19.4 ; and
(iii) whether a non-resident should obtain the benefit of restricted categories of exemption:
see 6.3(xvi).
18.9 Meaning
The matter has been considered by the courts on a number of occasions but comment is limited here in the
light of the above.
In ITC 1283 (1979) 41 SATC 36 the court agreed that the following factors should be borne in mind: (a)
the motives of the taxpayer and whether there was an intention to make a profit; and (b) the frequency of
the action which gives rise to the income and whether the earning of the income involves the conduct of a
series of actions.
With regard to the requirement that there be a profit-making intention it appears that while the presence of
such intention is a strong indication that a taxpayer is carrying on business, he could, admittedly in
exceptional circumstances, still be so doing even if such intention is absent.
As regards frequency it is nevertheless possible for an isolated transaction to constitute carrying on
business, as occurred in Stephan v CIR (1919) 32 SATC 54, (see 4.11).
Somewhat limited activity by way of the letting of property was held not to constitute carrying on business
in Estate G v COT (1964) 26 SATC 168 and ITC 1529 (1992) 54 SATC 252. Other precedents referred to
in the latter point to circumstances where a similar conclusion would be likely to be reached in relation to
directors’ fees and investment income.
D. INSURANCE PREMIUMS AND PROCEEDS
18.10 General
Reference to the taxability of proceeds and the deductibility of premiums, respectively, has already been
made elsewhere. In the case of an employer/employee relationship it is convenient to summarise both
aspects, as follows.
(a) Accident policy taken out by an employer against injury to an employee:
Premiums paid by the employer are deductible under section 15(2)(a) and are not treated as an
“advantage or benefit” in the hands of the employee.
If, under the policy, the benefits accrue to the employer they are taxable in his hands. If they
accrue to the employee or his estate, they are exempt under Schedule 3, paragraph 7(c).
(b) Term assurance policy under which an employer insures against the death of an employee
occurring within a specified period (after which the cover ceases):
Premiums are deductible and proceeds taxable in the hands of the employer.
(c) Whole life and endowment policies:
If, under the policy, the benefits are payable to the employer, premiums paid by him are not
deductible and are not an “advantage or benefit” in the hands of the employee.
If the benefits are payable to the employee or his estate, premiums paid by the employer are
deductible in his hands, under section 15(2)(a) and constitute an “advantage or benefit” in the hands of the
employee.
The proceeds are exempt, whether they accrue to the employer, the employee or his estate. If any
portion is paid by way of an annuity, however, it will be taxable under section 8(1)(a).
(d) In any instance where the employer is the beneficiary under the policy but is under a contractual
obligation to pay all or part of the proceeds to the employee, the payment is deductible in the employer’s

148
hands even if the benefit received by him was exempt, but will be taxed, as remuneration, in the hands of
the employee.
If the employer is under no contractual obligation, any payment to the employee or his estate will
be gratuitous and the question of deductibility again arises. If such payment is made to a continuing
employee it is deductible in the same way as any other remuneration. If it is made to an ex-employee it may
still be deductible (see 8.12). The ex-employee will in any event be taxable on the full amount.
Chapter 19
Withholding Taxes, etc.
Synopsis
19.1 Introduction
19.2 Non-resident shareholders’ tax
19.3 Resident shareholders’ tax
19.4 Non-residents’ tax on interest
19.5 Non-residents’ tax on fees
19.6 Non-residents’ tax on royalties
19.7 Non-residents’ tax on remittances
19.8 Residents’ tax on interest
19.9 Tobacco levy
19.10 Contractors to Government and to statutory corporations
19.11 Petroleum operators and holders of special mining leases
19.12 Informal traders tax
19.13 Automated financial transactions tax
19.4 Carbon tax
19.5 Banking institution levy
19.16 Intermediated money transfer tax
19.1 Introduction
Certain of the taxes listed above are not strictly withholding taxes but are, for convenience, considered in
this chapter. Pay as you earn (“PAYE”) is considered in chapter 21.
19.2 Non-resident shareholders’ tax (“NRST”) (ITA s 26 and Sch 9; FA s 15)
(a) Although dividends paid by companies ordinarily resident in Zimbabwe (and including amounts
distributed by private business corporations to members) are exempt from income tax in the shareholder’s
hands they may be subject to NRST. The current rates are 15% in respect of dividends from companies
listed on the Zimbabwe Stock Exchange and 20% in respect of other dividends. These rates are subject to
the terms of any double taxation agreement with the country of residence of the recipient: see chapter 20.

The tax constitutes “economic” double taxation within Zimbabwe since the profits out of which
the dividends are paid have generally already been liable to income tax in the company’s hands.
(b) The shareholders from whose dividends the tax must be withheld are any persons (such as
companies, individuals or trusts) or partnerships, not ordinarily resident in Zimbabwe.
For the above purposes a company is deemed to be ordinarily resident where its central
management and control is situated.
A recipient which is a life insurance company (in respect of its investment of Zimbabwe life
business funds), a pension fund, a benefit fund or a medical aid society escapes liability even if it is non-
resident.
(c) The tax is required to be withheld at the time of distribution of the dividend. “Distribution” is
deemed to be the date when it is paid, or credited to an account, or so dealt with that a shareholder becomes
entitled to it, whichever is the earliest. In the case of a dividend declaration to the effect that the dividend is
payable to shareholders registered on a certain future date, the then shareholders become entitled to the
dividend when that date is reached.
(d) “Dividend” is defined as any amount distributed to its shareholders by a company which is
ordinarily resident in Zimbabwe, but there are exclusions such as the following:
(i) building society dividends. (Dividends on “founding” shares are subject to the tax but
those are rarely encountered);
(ii) bonus shares;

149
(iii) repayment of subscribed capital (companies and private business corporations);
(iv) distributions, by a licensed investor, arising from his operations in an export processing
zone;
(v) distributions, by an industrial park developer, arising from the operation of his industrial
park.
(e) The amount of the dividend paid by a company is effectively reduced, for the purposes of NRST,
if it is paid out of substantially non-Zimbabwe receipts. More precisely, if a company’s receipts from
sources outside Zimbabwe exceed 15% of its total receipts, the dividend is reduced, for NRST purposes, to
the proportion which the receipts from sources within Zimbabwe bear to the company’s total receipts. The
measurement of such proportion of receipts is by reference to the “relevant accounting year” which is the
year ended 31 December (or other accounting year, see chapter 18) prior to the distribution of the dividend.
“Receipts” include both income and capital receipts, other than from capital issues or borrowings.
Example:
A non-resident becomes entitled to a dividend of $750 from a Zimbabwe company which had the following
receipts in its “relevant accounting year”:

Sales of goods from trading in Zimbabwe 100 000


Sales of goods from Zimbabwe to South African customers 20 000
Rent from immovable property in Zambia 32 500
Sale in Zimbabwe of obsolete capital assets 15 000
$167 500

Solution:
The receipts from sources outside Zimbabwe, at $32 500, exceed 15% of total receipts.
The formula results in:

= $604.40, which is subject to NRST at the appropriate rate, the balance of $145.60 being
payable gross.

(f) Every company which withholds NRST must provide its non-resident shareholder with a
prescribed certificate showing the gross dividend, the dividend on which tax is withheld and the tax
deducted.
(g) NRST deducted must be paid to the Collector of Taxes within 30 days after distribution,
accompanied by a prescribed form.
(h) If a dividend is paid free of the appropriate amount of NRST to the agent of a non-resident
shareholder, the agent must withhold and pay the NRST to the Collector, in a similar fashion to the above.
If, despite the above requirements, a non-resident shareholder receives a dividend free of NRST,
he is required to pay the relevant tax to the Collector within 30 days. (The collection mechanism might
prove difficult, although no doubt the Commissioner would appoint the paying company to recover any
shortfall out of future dividends.)
Penalties are provided for failure to withhold or pay NRST. These are that the company or agent
becomes personally liable for the tax and that the amount of the tax is increased by 15%.
Refunds of NRST may be obtained, provided that the claim is lodged within six years of payment,
in any of the following circumstances:
(i) where the tax has been overpaid;
(ii) if, with the approval of the Minister, the dividend is rescinded to comply with certain
exchange control requirements;
(iii) where the dividend is used, subject to conditions, to import essential goods into
Zimbabwe.
19.3 Resident shareholders’ tax (“RST”) (ITA s 28 and Sch 15; FA s 17)
(a) Although dividends paid by companies ordinarily resident in Zimbabwe (and including
distributions by private business corporations to members) are exempt from income tax in the shareholder’s
hands they may be subject to RST. The current rates are 15% in respect of dividends from companies listed
on the Zimbabwe Stock Exchange and 20% in respect of other dividends.
“Economic” double taxation arises as observed at 19.2(a).

150
(b) The Fifteenth Schedule is modelled on the Ninth (above). The definition of “dividend” is thus
identical for most commonly-encountered purposes as is, for example, the deemed time of “distribution”
and the mechanism for payment of the two taxes.
The term has, in the particular circumstances of an unreported case (referred to at 5.6.4) been held
not to include an interest-free loan to a shareholder.
(c) The shareholders from whose dividends the tax must be withheld are:
(i) a person (including, regardless of beneficiaries’ rights, a trust) ordinarily resident in
Zimbabwe, other than a Zimbabwe-incorporated company, a pension fund, a benefit fund or a medical aid
society; or
(ii) a partnership which is ordinarily resident in Zimbabwe.
Thus shareholders such as resident charities suffer the tax despite their accruals being entirely
exempt from income tax.
(d) For all of the above purposes a company is deemed to be ordinarily resident where its central
management and control is situated.
(e) There is no equivalent to the provision in the Ninth Schedule, containing the formula for reducing
NRST when the dividend is paid substantially out of non-Zimbabwe receipts. This would obviously not be
appropriate in relation to RST.
(f) There is again a six-year time limit on refund claims (which are available for overpayments only).
(g) There is also a totally distinct refund system, again subject to a six-year time limit, in respect of
those shareholders who are individuals and whose taxable income plus dividends does not exceed specified
amounts. Depending on the amounts, up to 100% of the RST on the dividend may, upon application, be
refundable.
(h) The main difference between NRST and RST occurs in the case of distributions to companies,
namely that NRST must be withheld where the recipient is a foreign-resident company but RST must not
be withheld where the recipient is a Zimbabwe-incorporated and resident company. Withholding of one or
the other tax occurs in any event if the recipient is, for example, an individual.
19.4 Non-residents’ tax on interest (“NRTI”) (ITA ss 29 and 94, and Sch 16; FA s 18)
(a) A non-resident recipient of interest from a source in Zimbabwe may be liable to income tax in
Zimbabwe on such interest. Liability to NRTI may also arise independently as outlined below. In such
cases the NRTI withheld is, as outlined below, allowed as a credit against income tax chargeable. The
credit is restricted to the amount of income tax attributable to the relevant interest and there is no refund of
any excess withheld.
Where, for example, a non-resident individual derives as his sole income from Zimbabwe, taxable interest
from a company, of (gross) $30 000 his ultimate income tax assessment would take account of the tax
already withheld, at 10% as indicated below, as follows.

Income tax on $30 000 say 10 815


Less NRTI 3 000
Tax payable $7 815

If he had incurred deductible expenditure totalling $22 000 (including interest on borrowed monies which
had been invested to yield the above) his assessment would reflect the following.

Taxable income $8 000


Income tax thereon say 2 884
Less NRTI 3 000
Tax refundable $ -

(b) The current rate of NRTI is 10%. This is subject to the terms of any double taxation agreement
with the country of residence of the recipient of the interest (though in no instance has it yet been reduced):
see chapter 20.
(c) The recipients from whose interest the tax must be withheld are most non-resident persons
(including, regardless of beneficiaries’ rights, a trust), and any non-resident partnership or foreign-
incorporated company. Non-resident means not ordinarily resident at the date of payment (or deemed
payment).

151
A recipient which is a company incorporated in Zimbabwe, or any pension fund, benefit fund, or
medical aid society, escapes liability even if it is non-resident.
(d) “Interest” for the purposes of the tax is defined as not only that from a source within Zimbabwe,
but as including interest payable by a person who is ordinarily resident or carries on business in Zimbabwe.
(The latter deeming provision can cause practical difficulties where, for example, the mechanism for
payment of interest by the Zimbabwe borrower is in the hands of the foreign lender.) The definition
includes also all distributions of dividends by building societies other than those (few) on which non-
resident shareholders’ tax is already payable.
Specifically excluded from the definition and therefore from the tax are :
(i) interest which is exempt from income tax in terms of paragraph 10 of the Third Schedule
to the Income Tax Act (such as that on Zimbabwe POSB accounts, building society class ‘C’ permanent
shares, etc., as observed for income tax purposes in chapter 6);
(ii) interest forming part of an annuity;
(iii) interest which is exempt from income tax and which is payable by a statutory corporation
specified by the Minister by notice in the Gazette;
(iv) interest payable to a payee whose receipts are exempt from income tax in terms of
paragraph 3 of the Third Schedule (again as indicated in chapter 6);
(v) interest payable by a licensed investor in respect of his operations in an export processing
zone;
(vi) interest payable by an industrial park developer in respect of the operation of his
industrial park.
(e) As with other provisions dealing with withholding taxes, the scheme of payment of the tax is that
liability falls in the first instance on the payer, failing whom on any agent receiving payment on behalf of a
payee and, in the last resort, on the payee himself. The mechanism for payment, with prescribed forms and
certificates, and the 30-day limit, are the same as for NRST.
There is again a six-year time limit on refund claims.
19.5 Non-residents’ tax on fees (“NRTF”) (ITA ss 30 and 95, and Sch 17; FA s 19)
(a) A non-resident recipient of fees from a source in Zimbabwe may be liable to income tax in
Zimbabwe on such fees. Liability to NRTF may also arise independently as outlined below.
In such cases the NRTF withheld is allowed as a credit against income tax chargeable. The credit
is restricted to the amount of income tax attributable to the relevant fees and there is no refund of any
excess withheld.
(The principle is the same as that in the example on NRTI at 19.4(a) but see also 20.2.8(b).)
(b) The current rate of NRTF is 20%. This is subject to the terms of any double taxation agreement
with the country of residence of the recipient of the fees.
(c) The recipients from whose fees the tax must be withheld are most non-resident persons, and any
non-resident partnership or foreign-incorporated company. Non-resident means not ordinarily resident at
the date of payment (or deemed payment).
A recipient which is a company incorporated in Zimbabwe escapes liability even if it is non-
resident.
(d) “Fees” for the purposes of NRTF are defined as those payable in respect of services of a technical,
managerial, administrative or consultative nature only. The definition embraces such fees not only from a
source within Zimbabwe but also those payable by a person or partnership ordinarily resident in Zimbabwe.
Specifically excluded from the definition and therefore from the tax are amounts payable in respect of:
(i) services rendered to an individual unconnected with his business affairs;
(ii) services rendered by any person in his capacity as an employee of the payer. Directors’
fees are, however, brought within the scope of the tax;
(iii) education or technical training;
(iv) the repair of goods outside Zimbabwe;
(v) services rendered to a licensed investor in respect of his operations in an export
processing zone;
(vi) services rendered to an industrial park developer in respect of the operation of his
industrial park.
Exemption may arise also in respect of any project specified for this purpose in the Gazette and
any project which is the subject of an agreement entered into by the Government of Zimbabwe.

152
(e) The scheme of payment is again that liability falls in the first instance on the payer, failing whom
on any agent receiving payment on behalf of a payee and, in the last resort, on the payee himself. The
mechanism for payment, with prescribed forms and certificates, and the 30-day limit, is the same as for
NRST.
There is again a six-year time limit on refund claims.
19.6 Non-residents’ tax on royalties (“NRTRoy”) (ITA ss 32 and 96, and Sch 19; FA s 21)
(a) A non-resident recipient of royalties from a source in Zimbabwe may be liable to income tax in
Zimbabwe on such royalties. Liability to NRTRoy may also arise independently as outlined below. In such
cases the NRTRoy withheld is allowed as a credit chargeable against income tax. The credit is restricted to
the amount of income tax attributable to the relevant royalties and there is no refund of any excess
withheld.
(The principle is the same as for NRTI and NRTF above. See also 20.2.8(b).)
(b) The current rate is 20%. This is subject to the terms of any double taxation agreement with the
country of residence of the recipient of the royalties.
(c) The recipients from whose royalties the tax must be withheld are most non-resident persons, and
any non-resident partnership or foreign-incorporated company. Non-resident means not ordinarily resident
at the date of payment (or deemed payment).
A “licensed investor”, however, is not required to withhold the tax in relation to his operations in
an export processing zone.
A recipient company incorporated in Zimbabwe escapes liability even if it is non-resident.
(d) “Royalties” for the purposes of NRTRoy are defined at length. In brief they include amounts for
the use of (or the right to use):
(i) any literary, etc., work, in which any copyright exists; and
(ii) any patented article, trade mark, etc..
Exemption may arise in respect of any project specified for this purpose in the Government
Gazette and any project which is the subject of an agreement entered into by the Government of Zimbabwe.
By definition, only royalties from a source in Zimbabwe fall within the scope of the tax but, as in
the case of interest and fees, the scope is expanded. Royalties are deemed to be from a Zimbabwe source if
either:
(i) the payer is a person or partnership ordinarily resident in Zimbabwe; or
(ii) they are payable by virtue of the use (or the right of use) in Zimbabwe of any property
included in the definition of “royalties”, i.e. copyrights, trade marks and the like.
(e) The scheme of payment is again that liability falls in the first instance on the payer, failing whom
on any agent receiving payment on behalf of a payee and, in the last resort, on the payee himself. The
mechanism for payment, with prescribed forms and certificates, and the 30-day limit, is the same as for
NRST.
There is again a six-year time limit on refund claims.
19.7 Non-residents’ tax on remittances (“NRTRem”) (ITA s 31 and Sch 18; FA s 20)
The current rate of NRTRem is 20%. It appears that the rate is not subject to the terms of double taxation
agreements concluded so far, since they apply only to taxes on income.
In brief a non-resident person or partnership remitting an amount from Zimbabwe to another country, for
expenditure incurred outside Zimbabwe in connection with or allocable to the carrying on by that person or
partnership of any trade within Zimbabwe, is required to pay the tax, under a prescribed form, within 30
days of the date of making the remittance. A “licensed investor” however, is not required to withhold the
tax in relation to his operations in an export processing zone.
The impact of the tax is mainly on foreign head office expenses. It does not relate to the remittance of
profits. Its effect, in the context of a foreign company operating through a branch in Zimbabwe, is that of a
countervailing tax to NRTF, which would apply if the operations were performed through a Zimbabwe
subsidiary company which paid fees to its parent.
There is again a six-year time limit on refund claims.
19.8 Residents’ tax on interest (“RTI”) (ITA ss 34 and 97, and Sch 21; FA s 22)
(a) RTI is required to be withheld at source, at the appropriate rate, on interest which accrues to
persons (i.e. including companies and trusts) ordinarily resident in Zimbabwe and which is payable by
locally-registrable “financial institutions” as defined, namely banks, discount houses, finance houses,
building societies and the Reserve Bank of Zimbabwe, on any loan or deposit.
(b) The rate is 30% reduced to 20% with effect from 1 January, 2001

153
(c) “Interest”, payable by financial institutions, includes for this purpose dividends payable by a
building society. It includes also income from Treasury bills.
The definition of “financial institutions” was expanded with effect from 1 January, 2001, to
include unit trusts, the Zimbabwe Development Bank and the Agricultural Bank of Zimbabwe Limited.
It excludes, however
(i) dividends on “founding-type” shares in building societies;
(ii) interest falling within the regulations applicable to class “C” shares in building societies;
(iii) interest payable to any other financial institution;
(iv) interest payable to the holder of a money-lender’s licence;
(v) interest payable to various bodies, including pension funds (see 6.2(b)), whose receipts
are exempt from income tax;
(vi) interest payable to an insurer registered in terms of the Insurance Act [Chapter 24:07];
(vii) interest payable on a foreign currency denominated account held by a taxpayer other than
a company or a trust, i.e. commonly an individual.
Clearly interest from bodies other than “financial institutions”, such as the Post Office Savings
Bank also falls outside the provision, as does the yield on, for example, Government and Municipal stocks.
Further, the tax otherwise applies only to “interest” as such. It is understood that the
Commissioner accepts that the term does not include the yield on bankers’ acceptances and, in the case of
negotiable certificates of deposit, any yield in the hands of a person trading them in the secondary market.
The definition of “interest” was therefore expanded to include “income from bankers” acceptances
and other discounted instruments traded by financial institutions”with effect from 1 January, 2001.
(d) As in the case of other withholding taxes there is a provision deeming the time of “payment” of
interest, various procedures are laid down and forms prescribed for use in accounting for the tax, and
penalties are imposed for non-compliance.
(e) There is again a six-year time limit on refund claims
19.9 Tobacco levy (ITA ss 16(1)(p) and 36A, and Sch 24; FA s 22A)
(a) In relation to tobacco sold on an auction floor an auctioneer is required:
(i) to withhold from the proceeds, and to pay, a levy at the rate of 2,5% (5% prior to 1
January 2000) of the gross price payable by the buyer; and
(ii) to recover from the buyer, and to pay, a levy at the rate of 2,5% (5% prior to 1 January
2000) of the above price.
With effect from 1 January, 2001 the levy, payable by sellers and buyers, was reduced to 1.5% in
each case.
With effect from 1 January, 2002 the levy is deductible in determining the respective taxable
incomes.
(b) Any auctioneer failing to comply incurs personal liability and a penalty.
The mechanism for payment, with prescribed forms and certificates, is the same as above for
NRST, though the time allowed for payment is shorter.
There is again a six-year time limit on refund claims.
(c) See also 11.10.3 regarding the income tax position.
19.10 Contractors to Government and to statutory corporations (ITA s 80)
Where a contractor (under a contract for $150 000 or more) has defaulted in submission of an income tax
return for the most recent year of assessment, 10% of any payment due to him is required to be withheld.
This is set off against his tax payable on assessment in due course and any excess is refundable.
19.11 Petroleum operators (ITA s 35) and holders of special mining leases (ITA s 36)
Exemptions from various withholding taxes have been granted through Statutory Instruments issued in
terms of the above sections. These are of limited interest in view of the very restricted number of operators
concerned.
19.12 Informal traders tax (ITA ss 36C and 97 and Sch 26: FA s 22C)
Small traders, such as “flea market” operators, are liable to a tax of 10% of rentals payable by them. This is
to be withheld and paid by the landlord; non-compliance may result in the latter incurring personal liability
for the tax and a penalty.
The tax constitutes, unlike most withholding taxes, an amount additional to the rent.
The tax is allowed as a credit against income tax payable by the trader in respect of his income from trade
or investment.
19.13 Automated financial transactions tax (ITA s 36B and Sch 25: FA s 22B)

154
Financial institutions are required to withhold and pay such tax to the Commissioner. The rate is $0,80 on
each stipulated transaction, namely withdrawals from and debits to any customer’s account by means of an
automated teller machine. With effect from 1 January, 2001 the tax on each transaction was increased to
$1.50. With effect from 1 January, 2003 the tax on each transaction was increased to $5.00.
This is not a tax on income though the Income Tax Act is being used as the vehicle for its imposition. It
will generally be deductible for income tax purposes, in the hands of the taxpayer incurring it, subject to the
same principles applicable to any other expenditure relating to trade.
19.14 Carbon tax
With effect from 1 January 2001 , a carbon tax is payable by insured persons on new and renewed policies
of vehicle insurance at the following rates:
Engine capacity Tax payable
Up to 1500cc $ 1 200
1501cc to 2000cc $ 2 400
2001cc to 3000cc $ 3 600
More than 3000cc $ 4 800
Diplomatic missions, persons connected with diplomatic missions and certain regional and international
organisations are exempt from liability for the tax.
Visitors to Zimbabwe arranging temporary vehicle insurance contracts are liable for carbon tax for each
month of such visitor’s stay, a part-month being deemed to be a month.
With effect from 1 January 2003 the rate is increased, as follows:
Engine Capacity Tax payable
Up to 1500 cc $  4 000
1501 cc to 2000 cc $  7 000
2001 cc to 3000 cc $10 000
More than 3000 cc $20 000
The tax, which was previously required to be collected by the insurer, is to be paid to the Zimbabwe
Revenue Authority by the owner of the vehicle or collected from him and paid by any agent, including the
insurer, appointed by the Authority.
19.15 Banking institution levy
This levy is introduced with respect to accounting years ending on a date which is accepted by the
Commissioner for the purposes of the year of assessment ended 31 December 2001 and subsequent years.
The banking institutions targeted are those registered or registrable in terms of the Banking Act [Chapter
24:20].
The amount due is 5% of the bank’s net profit at the end of its accounting year. The net profit element of
the calculation is reduced by the proportion that loans to small or medium enterprises (SMEs) bears to the
total loan portfolio. The term “net profit” has not been defined.
SMEs constitute the following :
(a) a company or co-operative registered in terms of the Companies Act [Chapter 24:03] but which is
not a subsidiary of another company as defined in section 143 of that Act; or
(b) a co-operative society registered in terms of the Co-operative Societies Act [Chapter 24:05]; or
(c) a private business corporation registered in terms of the Private Business Corporations Act
[Chapter 24:11].
It appears that banks could in some instances experience difficulty in determining the characterisation of
the borrower.
Normally the levy is payable not later than three months after the end of a bank’s accounting year but
extensions of this period may be granted upon application. All targeted banks are required to be registered
with the Commissioner within 30 days of the commencement of business.
With respect to the year ended 31 December 2001, the above provisions are varied, with registration being
required not later than 30 days from date of promulgation, namely 31 December 2001.
The due date of payment is not clear but the penalty for late payment could be up to 100% of the levy.
19.16 Intermediated money transfer tax
With effect from 1 January 2003 this tax is introduced at $5.00 per transaction. It arises whenever a
financial institution as defined mediates the transfer, otherwise than by cheque, of any local currency,
foreign currency, bill of exchange, promissory note, bank draft, postal order or money order between one or
more persons either physically, electronically or by any other means.

155
Chapter 20
Double Taxation Agreements
Synopsis
20.1 Introduction
20.2 Agreement with the United Kingdom
20.3 Agreement with Germany
20.4 Agreement with the Netherlands
20.5 Agreement with South Africa
20.6 Withholding taxes: table
20.7 Absence of double taxation agreement
20.1 Introduction (ITA s 91 and CGTA s 28)
(a) The comments in this chapter should be regarded as merely an introduction to the subject.
Reference to specialised works should be made for detailed commentary.
(b) Agreements are styled as being “for the avoidance of double taxation and the prevention of fiscal
evasion”. The latter function is effected partly through the application of the “arm’s length” principle, to
which reference is made below and partly through the exchange of information between the tax authorities
of the countries which are parties to an agreement. With regard to double taxation the danger to the
taxpayer arises from the frequent overlapping of tax jurisdictions in the case of cross-border transactions.
Where, for example, a taxpayer resident in country A conducts operations in, or derives investment income
from, country B, liability may well arise both in country A by reason of the taxpayer’s residence and in
country B by the application of the source principle.
(c) Most agreements between Zimbabwe and other countries have been entered into only since 1980.
They are in similar, though not identical, terms and are based on international norms contained in either the
OECD Model or the United Nations Model Convention. The agreement with South Africa, however, which
was entered into in 1965, is substantially different.
The main effect of all the agreements is either to restrict or, conversely, to preserve the power
which each country might otherwise have had to impose liability on residents of the other country. The
limitation takes various forms, such as (from the standpoint of the incurring of liability in Zimbabwe): (i) in
the case of business or professional profits of a non-resident, a requirement that liability arises if operations
in this country are conducted only in a particular way; (ii) in the case of investment income payable to a
non-resident, the imposition of ceilings on the rates of various withholding taxes; and (iii) the granting of
relief, subject to limits, where foreign income is being taxed in Zimbabwe, in recognition of tax payable on
that income in the country of origin.
In the case of a Zimbabwean taxpayer with foreign income there are, similarly, limitations on his
liability in the other country concerned.
(d) The first of the more recently-negotiated agreements was that with the United Kingdom, which
will therefore be used for general description purposes in this chapter. Limited comparisons with two other
more recent agreements will then be made, merely for illustration purposes, followed by references to that
with South Africa. Finally, a table is appended summarising the Zimbabwe withholding tax rates, some of
which will have been referred to in the narrative.
20.2 Agreement with the United Kingdom
20.2.1 Business profits (Articles 5, 7 and 9)
(a) A United Kingdom “enterprise”, i.e. an enterprise carried on by a resident (including a company
or an individual) of the United Kingdom, would ordinarily be taxable in Zimbabwe on its profits from a
source here (see chapter 3).
In terms of the agreement, however, the profits are taxable only in the United Kingdom unless the
enterprise carries on its business in Zimbabwe through a “permanent establishment” situated in this
country. Where its business is so conducted the enterprise is taxable in Zimbabwe on only such profits as
are attributable to the permanent establishment. Such attribution is made, in brief, on the basis of what the
permanent establishment might be expected to make if it were a separate enterprise operating
independently. This is one application of what is known as the “arm’s length” principle. Expenses incurred
for the purposes of the permanent establishment are deductible including an allocation of administrative
expenses incurred outside Zimbabwe.

156
(b) Since liability rests on the existence or otherwise of a permanent establishment the term is
extensively defined. It means “a fixed place of business through which the business of an enterprise is
wholly or partly carried on”.
The term is stated to include especially: (i) a place of management; (ii) a branch; (iii) an office;
(iv) a factory; (v) a workshop; (vi) a mine, an oil or gas well, a quarry or other place of extraction of natural
resources; and (vii) an installation or structure used for the exploration or exploitation of natural resources.
A building site or construction or installation project constitutes a permanent establishment only if
it lasts for more than six months.
Certain activities are excluded from the term, such as mere storage or display of goods, and the
maintenance of a fixed place of business solely for the purpose of purchasing goods.
(c) The above comments are framed in the context of a United Kingdom resident carrying on business
operations in Zimbabwe. The same principles would apply if a Zimbabwe resident operated in the United
Kingdom.
(d) Where the profits include items of income which are the subject of other articles of the agreement
the latter provisions remain effective. See 20.2.8 for comment on such inter-action of the articles.
(e) The above comments, applying articles 5 and 7, relate to a single enterprise of one country
operating through a branch, etc., in the other. Where there are separate enterprises, with some form of
common participation or control, as in the case of a holding company and its subsidiary, article 9 applies
the arm’s length principle to transactions between the two. If they make arrangements between themselves
which they would not make if they were dealing with an independent party, such as over or under-charging
for goods supplied, thus effectively shifting profits between countries, the authorities may, in taxing each
enterprise, ignore the abnormal conditions.
Such arrangements between connected taxpayers are examples of what is termed “transfer
pricing”.
20.2.2 Dividends (Article 10)
(a) United Kingdom dividends accruing to a Zimbabwean resident
As indicated in chapter 10 such dividends are liable to income tax in Zimbabwe at a flat rate of
20%. In most cases this is reduced by the granting of relief, in terms of Article 23, in recognition of United
Kingdom tax liability at 10% on the dividends.

The favourable United Kingdom tax rate could fall away in the (rare) case of the shares being
“effectively connected” with the individual’s permanent establishment (see 20.2.1) or fixed base (see
20.2.5) in the United Kingdom
(b) Zimbabwe dividends accruing to a United Kingdom resident
As indicated in chapter 19, such dividends are subject to non-resident shareholders’ tax in
Zimbabwe at the appropriate rate of either 15% or 20%. The rate is reduced to 5%, however, where the
recipient, being resident in the United Kingdom and subject to tax there on the dividends, is a company
which controls, directly or indirectly, at least 25% of the voting power in the Zimbabwe company.
This reduction in the rate would fall away in the event of the shares being effectively connected
with a permanent establishment in Zimbabwe.
20.2.3 Interest (Article 11)
(a) United Kingdom interest accruing to a Zimbabwe resident
Such interest is liable to income tax in Zimbabwe (see 3.2.7). Normal rates apply, subject to relief
in terms of Article 23 in recognition of United Kingdom tax liability on the interest. The rate of the latter
tax is restricted to 10%.
(b) Zimbabwe interest accruing to a United Kingdom resident
Liability to Zimbabwe tax arises but the rate is restricted to 10% of the gross interest. Thus if non-
residents’ tax has already been withheld by the payer no further liability arises when the recipient is
assessed to Zimbabwe income tax on interest forming part of his taxable income.
(c) See also 20.2.8.
20.2.4 Royalties and technical fees (Articles 12 and 13)
The royalties referred to are payments for the use of:(i) the copyright of literary, artistic or scientific work
(including films, tapes, etc.); (ii) patents, trade marks, formulae, etc.; and (iii) industrial “know-how”. They
do not refer to royalties in respect of the extraction of minerals. The technical fees are those for services of
a technical, managerial or consultancy nature. Fees payable to an employee of the payer are excluded.
(a) United Kingdom royalties and technical fees accruing to a Zimbabwe resident

157
In any instances where such royalties and fees attract Zimbabwe tax the position is the same as
that for interest: see 20.2.3.
(b) Zimbabwe royalties and technical fees accruing to a United Kingdom resident
Liability to Zimbabwe tax arises but the rate is restricted to 10% of the gross royalties or fees.
(c) See also 20.2.8.
20.2.5 Independent personal services (Article 15)
As observed earlier (see 20.2.1) Zimbabwe’s power to tax business profits is restricted to those instances
where the United Kingdom resident has a permanent establishment in Zimbabwe. It is similarly restricted
in the case of independent personal services to instances where such resident has a “fixed base” regularly
available to him in Zimbabwe for the purpose of performing his activities. If he has a fixed base Zimbabwe
may tax only such income as is attributable to it. There is no definition of fixed base but generally it will
equate to the concept of a permanent establishment.
The term “professional services”, used in the article, is stated to include “especially independent scientific,
literary, artistic, educational or teaching activities as well as the independent activities of physicians,
lawyers, engineers, architects, dentists and accountants”.
The article recognises that some of the types of income to which it relates may constitute technical fees and
thus fall also under Article 13 referred to above.
20.2.6 Dependent personal services (Article 16)
The article applies essentially to the earnings of employees (excluding directors’ fees and civil servants’
remuneration) where the general rule under the Zimbabwe Income Tax Act is that all amounts for services
rendered in Zimbabwe are liable to income tax in this country. This rule is modified in that Zimbabwe
withdraws its right to tax if:
(a) the recipient is present in Zimbabwe for a period (or aggregate periods) not exceeding 183 days in
the fiscal year (ended 31 December) concerned; and
(b) the employer is not a resident of Zimbabwe; and
(c) the remuneration is not borne by the employer’s permanent establishment or fixed base in
Zimbabwe.
It will be noted that all three of the conditions must be satisfied for the employee to escape Zimbabwe
liability.
There are other provisions affecting pensions.
20.2.7 Capital gains (Article 14)
As indicated in chapter 22 Zimbabwe imposes capital gains tax on gains (from a source in this country) on
disposals of, in general terms, immovable property and “marketable securities” (mainly stocks and shares).
Under the article Zimbabwe has relinquished any right to tax gains from sales of shares by a United
Kingdom resident save for shares in any company the assets of which consist principally of immovable
property.
20.2.8 Limitations on, and withdrawal of, reduced rates of tax
(a) The comments at 20.2.3 regarding interest, and at 20.2.4 regarding royalties and technical fees,
refer.
Where the amount paid exceeds, by reason of a special relationship, that which would normally be
paid between independent parties, the reduced rate of tax applies only to the normal amount. This is a
further instance of the application of the arm’s length principle to counter transfer pricing.
Where the income is “effectively connected” with a permanent establishment or fixed base of the
recipient the reduced rate is withdrawn and the income treated in the same way as business profits.
(b) In cases where the taxpayer’s profit margin is narrow the Zimbabwe tax burden may still prove to
be heavy since, despite the restriction of the rate to 10%, this is applied to the gross amount. The burden
stems from the impact of withholding taxes, considered in chapter 19, which similarly apply on the gross
amount. In the case of two United Kingdom enterprises earning technical fees from operations through
permanent establishments (or fixed bases) in Zimbabwe the following could result (for the year ended 31
December 1999).

Case 1 Case 2
Fees 1 000 000 1 000 000
Allowable expenditure 950 000 700 000
Taxable income $50 000 $300 000
Income tax

158
(36,05%) thereon 18 025 108 150
Non-residents’ tax already
withheld on fees: 10% on
$1 000 000 100 000 100 000
Excess non-refundable in Case 1 $81 975
Excess chargeable in Case 2 $8 150
Actual liability therefore $100 000 $108 150
Effective rate of tax 200% 36,05%

If either enterprise earned the fees without having a permanent establishment no income tax would
arise but liability to withholding tax would remain. The results are as follows.

Case 1 Case 2
Non-residents’ tax withheld on fees $100 000 $100 000
Effective rate of tax thereon 200% 33,3%

As indicated, non-resident recipients of fees may be adversely affected as compared with


Zimbabwe counterparts.
Disparities arise similarly in relation to interest and royalties which are also subject to withholding
tax.
20.2.9 Exchange of information (Article 26)
The main effect of this article in the case of Zimbabwe residents is that the Commissioner is likely to
receive information relating to, for example, interest and dividends accruing to them from investments in
the United Kingdom. Presumably there would also be an exchange of information in the event of, for
example, the detection of transfer pricing.
Neither country, however, assists the other in the collection of tax (see 20.5.1).
20.3 Agreement with Germany
Some of the areas where the agreement varies from that with the United Kingdom are as follows.
20.3.1 Business profits (Articles 5 and 7)
With regard to a permanent establishment in the context of a building site or construction or installation
project, although the six months rule is retained the attribution of profits excludes, for example, those on
delivery of goods either in connection with the activity or independently of it.
20.3.2 Capital gains (Article 13)
Zimbabwe has not relinquished its power to tax gains on certain shares.
20.4 Agreement with the Netherlands
20.4.1 Business profits (Articles 5 and 7)
There are again variations in the attribution of profits to certain permanent establishments.
20.4.2 Capital gains (Article 13)
(a) Zimbabwe has not relinquished its power to tax gains on certain shares.
(b) Specific recognition is accorded to the circumstances covered by section 15 of the Zimbabwe
Capital Gains Tax Act, i.e. the election to transfer at effectively “tax value” in schemes of group
reconstruction, etc..
20.5 Agreement with South Africa
20.5.1 General
As indicated at the beginning of this chapter, the agreement was entered into in 1965 and there are
accordingly a number of differences as compared with agreements based on the OECD and United Nations
Models.
Further, at the time the agreement was concluded, none of Zimbabwe’s withholding taxes had been
enacted. The Zimbabwe taxes which are the subject of the agreement are, however, not only “. . . income
tax and . . . all other taxes on persons or on the incomes of persons . . .” but also (Article 1) “. . . any other
taxes of a substantially similar character imposed . . . subsequently to the date of signature of this
Agreement”. It is understood that the application of the agreement to Zimbabwe’s withholding taxes is
accepted by the Commissioner.
As regards a refusal by South Africa to assist with the collection of tax see COT, Federation of Rhodesia, v
McFarland (1965) 27 SATC 15.
20.5.2 Business profits (Articles 2 and 3)

159
The concept of a permanent establishment and the attribution of profits thereto are broadly the same as in
the United Kingdom agreement.
The definition of “permanent establishment” is very similar though there is no specific clarification
regarding building sites or exploration, etc., of natural resources.
Management charges are specifically excluded from those types of income which are required to be linked
with a permanent establishment. A South African enterprise, for example, earning management charges in
Zimbabwe, would therefore be liable to Zimbabwe income tax, even in the absence of a permanent
establishment in this country.
20.5.3 Dividends and interest
There are no special provisions. South African tax applicable to a dividend or interest accruing to a
Zimbabwe resident would rank for relief, in terms of Article 13, against Zimbabwe tax payable on such
income.
20.5.4 Royalties (Article 6)
The royalties correspond largely with those referred to at 20.2.4.
Royalties accruing to a Zimbabwe resident which are subject to tax in South Africa are exempt in
Zimbabwe, and vice versa. This is a totally different result as compared with the position under agreements
based on, for example, the OECD Model, as described at 20.2.4.
20.5.5 Technical fees
There are no special provisions.
It appears that technical fees earned by a company from a source in Zimbabwe may, if they are taxable in
South Africa, escape liability to income tax and withholding tax in Zimbabwe unless they arise from the
operations of a permanent establishment in this country.
20.5.6 Personal and professional (dependent and independent) services (Article 9)
Zimbabwe’s general rule regarding employees is again modified in that Zimbabwe relinquishes its right to
tax if:
(a) the recipient is present in Zimbabwe for a period (or aggregate periods) not exceeding 183 days in
the fiscal year (ended 31 December) concerned; and
(b) the services are performed for or on behalf of a person who is a resident of South Africa; and
(c) the remuneration is subject to South African tax.
The above applies equally to profits from professional services regardless, it appears, of the existence or
otherwise of a permanent establishment.
There are other provisions affecting pensions and civil servants’ remuneration.
20.5.7 Capital gains
There are no provisions. South Africa has not had such a tax, though its introduction is imminent.
20.6 Withholding taxes: table
The percentage rates (4), taking account of agreements, are as follows.

Dividends Interest Royalties Technical


(1) Fees
Bulgaria 10(2) 10 10 10
Canada 10(2) 15(3) 10 10
France 10(2) 10 10 10
Germany 10(2) 10 7,5 7,5
Malaysia 10(2) 10 10 10
Mauritius 10(2) 10 15 0/20
Netherlands 10(2) 10 10 10
Norway 15(2) 10 10 10
Poland 10(2) 10 10 20
South Africa (1) 10 20 20
Sweden 15(2) 10 10 10
United Kingdom 5(2) 10 10 10
Other (1) 10 20 20
(1) General rate: Shares listed on the Zimbabwe Stock Exchange 15%
Other shares: 20%
(2) Only on shareholding or voting power, by a company, of at least 25%; otherwise the rates
remain as in (1).

160
(3) Therefore Zimbabwe’s rate remains at 10%.
(4) Non-residents’ tax on remittances remains at 20% in all cases.
Agreements are under negotiation with Botswana, Indonesia, Namibia, Seychelles, Tanzania, Tunisia,
Yugoslavia, Zambia and (renegotiation) South Africa.

20.7 Absence of double taxation agreement (s 93)


Where foreign income, from a country with which there is no agreement, is taxable in Zimbabwe by virtue
of it being deemed to be from a source here (see 3.2) Zimbabwe in any event grants unilateral relief. To
some extent, therefore, the result may be similar to that in the case of agreements.
In ITC 1535 (1992) 54 SATC 356 the court held that Zimbabwe was obliged to grant relief on income, in
that instance a royalty, which was designated in the Act as being deemed to be from a source in this
country, despite the possibility, that is, of it being from a true source here.
Chapter 21
Administration, Tax Collection, etc.
Synopsis
21.1 Returns and information
21.2 Assessments and penalties
21.3 Representative taxpayers
21.4 Objections and appeals
21.5 Payment of tax
21.6 Offences
21.1 Returns and information (mainly ss 37 to 44)
21.1.1 General
The Commissioner is required to give public notice every year of the classes of persons who must lodge
income tax returns. This provision is important because, although in practice most taxpayers are issued with
returns, the effect of the notice is to place the onus on every person in the classes stipulated to apply for a
return if one is not received. Such person is not relieved of penalties by reason simply of his not having
received either a return or a request to submit a return.
The Commissioner’s notice, which is published in the Gazette and in the local Press, usually includes the
following categories of persons, who are thus required to submit a return, in addition to those to whom a
form has been sent:
(a) every individual whose income for the year ended 31 December exceeded specified limits namely,
for 1999, $24 000;
(b) in their representative capacities, every public officer of a company, trustee of a trust, executor of
a deceased estate and administrator of an insolvent estate.
The section requires submission of returns within thirty days of the date of the notice but the Commissioner
is permitted to grant extensions of time for good cause.
The consequences of failure to submit a return within the time allowed can be serious, as penalties may be
imposed upon subsequent assessment of the taxable income for the year. Prosecution may ensue in cases of
repeated default.
In the case of every return submitted the signatory is deemed to be aware of its contents. Thus in the event
of a return being incorrect it is generally not possible for a signatory to shelter behind the fact that he
engaged a professional adviser to complete the return.
Where a person is unable to furnish a return, as where accurate figures are not available, the
Commissioner may accept a return of estimated income or may himself make an estimate. He is
empowered not to impose a penalty if he is satisfied that the reasons for the failure to render a return are
genuine.
With regard to: (i) the submission of returns by partnerships, see chapter 14; and (ii) the submission of
supporting accounts drawn up for a year ended other than 31 December, see chapter 18.
21.1.2 Further aspects
(a) Income of a person’s minor child (s 38)
Each parent must reflect in his return the amount of certain income of his minor child, i.e. either
that which has accrued directly or indirectly from the parent or the latter’s spouse, or which is the subject of
specific provisions of the Act (see 2.7.3 and 2.7.4.)

161
(b) Further returns, information, etc.
A person must furnish further returns and information when called upon to do so (s 39). Many
employers are, for example, required to lodge an annual return of general information, which includes
items such as salary paid to each employee, interest, fees, rents and commissions payable.
(c) Every company must file a copy of its memorandum and articles of association within 30 days of
its incorporation and furnish copies of subsequent alterations (s 42).
(d) If the nature of a return is such that a balance sheet and profit and loss account are required to
support it these must be lodged, authenticated by the signature of the person making the return. Such
accounts must be accompanied by a statement, by the person who prepared them, as to the extent of his
examination of the books. A prescribed form is issued for this purpose (s 43).
(e) The Commissioner is granted various other powers of action, which are usually used only where it
is suspected that a taxpayer’s return, or failure to submit a return, is fraudulent, but which may be employed
in relation to any taxpayer.
21.2 Assessments and penalties
21.2.1 Assessments : general
The term “assessment” has a technical meaning in that it is defined in section 2(1) of the Act as:
(a) the determination of taxable income and of the credits (in terms of the Finance Act) to which a
person is entitled; or
(b) the determination of an assessed loss ranking for deduction.
In ITC 1535 (1992) 54 SATC 356, however, in the context of the Commissioner’s power to raise an
additional assessment (see 21.2.6(a)) it was held at 361 that the term covered “the case where an amount
which should have been charged to tax is not so charged because relief from double taxation has
wrongfully been granted ....”.
(In everyday parlance the term is used also to denote the notice of assessment, issued on the
Commissioner’s behalf, reflecting the above determination.)
Many assessments are made routinely on the basis of the taxpayer’s return and accounts, with perhaps
minor adjustments to the amount declared by him.
21.2.2 Provisional assessments (s 37(14))
The above term does not appear in the Act itself but is used as a convenient description of authorised
procedures. Where a person is unable initially to furnish a required return the Commissioner may issue an
assessment on the basis of either the taxpayer’s, or his own, estimate of the income. This is generally done
in conjunction with the granting of an extension of time for the submission of a return and thus without the
imposition of penalties. The assessment is adjusted once a return of actual income is furnished.
21.2.3 Assessments “subject to audit”
The Commissioner has adopted a procedure of issuing notices of assessment bearing the above
endorsement. There is no statutory reference to this. It is understood that the Commissioner accepts that in
these circumstances an “assessment” as defined in section 2(1) of the Act has been made, i.e. that there has
been a determination of taxable income, etc., and that all the normal consequences follow, including the
commencement of the prescription period on the issuing of an additional assessment (see 21.2.6). The
endorsement thus amounts merely to a notification that a fuller examination of the return and any accounts
may still be undertaken. It appears, however, in the light of the decision in ITC 1480 (1990) 52 SATC 276,
that a taxpayer may not be so protected against the issue of an additional assessment, in which event the
position is unsatisfactory.
21.2.4 Estimated assessments (s 45)
If a taxpayer either fails to submit a return or submits a return which is not satisfactory, or if a taxpayer is
about to leave Zimbabwe, the Commissioner may issue an estimated assessment.
The section contemplates also quite different circumstances, namely where the taxpayer is unable to furnish
an accurate return. He and the Commissioner may then agree on an estimated taxable income (or assessed
loss). In this event, however, the agreed amount will not be subject to later objection by the taxpayer,
though the Commissioner may amend the figure if he finds that the taxpayer withheld relevant information.
In both of the above cases penalties may be imposed by the Commissioner, as considered below.
21.2.5 Penalties (s 46)
(a) The Commissioner is empowered to impose a penalty (technically “additional tax”) in any
instance where a person either fails to submit a return or submits an incorrect return. The maximum
penalty is 100% of the tax applicable to the default or, more precisely:

162
(i) if the taxpayer defaults in submission of a return: 100% of the tax applicable to his entire
taxable income (the latter in these circumstances probably being estimated by the Commissioner);
(ii) if the taxpayer, though he submits a return, omits an amount which ought to have been
included, or makes an incorrect statement, or fails to disclose facts which should have been disclosed, or
claims a deduction which is not permissible, or wrongly claims a personal credit: 100% of the tax which he
would have escaped through his omission, etc..
There is also an alternative maximum penalty equal to the maximum fine which could be imposed
for the offence of failing to submit a return. This would apply to a taxpayer who, despite having defaulted,
would otherwise escape punishment by reason of his taxable income being lower than the amount liable to
tax.
The Commissioner is empowered to remit the whole or part of such penalties if he considers, in
the circumstances in (i) above, that the default was not due to any intent to defraud the revenue or to
postpone payment of tax or, in the circumstances in (ii), that the omission, etc., was not due to any intent to
evade tax.
As in the case of estimated or “provisional” assessments, considered at 21.2.2, there is again
provision for agreement to be reached between the Commissioner and the taxpayer, this time as regards the
amount of the penalty to be imposed. Again the agreed amount is not subject to later objection by the
taxpayer though the Commissioner may amend the penalty if he finds that the taxpayer withheld relevant
information.
The opportunity to reach an agreement, or settlement, which occurs in practice by negotiation,
often produces at least some mitigation of the penalty even in cases of evasion. The result, nevertheless,
can be severe.
(b) The practice of settlement is a factor which has contributed to there having been few appeals in
Zimbabwe on this subject.
An instance arose, however, in ITC 1334 (1981) 43 SATC 98, which was decided at a time when the
maximum penalty was 200%. The taxpayer company had purported to have purchased a motor vehicle in a
year of assessment and had claimed special initial allowance in relation to that year. The Commissioner, in
disputing such fact and rejecting the claim, imposed the maximum penalty. On appeal the company failed
to convince the court (i) that its statement regarding the date of purchase was correct and (ii) that such
statement was not made with intent to evade the payment of tax. Given the existence of the intent to evade,
the judgment contains the comment, at 106 “. . . then the Commissioner, and therefore the court, has no
discretion to remit any part of the penalties”.
Conversely in ITC 1669 (1999) 61 SATC 479, concerning whether items of machinery and equipment had
been brought into use (see 9.1(c)) the court remitted the penalty in full because the point was an arguable
one.
(c) The provisions in the South African Act in this context, although similar to those referred to
above, are not identical in that remission is specifically contemplated, even in cases of evasion, if the
Commissioner is of the opinion that there were extenuating circumstances. Decisions of the South African
courts are therefore not strictly appropriate in the context of the Zimbabwean Act, but are of interest,
(bearing in mind also that they relate to a permitted maximum penalty of 200%).
ITC 1577 (1994) 56 SATC 236 laid down three alternative circumstances where it was considered that
evasion could be identified, namely: (i) where evasion is the taxpayer’s aim (“direct”); (ii) where evasion is
not his closest aim but is needed in order to achieve such aim (“awareness of certainty”); and (iii) where he
pursues such aim despite evasion being a possible result (“awareness of possibility”).
The tests laid down in ITC 1430 (1988) 50 SATC 51 would also warrant attention in this country. In that
case the taxpayer, having lodged an appeal against the imposition of penalties (his omissions of income,
and the basic tax thereon, not being disputed) had died prior to the hearing of the appeal.
The court identified three factors underlying the appropriateness of the imposition of penalties, i.e.,
punishment of the taxpayer, the deterrent effect upon him and the deterrent effect upon other taxpayers.
Having regard to the taxpayer’s death (and the insolvency of his estate) the first and second factors had no
application and the third only little. The penalties, having preference, would have been recoverable by the
Commissioner at the expense of other creditors. Even had the estate been solvent it would have been the
beneficiaries who would have suffered.
It was accordingly ordered that the penalties should be remitted but nevertheless (and somewhat in conflict
with the above) that a reduced amount should stand, calculated by reference to the amount of interest lost
by the State by reason of the late payment now arising.

163
(The general reasoning in relation to penalties in this case was adopted by the Special Court in ITC 1669,
referred to above, where the taxpayer company was in provisional liquidation.)
The principle that the loss of interest suffered by the State should be taken into account has been
applied also in other cases where the taxpayer’s blameworthiness has been found to be only moderate, as
in KBI v Mabotsa (1993) 55 SATC 98.
Taxpayers’ lack of literacy, etc., has availed them to varying degrees in cases such as ITC 1331
(1981) 43 SATC 76, CIR v Da Costa (1985) 47 SATC 87, and ITC 1540 (1992) 54 SATC 400.
An instance where the court found that a claim for overseas travelling expenses had been made with intent
to evade taxation, warranting no remission of penalty, arose recently in ITC 1658 (1999) 61 SATC 231.
Factors which were taken into account included the length of the trips, their timing, the absence of resultant
business and the lack of likelihood of business being concluded.
21.2.6 Additional and reduced assessments (ss 47, 48 and 49)
The Commissioner may adjust assessments, and assessments of loss, so as to increase or reduce the income
assessed or the deductions or credits allowed. Such reopening is, however, subject to restriction, as follows.
(a) Additional assessments (s 47)
(i) There are two circumstances in which the Commissioner may never issue an additional
assessment (or a reduced assessment of loss).
The first is where this would constitute a variation of an objection decision which he had
already made in respect of the taxpayer. It has been held, in a case involving the disallowance of
unproductive expenditure, that where the Commissioner has based his decision in an objection on the use of
a formula, any subsequent attempt by him to increase the disallowance through the adoption of a different
formula would constitute a variation of his decision and would be precluded: ITC 1252 (1976) 38 SATC
176.
It appears however that, after a decision in any objection, the limits on the
Commissioner’s power relate only to the matter which was under consideration, and that he would not be
precluded from raising an additional assessment in relation to some other matter.
The second is where the original assessment was made not merely in error but in
accordance with the practice generally prevailing at the time. A taxpayer relying on the latter expression for
protection may, however, experience difficulty since it has been held that “generally prevailing” signifies a
practice “... which is applied generally in the different offices of the Department in the assessment of
taxpayers ...” and further that “... in seeking to establish such a practice in regard to a particular aspect of
tax assessment it would not be sufficient to show that the practice was applied in merely one or two
offices”: CIR v SA Mutual Unit Trust Management Company Ltd (1990) 52 SATC 205, and see also ITC
1495 (1991) 53 SATC 216.
(ii) In other instances there has been a longstanding general time limit of three years from the
end of the year of assessment to which the original assessment relates, though the limit has been six years
in cases of fraud, misrepresentation or wilful non-disclosure of facts. (The latter, by their very nature,
almost invariably attract penalties, as above.) The three-year limit has been inapplicable also if the
additional assessment resulted from the Commissioner’s carrying out of the terms of a double taxation
agreement.
These time limits have been held not to apply to estimated or “provisional” assessments
issued in terms of section 37(14) (see 21.2.2): ITC 1442 (1989) 51 SATC 19. ITC 1442 is authority also for
the rule that for an additional assessment to be valid it must be received by the taxpayer, and not merely
issued by the Commissioner, within the statutory period. In terms of ITC 1665 (1999) 61 SATC 413 the
burden of proof regarding the date of receipt rests, in South Africa, on the taxpayer. The same position is
likely to obtain in Zimbabwe.
The time limit in the case of the first of the above categories, which may conveniently be
styled “innocent error”, was with effect from 28 November 1997 extended from three to six years, while
the limit was, with effect from that date, removed altogether in cases of fraud etc.. Since, however, the
amendments did not specify that they were to have retrospective effect it is submitted that the current
position is that the Commissioner may not reopen years: (i) in the case of “innocent error”, earlier then the
year ended 31 March 1995; and (ii) in the case of fraud, etc., earlier than the year ended 31 March 1992.
With the passage of time the new six-year limit for innocent error will, of course, move annually but the
Commissioner will be able to reopen indefinitely, to the year ended 31 March 1992, in fraud, etc., cases.
(iii) With regard to the rule in fraud, etc., cases there is a requirement that the Commissioner
be satisfied that the adjustment is necessitated by reason of fraud, etc.. This wording appears to be

164
sufficiently close to that in the South African Act for cases such as SIR v Trow (1981) 43 SATC 189 and
ITC 1563 (1993) 55 SATC 315 to apply in favour of the taxpayer, in Zimbabwe, in obliging the
Commissioner to be satisfied (not merely to suspect) and to give his reasons.
Further, when the Commissioner relies on non-disclosure, the taxpayer will, unlike the
position in South Africa (ITC 1518 (1992) 54 SATC 113) still escape unless his conduct was wilful.
As to the meaning of “non-disclosure”, this has been held to apply to, for example, a
failure to reveal the granting of abnormal discounts between related companies (ITC 1582 (1995) 57 SATC
27) and to a taxpayer’s use of the term “running of factory” to describe consultancy fees relating to a post-
merger rationalisation (ITC 1594 (1995) 57 SATC 259). It appears that a similar result would arise in
Zimbabwe despite those cases having been decided in relation to a reference to material facts.
(iv) The procedures are strictly observed by the courts. In ITC 1671 (2000) 62 SATC 39 the
(South African) Commissioner had, outside the period for raising an additional assessment, cancelled an
“original” assessment and issued another. This procedure was rejected. It is considered that in Zimbabwe
the Commissioner would similarly be unable to “restart the clock”.
(b) Reduced assessments (ss 48 and 49)
No reduced assessment (or additional assessment of loss) may be obtained by a taxpayer if the earlier
assessment was accepted by him and was in accordance with the “practice generally prevailing”. If the
existence of such a practice were relied upon by the Commissioner (in order to resist a claim for a reduced
assessment) it appears that he would have to comply with the same test as is referred to in (a) above.
In other instances there has been a longstanding general time limit of three years from the date of the notice
of assessment in question, this measurement thus favouring the taxpayer in comparison with that for
additional assessments. The three-year limit has been inapplicable if the reduced assessment resulted from
the Commissioner’s carrying out of a double taxation agreement.
The time limit, was, with effect from 28 November 1997, extended from three to six years. The comments
in (a) above regarding the corresponding new six-year limit, apply.
No objection may be lodged against a reduced assessment. The effect is to confine rights of objection to the
original and any additional assessment.
21.2.7 Access to assessments (s 52)
Assessment notices are not open to public inspection but a taxpayer is entitled to certified copies of his own
assessments.
21.3 Representative taxpayers (ss 53 to 61)
21.3.1 Meaning
The term is defined in the Act to identify those persons upon whom liability and duties are imposed where
the taxpayer is unable to deal personally with tax matters, such as companies, trusts and any individual who
is absent from Zimbabwe. In the case of companies the representative is the “public officer”. In case of an
insolvent estate or a trust it is the trustee, and in the case of a deceased estate it is the executor (each of the
latter, along with company liquidators, insolvency administrators, etc., being covered by the wide-ranging
definition of “trustee” in section 2(1)).
All responsibilities and duties under the Act are still legally binding on the taxpayer despite the existence of
a representative.
21.3.2 Liability and duties
The liability and duties of a representative taxpayer extend to all the liabilities and duties which would fall
on him if the income were his own, but assessments will be made on him in his representative capacity
only. In particular, taxes payable can be recovered only to the extent that the representative taxpayer
possesses or controls funds belonging to the taxpayer whom he represents (see 21.5.4). The representative
has the right to recover from the taxpayer, or from the taxpayer’s funds, any taxes paid on his behalf.
If the representative disposes of funds or money out of which the tax could have been paid he will be
personally liable for any unpaid tax (s. 56). In terms of the decision in Esselmann v Secretary of Finance
(1991) 53 SATC 285, liability for payment of tax arises only, as might be expected, at the point when a
proper assessment has been raised and properly served. It seems clear, however, that the existence of
annual payment dates similarly creates such liability.
21.3.3 Appointment or declaration of agent (s 2 (def’n) and s 58)
If the Commissioner thinks it necessary he may, for the purposes of the Act, declare any person as the agent
of any other person. He usually takes this action if the Collector of Taxes has been unable to recover tax.
Thus, an employer may be declared agent for an employee; a building society or bank as agent for a

165
depositor; and so on. In all cases the agent is responsible for paying the tax but only out of money which is
due to the taxpayer. The powers are wide and cover for example PAYE, interest and penalties.
It appears from Regering van die Republiek van Suid Afrika v Raubex Konstruksie (Edms) Bpk (1994) 56
SATC 167, that it is not necessary for such declaration to be in writing.
21.3.4 Public officers of companies (s 61)
Every company carrying on a trade or with an office or place of business in Zimbabwe must always be
represented by a public officer who resides in Zimbabwe.
The appointment must receive the approval of the Commissioner and he will normally approve only a
director or manager of the company, or some other senior person acquainted with its day-to-day affairs. In
default of an appointment within one month of the establishment of an office or place of business he has
power to designate such a person as public officer.
Within the same period, every company must supply an address for delivery of documents. If there are any
changes in the appointment or the address, notification must be given within 30 days.
Penalties are prescribed for companies and their officials not complying with the section and for individuals
not carrying out properly the duties of a public officer.
21.4 Objections and appeals
21.4.1 Objections: General (s 62)
The circumstances in which a right of objection arises and the manner in which objections should be lodged
are:
(a) when the assessment does not comply with the Act. Obvious examples are the Assistant
Commissioner’s inclusion of amounts considered by the taxpayer not to be taxable, rejection of claimed
deductions, etc.;
(b) when the Commissioner has determined a matter which the Act puts within his discretion,
provided such matter is listed in the Eleventh Schedule. This includes, for instance, the determination of
whether or not:
(i) a medical aid society should be approved ;
(ii) penalties applicable to omissions of income, etc. should be waived;
(iii) penalties arising out of a taxpayer’s failure to withhold and pay various amounts,
including PAYE, should be waived.
Under section 68 the taxpayer is precluded from objecting to decisions not listed in the Eleventh
Schedule;
(c) where the question of relief from double taxation arises, i.e. either:
(i) in the case of Zimbabwean income liable in Zimbabwe to both withholding tax and
income tax in the hands of a non-resident; or
(ii) in the case of foreign income liable to both Zimbabwean and foreign taxes in the hands of
a Zimbabwean resident.
An objection must be lodged in writing within 30 days after the date of issue of an assessment, unless the
Commissioner agrees that there are reasonable grounds for delay. This decision is not subject to objection.
It is advisable that the full grounds be set out in the initial letter of objection since section 65(4) could
restrict the taxpayer, in any subsequent appeal, to those grounds.
There is no provision obliging the Commissioner to furnish his decision within a specified period.
Taxpayers are, in fact, prejudiced by a provision that, in the event of a decision not having been furnished
within six months, the objection is deemed to have been disallowed. They could thus find themselves, in
order to preserve their rights, having to commence appeal proceedings, as below, with the attendant
inconvenience and cost. The provision also condones dilatoriness on the part of officials.
The use of the objection procedure as the avenue for an appeal may be instrumental also as the means of
challenging the validity of the related tax debt; see section 79 of the Act, and Van Zyl N.O. v The Master
and Another (1987) 49 SATC 165. (A challenge on a different basis succeeded in CIR v Bowman N.O.
(1990) 52 SATC 69 but the circumstances there were unusual.)
21.4.2 Burden of proof (s 63)
In any objection or appeal the burden of proof that an amount is not liable to tax, or is deductible, or ranks
for credit, is placed on the taxpayer. The significance of this burden has been illustrated in a number of
cases.
Reliance Land and Investment Co Ltd v CIR (1946) 14 SATC 47, dealt with an equivalent provision. It was
held that the section meant that when a particular accrual has been determined by the Commissioner to be
an accrual of income, the court must treat it as such until the taxpayer establishes on a balance of

166
probability that it is an accrual of a capital nature. A similar standard had been laid down earlier in CIR v
Goodrick (1942) 12 SATC 279.
In ITC 1423 (1987) 49 SATC 85 the Commissioner had estimated the taxable incomes of a company and
its directors as he considered its books and records to be unsatisfactory. The appeal failed as the taxpayers
were unable to produce evidence to rebut the Commissioner’s estimates. The court quoted with approval
from an earlier judgment to the effect that the Commissioner’s estimates potentially stand even if it is
shown that an element of the assessment is a guess or is very probably wrong: “. . . It is prima facie right
and remains right until (the taxpayer) shows it is wrong. The taxpayer must, as a general rule, show not
only negatively that the assessment is wrong, but also, positively, what correction should be made to make
it right or more nearly right.”. And further: “Proof is not achieved by a taxpayer submitting that his guess is
as good as the assessor’s; more is required.”.
In the case of an accrual or receipt such burden is imposed on the taxpayer, however, only after the
Commissioner has proved that an accrual or receipt has occurred: CIR v Butcher Bros (Pty) Ltd (1945) 13
SATC 21, followed in ITC 1545 (1992) 54 SATC 464.
21.4.3 Appeal procedure and related matters (ss 64 to 70 and Sch 12)
Any taxpayer whose objection has been wholly or partly disallowed, or deemed to have been disallowed,
by the Commissioner has a right of appeal. Any taxpayer wishing to appeal must do so in writing within 21
days after the Commissioner has issued his ruling, or after the deemed disallowance and must say whether
he wishes the case to be heard in the Special Court or in the High Court. The cost of taking a case to the
Special Court may be lower in view of the more relaxed rules regarding representation. Both the taxpayer
and the Commissioner may have the right to take a further appeal to the Supreme Court.
If an assessment is under objection or on appeal, the obligation to pay the assessed tax is not deferred. The
Commissioner has, however, the power to allow payment to be delayed. Where this is exercised it is
usually made subject to the condition that interest will be payable from the original date on unpaid tax
eventually found to be due.
21.5 Payment of tax
21.5.1 General (s 71)
The Commissioner is empowered to fix the period or periods within which tax must be paid (though mostly
liability will be governed by sections, considered below, relating to “annual payment dates” and
“employees’ tax” respectively). In any other instances he generally allows 30 days after the issue of the
assessment but deferments or arrangements to pay by instalments may be agreed, by the Collector of Taxes,
on behalf of the Commissioner.
Unless the Commissioner recognises special circumstances interest is payable on overdue tax. The current
rate is 35% (S.I. 15 of 1995).
The Commissioner, however, has no liability to pay interest where refunds of tax are delayed (COT v F
Kristiansen (Pvt) Ltd (1995) 57 SATC 238) save where the tax has been levied under a provision which
transpires to have been unconstitutional (Ellis NO v COT (1995) 57 SATC 282).
Although the Commissioner may waive interest on overdue tax he has no power to waive the imposition of
tax. Matters such as writing off its recovery, insolvency of the debtor, etc., rest on other statutes.
In a case on the question of set off of debts the court held that the taxpayer company was not entitled to set
off, against tax payable by it, an amount due to it by the State under a guarantee of an overdue debt owed to
the taxpayer by a state-owned corporation: COT v First Merchant Bank Ltd (1997) 59 SATC 174.
21.5.2 Annual payment dates (s 72)
Save for instances of liability to employees’ tax, provisional tax is payable on each of three annual payment
dates (“APDs”) in the year after the year of assessment, namely the following 28 February, 30 June and 30
November.
The taxpayer must estimate his liability and pay to his Collector of Taxes 50% of it on the first APD and
25% on each of the second and third dates. Upon receipt of an assessment, or at any earlier opportunity,
any shortfall on instalments should be made good in order to minimize interest payable.
Although the Commissioner is empowered to prescribe, by notice in the Gazette, lower limits of taxable
income to which APDs will not apply, no such notices have yet been published. He is empowered also to
vary a particular taxpayer’s APDs by written notice.
21.5.3 Employees’ tax (PAYE)
(a) General (s 73 and Sch 13)
The PAYE system is merely a means of advance collection, on a monthly, weekly, etc., basis, of
income tax (plus, in practice, the AIDS levy) due by employees on their remuneration. The operation of the

167
system is described in detail in booklets issued free of charge to employers and employees by the
Commissioner. Briefly the position is as follows.
Any employer of an employee whose annual remuneration, including benefits, exceeds the ceiling
of the “zero-rate band” (see 10.2: $30 000 for the year ending 31 December 2000) is required to register for
PAYE purposes with the Collector of Taxes. A similar obligation extends to, for example, administrators of
pension funds and, in respect of agents’ commission, to insurers.
Employers, administrators, etc., are required to withhold, from employees’ remuneration, tax in
accordance with tables published by the Commissioner. The amount to be withheld may, however, be
varied in accordance with any directive issued by the employee’s Assistant Commissioner of Taxes. The
amount withheld is credited against the income tax due upon ultimate assessment of the employee’s return
for the year.
The rate of PAYE varies according to the tables and directives referred to above and further in any
year when a surcharge or levy is imposed.
The tax must be paid by the employer to his Collector of Taxes by the 15th of the month following
that in which the income accrued. Late payment attracts penalties and interest which can be severe.
(b) Final deduction system (“FDS”) (s 37(12) and para 20A of Sch 13)
With effect from 1 January 2000 legislation was introduced to reduce the submission of returns by
taxpayers whose taxable income consists solely of remuneration subject to PAYE. The Commissioner is
empowered to direct any employer to withhold PAYE from each employee’s remuneration in such a way as
to ensure that the amount withheld in any year of assessment is as nearly as possible the same as the
income tax payable by the employee concerned for that year of assessment. The system, which is described
in detail in booklets issued free of charge by the Commissioner, will embrace, for example, the deduction
of personal credits, and refunds of excess, and the collection of additional, PAYE by the employer.
21.5.4 Persons by whom tax is payable (ss 74 and 77)
(a) Every representative taxpayer is liable for tax on income over which he has control.
(b) As has been seen (in chapter 2) tax is payable by a person not only on the income which accrues to
him but also on income which is deemed to accrue to him, i.e. despite its actually accruing to another.
Payments in the latter circumstances may, however, then be recovered from the true recipient of the
income.
(c) An individual is not normally liable in respect of the tax debt of a “relation” as defined (commonly
a near relative) and nor is a company normally liable for the tax debt of an associated company. If,
however, such party transfers any asset to the other with the intention of avoiding recovery of tax, the
recipient is deemed to be chargeable with the tax up to the fair market value of the asset.
(d) In the case of a partnership each partner is personally liable for the tax on his taxable income,
including that from any other activities. If, however, after his other assets have been exhausted, a debt
remains in relation to his taxable income from the partnership, this may be collected from the partnership,
but only to the extent of the value of his interest in the partnership assets.
21.5.5 Minimum tax payable and refundable (s 76 and para 18(1)(a) of Sch 13)
If the tax payable for any year of assessment is less than $100 it is waived. Similarly, if an employee has
been subject to PAYE, any shortfall of less than $100 in deductions for a year is waived. Where an
employee’s PAYE deductions exceed his liability on subsequent assessment the excess is refundable
provided that it amounts to $100 or more.
21.5.6 Tax reserve certificates
The Tax Reserve Certificates Act [Chapter 23:10] and regulations thereunder authorise the issue by
Government of certificates in multiples of at least $10 up to a maximum holding of $250 000. They bear
interest at 21,75% annually for each complete month.
21.6 Offences (ss 81 to 86)
The circumstances in which an offence will have been committed under the Act include:
(a) failing or neglecting to submit a return;
(b) refusing or neglecting to give information, to attend an interview, to answer a question or to
produce documents;
(c) failing to reflect any portion of gross income or to disclose any material fact, either in one’s own
return or a return prepared on behalf of another person;
(d) failing to keep proper books and accounts. These must be in English and retained for six years
from the date of the last entry therein; retention of a photographic reproduction is, however, acceptable. (A
person in receipt only of salary, wages or other personal remuneration is not required to keep books);

168
(e) giving incorrect information or submitting incorrect returns or certificates;
(f) failing to file the memorandum and articles of association of a company;
(g) failing to give security, if required;
(h) making false entries in the books of account.
The possible fines and periods of imprisonment range from (i) $1 000, or three months, to (ii) $10 000, or
three years, or (iii) both such amount and period.
Chapter 22
Capital Gains Tax
Synopsis
22.1 Introduction
22.2 Calculation of the capital gain
22.3 Source
22.4 Gross capital amount
22.5 Exemptions
22.6 Deductions
22.7 Losses
22.8 Damage or destruction
22.9 Fair market price
22.10 Reconstructions, mergers, etc.
22.11 Transfer of individual’s immovable property
22.12 Sale under suspensive conditions
22.13 Instalment sale
22.14 Reduction in cost
22.15 Transfers between husband and wife
22.16 Principal private residence: “rollover” relief
22.17 Business property: “rollover” relief
22.18 Trusts
22.19 Tax avoidance
22.20 Double taxation agreements
22.21 Administration, tax collection, etc.
22.22 Examples
22.23 Capital gains withholding tax
22.1 Introduction
In terms of the Capital Gains Tax Act [Chapter 23:01] capital gains tax is chargeable on gains from a
source within Zimbabwe from the sale, or deemed sale, of “specified assets”. The latter term is defined in
section 2(1) as:
(1) immovable property; and
(2) any marketable security, again a defined term, i.e.:
(a) any bond capable of being sold in a share market or exchange; or
(b) (i) any debenture, share or stock; or
(iii) any right possessed by reason of a person’s participation in any unit trust;
whether or not capable of being sold in a share market or exchange.
The term “immovable property” is not defined. It therefore bears its ordinary meaning and relates to, for
example, land, buildings and other permanent structures. In terms of ITC 1610 (1997) 59 SATC 79 it does
not include incorporeals (in that instance an option over immovable property). It appears similarly,
therefore, to exclude mining claims. There are, however, provisions regarding assignment of deeds of sale.
The term “share” includes a member’s interest in a private business corporation.
No other sales of assets attract the tax.
In the case of taxable sales, liability arises regardless of the date of acquisition.
The rates of tax, which are fixed in the Finance Act, apply to sales by a “person” including a company and,
where appropriate, a trust. The general rate is 20% but there is a 10% rate on sales of: (i) any security listed
on the Zimbabwe Stock Exchange; and (ii) a “principal private residence” in the case of a sale by an
individual who is over 59 years of age.
Subject to exceptions stated below the tax becomes payable in respect of the year of assessment in which
the sale takes place. As will be seen, deferment of liability may arise, as in the case of the various elections

169
which are available. Such elections are required to be made at the time of submission of the returns
reflecting the sales.
22.2 Calculation of the capital gain
The pattern follows that of the Income Tax Act. “Gross capital amount” is the counterpart of “gross
income”, “capital amount” that of “income”, and “capital gain” that of “taxable income”. Many technical
definitions are effectively re-used. Various other general concepts, such as those of “accrued”, “incurred”
and “to the extent that”, which are common to both Acts, appear likely to be interpreted by reference to
principles recognised for income tax purposes, bearing in mind that it was observed in ITC 1395 (1985) 47
SATC 123, that the two statutes are in pari materia.
The first step is the establishment of the “gross capital amount” which is defined in section 8(1). This is the
total amount derived (or deemed to be derived) by a person in any year of assessment from a source within
Zimbabwe from the sale of specified assets. Amounts falling within the scope of income tax, such as from
the recoupment of capital allowances, and proceeds from disposals of speculative property or shares, are
excluded since the same amount might otherwise be taxed twice.
Secondly the “capital amount” is established by deducting from the gross capital amount any items which
are specified as exempt.
Finally the capital gain is established by deducting from the capital amount the amounts specified. These
are:
(a) the cost of the asset sold, i.e. expenditure on the acquisition or construction of specified assets
(other than expenditure allowed for income tax purposes, commonly by way of special initial, wear and
tear, and scrapping allowances);
(b) the cost of additions, etc., to such assets, i.e. expenditure on additions, alterations or improvements
(other than expenditure allowed, and the above allowances granted, for income tax purposes);
(c) a 15% annual allowance (5% in respect of sales before the year of assessment which commenced
on 1 April 1991 and thereafter 10% in respect of sales before the year of assessment which commenced on
1 April 1996. This remains relevant in some past instances of group reconstructions: see example 4 at
22.22);
With effect from 1 January, 2002 in respect of sales in the year of assessment or in any subsequent
year the annual allowance is increased to 30% with effect from the year of acquisition, etc.
(d) direct selling expenses such as agent’s or broker’s fees and advertising costs;
(e) bad debts in respect of amounts included in the taxpayer’s gross capital amount in the current or
any prior year;
(f) certain legal costs incurred in connection with an appeal to the courts against a capital gains tax
assessment.
Where a person’s total capital gain for a year of assessment, after the application of the above, is $5 000 or
less, such capital gain is generally reduced to nil and no liability arises for that year. (The exceptions are the
subject of 22.12 and 22.13.)
With regard to the annual allowance referred to above the effect is to allow not only the cost of the asset but
an additional 15% of such cost, calculated for each assessment year from that in which it was acquired to
that in which it is sold, part of a year thus counting as a year. (See further comment below.) The allowance
on the cost of additions runs from the year in which such additions, etc., are made. In the case of both the
original asset and the additions the allowance arises despite such cost having ranked for the various income
tax allowances available to, for example, industrialists, farmers and miners. (See examples at 22.22.) There
is no allowance, however, in the case of sales of shares in building societies.
As in the case of income tax the year of assessment ends on 31 December and the Commissioner similarly
may accept, for capital gains tax purposes, an accounting year other than 31 December. As will be apparent
from some of the examples at 22.22 the change in the tax year end from 31 March to 31 December, to
which reference is made at 18.1, may affect the calculation of the number of years for which the 15%
allowance is available, since the nine-month period from 1 April to 31 December 1997 counts as a year.
22.3 Source
Although, as indicated earlier, many technical definitions contained in the Income Tax Act are applicable
also for capital gains tax purposes, this does not apply to the concept of deemed source. Only gains from a
true source in Zimbabwe are liable.
The question of whether an amount was from a source here has been considered by the Special Court in the
context of the sale by an investment company of some of its shares. It was held that income tax principles
had to be applied and that the matter turned on where the seller carried on its “activities” (i.e. despite

170
holdings on capital account commonly being relatively passive. The court thus found that the seller in that
case was liable on a gain, on the sale of shares in a company registered on a foreign stock-exchange, which
had resulted from the carrying on of the seller’s activities in Zimbabwe: ITC 1395 (1985) 47 SATC 123.
The principle can be important in converse circumstances also, i.e. where a taxpayer whose activities are
conducted outside Zimbabwe sells shares in a Zimbabwe-registered company. On the basis of the court’s
approach the gain (or loss) would not be from a source in Zimbabwe.
In the case of immovable property it appears (i) that a gain on a sale of property situated in this country
would be from a source in Zimbabwe regardless of the base of operations of the seller and (ii) that a gain on
a sale of immovable property situated in another country would be from a source outside Zimbabwe despite
the seller’s operations being conducted in this country.
22.4 Gross capital amount
22.4.1 Disposals otherwise than by way of sale (s 8(2)(b))
Where a person disposes of a specified asset otherwise than by way of sale, such disposal is deemed to be a
sale at an amount which, in the opinion of the Commissioner, is equal to the fair market price of such asset.
Donations of such assets thus attract the tax in the donor’s hands despite there being no proceeds from
which to meet it.
Further, as indicated at 2.2, an exchange of a specified asset for a consideration other than cash constitutes
a realisation and thus a disposal. See, however, 22.10.2.
22.4.2 Expropriation of assets (s 8(2)(c))
Where a taxpayer’s specified asset is expropriated, it is deemed to have been sold by him for an amount
equal to the amount received by way of compensation.
22.4.3 Assets sold in execution of a court order (s 8(2)(d))
Where a taxpayer’s specified asset is sold in execution of the order of a court, the Act overcomes any
technical question as might arise in relation to accrual by deeming the sale price to accrue to the owner.
22.4.4 Maturity or redemption of stock (s 8(2)(e))
Where an amount accrues by reason of the maturity or redemption of a specified asset, such as the maturing
of a quoted bond which had been purchased at a discount, the asset is deemed to have been sold for that
amount. The provision applies also to any amount accruing in circumstances which are considered by the
Commissioner to be similar to those stated.
It is, of course, possible for a taxpayer to have acquired the bond with a view to deriving the profit on
maturity by way of his return on the investment. In this event such profit will be on revenue account and be
subject to income tax.
22.4.5 Foreign exchange differences (s 8(2)(a))
Where the proceeds of disposal of a specified asset are receivable in a foreign currency the amount received
in local currency is (unless the debt is settled immediately) likely to be affected by fluctuations in the
exchange rate. In this event it is the amount actually received in Zimbabwe currency which forms the gross
capital amount.
If the receipt and the accrual occur in different years of assessment, effect must be given to the increase or
reduction in the gross capital amount in the year of assessment in which the amount accrued. This rule as to
the year in which the difference is to be brought to account thus differs from that for income tax purposes.
22.4.6 Issue of shares
There is no provision deeming this to be a disposal.
22.4.7 Alternative: “gross income” for income tax purposes (s 8(1)(a))
In view of the lower rates of capital gains tax, but bearing in mind the possible presence of an assessed loss
for income tax purposes for a particular year of assessment, it is likely, as was observed at 4.1, that capital
gains tax liability on a disposal of a specified asset in that year will be less than had the profit been subject
to income tax.
The option of proving that such an amount constitutes “gross income”, placing it outside the scope of
capital gains tax, is not open to those classes of taxpayer, such as building societies, as are exempt from
income tax but subject to capital gains tax.
22.4.8 Deemed accruals (s 9)
Gains are deemed to accrue to a taxpayer despite their being reinvested, etc. (as considered for income tax
purposes at 2.7.1).
Partners’ gains are deemed to accrue on the partnership accounting date as discussed at 14.3.
22.5 Exemptions (s 10)
22.5.1  Receipts and accruals which are exempt from the tax include the following:

171
(a) amounts accruing on the realisation or distribution, by the executor of a deceased estate, of a
specified asset forming part of the estate. This effectively exempts deceased estates from capital gains tax
in the post-death period although additional estate duty might arise.
(Similarly no liability would have arisen in the period to date of death since there was no disposal
by the deceased.);
(b) proceeds from the sale of marketable securities being bonds or stock in respect of loans to
Government, local authorities or statutory corporations. Sales (including redemptions) of, for example,
Government stock, thus enjoy exemption;
(c) amounts accruing on the sale of specified assets by a person carrying on life insurance business.
(These fall within the formula for income tax purposes: see chapter 13);
(d) amounts accruing on the sale of related specified assets by a “licensed investor”. (See chapter 10
regarding export processing zones.);
(e) amounts accruing on the sale of related specified assets by an “industrial park developer”. (See
chapter 10.)
With effect from 1 January, 2002 the sale of any marketable security which is listed on the
Zimbabwe Stock Exchange is exempt.
22.5.2  Organisations which are exempt from the tax include the following:
(a) local authorities;
(b) friendly, benefit and medical aid societies;
(c) clubs, societies, institutes and associations which are prohibited from distributing income to
members;
(d) ecclesiastical, charitable and educational institutions of a public character;
(e) pension and benefit funds;
(f) any statutory corporation which is declared by the Minister, by notice in the Gazette, to be exempt
from income tax;
(g) trade unions;
(h) trusts of a public character.
As was observed earlier, exemption is not conferred on, for example, building societies.
22.6 Deductions
22.6.1 Expenditure on acquisition or construction (s 11(2)(a))
As indicated above, expenditure which is allowable for income tax purposes, does not qualify for
deduction. In the case of a sale of business premises, therefore, the question of the deductibility of
mortgage bond interest, on the grounds that it was part of the cost of acquisition or construction, does not
normally arise since the expenditure would have been allowed in arriving at taxable income. Such interest
on an individual’s private residence would not have been so allowed. It is deductible for capital gains tax
purposes, and ranks for the 15% annual allowance, in terms of a judgment of the Supreme Court
(unpublished in SATC reports). There appears to be no reason why the same principle should not apply to a
sale, on capital account, of shares acquired out of interest-bearing borrowings.
Incidental costs, such as legal or agency fees and transfer duty, constitute expenditure on acquisition.
22.6.2 Inherited assets (s 11(2)(a)(i))
Where the person disposing of a specified asset acquired it by way of inheritance he is deemed to have
expended the amount at which the specified asset was valued in the deceased estate concerned (even if the
death took place prior to the introduction of the tax on 1 August 1981). The valuation will thus affect not
only the level of any duty in the estate but also the ultimate capital gains tax liability of the beneficiary
upon sale by him.
22.6.3 Other acquisitions (s 11(2)(a)(ii))
Where the person disposing of a specified asset acquired it otherwise than by way of purchase or
inheritance, (commonly by way of donation) the amount to be deducted depends on the date of acquisition,
i.e.:
(a) prior to the introduction of the tax on 1 August 1981: the fair market value at the time the asset
was acquired;
(b) on or after 1 August 1981: any amount included in respect of such asset, for income tax or capital
gains tax purposes, in the hands of the person disposing of the asset.
22.6.4 Apportionment of expenditure (s 11(2)(a))
Expenditure is allowable “to the extent to which” it is incurred on a specified asset which is sold during the
year of assessment.

172
Thus where, for example, two pieces of land have been purchased for a lump-sum, and one is sold,
apportionment of the purchase price will arise.
22.6.5 Shares in a property-owning company (s 11(2)(b))
In the case of the sale of shares in a company which owns immovable property, any expenditure incurred
personally, by the seller of the shares, on additions or alterations to the property, is deemed to be
expenditure incurred by him on additions to the shares. This provision applies in practice mainly to private
town-house schemes and to other instances where an individual improves (and often occupies) a residence
owned by his company.
22.6.6 Deductions under more than one section (s 11(4))
Where a deduction could be allowed under more than one provision (whether in the same or in a different
year of assessment) the taxpayer is not entitled to claim that an amount be deducted more than once. He
must elect under which provision he wishes to claim the deduction.
22.6.7 Improvements to leased property (s 11(5))
Where the owner of immovable property has been subjected to income tax in respect of expenditure on
improvements carried out by a lessee (as in the case of obligatory improvements in terms of a lease
agreement: see 5.12) the owner is deemed to have incurred the expenditure which thus becomes deductible
upon his sale of the property.
22.6.8 Circumstances in which no deduction may be made (s 12)
No deduction is allowable in respect of expenditure on specified assets the sale of which is exempt from
tax. This provision prevents claims for deductions in respect of, for example, the cost of Government stock.
22.6.9 Foreign exchange differences (s 11(1))
The comments at 22.4.5 apply also, in principle, to expenditure incurred on the cost of a specified asset
which is sold. Effect is given to the adjustment (i.e. the increase or reduction in cost) in the year in which
the liability was incurred.
22.6.10 Proof of expenditure (s 25(2)(b))
As in the income tax context (see 21.4.2) in any objection or appeal the burden of proof that any receipt is
exempt, or expenditure deductible, is placed on the taxpayer. Having regard to the practicalities, from any
taxpayer’s point of view, full records should be kept, perhaps for many years.
Where a building has been demolished, prior to the sale of the property on which it stood, a claim for
deduction of its cost of construction could be difficult to sustain in the face of the argument that only the
land is now the “specified asset” which is being sold. See, however, the reference at 9.3.4 to ITC 1467
(1990) 52 SATC 28.
22.6.11 Bad debts (s 11(2)(e))
Where any proceeds prove to be irrecoverable, having been brought to account for capital gains tax
purposes, the amount of the bad debt is deductible in the seller’s hands. If the debt is proved to be bad only
in a subsequent year of assessment, the seller will merely have an assessed capital loss for that year (unless
he has gains at that stage from other disposals).
Unlike the income tax position there is no allowance for doubtful debts in the capital gains tax context.
22.7 Losses (s 11(3))
22.7.1 General
Any loss (computed in terms of the Act) incurred on disposal of an asset, is set off against any computed
gain on disposal of another asset in the same year of assessment. This applies regardless of whether the
assets are of kinds which attract different rates of tax; a loss on locally-quoted shares, for example, is set off
against a gain on immovable property, and vice versa.
Where in any year the deductions to which a taxpayer is entitled exceed his capital amount, this potentially
gives rise to an assessed capital loss for that year. As in the case of gains, however, generally a yardstick
comes into play, in that if the net loss on all transactions for the year is $5 000 or less it is reduced to nil.
(The exceptions are the subject of 22.12 and 22.13.)
Where an assessed capital loss is established it is carried forward to be set off against capital gains arising
in future years. Unlike the income tax position there is no time limit.
Capital losses are assessed separately from losses for income tax purposes and there is no set-off between
the two systems.
As in the income tax context there is usually no transfer of losses between taxpayers. There is an exception
where a company is converted to a private business corporation or vice versa.
22.7.2 Change in shareholding of a company with an assessed capital loss

173
As is the case with income tax assessed losses there are provisions against trafficking in assessed capital
losses. If the shares in a company change hands and the Commissioner considers that the change has been
made with a view to taking advantage of its loss he may disallow such loss. The same goes for changes in
the shareholding of a company controlling a company with an assessed capital loss.
22.7.3 Further aspects
Other areas of comparison with the income tax context (which are of limited interest) are:
(a) there is a similar prohibition on the carrying forward of an assessed capital loss in the case of
insolvency or assignment; but
(b) there is no similar ability to transfer a loss from a foreign-incorporated company.
22.8 Damage or destruction (s 13)
22.8.1 General
Where amounts accrue to a person in respect of damage to or destruction of a specified asset, commonly
insurance recoveries in respect of buildings which have been damaged or destroyed, capital gains tax
consequences arise as follows.
If the recovery does not exceed the original cost of the asset such cost is reduced accordingly, as is the
calculation of the 15% annual allowance, from the commencement of the year of assessment of the
recovery.
Where the recovery exceeds the cost a deemed sale arises and capital gains tax liability may be incurred at
that stage. Further liability may, of course, arise subsequently on any actual sale by the property owner.
See examples at 22.22.
22.8.2 Subsequent replacement or repair
Where a person, whose property has been damaged or destroyed, spends the amount recovered on
replacement or repair of the property within two years of the damage or destruction, the amount recovered
is not taxable.
If only a portion of the recovery is so spent proportionate liability arises.
If, in due course, the replacement building is sold, no deduction is allowable for the cost met out of the
untaxed recovery.
(See example at 22.22. The application of this section may, however, lead to strange results.)
22.9 Fair market price (s 14)
Where a person purchases a specified asset at a price in excess of the fair market price, or where he sells a
specified asset at less than the fair market price, the Commissioner may, for the purpose of determining the
capital gain or assessed loss of such purchaser or seller, determine the fair market price at which such
purchase or sale shall be taken into account. In ITC 1554 (1993) 55 SATC 115, it was held that the
Commissioner must produce some indication of an asset’s fair market price and that he could not arbitrarily
reallocate prices between assets even if the allocation by the parties to a transaction appeared unrealistic.
It is submitted that the Commissioner’s power in this respect is not automatic and that he would be subject
to the same limitations as have been imposed by the courts in the context of income tax: see 17.3.1.
As also observed at 17.3.1 where the Commissioner invokes the section the effect is double taxation in the
hands of the two parties.
22.10 Reconstructions, mergers, etc. (s 15)
22.10.1 Companies under the same control (s 15(1))
Provision is made regarding transfers of specified assets between companies under the same (merely
majority) control, in the course of group reconstructions, mergers and business operations which the
Commissioner accepts as being of a similar nature. The effect is that, subject to election by the transferor
and transferee companies effectively to use “tax values” for tax purposes regardless of the actual
consideration, no capital gains tax arises on such “internal” transfer of an asset but full liability (or loss) is
swept up in the hands of any company subsequently selling outside the group. See example at 22.22.
Section 15(1) is thus in this respect the counterpart, in relation to transfers of immovable property, to
paragraph 8(3) of the Fourth Schedule to the Income Tax Act, discussed at 9.5.
In relation to immovable property, as observed at 9.5, the election may be made under either Act or under
both. The decision will probably be affected mainly by the existence of any assessed loss for income tax
purposes or assessed capital loss, in the hands of the transferor, which could be set off if the election were
not exercised.
A company receiving any specified asset under an election carries an inherited liability which continues to
be latent if the shares in that company change hands. It is only when the asset is resold by that company

174
that the liability becomes actual in its hands, affecting the underlying interests of the new shareholders
(again as discussed at 9.5).
(The same election is available on conversion of a company to a private business corporation or vice versa.)
22.10.2 Share swaps (s 15(2))
As already observed an exchange of shares constitutes a disposal, which normally gives rise to capital gains
tax consequences.
Where, however, marketable securities such as shares, issued by a “company involved in the scheme” as
above, are exchanged for no cash consideration for shares, etc., “issued by another such company”, any
shareholder concerned also may make an election and thus not incur liability on the exchange forming part
of the scheme. This confers, on shareholders such as individuals or non-group companies, benefits which
would otherwise be enjoyed only by companies within the group concerned. There must firstly, however,
be a scheme of reconstruction, or merger, etc., with the tax-free exchange of, probably, minorities’ shares
following on the coat-tails. The Commissioner has in the past adopted a fairly broad approach to the
recognition of schemes such as mergers.
22.10.3 Winding up of foreign company (s 15(1))
There is a further deferment provision though it covers a rare occurrence. It favours a foreign company,
which has carried on its principal business in Zimbabwe, and which is about to be wound up in order to
transfer all of its assets and operations to another company (local or foreign), the sole consideration being a
proportionate issue of shares in the latter solely to members of the former. By election the assets may be
transferred, and shares exchanged, without liability arising at the time.
22.11 Transfer of individual’s immovable property (s 17)
Subject to conditions stated below, where an individual transfers any immovable property to a company
controlled by him (through holding a majority of its shares or otherwise) the parties may elect, regardless of
the actual price, to use a figure for tax purposes the effect of which is that no capital gains tax arises on
such transfer. Full liability (or loss) is swept up on any resale to any purchaser other than a company under
the same control.
The principle is the same as that considered at 22.10.1.
The conditions to be satisfied are that the individual must have used the property for the purposes of his
trade and that the Commissioner is satisfied that the company will continue to use it for the purposes of its
trade.
22.12 Sale under suspensive conditions (s 18)
22.12.1 General
The taxation of gains from sales of specified assets under suspensive conditions including, commonly, deed
of sale agreements, is governed by a formula, the effect of which is to spread such gain in instalments over
the period during which the proceeds fall due. (See example at 22.22.)
Where the capital amount of a person for any year of assessment includes any amount to which this section
relates such person is denied the benefit of the $5 000 minimum.
Sales of property (on capital account) are the commonest instances, but the provisions would apply also to
a sale of a marketable security under conditions whereby the passing of ownership was suspended.
22.12.2 Deed of sale: assignment of rights (ss  2(1)(def’n), 8(2)(f), 11(6) and 18(4))
A person who has been purchasing, for example, immovable property through the medium of a deed of
sale, may proceed to transfer his rights under the deed. He is:
(a) deemed to have sold the property for the full proceeds of the rights; and
(b) allowed to treat as a cost price the amount, (including interest) payable by him under the deed of
sale.
His capital gain is calculated in the usual way: see example at 22.22.
If the proceeds of the cession are receivable by instalments the gain is spread for tax purposes over the
relevant period.
22.13 Instalment sale (s 19)
Although (unlike section 18, referred to above) no specified formula is provided, it is understood that,
where the proceeds of sales of specified assets are receivable in instalments, the Commissioner taxes gains
from credit sales in a manner similar to that set down in that section, i.e. by spreading such gain over the
period during which the instalments fall due.
Again, in cases falling under this section, the taxpayer is denied the benefit of the $5 000 minimum.
22.14 Reduction in cost (s 20)

175
Probably the commonest application of this section is in relation to amounts received by a shareholder by
way of reduction in a company’s share capital. Where this occurs the cost of the shares in the hands of the
shareholder is reduced accordingly, as is the subsequent calculation of the 15% annual allowance, for the
purpose of deduction against any future sale of the shares by the shareholder. (See example at 22.22.)
Where, exceptionally, the amount received by way of reduction exceeds the cost of the shares, a deemed
sale arises and capital gains tax liability may be incurred at that stage. Further liability may, of course, arise
on any subsequent actual sale by the shareholder.
22.15 Transfers between husband and wife (s 16)
Where the ownership of any specified asset is transferred between husband and wife they may elect,
regardless of the actual price, to use a figure for tax purposes the effect of which is that no capital gains tax
arises on such transfer.
The principle is the same as that considered at 22.10.1. (See also 9.6 regarding the corresponding income
tax position.)
The provision applies also to a transfer of a principal private residence, by a husband to his former wife, in
terms of, for example, a divorce settlement.
22.16 Principal private residence: “rollover” relief (s 21)
(a) Any capital gain on a sale of an individual’s sole or main residence (called his principal private
residence) is, if he so elects, not subject to capital gains tax if an amount at least equal to the proceeds is
expended, before the end of the next assessment year, on the purchase or construction of a replacement
residence on land owned by him in Zimbabwe. This gain is “rolled over”, or deferred. If the amount
expended on the purchase or construction is less than the proceeds from the previous residence the
proportionate balance is subject to tax. These situations, and the “roll-over” consequences, are illustrated in
examples at 22.22.
(b) There is a lengthy definition of the term “principal private residence”, referred to below as his
“PPR”. A property qualifies only if it passes one or other of the stipulated tests.
Thus where it is established that the dwelling was the seller’s sole or main residence throughout
the period for which he owned it, that is sufficient. The same applies where the dwelling has been the
seller’s sole or main residence for a period of four years or more immediately before the sale. (The
Commissioner is given a discretion to accept a shorter period.) A dwelling will qualify also if it is
established that it has been regarded by the seller as his sole or main residence even though he has been
prevented from residing in it in consequence of his employment. The Commissioner is given a discretion to
accept other causes of absence.
Land, such as gardens and grounds surrounding the dwelling, and associated buildings such as
garages, storerooms, staff quarters, and, it appears, secondary dwelling houses such as relatives’ cottages,
also qualify, provided that they are not disposed of separately. There is an initial statutory limit of two
hectares on the size of the surrounding land but the Commissioner may accept a larger area.
The definition recognises also an allocation of a portion of a transfer price to a dwelling which
changes hands and which forms an undivided part of property which is not mainly residential. This would
appear to apply to, for example, a homestead on a farm owned by an individual.
Where, for example, a block of townhouses or flats is owned by a company, and units are
occupied as residences by shareholders, an individual’s acquisition or disposal of a share in the company,
conferring his right of occupation, is recognised as effectively representing a dwelling. The Commissioner
does not, however, accept that rollover relief applies to shares in cases where an individual’s holding
entitles him to occupation of an entire building, such as a house, owned by the company concerned.
(c) With regard to the requirement, referred to above, that expenditure on the replacement residence
be incurred by the end of the next following assessment year, a sale of the existing PPR will usually have
occurred prior to the acquisition or construction of another. An individual may find, however, that in order
not to miss an opportunity in the market, he first purchases the replacement while his sale is still
proceeding. It is understood that the Commissioner accepts that rollover relief remains available in these
circumstances provided that all the other requirements are satisfied.
(d) With regard to the comparison between the proceeds from the existing residence and the
expenditure on the replacement, the Commissioner accepts that the latter amount comprises both the initial
price and the cost of any additions effected within the stipulated period.
(e) A taxpayer would be entitled to rollover relief on more than one occasion in the event of his
having successive PPRs.

176
(f) Variations arise in cases where assets are held in the joint names of spouses. Where, for example,
a PPR owned by the husband is replaced by one jointly-owned, the Commissioner treats only one-half of
the husband’s proceeds of sale as having been expended, reducing the rollover relief accordingly.
(g) As will have been observed, “rollover-type” provisions such as this, and that considered at 22.17,
depend for their operation on the replacement of the asset. The concept differs therefore from some of the
other types of deferment provisions, which merely require conditions to be satisfied, namely those relating
to reconstructions, etc., of groups of companies (considered at 22.10), transfer of an individual’s business
property to his company (22.11) and transfers between husband and wife (22.15).
22.17 Business property: “rollover” relief (s 22)
The same principles as those considered at 22.16(a) and (c) to (g) apply to the sale to a third party by any
taxpayer, of immovable property previously used for the purposes of the latter’s trade.
Thus not only individuals but also other taxpayers, including companies, may elect to “roll over” a gain,
provided that expenditure on a replacement property is incurred within the time limit.
22.18 Trusts
The effect of section 2(2)(a) and (b) of the Act is that either the trust itself or a beneficiary may incur
liability on a gain arising from a disposal of trust assets. In the absence of a specific deeming provision
imposing liability on a donor, this rule applies not only to a trust created mortis causa but also, in the case
of an inter vivos trust, during the donor’s lifetime (unless the Commissioner succeeds in invoking the
general tax avoidance provisions). The position appears to be as follows.
(a) Where the beneficiary has no vested right to the capital (such as being dependent on the exercise
of the trustee’s absolute discretion) liability arises in the trust’s hands on the gross capital amount, the latter
being either the proceeds of a sale to a third party or (see 22.4.1) the market value of a distribution to a
beneficiary. The cost to the trust for tax purposes would depend on how and when it acquired the asset. The
principles at 22.6.2 and 22.6.3 refer. The amount so determined would rank for the 15% allowance in the
trust’s hands.
(b) Where the beneficiary has a vested right to the capital the distribution would not constitute a
disposal and no gross capital amount would arise in the hands of the trust.
22.19 Tax avoidance (s 29)
The Act incorporates, for capital gains tax purposes, the provisions of section 98 of the Income Tax Act
relating to tax avoidance. Comment on the latter and on various aspects of case law is contained in chapter
17.
The only capital gains tax case to have reached the Zimbabwe courts is Sommer Ranching (Pvt) Ltd v COT
(1999) 61 SATC 472.
The taxpayer company owned the entire share capital in a subsidiary which in turn owned farm land. The
taxpayer received an offer for the land, such offer being subject to the offeror retaining a right to purchase,
instead, the shares in the subsidiary. The subsidiary, having revalued the land, declared a dividend which
was credited to the taxpayer’s loan account. When the sale was concluded it was the taxpayer’s contention
that the sale was partly of the (taxable) shares but, substantially, of the (non-taxable) loan.
The Supreme Court, noting that the loan account had been specially created, considered, as had been found
in the Special Court in ITC 1631 (1998) 60 SATC 63, that to accord with the substantive nature of the
transaction the capital gain had to reflect the full proceeds as representing a sale of land. The Special
Court’s reduction of the penalty to 10% of the tax payable (the Commissioner having imposed 100%) was
also upheld.
(In the Special Court the President had found in favour of the taxpayer on the “normality” aspect of section
29. Since the Commissioner did not cross-appeal no finding on this ground by the Supreme Court was
needed.)
22.20 Double taxation agreements (s 28)
General comment on such agreements appears in chapter 20. In the Zimbabwe capital gains tax context
they are again of importance whenever a disposal of a specified asset, giving rise to a gain from a source
within Zimbabwe, is effected by a resident of another country with which Zimbabwe has an agreement
containing relevant articles.
In the case of the agreement with the United Kingdom, for example, (Article 14) Zimbabwe has
surrendered its taxing power over any gain on a disposal of shares, save where they are shares in a
company the assets of which consist principally of immovable property situated in Zimbabwe. In the latter
case (assuming that the gain is from a source within this country) Zimbabwe’s taxing power is retained, as
it is in the case of a gain on a disposal of immovable property itself situated in Zimbabwe. “Immovable

177
property” is defined as having the meaning which it has under the law of the country in which the property
is situated. The meaning is expanded to include also “property accessory to immovable property, livestock
and equipment used in agriculture and forestry” and various property-related rights.
Some other agreements are couched in substantially the same terms.
In certain agreements, however, Zimbabwe has retained its power to tax gains, from a source in Zimbabwe,
on any disposal of shares in any company resident in this country, and is not restricted as above.
22.21 Administration, tax collection, etc.
22.21.1 Administration (ss 23, 24, 25 and 27)
The Act incorporates provisions from the Income Tax Act, or enacts their equivalent, regarding matters
such as returns, assessments, penalties, representative taxpayers, objections, appeals and offences.
22.21.2 Payment of tax (s 26)
The Commissioner may fix the period or periods within which tax must be paid. He generally allows 30
days after the issue of the assessment but deferments or arrangements to pay by instalments may be agreed
by the Collector of Taxes on behalf of the Commissioner. Unlike the position for income tax there is no
system of annual payment dates.
Unless the Commissioner recognises special circumstances, interest is payable on overdue tax. The current
rate is 35% (S.I. 17 of 1995).
22.22 Examples
Example 1: Sale of farm, etc.
Mr E, who draws up his financial statements to 31 December each year, commenced farming in January
1993, when he bought farm land and existing buildings (the latter constituting “farm improvements” for
income tax purposes) for $5 000 000, of which $2 250 000 was agreed as the price of the land and $2
750 000 as that of the buildings. On 1 January, 1995, he spent $300 000 on building a dam and $500 000 on
the construction of further farm improvements which were immediately brought into use. In April 1996 he
purchased, for $7 500 000, a block of flats which had been constructed in 1973. On 1 May 1999, Mr E sold
his properties for $25 000 000 (after deduction of selling expenses).
His capital gains tax payable for 1999, assuming maximum income tax allowances for the years to the date
of sale, is calculated as follows.

Proceeds 25 000 000
Eliminate income tax recoupment on:
Farm improvements purchased:
W&T as below (1) 1 100 000
Farm improvements constructed:
SIA as below (1) 500 000 1 600 000
23 400 000
Less:
(i) Cost:
Farm land 2 250 000
Farm improvements purchased 2 750 000
Eliminate W&T allowed:
5% for 8 years (1) 1 100 000 1 650 000
Dam constructed 300 000
Eliminate Sch.7 deduction
allowed (1) 300 000 –
Farm improvements constructed 500 000
Eliminate SIA and W&T allowed (1) 500 000 –
Flats 7 500 000
(ii) 15% on:
$2 250 000 for 8 years 2 700 000
$2 750 000 for 8 years (2) 3 300 000
$300 000 for 6 years (2) 270 000
$500 000 for 6 years (2) 450 000
$7 500 000 for 4 years  4 500 000 22 620 000
Capital gain $780 000
Tax at 20% $156 000

178
Notes

(1) The income tax allowances and recoupments respectively are eliminated from both sides of the
capital gains tax equation.
(2) The annual 15% allowance is granted, for capital gains tax purposes, on the cost regardless of
whether it has ranked for income tax allowances.

Example 2: Sale of shares


Reverting to example 1, if Mr E had instead:
(i) formed a company with an issued share capital of $5 000 000 in January 1993, which
proceeded to purchase the farm and the flats and to incur the other expenditure set out above; and
(ii) disposed of his entire interest in the company on 1 May 1999 for (net) $25 000 000, being
$17 500 000 for the sale of his shares and $7 500 000 for the cession of the amount at which his loan
account then stood;
the capital gains tax payable by Mr E for 1999 would be calculated as follows.

Proceeds 25 000 000
Eliminate portion attributable to loan account 7 500 000
Attributable to shares 17 500 000
Less:
Cost 5 000 000
15% on $5 000 000 for 8 years 6 000 000 11 000 000
Capital gain $6 500 000
Tax at 20% $1 300 000

Note
Whether it is preferable for a 100% shareholder to sell his shares or to cause his company to sell its assets
depends on the facts in each case. Careful planning is necessary, bearing in mind also that a dividend
distributing the profit on the sale of the assets will, unless the recipient is a Zimbabwe company, suffer
withholding tax.
Example 3: Sale of mine
Reverting to the example at 12.12, the position was:

P/ship’s
Sale price cost price
Mine buildings  1 250 000 200 000
House occupied by Mr A 500 000 80 000
Mine manager’s house 400 000 70 000
Underground development 1 000 000 200 000
Plant and equipment 750 000 175 000
Reservoir 550 000 225 000
Claims 950 000 40 000
Goodwill 250 000 –
$5 650 000 $990 000

Assuming that the cost of the specified assets sold by Messrs A and B ranks for the annual allowance for 11
years the result in their hands would be as follows.

Proceeds 5 650 000
Eliminate:
Non-specified assets:
Plant and equipment 750 000
Claims 950 000
Goodwill 250 000
Recoupment for income tax purposes

179
on specified assets:
Buildings 1 250 000
Mr A’s house 93 750

Mine manager’s house 400 000


Underground development1 000 000
Reservoir 550 000 3 293 750 5 243 750
406 250
Less:
Cost: Total 990 000
Eliminate non-specified assets, i.e.
plant and equipment, claims (and goodwill nil) 215 000
775 000
Eliminate allowances for income
tax purposes:
Buildings 200 000
Mr A’s house 15 000
Mine manager’s house 70 000
Underground development200 000
Reservoir 225 000 710 000 (65 000)
15% on $775 000 for 11 years (1 278 750)
Assessed capital loss ($468 750 to each partner) ($937 500)

Example 4: Reconstruction scheme


A Ltd and B Ltd are fellow subsidiaries (of a family-held holding company) in the retail trade, the group’s
financial year ending on 31 December. A Ltd purchased a shop (which had been constructed in 1970) for
$450 000 in September 1992, and transferred it to B Ltd, in May 1994, for $1 250 000 (see note 1) both
companies exercising valid elections regarding a scheme of reconstruction (see note 2). In September 1994
B Ltd, contemplating long-term operations in the building, completed improvements at a cost of $4
000 000. In June 1999 B Ltd sold the property, in response to an unsolicited offer from a third party, for
$10 000 000. Selling expenses were $500 000 and B Ltd had an assessed capital loss brought forward from
the previous year of $750 000.
Tax payable by each company
In view of the election no liability would be incurred by A Ltd.
B Ltd

Proceeds 10 000 000
Less:
Cost to A Ltd 450 000
Improvements 4 000 000
15% on:
$450 000 for 8 years 540 000
$4 000 000 for 6 years 3 600 000
Selling expenses 500 000 9 090 000
Capital gain for the year 910 000
Less: Assessed capital loss brought forward 750 000
Capital gain $160 000
Tax at 20% $32 000

Notes

(1) In view of the election the price at which A Ltd transferred the property to B Ltd becomes
irrelevant for tax purposes and does not figure in the calculations. The transfer is deemed to have been

180
made at, effectively, “tax value”, i.e. A Ltd’s cost, plus the 15% allowance to date of transfer. See also,
however, note 2.
(2) The decision of the two companies to exercise the election in May 1994 would have been taken in
order to obtain the saving, at that stage, of the tax which would have been payable, had the companies
foregone the election, as follows.
A Ltd
Proceeds 1 250 000
Less: Cost 450 000
10 % (on sale prior to the 1996
year of assessment) on $450 000
for 3 years 135 000 585 000
Capital gain $665 000
Tax at 30% $199 500
The subsequent resale by B Ltd would have resulted in the following.
B Ltd
Proceeds 10 000 000
Less: Cost to B Ltd  1 250 000
Improvements 4 000 000
15% on:
$1 250 000 for 6 years 1 125 000
$4 000 000 for 6 years 3 600 000
Selling expenses 500 000 10 475 000
Capital loss for the year 475 000
Add: Assessed capital loss brought forward 750 000
Assessed capital loss $1 225 000

Example 5: Exchange of shares for no cash consideration


The shares in Propco Ltd, a property-owning company, have been owned as to 90% by OY Ltd, a wholly-
owned subsidiary of OZ Ltd, and as to 10% by Mr A. A transfer of Propco Ltd’s property to OY Ltd is
accepted as being in the course of a scheme of reconstruction of the OZ group and as qualifying for the
election in terms of section 15(1)(b).
If, as part of the scheme, Mr A were offered shares in OY Ltd in exchange for his shares in Propco Ltd, it is
submitted that the election in terms of section 15(2) to transfer at “tax value” would be available to him on
the grounds that he was exchanging a marketable security issued by “a company involved in the scheme”,
i.e. Propco Ltd, for a marketable security issued by “another such company”, i.e. OY Ltd. Thus:

Value of OY Ltd shares received by Mr A


in, say, October 1999 1 000 000
Less: Cost of his Propco Ltd shares
in, say, February 1997 200 000
15% thereon for 4 years 120 000 320 000
Actual capital gain $680 000
His deemed sale price however, if he so elects, is $320 000, resulting in no liability.

(As regards a future disposal by Mr A of his shares in OY Ltd the section makes no specific provision as to
their deemed cost.)
Example 6: Principal private residence: full “roll-over” relief
Mr L’s facts are that he purchased his principal private residence (“PPR”) in March 1984 for $30 000, sold
it in May 1989 for $90 000 and immediately purchased another for $120 000.
Had he so elected, no capital gains tax liability would have arisen in the year ended 31 March 1990 because
the amount expended on his second PPR exceeded the proceeds from the first. The gain which was “rolled
over”, i.e. suspended, would have been calculated as follows.

Proceeds 90 000
Less: Cost 30 000
5% (on sale prior to 1992 year of

181
assessment) on $30 000 for 7 years 10 500 40 500
Capital gain $49 500

Assuming that he had exercised the election in respect of the first PPR, if he had sold the second PPR in
November 1999 for $2 000 000 and not replaced it by 31 December 2000, his liability for the year ended 31
December 1999 would be calculated as follows.

Proceeds of second PPR


2 000 000
Less: Cost of second PPR 120 000
Reduced by gain on first PPR
rolled over 49 500
70 500
15% on $120 000* for 11 years 198 000 268 500
Capital gain $1 731 500
Tax payable at (assuming that Mr L was over 59 years of age at 1 January 1999) 10%

$173 150

* Understood to be the Commissioner’s approach to the provisions of section 21(3). The


effect is the same as if the rolled-over gain from the first PPR were added to the proceeds of the second
PPR. The taxable gain calculated in that manner would similarly have been $1 731 500, being $2 049 500
(i.e. $2 000 000 + $49 500) – $318 000 (i.e. $120 000 + $198 000).
Example 7: Principal private residence: partial “roll-over” relief
Mr M’s facts are that he purchased his principal private residence (“PPR”) in April 1980 for $20 000,
improved it in September 1990 at a cost of $80 000, sold it in November 1999 for $2 500 000 and
immediately purchased a smaller PPR for $2 000 000. He made the election for “roll-over” relief.
Partial capital gains tax liability would arise for the year ended 31 December 1999 because the amount
expended on his second PPR was less than the proceeds from the first. Such liability and the gain which
was “rolled over” would have been calculated as follows.

Proceeds 2 500 000
Less: Cost 20 000
Improvements 80 000
15% on:
$20 000 for 20 years 60 000
$80 000 for 10 years 120 000 280 000
Capital gain 2 220 000
Portion rolled over 1 776 000
Balance taxable in year ended 31 December 1999 $444 000
Tax payable at (assuming that Mr M was under 59
years of age at 1 January 1999) 20% $88 800

In the event of a resale of the second PPR, without replacement within the time allowed, the rolled-over
gain of $1 776 000 will be brought to account as illustrated in example 6.
Example 8: Suspensive sale
Mr G bought a house for $200 000 in January 1991 and sold it under an eight-year suspensive sale
agreement in July 1999 for $2 400 000. The purchaser paid a deposit of $300 000 and undertook to pay
$300 000 on each anniversary.
Mr G’s liability is calculated as follows.

Year ended 31December 1999


Proceeds 2 400 000
Less: Cost 200 000
15% on $200 000 for 10 years 300 000 500 000
Capital gain 1 900 000

182
Less: Allowance for instalments not yet due:
1 662 500
Taxable in that year $237 500
Year ended 31 December 2000
Previous year’s allowance deemed to be an accrual 1 662 500
Less: Allowance for instalments not yet due:
1 425 000
Taxable in that year $237 500

These calculations would normally proceed to the year ending 31 December 2006 at which stage the entire
gain of $1 900 000 would have been taxed.
Assuming, however, that the purchaser defaulted on his instalment which was due in July 2001, the sale as
a result being cancelled, no further allowances are granted and Mr G’s liability would be calculated as
follows.

Amounts received by him (in July 1999 and 2000) 600 000


Amounts taxed (in years ended 31 December 1999 and 2000) 475 000
Gain taxable in year ending 31 December 2001 $125 000

In the event of a seller having been taxed on an aggregate amount greater than that received by him the
excess constitutes an assessed capital loss which is available to be set-off against other capital gains in the
current or a future year.
(The above example uses an asset which did not qualify for income tax allowances. Where an asset which
has so qualified is the subject of a suspensive sale agreement, the gain is calculated in the usual way, with
the elimination of income tax recoupments and deductions from both sides of the equation as illustrated in,
for instance, example 1. The gain is then, as shown above, effectively taxed over the period of the
agreement.)
Example 9: Cession of deed of sale
On 31 October 1997 Mr PH purchased a flat, on a four-year deed of sale, for $600 000, payable by way of a
deposit of $120 000 and an instalment of $120 000 on each anniversary. Interest at 40% per annum was
charged on the outstanding balance. On 31 July 1999 he accepted a cash offer of $1 000 000 for his rights
under the deed, his creditor agreeing to the cession. Interest payable by Mr PH was $32 000 for the period
ended 31 December 1997, $184 000 for the year ended 31 December 1998 and $84 000 for the period
ended 31 July 1999.
Mr PH’s capital gains tax liability would be calculated as follows.

Proceeds of deemed sale of property 1 000 000


Less: Deemed cost (2 x $120 000) 240 000
Interest 300 000
15% on $120 000 for 3 years 54 000
15% on $120 000 for 2 years 36 000
15% on $32 000 for 3 years 14 400
15% on $184 000 for 2 years 55 200
15% on $84 000 for 1 year 12 600 712 200
Capital gain $287 800
Tax at 20% $57 560

Notes

(1) Had the proceeds of the cession been receivable in instalments Mr PH’s capital gain would have
been spread as in example 8 on suspensive sales.
(2) The Act contains no provisions regarding a subsequent sale of the property by the party to whom
Mr PH ceded his rights. It is understood that the Commissioner accepts that the consideration for the
cession, in the above case $1 000 000, forms part of the cost, along with subsequent instalments met by that
party.

183
Example 10: Damage or destruction
Part 1
NA Ltd, which leased five undivided hectares of land from Mr N, constructed a factory at a cost of
$4 000 000 and brought it into use, claiming SIA, in the year ended 31 March 1996. In the year ended 31
December 1999, at which stage the factory was insured for $9 000 000, it was destroyed by fire and the full
proceeds received.
NA Ltd’s capital gains tax liability for the year ended 31 December 1999 would be calculated as follows.

Insurance proceeds (being in excess of cost


resulting in a deemed sale) 9 000 000
Eliminate income tax recoupment 4 000 000
5 000 000
Less: Cost of specified asset 4 000 000
Eliminate capital allowances 4 000 000
15% on $4 000 000 –
for 5 years 3 000 000 3 000 000
Capital gain $2 000 000
Tax at 20% $400 000

Part 2
Had NA’s factory merely been damaged and the insurance recovery been $1 500 000 (resulting in no
deemed sale) the position with regard to the calculation of the 15% allowance against an actual future sale
would have been (assuming no income tax recoupment: see 9.3.4):
On $4 000 000 for four years from March 1996 to December 1998; and
On $2 500 000 for 1999 and subsequent years.
Part 3
NA Ltd might, instead, have constructed (similarly in the year ended 31 March 1996) a factory and, say, a
related warehouse on its own land acquired at the beginning of that year, with costs as follows:

Land 3 000 000
Factory 4 000 000
Warehouse 2 400 000
$9 400 000

The result of recovering $9 000 000 on destruction of the factory in the year ended 31 December 1999
would be similar in principle to that in Part 2 above. This is because the entire property, and not merely the
factory, constitutes the specified asset. There would again be no deemed sale and the 15% allowance
against a future sale would be calculated:
On $9 400 000 for four years from March 1996 to December 1998; and
On $400 000 for 1999 and subsequent years.
Example 11: Damage or destruction: repair or replacement
In the year ended 31 March 1996 NB Ltd acquired land at a cost of $2 000 000 and constructed thereon, at
a cost of $10 000 000, a factory which it brought into use in that year and on which SIA and thereafter
accelerated W&T was allowed. When the factory was destroyed by fire on 30 November 1999 $15 000 000
was recovered by way of insurance.
Assuming that those proceeds were wholly expended on a replacement factory during the year ending 31
December 2000 no liability would arise in the year ended 31 December 1999 as the 2000 expenditure
exceeds the original cost of the entire property.
In the event of a subsequent sale of the property, including the replacement factory, the $15 000 000 would
not rank as a cost for capital gains tax purposes.
Example 12: Reduction in share capital
In February 1987, Mr PQ subscribed at par for 10 000 shares of $2 each in a company listed on the
Zimbabwe Stock Exchange. In June 1993 and again in June 1998 the company’s share capital was reduced
by 50c. a share. On each occasion Mr PQ was repaid $5 000. As his only such transaction during the year
ending 31 December 1999, he sold the shares in July for $50 000.
His capital gains tax liability for the year ended 31 December 1999 would be calculated as follows.

184
Gross capital amount and capital amount 50 000
Less: Cost 20 000
Less reductions in share capital 10 000
10 000
15% on $20 000 for 7 years (February
1987 to 31 March 1993) 21 000
15% on $15 000 for 6 years (1 April
1993 to 31 December 1998) 13 500
15% on $10 000 for 2 years (1 January
1998 to December 1999) 3 000 47 500
Capital gain $2 500
Tax at 10% (But the gain is $5 000 or less) $        -

The cost of the shares on which the 15% allowance is calculated is deemed to be reduced with effect from
the commencement of the year of assessment in which any amount is received in reduction of capital.
22.23 Capital gains withholding tax (CGTA ss 22A to 22L: FA s 39)
(a) Such tax (“CGWT”) is required to be withheld, and paid to the Commissioner, at the rate of:
• 10% of the sale price in the case of immovable property;
• 5% of the sale price in the case of a marketable security listed on the Zimbabwe Stock
Exchange; and
• 10% of the sale price in the case of any other marketable security.
Persons who are termed “depositaries”, and who are required to register with the Commissioner,
are responsible for such withholding and payment. They include in particular:
• in the case of immovable property: conveyancers, legal practitioners and estate agents;
• in the case of marketable securities: stockbrokers and financial institutions.
Such payment is due by the end of the month following that in which payment of, commonly, the
proceeds of sale is made by the above parties to the seller.
There are provisions also whereby: (i) liability to withhold falls on any agent receiving an amount
from which a depositary has failed to withhold: (ii) liability falls on the seller where neither the depositary
nor an agent has withheld; (iii) personal liability falls on the depositary or agent; (iv) liability for a 15%
penalty arises.
(b) The application of the provisions relating to marketable securities have been suspended until such
date as the Minister may specify by notice in the Gazette.
(c) No CGWT needs to be withheld if either a depositary or the seller obtains in advance (subject to
various requirements) a clearance certificate from the Commissioner. Examples of circumstances where
this could arise include reconstructions of groups of companies and rollover relief in respect of a principal
private residence, both of which are considered earlier in this chapter.
(d) No withholding is required in respect of the sale of any specified asset which was concluded
before 1 January 1999. Nevertheless any payment of CGWT in respect of any such sale is stipulated as
having been validly made.
(e) CGWT is not a final tax on the seller. Once the seller’s actual liability is assessed in accordance
with the general provisions of the Capital Gains Tax Act (i.e. using all of the “normal” rules and rates) the
CGWT already paid is allowed as a credit; any excess is refundable and any shortfall payable. There is a
six-year time limit on claims for refund of overpayment.
(f) The Commissioner is empowered (subject to the abovementioned suspension) to require
stockbrokers and financial institutions to submit returns reflecting details of sales, though their agency, of
marketable securities.
(He is empowered also to require submission of returns by the Registrar of Deeds regarding
transfers of immovable property.)
(g) With effect from 1 January 2003 a restriction on transfers of specified assets is introduced. If the
above tax has not been withheld no registration may be effected by the Registrar of Deeds or the Registrar
of Companies respectively unless a certificate issued by the Zimbabwe Revenue Authority is lodged stating
that any related capital gains tax due has been paid.

185
Table of Cases

A  B  C  D  E  F  G  H  I  J  K  L  M  N  O  P  Q  R  S  T  U  V  W  X  Y  Z

A
A v COT (1954) 19 SATC 29 8.5
A v COT (1988) 50 SATC 159 2.4
"A" Company, COT v (1979) 41 SATC 59 8.7
AB v COT (SR) (1921) 1 SATC 77 4.3.1
Abbott v Philbin, HL (1960) 39 TC 82 5.2(e)
African Life Investment Corporation (Pty) Ltd v CIR (1969) 31 SATC 163 4.3.2
African Products Manufacturing Co Ltd, CIR v (1944) 13 SATC 164 8.2
Amalgamated Zinc (de Bavay's) Ltd v FC of T (1935) 54 CLR 295 7.4(k)
Anglo-Persian Oil Co Ltd v Dale (1931) 16 SATC 253 7.4(h), 7.6.1(b)
Armstrong v CIR (1938) 10 SATC 1 15.4.1, 15.4.3
Atherton v British Insulated and Helsby Cables Ltd, HL(1925) 10 TC 155 7.6.1(a)
Automated Business Systems (Pty) Ltd v CIR (1986) 48 SATC 41 9.2.3
Avenue Buildings (Pvt) Ltd, COT v (1963) 25 SATC 366 7.4(e)
B
B v COT (1955) 19 SATC 353 8.2
Barnato Holdings Ltd v SIR (1978) 40 SATC 75 4.5
BAT v COT (1995) 57 SATC 271 7.3(b)
Berea Park Avenue Properties (Pty) Ltd v CIR (1995) 57 SATC 167 4.3.3(c), 4.3.3(f), 4.4(b)
Berea West Estates (Pty) Ltd v SIR (1976) 38 SATC 43 4.3.5
Black, CIR v (1957) 21 SATC 226 3.1.3(a), 3.1.12
Booysens Estates Ltd, COT v (1918) 32 SATC 10 4.4(c)
Borstlap v SBI (1981) 43 SATC 195 7.6.2(a)
Bourke, Estate AG v CIR (1991) 53 SATC 86 4.7
Bowman N.O., CIR v (1990) 52 SATC 69 21.4.1
Boyd v CIR (1951) 17 SATC 366 3.1.2, 3.1.5
British United Shoe Machinery (SA) (Pty) Ltd, COT v (1964) 26 SATC 163 3.1.9
Brollo Properties (Pty) Ltd, G, CIR v (1994) 56 SATC 47 7.4(e)
Brookes Lemos Ltd v CIR (1945) 14 SATC 295 2.3
Brown Bros Ltd, CIR v (1955) 20 SATC 55 4.15
Building Contractors v COT (1941) 12 SATC 182 2.4
Burgess v CIR (1993) 55 SATC 185 7.4(a)
Burmah Steam Ship Co. Ltd v CIR (UK)(1931) 16 TC 67 4.7
Burman v CIR (1991) 53 SATC 63 7.6.2(e)
Butcher Bros (Pty) Ltd, CIR v (1945) 13 SATC 21 5.4(b), 21.4.2
C
C, COT v (1981) 43 SATC 9 9.2.4
C v COT (1984) 46 SATC 57 2.3
C v COT (1973) 35 SATC 241 8.25
Cactus Investments (Pty) Ltd v CIR (1999) 61 SATC 43 2.4
Cadac Engineering Works (Pty) Ltd, SIR v (1965) 27 SATC 61 7.6.1(b)
Cape Consumers (Pty) Ltd, C: SARS v (1999) 61 SATC 91 2.4
Cape Lime Company Ltd, SIR v (1967) 29 SATC 131 9.2.3

186
Carron Co, CIR (UK) v, HL (1968) 45 TC 18 7.6.1(b)
Cathcart, COT v (1964) 27 SATC 1 7.4(b)
Charkay Properties (Pty) Ltd, SIR v (1976) 38 SATC 159 9.2.4
CIR: See name of other party
Conhage (Pty) Ltd, CIR v (1999) 61 SATC 391 17.2.6
Constantia Heights (Pty) Ltd v SIR (1979) 41 SATC 77 4.3.3(a), 4.3.6
Contour Engineering (Pty) Ltd, C: SARS v (1999) 61 SATC 447 7.4(k)
Cooper v COT (1952) 18 SATC 259 8.7
COT: See name of other party
Cowley, CIR v (1960) 23 SATC 276 5.2(a)
Creative Productions (Pty) Ltd, CIR v (1999) 61 SATC 106 7.6.2(e)
C: SARS: See name of other party
CTBR: 4 CTBR (NS) Case 127 7.4(k)
D
D v COT (unreported) 3.1.12
Da Costa, CIR v (1985) 47 SATC 87 21.2.5
Datakor Engineering (Pty) Ltd, CIR v (1998) 60 SATC 503 5.11
De Beers Consolidated Mines Ltd v Howe (Surveyor of Taxes) (1906) 5 TC 198 18.6
De Beers Holdings (Pty) Ltd v CIR (1985) 47 SATC 229 7.6.2(d)
Deary v Deputy CIR (1920) 32 SATC 92 4.13
Delagoa Bay Cigarette Co. Ltd, CIR v (1918) 32 SATC 47 4.17, 7.4(c)
Delfos, CIR v (1933) 6 SATC 92 2.5
Drakensberg Garden Hotel (Pty) Ltd, CIR v (1960) 23 SATC 251 7.7(f)
Dunn: See William Dunn
Durban North Traders Ltd v CIR (1956) 21 SATC 85 4.3.3(e), 4.5
E
Eaton Hall (Pty) Ltd, SIR v (1975) 37 SATC 343 9.1
Edgars Stores Ltd v CIR (1988) 50 SATC 81 7.3(a)
Elandsheuwel Farming (Edms) Bpk v SBI (1978) 39 SATC 163 4.2, 4.4(e)
Elite Wholesale (Rhodesia) (Pvt) Ltd v COT (1955) 20 SATC 33 17.3.1
Ellis NO v COT (1995) 57 SATC 282 21.5.1
Elma Investments CC, CIR v (1996) 58 SATC 295 7.4(e)
Endeavour Foundation and UDC Ltd v COT (1995) 57 SATC 297 5.6.6, 6.2, 6.3
Epstein, CIR v (1954) 19 SATC 221 3.1.3(a), 3.1.3(e), 14.5
Esselmann v Secretary of Finance (1991) 53 SATC 285 21.3.2
Essential Sterolin Products (Pty) Ltd v CIR (1993) 55 SATC 357 3.1.3(a), 3.1.13
Evans Medical Supplies Ltd v Moriarty, HL (1957) 37 TC 540 4.14
F
Feldman Ltd, L v SIR (1969) 31 SATC 121 7.1
Ferera, COT v (1976) 38 SATC 66 17.2.6
Financier v COT (1950) 17 SATC 34 7.7(f)
First Merchant Bank Ltd, COT v (1997) 59 SATC 174 21.5.1
Flemming v KBI (1995) 57 SATC 73 8.2
Formscaff Investments (Pty) Ltd v CIR (1993) 55 SATC 251 9.2.3
Furniss v Dawson (1984) 1 All ER 530 (HL)17.1
G
G v COT (1972) 34 SATC 62 2.5
G, COT v (1981) 43 SATC 159 2.3
G, Estate, v COT (1964) 26 SATC 168 18.9
Gallagher, SIR v (1978) 40 SATC 39 17.2.3
General Motors SA (Pty) Ltd, CIR v (1981) 43 SATC 249 7.6.2(f)
Genn & Co. (Pty) Ltd, CIR v (1955) 20 SATC 113 2.3, 7.6.1(b)
George Forest Timber Co. Ltd, CIR v (1924) 1 SATC 20 4.3.2, 4.6, 7.6.1(a), 7.6.1(b)
Gerber v CIR (1989) 51 SATC 183 7.6.2(d)
Geustyn, Forsyth and Joubert, SIR v (1971) 33 SATC 113 17.2.5
Glen Anil Development Corporation Ltd v SIR (1975) 37 SATC 319 8.27(f)

187
Glenboig Union Fireclay Co. Ltd v IRC (1921) 12 TC 427 4.7
Godden v A Wilson's Stores (Holdings) Ltd (1962) 40 TC 161 7.4(k)
Golden Dumps (Pty) Ltd, CIR v (1993) 55 SATC 198 7.3(c)
Goodrick, CIR v (1942) 12 SATC 279 21.4.2
Greenband Properties (Pty) Ltd v CIR (1981) 43 SATC 151 4.4(e)
Guardian Assurance Company South Africa Ltd, CIR v (1991) 53 SATC 129 13.6
H
H v COT (1972) 34 SATC 39 17.2.6
H v COT (1981) 43 SATC 22 11.10.5
H Ltd v COT (1972) 34 SATC 57 8.27(f)
Hancock v General Reversionary & Investment Co. Ltd (1918) 7 TC 358 8.12
Heather v PE Consulting Group Ltd (1973) 48 TC 293 15.4.3
Hepker, COT v (1933) 6 SATC 87 4.3.1
Hersamer (Pty) Ltd, SIR v (1967) 29 SATC 53 9.2.3
Hicklin v SIR (1980) 41 SATC 179 17.2.4
Hiddingh v CIR (1941) 11 SATC 205 2.4
Hilda Holt Will Trust v CIR (1993) 55 SATC 1 15.4.3
Hilewitz, CIR v (1998) 60 SATC 86 7.6.2(e)
Hogan, KBI v (1993) 55 SATC 329 5.1
Holley v CIR (1947) 14 SATC 407 15.2.3
Hulett v CIR (1944) 13 SATC 58 2.7
I
Illovo Sugar Estates Ltd, CIR v (1951) 17 SATC 387 4.7
ITC 56 (1926) 2 SATC 178 3.2.3
ITC 77 (1927) 3 SATC 72 3.1.11
ITC 106 (1927) 3 SATC 336 3.1.4
ITC 124 (1928) 4 SATC 119 14.4.1
ITC 210 (1931) 6 SATC 59 5.6.2
ITC 239 (1932) 6 SATC 358 15.4.3
ITC 250 (1932) 7 SATC 46 3.1.4
ITC 321 (1935) 8 SATC 236 4.16
ITC 350 (1936) 9 SATC 69 14.4.2
ITC 414 (1938) 10 SATC 249 7.4(h)
ITC 417 (1938) 10 SATC 264 15.4.3
ITC 597 (1945) 14 SATC 264 4.9
ITC 617 (1946) 14 SATC 474 8.2
ITC 631 (1946) 15 SATC 100 9.3.3
ITC 636 (1947) 15 SATC 120 15.4.3
ITC 675 (1949) 16 SATC 238 2.3
ITC 678 (1949) 16 SATC 348 7.4(e)
ITC 679 (1949) 16 SATC 349 5.6.6
ITC 701 (1950) 17 SATC 108 5.2(a)
ITC 721 (1951) 17 SATC 485 4.16
ITC 740 (1951) 18 SATC 219 4.4(b)
ITC 761 (1952) 19 SATC 103 5.1
ITC 767 (1953) 19 SATC 206 5.5
ITC 774 (1953) 19 SATC 311 9.1
ITC 795 (1954) 20 SATC 107 9.3.3
ITC 807 (1955) 20 SATC 338 7.6.2(e)
ITC 808 (1955) 20 SATC 343 4.15
ITC 819 (1955) 21 SATC 71 7.6.1(b)
ITC 826 (1956) 21 SATC 189 3.1.2, 5.1
ITC 833 (1956) 21 SATC 324 7.7(b)
ITC 862 (1958) 22 SATC 301 4.3.3(e)
ITC 870 (1959) 23 SATC 79 7.6.2(e)
ITC 915 (1960) 24 SATC 219 8.2

188
ITC 919 (1959) 24 SATC 236 15.4.3
ITC 938 (1960) 24 SATC 375 5.2(a)
ITC 947 (1961) 24 SATC 463 6.3
ITC 955 (1961) 24 SATC 631 9.3.3
ITC 963 (1961) 24 SATC 705 17.2.2
ITC 976 (1961) 24 SATC 812 4.12
ITC 992 (1963) 25 SATC 129 7.6.1(b)
ITC 1017 (1963) 25 SATC 337 7.2
ITC 1019 (1962) 25 SATC 411 7.6.1(b)
ITC 1033 (1959) 26 SATC 73 2.7
ITC 1038 (1963) 26 SATC 123 8.2
ITC 1049 (1964) 26 SATC 229 9.2.3
ITC 1063 (1965) 27 SATC 57 7.6.1(b)
ITC 1088 (1966) 28 SATC 202 3.2.3
ITC 1094 (1966) 28 SATC 275 7.3(f)
ITC 1102 (1967) 29 SATC 28 3.1.11, 3.2.3
ITC 1103 (1967) 29 SATC 35 3.1.3(b)
ITC 1104 (1967) 29 SATC 46 3.1.11, 3.2.3
ITC 1105 (1967) 29 SATC 116 3.2.3
ITC 1116 (1968) 30 SATC 98 7.7(f)
ITC 1117 (1968) 30 SATC 130 7.3(a)
ITC 1121 (1968) 30 SATC 171 7.3(a)
ITC 1123 (1969) 31 SATC 48 8.27(f)
ITC 1124 (1969) 31 SATC 53 7.4(e), 7.6.1(b)
ITC 1127 (1969) 31 SATC 114 3.1.11, 3.2.3
ITC 1130 (1969) 31 SATC 148 3.2.3
ITC 1139 (1970) 32 SATC 83 7.4(e)
ITC 1173 (1972) 34 SATC 113 17.2.6
ITC 1174 (1972) 34 SATC 135 3.2.4
ITC 1177 (1972) 34 SATC 147 11.10.5
ITC 1185 (1972) 35 SATC 122 4.3
ITC 1190 (1973) 35 SATC 188 4.14
ITC 1194 (1973) 35 SATC 227 4.3.3(b)
ITC 1199 (1974) 36 SATC 16 7.4(d)
ITC 1206 (1974) 36 SATC 56 7.7(b)
ITC 1208 (1974) 36 SATC 80 4.11
ITC 1209 (1974) 36 SATC 84 8.27(f)
ITC 1212 (1974) 36 SATC 108 7.4(d)
ITC 1214 (1974) 36 SATC 155 7.4(j)
ITC 1221 (1974) 36 SATC 233 7.4(b)
ITC 1244 (1976) 38 SATC 7 4.3.5
ITC 1246 (1975) 38 SATC 21 9.2.3
ITC 1247 (1976) 38 SATC 27 9.2.3
ITC 1248 (1976) 38 SATC 35 9.2.4
ITC 1252 (1976) 38 SATC 176 21.2.6
ITC 1253 (1976) 38 SATC 181 7.7(b)
ITC 1264 (1977) 39 SATC 133 8.2
ITC 1267 (1977) 39 SATC 146 7.6.1(b)
ITC 1269 (1977) 40 SATC 60 16.4.5
ITC 1270 (1977) 40 SATC 65 7.6.2(e)
ITC 1271 (1977) 40 SATC 123 4.11
ITC 1277 (1978) 40 SATC 204 4.3.3(a)
ITC 1283 (1979) 41 SATC 36 18.9
ITC 1286 (1979) 41 SATC 98 9.2.1
ITC 1292 (1979) 41 SATC 163 7.4(g)
ITC 1313 (1980) 42 SATC 197 9.2.4

189
ITC 1315 (1980) 42 SATC 219 9.2.3
ITC 1330 (1981) 43 SATC 65 3.2.3
ITC 1331 (1981) 43 SATC 76 21.2.5
ITC 1333 (1981) 43 SATC 90 9.1
ITC 1334 (1981) 43 SATC 98 21.2.5
ITC 1338 (1981) 43 SATC 171 4.8
ITC 1341 (1981) 43 SATC 215 4.8
ITC 1342 (1981) 43 SATC 224 7.4(g)
ITC 1344 (1982) 44 SATC 19 7.6.2(e)
ITC 1346 (1982) 44 SATC 31 2.4
ITC 1347 (1982) 44 SATC 33 8.27(f)
ITC 1348 (1982) 44 SATC 46 4.4(b)
ITC 1350 (1982) 44 SATC 54 4.3.3(h)
ITC 1352 (1982) 44 SATC 64 18.2.5
ITC 1354 (1982) 44 SATC 118 4.3.3(f)
ITC 1358 (1982) 44 SATC 155 14.4.2
ITC 1359 (1982) 44 SATC 165 5.2(a)
ITC 1363 (1983) 45 SATC 17 7.6.2(e)
ITC 1364 (1983) 45 SATC 23 17.2.6
ITC 1365 (1983) 45 SATC 27 7.6.1(b)
ITC 1366 (1983) 45 SATC 33 13.6
ITC 1372 (1983) 45 SATC 180 5.4(b)
ITC 1373 (1983) 45 SATC 189 11.1
ITC 1374 (1983) 45 SATC 196 5.6.2
ITC 1378 (1983) 45 SATC 230 2.4
ITC 1381 (1984) 46 SATC 75 7.4(f)
ITC 1385 (1984) 46 SATC 111 7.4(g)
ITC 1386 (1984) 46 SATC 116 5.2(a)
ITC 1389 (1985) 47 SATC 57 11.10.2
ITC 1391 (1985) 47 SATC 66 4.3.3(b)
ITC 1392 (1985) 47 SATC 75 14.4.2
ITC 1394 (1985) 47 SATC 119 7.7(m)
ITC 1395 (1985) 47 SATC 123 3.1.12, 22.2, 22.3
ITC 1403 (1985) 47 SATC 226 7.6.2(c)
ITC 1406 (1986) 48 SATC 12 4.4(e)
ITC 1408 (1986) 48 SATC 21 8.2
ITC 1410 (1986) 48 SATC 32 7.7(b)
ITC 1411 (1986) 48 SATC 35 2.4
ITC 1413 (1986) 48 SATC 167 15.4.3
ITC 1415 (1986) 48 SATC 179 2.4
ITC 1419 (1987) 49 SATC 45 7.6.2(e)
ITC 1423 (1987) 49 SATC 85 21.4.2
ITC 1424 (1987) 49 SATC 99 11.10.5
ITC 1427 (1988) 50 SATC 25 4.11
ITC 1428 (1988) 50 SATC 34 7.4(e)
ITC 1430 (1988) 50 SATC 51 21.2.5
ITC 1432 (1988) 50 SATC 70 4.3.3(a)
ITC 1433 (1988) 50 SATC 73 7.6.1(b)
ITC 1434 (1988) 50 SATC 77 7.4(f)
ITC 1435 (1988) 50 SATC 117 9.3.4
ITC 1436 (1988) 50 SATC 122 4.4(b)
ITC 1437 (1988) 50 SATC 154 7.7(b)
ITC 1441 (1989) 51 SATC 10 18.1, 18.2.5
ITC 1442 (1989) 51 SATC 19 21.2.6
ITC 1444 (1989) 51 SATC 35 7.3(a)
ITC 1450 (1989) 51 SATC 70 15.4.3

190
ITC 1451 (1989) 51 SATC 93 4.12
ITC 1452 (1989) 51 SATC 97 7.6.1(b)
ITC 1456 (1989) 51 SATC 125 9.3.3
ITC 1457 (1989) 51 SATC 131 8.2
ITC 1464 (1989) 51 SATC 205 5.5
ITC 1465 (1990) 52 SATC 1 9.2.3
ITC 1467 (1990) 52 SATC 28 9.3.4, 22.6.10
ITC 1471 (1990) 52 SATC 96 4.4(b)
ITC 1480 (1990) 52 SATC 276 21.2.3
ITC 1481 (1990) 52 SATC 285 4.3.5
ITC 1482 (1990) 52 SATC 298 4.3.3(a)
ITC 1485 (1990) 52 SATC 337 7.3(b)
ITC 1490 (1991) 53 SATC 108 7.4(d)
ITC 1491 (1991) 53 SATC 115 3.1.13
ITC 1494 (1991) 53 SATC 206 4.3.2, 11.8
ITC 1495 (1991) 53 SATC 216 7.3(a), 21.2.6
ITC 1496 (1991) 53 SATC 229 7.3(b), 17.2.5
ITC 1504 (1991) 53 SATC 349 7.4(e)
ITC 1507 (1991) 53 SATC 425 4.4(e)
ITC 1508 (1991) 53 SATC 442 4.11
ITC 1509 (1992) 54 SATC 18 4.3.1, 4.3.3(f)
ITC 1510 (1992) 54 SATC 30 4.10
ITC 1511 (1992) 54 SATC 39 9.2.1, 9.2.2, 9.2.7
ITC 1512 (1992) 54 SATC 45 4.8
ITC 1513 (1992) 54 SATC 56 17.2.6
ITC 1514 (1992) 54 SATC 65 8.27(f)
ITC 1516 (1992) 54 SATC 101 7.3(a)
ITC 1517 (1992) 54 SATC 109 7.4 (e)
ITC 1518 (1992) 54 SATC 113 7.4(j), 17.2.6, 21.2.6
ITC 1521 (1992) 54 SATC 175 7.4(e)
ITC 1524 (1992) 54 SATC 201 8.25
ITC 1525 (1992) 54 SATC 209 4.6
ITC 1528 (1992) 54 SATC 243 7.6.1(b)
ITC 1529 (1992) 54 SATC 252 18.9
ITC 1530 (1992) 54 SATC 261 7.4(j)
ITC 1535 (1992) 54 SATC 356 20.7, 21.2.1
ITC 1537 (1992) 54 SATC 383 8.2
ITC 1538 (1992) 54 SATC 387 9.1, 9.3.3
ITC 1540 (1992) 54 SATC 400 21.2.5
ITC 1541 (1992) 54 SATC 408 4.3.3(c), 4.4(e)
ITC 1543 (1992) 54 SATC 446 4.2, 4.3.6
ITC 1545 (1992) 54 SATC 464 2.3, 4.17, 7.3(c), 21.4.2
ITC 1546 (1992) 54 SATC 477 9.3.2
ITC 1547 (1993) 55 SATC 19 4.3.3(h)
ITC 1549 (1993) 55 SATC 31 4.8
ITC 1553 (1993) 55 SATC 105 7.4(e)
ITC 1554 (1993) 55 SATC 115 17.3, 22.9
ITC 1557 (1993) 55 SATC 218 2.4, 4.7
ITC 1559 (1993) 55 SATC 286 9.2.3
ITC 1563 (1993) 55 SATC 315 21.2.6
ITC 1565 (1994) 56 SATC 18 6.2(b)
ITC 1566 (1994) 56 SATC 34 7.4(e)
ITC 1570 (1994) 56 SATC 120 15.4.1
ITC 1577 (1994) 56 SATC 236 21.2.5
ITC 1580 (1994) 56 SATC 275 4.3.3(d)
ITC 1582 (1995) 57 SATC 27 17.2.6, 21.2.6

191
ITC 1583 (1995) 57 SATC 58 7.4(e)
ITC 1585 (1995) 57 SATC 81 3.1.3(a), 3.2.3
ITC 1587 (1995) 57 SATC 97 7.3(b)
ITC 1588 (1995) 57 SATC 148 7.3(b)
ITC 1589 (1995) 57 SATC 153 7.2, 7.4(e)
ITC 1590 (1995) 57 SATC 160 5.2(a), 5.6.1, 5.6.2
ITC 1591 (1995) 57 SATC 212 9.2.3
ITC 1592 (1995) 57 SATC 247 7.4(g)
ITC 1594 (1995) 57 SATC 259 7.6.1(b), 21.2.6
ITC 1596 (1995) 57 SATC 341 9.2.5
ITC 1597 (1996) 58 SATC 27 4.3.3(c)
ITC 1598 (1996) 58 SATC 35 7.6.1(b)
ITC 1600 (1996) 58 SATC 131 7.4(a)
ITC 1601 (1996) 58 SATC 172 8.22
ITC 1603 (1996) 58 SATC 212 7.4(e)
ITC 1604 (1996) 58 SATC 263 7.4(e), 7.6.1(b)
ITC 1606 (1996) 58 SATC 328 17.1
ITC 1607 (1996) 58 SATC 340 4.12
ITC 1608 (1997) 59 SATC 63 4.2, 4.3.4, 4.10, 6.3
ITC 1611 (1997) 59 SATC 126 17.1
ITC 1612 (1997) 59 SATC 180 4.10
ITC 1615 (1997) 59 SATC 264 5.5
ITC 1616 (1997) 59 SATC 272 4.2
ITC 1618 (1997) 59 SATC 290 2.4, 17.4
ITC 1619 (1997) 59 SATC 309 9.1
ITC 1620 (1997) 59 SATC 316 7.4(e)
ITC 1621 (1997) 59 SATC 324 7.6.2(e)
ITC 1624 (1997) 59 SATC 373 2.3
ITC 1625 (1997) 59 SATC 383 7.4(e), 17.2.6
ITC 1626 (1998) 60 SATC 17 5.6.4
ITC 1629 (1998) 60 SATC 53 11.1
ITC 1631 (1998) 60 SATC 63 22.19
ITC 1634 (1998) 60 SATC 235 2.4(b), 5.10
ITC 1638 (1998) 60 SATC 423 4.3.3(f)
ITC 1641 (1998) 60 SATC 493 7.2
ITC 1643 (1999) 61 SATC 12 7.4(e)
ITC 1645 (1999) 61 SATC 31 2.4
ITC 1648 (1999) 61 SATC 58 7.6.1(b)
ITC 1650 (1999) 61 SATC 72 7.6.2(e)
ITC 1653 (1999) 61 SATC 120 7.4(g)
ITC 1658 (1999) 61 SATC 231 21.2.5
ITC 1659 (1999) 61 SATC 239 4.3.3(h)
ITC 1660 (1999) 61 SATC 249 8.22
ITC 1661 (1999) 61 SATC 353 7.4(b)
ITC 1662 (1999) 61 SATC 357 5.8(f)
ITC 1663 (1999) 61 SATC 363 6.3(vii)
ITC 1665 (1999) 61 SATC 413 21.2.6
ITC 1667 (1999) 61 SATC 439 8.22
ITC 1669 (1999) 61 SATC 479 2.4, 9.1, 17.2.6, 21.2.5(b), (c)
ITC 1670 (2000) 62 SATC 34 2.4
ITC 1671 (2000) 62 SATC 39 21.2.6
ITC 1672 (2000) 62 SATC 47 7.4(k), 17.4
J
J v COT (1993) 55 SATC 62 7.4(g)
Jeffrey v Rolls-Royce Ltd, HL (1962) 40 TC 443 4.14
Joffe and Co. (Pty) Ltd, A.S., v CIR (1946) 13 SATC 354 7.4(b)

192
John Bell and Co. (Pty) Ltd v SIR (1976) 38 SATC 87 4.4(a)
Johnstone and Co. Ltd, W.F., v CIR (1951) 17 SATC 235 7.4(h)
Jones v CIR (UK) (1920) 7 TC 310 4.13
K
K v COT (1993) 55 SATC 276 3.2.3
KBI: See name of other party
Kempton Furnishers (Pty) Ltd, SIR v (1974) 36 SATC 67 8.7
King, CIR v (1947) 14 SATC 184 17.2.2, 17.2.6
Knuth and Industrial Mouldings (Pty) Ltd, C:SARS v (2000) 62 SATC 65 4.4(b), 4.6, 4.11
Kohler v CIR (1949) 16 SATC 312 2.7
Kristiansen (Pvt) Ltd, F, COT v (1995) 57 SATC 238 21.5.1
Kuttel, CIR v (1992) 54 SATC 298 18.5
L
L v COT (1951) 18 SATC 137 5.1
L v COT (1975) 37 SATC 116 17.2.6
L v COT (1992) 54 SATC 91 7.4(f), 7.7(b)
Lace Proprietary Mines Ltd v CIR (1938) 9 SATC 349 4.3.6
Ladysmith (Erf 3183/1) and Another v CIR (1996) 58 SATC 229 5.5, 17.4
Lamb v CIR (1955) 20 SATC 1 3.1.5
Lambson v CIR (1946) 14 SATC 57 15.2.3
Land Dealing Company v COT (1959) 22 SATC 310 4.4(c), 5.8(e)
Lever Bros and Unilever Ltd, CIR v (1946) 14 SATC 1 3.1.1, 3.1.7
Levy, COT (SR) v (1952) 18 SATC 127 4.2, 4.5
Liquidator, Rhodesia Metals Ltd v COT (1938) 9 SATC 363 3.1.6, 3.1.10
Local Investment Co. v COT (1958) 22 SATC 4 7.2
Lourens Erasmus (Edms) Bpk, SBI v (1966) 28 SATC 233 9.2.3
Louw, CIR v (1983) 45 SATC 113 17.2.5
M
M Ltd v COT (1958) 22 SATC 27 3.1.3(a), 3.1.12
Mabotsa, KBI v (1993) 55 SATC 98 21.2.5
Maguire v COT (1966) 28 SATC 146 2.5
Malan v KBI (1983) 45 SATC 59 4.5
Malcolmess Properties (Isando) (Pty) Ltd, CIR v (1991) 53 SATC 153 4.4(e)
Manganese Metal Company (Pty) Ltd, CIR v (1996) 58 SATC 1 5.5, 7.6.1(b)
Matla Coal Ltd v CIR (1986) 48 SATC 223 4.3.3(i)
McFarland, COT, Federation of Rhodesia, v (1965) 27 SATC 15 20.5.1
Metal Pressing and Enamelling Co (Pvt) Ltd v COT (1970) 32 SATC 229 8.25
Meyerowitz v CIR (1963) 25 SATC 287 17.2.6
Middelman, CIR v (1990) 52 SATC 323 4.3.3(d), 4.5
Millin v CIR (1928) 3 SATC 170 3.1.10
Milstein, CIR v (1942) 11 SATC 279 5.1
Mining and Manufacturing Co. v COT (1944) 13 SATC 146 3.1.3(d)
Modified Investments (Pty) Ltd, CIR v (1981) 43 SATC 257 4.4(d)
Moodie v CIR, Transkei, and Another; CIR, Transkei, and Another v
Moodie and Another (1993) 55 SATC 164 2.4
Mooi v SIR (1971) 34 SATC 1 2.4, 5.2(e)
Morgan v Tate and Lyle Ltd (1954) HL 35 TC 367 7.4(b)
Morrison v CIR (1949) 16 SATC 377 4.10
Munro, Estate v CIR (1925) 1 SATC 163 15.4.1, 15.4.3
Murray v Goodhews (1977) 52 TC 86 5.2(a)
Musker v English Electric Co. Ltd, HL (1964) 41 TC 556 4.14
Myerson, CIR v (1947) 14 SATC 300 5.4(b)
N
Nasionale Pers Bpk, KBI v (1986) 48 SATC 55 7.3(a)
Natal Estates Ltd v SIR (1975) 37 SATC 193 4.3.6, 4.4(c)
Natal Laeveld Boerdery BK v KBI (1998) 60 SATC 81 7.4(e)

193
National Co-operative Dairies Ltd v CIR (1992) 54 SATC 1 9.2.3
N'Changa Consolidated Copper Mines Ltd, COT v (1964) 26 SATC 37 7.6.1(b)
Nedbank Ltd, CIR v (1986) 48 SATC 73 4.3.3(g)
Nel, CIR v (1997) 59 SATC 349 4.3.4
Nemojim (Pty) Ltd, CIR v (1983) 45 SATC 241 7.6.2(d)
New State Areas Ltd v CIR (1946) 14 SATC 155 7.6.1(a), 7.6.1(b)
Newfield, COT v (1970) 32 SATC 157 14.3.1
Niko, CIR v (1940) 11 SATC 124 4.3.3(h)
Nussbaum, CIR v (1996) 58 SATC 283 4.3.2
O
Ovation Recording Studios (Pty) Ltd v CIR (1990) 52 SATC 163 9.2.3
Ovenstone v SIR (1980) 42 SATC 55 2.7
Overseas Trust Corporation Ltd v CIR (1926) 2 SATC 71 3.1.3(a), 3.1.12, 4.3.2
Oxford, Chancellor, Masters and Scholars of the University of, v CIR
(1996) 58 SATC 45 6.2
P
P, COT v (1978) 40 SATC 215 4.5
Palabora Mining Company Ltd v SIR (1973) 35 SATC 159 7.6.1(b)
Parkwell House (Pvt) Ltd, COT v (1972) 34 SATC 105 8.15, 9.1
Paul, CIR v (1956) 21 SATC 1 4.4(b)
Pick'n Pay Employee Share Purchase Trust, CIR v (1992) 54 SATC 271 4.3.2, 15.4.3
Plate Glass and Shatterprufe Industries (Finance Company) (Pty) Ltd v SIR
(1979) 41 SATC 103 7.6.2(e)
Polonsky, CIR v (1942) 12 SATC 11 15.4.1
Port Elizabeth Electric Tramway Co. Ltd v CIR (1936) 8 SATC 13 7.3(a), 7.4(b)
Processing Enterprises (Pvt) Ltd, COT v (1975) 37 SATC 109 9.2.3
Producer v COT (1948) 15 SATC 405 7.7(f)
Professional Suites Ltd v COT (1960) 24 SATC 573 5.5
R
R, COT v (1966) 28 SATC 115 3.1.2, 5.1
R Ltd and K Ltd v COT (1983) 45 SATC 148 17.2.3, 17.2.6, 17.3.3
Ramsay, WT, v IRC (1981) All ER 865 (HL) 17.1
Rand Mines (Mining & Services) Ltd v CIR (1997) 59 SATC 85 7.6.1(b)
Raubex Konstruksie (Edms) Bpk, Regering van die Republiek van Suid Afrika v
(1994) 56 SATC 167 21.3.3
Realisation Co. v COT (1950) 17 SATC 139 4.3.5
Reliance Land and Investment Co. Ltd v CIR (1946) 14 SATC 47 21.4.2
Relier (Pty) Ltd v CIR (1998) 60 SATC 1 5.5, 17.4
Rendle, COT v (1965) 26 SATC 326 7.3(d), 7.4(b)
Rex Tearoom Cinema (Pty) Ltd v CIR (1946) 14 SATC 76 5.5
Rezende Gold and Silver Mines (Pvt) Ltd, COT v (1975) 37 SATC 39 4.16
Rhodesia Metals, Liquidator v COT (1938) 9 SATC 363,
and Rhodesia Metals Ltd (in liquidation) v COT (1940) 11 SATC 244 2.6, 3.1.6, 3.1.10
Rhodesia Railways v CIT Bechuanaland (1933) 6 SATC 225 8.2
Rhodesia Railways Ltd and Others v COT (1925) 1 SATC 1338.2
Richards Bay Iron & Titanium (Pty) Ltd and Another v CIR (1996) 58 SATC 55 5.8(g)
Richmond Estates (Pty) Ltd, CIR v (1955) 20 SATC 355 4.4(d)
Ridgeway Hotel Ltd, COT v (1961) 24 SATC 616 5.5
Rile Investments (Pty) Ltd, SIR v (1978) 40 SATC 135 4.5
Robin Consolidated Industries Ltd v CIR (1997) 59 SATC 1997.4(k)
Rolls-Royce Ltd, Jeffrey v, HL (1962) 40 TC 443 4.14
Ropty (Edms) Bpk v SBI (1981) 43 SATC 141 4.5
Rosen, SIR v (1971) 32 SATC 249 15.4.3
Royal Insurance Co. Ltd v Stephen (1928) 14 TC 22 2.2
Ruhamah Property Company Ltd v FC of T, 41 CLR 148 4.3.6
S

194
S (Pvt) Ltd v COT (1969) 31 SATC 77 8.27(f)
SA Marine Corporation Ltd v CIR (1955) 20 SATC 15 4.15
SA Mutual Unit Trust Management Company Ltd, CIR v (1990) 52 SATC 205 7.6.2(d), 21.2.6
Sacks v CIR (1946) 13 SATC 343 14.3.1
Safranmark (Pty) Ltd, SIR v (1981) 43 SATC 235 9.2.3
SBI: See name of other party
Seeff Properties CC v CIR (1998) 60 SATC 407 7.6.1(b)
Sentra - Oes Koöperatief Bpk v KBI (1995) 57 SATC 109 7.6.2(e), 13.6
Shapiro, CIR v (1928) 4 SATC 29 7.4(e)
Shein, COT v (1958) (FC) 22 SATC 12 3.1.11
Silverglen Investments (Pty) Ltd, SIR v (1968) 30 SATC 199 2.5
SIR: See name of other party
Smith v CIR (1968) 30 SATC 35 7.6.1(b)
Smith, DG, CIR v (1998) 60 SATC 397 7.4(e)
Snyman en Webster, KBI v (1994) 56 SATC 149 4.13
Solaglass Finance Co. (Pty) Ltd v CIR (1991) 53 SATC 1 7.6.2(e)
Sommer Ranching (Pvt) Ltd v COT (1999) 61 SATC 472 22.19
Southern v Borax Consolidated Ltd (1940) 23 SATC 597 7.6.1(b)
Standard Bank of SA Ltd, CIR v (1985) 47 SATC 179 7.7(f)
Stander v CIR (1997) 59 SATC 212 5.2(a), 5.6.1
Stephan v CIR (1919) 32 SATC 54 4.11, 18.9
Stone v SIR (1974) 36 SATC 117 7.6.2(e)
Stott, CIR v (1928) 3 SATC 253 4.4(b)
Strathmore Exploration and Management Ltd, CIR v (1956) 20 SATC 375 4.3.6, 4.12
Strathmore Holdings Ltd v CIR (1959) 22 SATC 203 4.3.3(c)
Strong & Co. of Romsey Ltd v Woodifield (Surveyor of Taxes) HL (1906) 5 TC 215 7.4(a)
Struben Minerals (Pty) Ltd, SIR v (1966) 28 SATC 248 4.3.5, 4.16
Sub-Nigel Ltd v CIR (1948) 15 SATC 381 1.4, 7.1, 7.3(e)
Sunnyside Centre, CIR v (1996) 58 SATC 319 7.4(e)
T
T v COT (1978) 40 SATC 179 4.3.3(e), (f), 5.8(d)
Taeuber and Corssen (Pty) Ltd v SIR (1975) 37 SATC 129 4.8
Taxpayer v COT Botswana (1980) 43 SATC 118 2.4
Ticktin Timbers CC v CIR (1999) 61 SATC 399 7.4(e)
Timberfellers (Pty) Ltd v CIR (1997) 59 SATC 153 7.4(k)
Tobacco Father v COT (1950) 17 SATC 3957.4(j)
Tod, CIR v (1983) 45 SATC 1 4.3.3(b)
Transvaal Associated Hide and Skin Merchants v COT Botswana
(1967) 29 SATC 97 3.1.3(b), 3.1.3(e)
Transvaalse Suikerkorporasie Bpk, KBI v (1987) 49 SATC 11 4.7
Trow, SIR v (1981) 43 SATC 189 21.2.6
Tuck v CIR (1988) 50 SATC 98 4.2, 4.8
U
Union Corporation Ltd and Others v CIR (UK) (1953) 34 TC 207 18.6
Union Government v Leask's Executor 1918 AD 458 15.2.3
United Aircraft Corporation v FC of T, 2 AITR 458 3.1.13, 3.2.9
V
Van Blerk, C: SARS v (2000) 62 SATC 131 4.4(b)
Van Blommestein, KBI v (1999) 61 SATC 145 15.2.3
Van der Walt, KBI v (1986) 48 SATC 104 7.4(f)
Van Zyl N.O. v CIR (1997) 59 SATC 105 2.6
Van Zyl N.O. v The Master and Another (1987) 49 SATC 165 21.4.1
Verrinder Ltd v CIR (1948) 16 SATC 48 4.7
Volskas Bpk v Meyer 1966 (2) SA 379 (T) 7.7(f)
VRD Investments (Pty) Ltd, CIR v (1993) 55 SATC 368 7.2, 7.6.1(b)
W

195
Wandrag Asbestos (Pty) Ltd, CIR v (1995) 57 SATC 123 8.25
Watermeyer, CIR v (1965) 27 SATC 117 5.1
Westminster Bank Ltd v Osler (1933) 17 TC 381 2.2
Widan, CIR v (1954) 19 SATC 3412.7
Wilkins v Rogerson (1960) 39 TC 344 5.2(a)
William Dunn & Co. Ltd, COT v (1918) 32 SATC 333.1.7
Wisdom v Chamberlain (1968) 45 TC 92 4.3.4
Woolidge, C: SARS v (2000) 62 SATC 1 2.7
X
X v COT, Y v COT, Z v COT (1977) 40 SATC 21 4.11, 5.2(e)

Indexes

• Income Tax • Capital Gains Tax • Estate Duty •

Income Tax

A  B  C  D  E  F  G  H  I  J  K  L  M  N  O  P  Q  R  S  T  U  V  W  X  Y  Z

This index includes references to other taxes falling under the Income Tax Act.
A
Abnormality
Tax avoidance: see chapter 17
Accident policy: see Insurance
Accounting profit
Taxable income contrasted with 1.4, 7.1
Accounting years
See generally chapter 18
Partnership 14.3
Accounts: submission of
See generally 21.1.2
Partnership: see chapter 14
Accrual/accrued
See also “deemed accrual”
Capital nature 4.1
Definition of “gross income” 2.1
Meaning2.4
Received: compared with 2.5
Additional assessments: see chapter 21
Administration
(See in particular chapter 21)
Department of Taxes 1.1
Legislation 1.1
Aerial and geophysical surveys: farmer’s 11.7.2
Agent
Appointment of, for collection of tax 21.3.3
“Agricultural bond”
Interest on: exemption 6.3(xiv)
AIDS levy: see chapter 10
Alimony
Deductibility 7.7(b)
Exemption 6.3(xvii)
Allocation
See also Apportionment
Of purchase price of assets9.4

196
Allowance
Benefit in respect of employment 5.6.6, 6.3(xix)
Amount
Definition of “gross income” 2.1
Meaning2.2
Annual payment dates 21.5.2
Annuities:
See also Retirement annuity
Deceased estates and trusts:see chapter 15
Deemed source in Zimbabwe: purchased 3.2.8
Deemed source in Zimbabwe: services rendered 3.2.6
“Gross income” 5.1
Payable by employer to former employee: deductibility 8.12
Source in Zimbabwe 3.1.2
“Annuity on retirement” 16.7.2, 16.7.3
Appeal
Legal expenses: deductibility 8.21
Right of, against disallowance of objection 21.4
Apportionment
Accruals: capital and revenue 4.8(b)
Accruals: pensions 3.2.6
Accruals: remuneration 3.1.11
Accruals: source: business profits 3.1.3
Credits (individuals’ personal) 10.3
Expenditure 7.2
Wear and tear allowance, scrapping allowance and recoupment 9.3.2, 9.3.3, 9.3.4
Approved employee share ownership trusts 15.4.3(e)
Approved Tourist Development Zone 10.26
“Arm’s length” principle
Double taxation agreements 20.2.1, 20.2.8
Tax avoidance 17.2.1, 17.2.5, 17.3.2
Arrear contributions: to pension fund by employee:
Deductibility 16.5
Articles
Allowances: see chapter 9
Meaning9.2.4
“Ascertained beneficiary”: see chapter 15
Assessed losses
See in particular 8.27
Death of individual 15.2.3(a)
Employment: prohibition against set-off 8.27(b)
Insolvency, etc., of individual 8.27(e), 15.3(c)
Partner 14.1
“Six-year rule”:
Generally 8.27(c)
Miners 8.27(d), 12.1
Tax avoidance context 17.3.3
Assessment: see chapter 21
Assigned estate: see chapter 15
Associations
Exempt 6.2
Automated financial transactions tax 19.13
Avoidance: see Tax avoidance
B
Bad debts
Deduction 8.7

197
Recovery 5.10
Banking Institution Levy 19.15
“Beneficiary with a vested right”: see chapter 15
Benefit fund
See generally chapter 16
Accruals from: exemption 6.3(vi)
Accruals of: exemption 6.2
Exemption from non-resident shareholders’ tax 19.2
Exemption from non-residents’ tax on interest 19.4
Exemption from resident shareholders’ tax 19.3
Lump sum payment from 10.7
Benefits in respect of services rendered
See also appropriate titles
General 5.6.1
Valuation 5.6.1
Boarding house
“Commercial building” definition 9.2.1
Bonus
Exemption 6.3(ii)
Provision: whether incurred 7.3(a)
“BOOT” arrangements 10.20, 10.21
Boreholes: farmer’s: capital allowances 11.7.2
Branch profits tax 10.10
Building societies
Accruals of: exemption 6.2(b)
Class “C” permanent shares: exemption 6.3(xii)
Non-resident shareholders’ tax on distributions 19.2
Bursaries
Accruals: exemption 6.3(i)
Expenditure: technical education: deductibility 8.11
Business operations: sales
Deemed source in Zimbabwe 3.1.3, 3.2.2
Source in Zimbabwe 3.1.3
C
Canals
“Farm improvement” definition 9.2.2
Canteen
“Industrial building” definition 9.2.3
Capital accruals
See generally chapter 4
Advantage of proving 4.1
Capital gains tax: alternative liability to 4.1
Capital allowances
See chapter 9
See also appropriate titles
Accounting year end differing from tax year
See chapter 18
Assets used for non-Zimbabwe trade 7.7(end)
Farmers 11.7
Capital expenditure and losses
See chapter 7, in particular 7.6
See also appropriate titles
Capital redemption allowance (miners)
See generally chapter 12
Carbon Tax 19.14
Car-parks

198
“Industrial building” definition 9.2.3
Carrying on business
Meaning18.9
Cash in lieu of leave
Accrual 5.2(b)
Casino
“Industrial building” definition 9.2.3
Cessation of business
Deductions 7.4(k)
Scrapping allowance and recoupments 9.3.3
Cession
Right to income 2.4(c)
Charging Act
Meaning1.1
Charitable donations
Deductibility 7.4(j)
Charitable institutions
Accruals of: income tax exemption 6.2(b)
Dividends receivable: liable to resident shareholders’ tax 19.3
Chiefs
Allowances of: exemption 6.2(f)
Child: see Minor child
Civil servants
Allowances: exemption 6.2(f)
Clubs
Sports, etc.: accruals of: exemption 6.2(b)
Collection of tax: see chapter 21
Commercial building
Allowances: See chapter 9
Meaning9.2.1
Commissioner of Taxes
Administration of Act generally: See chapter 21
Administration of Department 1.1
Practice notes: see Departmental practices
Commutation
Pensions and annuities 16.6, 16.7
Remuneration 5.2(d)
Retirement annuity 16.6
Company
Accounting year 18.3
Liquidation: assessed loss 8.27(e)
Ordinary residence 18.6
“Person”: constitutes 2.6
Public officer: appointment 21.3.1, 21.3.4
Compensation: accruals
Capital or revenue 4.7
Exemptions 6.3(iv), (vi)
Loss of employment 5.2(c)
Compensation: deduction
Damage or death 7.4(b)
Landlord’s eviction of tenant 7.6.1(b)
Competition
Cost of elimination of: deductibility7.6.1(b)
Compulsory improvements: see Improvements under a lease agreement
Computer operations
Building used for: “industrial building” definition 9.2.3

199
Concessions by creditors
Recoupment 5.11
“Conduit pipe” principle
Estates and trusts: see chapter 15
Connection fees
Deductibility 7.6.1(b)
Contour ridging
Farmer’s 11.7.2
Contractors to Government and statutory corporations 19.10
Contractual rights
Cost of acquisition of: deductibility 7.6.1(b)
Control of a company: meaning 9.5.3
Conventions: see Trade conventions and missions
Co-operatives
Taxable income: special deduction 8.19
Cost price
Fixed assets 9.1
Trading stock 5.8
Councillors (urban, etc.)
Allowances: exemption 6.2(f)
Creditors: see Concessions by creditors
Credits
Double taxation relief: withholding taxes and foreign taxes: see chapters 10, 19
and 20
Individuals’ personal: see chapter 10
Crops
See generally chapter 11
“Trading stock”: whether constitute 11.3
Valuation 11.3
D
Dam
Farmer’s: capital allowances 11.7.2
Damages
Accruals: capital or revenue 4.7
Deductibility 7.4(b)
Death
Partner: taxable income 14.3.2
Taxpayer: credits (personal) 10.3
Deceased estate
(See in particular chapter 15)
Executor’s submission of returns 21.1.1
“Person”: constitutes 2.6, 15.1
Deductions: expenditure and losses
See in particular chapters 7 and 8
See also appropriate titles
Deemed accrual
“Gross income” definition 2.1
Income not paid over 2.7.1
Meaning2.7
Minor child 2.7.3, 2.7.4
Partnership income 2.7.2
Trusts, settlements, etc. 2.7.5, 2.7.6
Deemed source in Zimbabwe
See in particular chapter 3
See also appropriate titles
“Gross income” definition 2.1

200
Defence forces
Allowances: exemption 6.2(f)
Departmental practices
Published by Commissioner 1.3
Deposits
“Gross income” definition 2.3
Designated area grants
“Gross income” definition 5.15
Diplomats
Accruals of: exemption 6.2(f)
Directors’ fees
Deemed source in Zimbabwe 3.2.4
Excessive: disallowance 7.3(d), 7.4(j)
Source in Zimbabwe 3.1.4
Disabled persons credit 10.3
Disposal of income
After accrual 2.4(c)
Disputed expenditure
When incurred 7.3(c)
“Dividend stripping”
Losses: deductibility 7.6.2(d)
Dividends
Deduction of Zimbabwe tax from 8.31
Deemed source in Zimbabwe 3.2.7
Distribution costs 7.4(e)
Double taxation agreements: restrictions on rate of tax: see chapter 20
Exemption 6.3(viii)
Expenditure applicable to exempt dividends: deductibility 7.7(f)
Expenditure applicable to foreign dividends: deductibility 7.7(n)
Foreign:
See generally chapters 10 and 20
General rate of income tax: companies 10.9
General rate of income tax: individuals 10.5
General rate of income tax: trusts 10.24
Non-resident shareholders’ tax on:
Double taxation agreements: restrictions on rates: see chapter 20
Generally 19.2
Resident shareholders’ tax on 19.3
Source in Zimbabwe 3.1.5
Donations
Deductibility 7.4(j), 8.13
Trading stock
Farmer’s 11.4
Tax avoidance 11.4, 17.3.3
Double taxation
Of same amount 2.5, 17.2.6(b), 17.3.1
Double taxation agreements: see chapter 20
Double taxation relief
See generally chapters 10, 19 and 20
Doubtful debts
Deduction 8.7
Driveways
“Industrial building” definition 9.2.1
Drought relief: farmers 11.6
E
Educational costs

201
Accruals: exemption 6.3(i)
Technical education grants: deductibility 8.11, 8.13
Educational institutions
Accruals of: exemption 6.2(b)
Elderly persons credit 10.3
Embankment
Farmer’s: capital allowances 11.7.2
Employee
See also Remuneration
Use of own assets: deduction 7.4(f), (l)
Employee share trusts
Taxability generally 15.4.3(e)
Employees’ savings schemes
Accruals of: exemption 6.2(b)
Employees’ tax
See Pay as you earn
Employment:
See Remuneration
See also Services rendered
Employment creation incentive 8.26
Endowment:
See Insurance
Entertainment
Allowance received: exemption 6.3(xix)
Expenditure and allowances incurred: deductibility 7.7(m)
Estates
See Deceased, etc.
Epidemic disease relief: farmers 11.6
Evasion: see Tax evasion
Excessive expenditure
Deductibility 7.3(d), 7.4(j)
Exchange
Of assets 2.2
Exchange rates: profits and losses
Accrual: meaning2.4(a)
Capital allowances: cost of assets 9.1
Capital or revenue nature: of losses 7.6.2(f)
Capital or revenue nature: of profits4.15
Variations: relevant year : losses 8.30
Variations: relevant year : profits 5.18
Executor in deceased estate
See generally chapters 15 and 21
Exemptions
See in particular chapter 6
See also appropriate titles
Expenditure
See in particular chapters 7 and 8
See also appropriate titles
Not yet incurred: allowance for 8.22
Prior to commencement of business: deductibility 8.15
Recoverable under contract of insurance or indemnity7.7(c)
Experiments
Expenditure: deductibility 8.10
Export incentive
Exemption 6.3(xviii)
Manufacturing companies 10.19

202
Export-market development expenditure 8.25
Export processing zones
See generally chapter 10
“Licensed investor”: exemption of accruals 6.3(xx)
“Extent”
Expenditure incurred 7.2
F
Fair market price
Tax avoidance 17.3.1, 17.3.2
Transfer of assets: determination by Commissioner 9.4
“Farm improvement”
Meaning9.2.2
Farming/Farmers
See generally chapter 11
See also appropriate titles
Deceased estate 15.2.2
Fourth and Seventh Schedule allowances contrasted 11.7.1
Trade or hobby 11.10.5
Fees
Double taxation agreements: restrictions on rate of tax: see chapter 20
Non-residents’ tax on 19.5
Fencing
Farmer’s: capital allowances 11.7.2
“Industrial building” definition 9.2.3
Financial year end: see Accounting years
Fines
Deductibility 7.4(d)
Fixed and floating capital
Accruals4.6
Expenditure 7.6.1(b)
Losses 7.6.2(e)
Fixed standard values
See Livestock
Flats
“Commercial building” definition 9.2.1
Floating capital
See Fixed and floating capital
Flower growing
Capital or revenue expenditure 11.10.1
Foreign currency denominated account
Interest on: exemption 6.3(xiii)
Foreign exchange: see Exchange rates
Friendly societies
Accruals of: exemption 6.2(b)
G
Gambling
Capital or revenue accruals 4.10
“General deduction formula”
Meaning: see chapter 7
Gifts
See also Donations
Capital or revenue accruals 4.12
From employer and others 5.2(a)
“Golden handshakes”
Deductibility 7.4(h)
Taxability 5.2(c)

203
Goodwill
Acquisition of 7.6.1(a)
Lease premiums distinguished 5.4
Partnership 14.4.2
Sale of 4.13
Government agencies
Accruals of: exemption 6.2(c)
Government loans
Interest: exemption 6.3(xi)
Grants
Accruals: designated area scheme 5.15
Accruals: “gross income” definition 5.13
Technical education: deductibility 8.11
Gross income
See in particular chapters 2 to 5
See also appropriate titles
Growing crops
See Crops
Growth point areas 9.7
H
Hire-purchase
Method of assessment 8.29
Purchases: capital assets: allowances 9.1
Sales: meaning of “accrued” 2.4(a)
Hobbies 4.10, 7.4(g), 8.17, 11.10.5
Hospitals, etc., and related housing
Capital allowances: farmers 9.2.2
Capital redemption allowance: miners 12.2
Hotels
Licensed: “industrial building” definition 9.2.3
Unlicensed: “commercial building” definition 9.2.1
Housing
See also Staff housing
Benefit in respect of employment 5.6.2
“Commercial building” definition 9.2.1
“Industrial building” definition 9.2.3
Husband and wife: transfers of assets between
Capital allowances 9.6
Mining 12.10
Payment of tax 21.5.4
I
Illegal trade
Capital or revenue accruals 4.17
Deductions 7.4(c)
Immovable property
Rentals: source in Zimbabwe 3.1.9
Sales: capital or revenue nature: see chapter 4
Sales: capital or revenue nature: source in Zimbabwe 3.1.6
Implements
Capital allowances 9.2.4
Improvements under a lease agreement
Accrual: “gross income” definition 5.5
Expenditure 8.5
Recoupment if applied against purchase price (tenant) 5.12
Income
See generally chapter 2

204
See also appropriate titles
Income tax
Calculation of: see in particular chapter 10
Collection of: see generally chapter 21
Deductibility 7.7(d)
Dividends: deduction of tax from 8.31
Rates
See in particular chapter 10
Fixing of 1.1
Growth point areas 9.7.4
Returns
See generally chapter 21
Fees for preparation of: deductibility 7.4(i)
Issue of 1.1, 21.1.1
Incurred
Meaning7.3
Individual
See in particular chapter 10
Accounting year 18.2
Constitutes a “person” 2.6
Ordinary residence 18.5
Industrial building
Meaning9.2.3
Industrial park developer
See generally chapter 10
Exemption of accruals 6.3(xxi)
Informal traders tax 19.12
Information
Required by Commissioner: See generally chapter 21
Inheritances
Capital or revenue accruals 4.12, 15.2.1
Insolvent estate
See chapter 15
Administrator’s duties: see chapter 21
Assessed loss 8.27(e)
Constitutes a “person” 2.6, 15.1
Instalment-credit sales
Meaning of “accrued” 2.4(a)
Method of assessment 8.29
Insurance
See generally chapter 18
Benefits: exemption 6.3(vi)
Premiums: deductibility generally 7.4(a)
Premiums: farmer: condition for loan 11.10.2
Proceeds: taxability 4.9
Insurance companies
See in particular chapter 13
Life: non-resident shareholders’ tax: exemption 19.2
Residents’ tax on interest: exemption 19.8
Intermediated money transfer tax 19.16
Intention
Capital or revenue accruals: see chapter 4
Interest: accruals
Deemed source in Zimbabwe 3.2.7
Double taxation agreements: restrictions on rate of tax: see chapter 20
Exemptions: see generally 6.3

205
Source in Zimbabwe 3.1.7
Interest: deductions
See generally chapter 7
Apportionable 7.2
Capital or revenue purposes 7.4(e), 7.6.1(b)
Dividends payable: applicable to 7.4(e)
Double taxation agreements: restrictions on rate of tax: see chapter 20
Exempt income: applicable to 7.2, 7.7(f)
Incurred during building construction period 8.15, 9.1
Loss on rentals 7.6.2(a)
Notional 7.7(h)
Overdue tax 7.7(d)
When incurred 7.3(b)
Interest payable on overdue tax: see chapter 21
Interest: withholding taxes on: see chapter 19
International agreements
Organisations: exemption 6.2(c)
Persons referred to: exemption 6.2(d)
Intestacy
See chapter 15
Isolated transactions
Capital or revenue accruals 4.11
J
Judges
Allowances: exemption 6.2(f)
K
“Know-how”: accruals from
Capital or revenue 4.14
Deemed source in Zimbabwe 3.2.9
Premium receivable 5.4
Source in Zimbabwe 3.1.13
“Know-how”: expenditure
Premiums deductible 8.4
Knowledge: expenditure on
Generally 7.6.1(b)
Technical education grants8.11
L
Lease improvements
See Improvements under a lease agreement
Lease premium
See Premium
Lease-hire
Passenger motor vehicle 7.7(k)
Recoupment 5.12(i)
Leave pay
Accrual 5.2(b)
Expenditure: when incurred 7.3(a)
Legal expenses: deductibility
Generally 7.4(b), 7.6.1(b)
Income tax appeals 8.21
“Licensed investor”: see Export processing zones
Life insurance: see Insurance and Insurance companies
Liquidation of company
Assessed loss 8.27(e)
Liquidator’s duties: see generally chapter 21
Livestock: Farmers’

206
See generally chapter 11
Deceased estate 15.2.2
Loan
Benefit in respect of employment 5.6.4
Loss of: deductibility 7.6.2(e)
Loan-raising fees
Capital allowances 9.1
Deductibility 7.6.1(b)
Local authorities
Accruals of: exemption 6.2(a)
Loss of office
Accrual 5.2(c)
Expenditure 7.4(h)
Losses
See in particular 7.6.2, 8.27 and 11.10.5
See also Assessed losses
Lump sums from pension and benefit funds
See generally chapter 16
Example10.7
M
Machinery
Capital allowances 9.2.4
Removal expenses 9.3.2(g)
“Mainly”
“Industrial building” definition 9.2.3
Management expenses
Apportionable to exempt accruals 7.2, 7.7(f)
Manufacturing
“Industrial building” definition 9.2.3
Married
Community of property: deceased estate 15.2.3(c)
Married woman
Payment of tax 21.5.4
Separate taxpayer10.1
Transfer of assets to or from husband: capital allowances 9.6
Transfer of assets to or from husband: mining 12.10
Medical aid society
Accruals of: exemption 6.2(b)
Approval by Commissioner 16.3
Benefits from: exemption 6.3(vi), 16.3
Contributions: employer’s
Deductibility 16.3
Exemption 6.3(vii)
Credit, personal: see chapter 10
Exemption: from non-resident shareholders’ tax 19.2
Exemption: from non-residents’ tax on interest 19.4
Exemption: from resident shareholders’ tax 19.3
Medical expenses
Credit, personal: see chapter 10
Paid by employer: exemption 6.3(vii)
Members of Parliament
Allowances: exemption 6.2(f)
Memorandum and articles of association
Copy required by Commissioner 21.1.2
Cost of alterations: deductibility 7.6.1(a)
Objects clause: intention of company 4.3.6

207
Mergers:
Companies generally: transfer of assets: capital allowances 9.5
Life insurance 13.5.2
Mining 12.9
Mineral
Meaning12.6.1
Mining/miners
See generally chapter 12
See also appropriate titles
Assessed loss: “six year rule” 8.27(d)
Claims: sale of: income tax 12.8
Operations: meaning 12.6.2
Minor child
Income of 2.7, 21.1.2
Tax avoidance by parent 2.7.3, 2.7.4
Moneylending
Losses on: deductibility 7.6.2(e)
Motor vehicle
Benefit in respect of employment 5.6.3
N
Natural Resources Board
Accruals of: exemption 6.2(a)
Nominee
Whether a “person” 2.6
Non-contiguous mine
Capital redemption allowance 12.4.2
Non-profit-sharing bodies
Accruals of: exemption 6.2(b)
Non-resident shareholders’ tax
See generally 19.2
Double taxation agreements: restrictions on rates: see chapter 20
Non-residents
Credits (personal)10.3
Interest: exemption 6.3(xvi)
Non-residents’ tax on fees, interest, remittances and royalties
See generally 19.4 to 19.7
Double taxation agreements: restrictions on rates: see chapter 20
O
Objections 21.4
Obligatory improvements: see Improvements under a lease agreement
Offences: see chapter 21
Office: “commercial building” definition 9.2.1
Options
Capital or revenue accruals 4.16
Shares 5.2(e)
Orchards
Farmer’s 11.9
Ordinarily resident and ordinary residence
See generally chapter 18
Credits (personal): effect on: see chapter 10
Deceased estate 15.2.3(d)
Insolvent estate 15.3(d)
Trust 15.4.3(d)
P
Partnership
See in particular chapter 14

208
Payment of partner’s tax 21.5.4
“Person”: meaning 2.6
Trade conventions and missions 8.18
“Passage benefit”
Benefit in respect of employment 5.6.5
“Passenger motor vehicle”
Capital allowances 9.2.5
Capital redemption allowance: miners 12.2
Meaning9.2.5
Recoupment 9.3.4(c)
Rentals payable: deductibility 7.7(k)
Scrapping allowance 9.3.3(d)
Patents
Premiums: payable 8.4
Premiums: receivable 5.4
Pay as you earn (“PAYE”)
Credit against tax payable on assessment: see chapter 10
System generally 21.5.3
Payment of tax 21.5
Penalties
Default in submission of return 21.1.1, 21.2.5
Tax avoidance/evasion: see chapter 17
Withholding taxes: see appropriate titles
Pension: accrual
See generally chapter 16
Commutation 16.6, 16.7
Deemed source in Zimbabwe: services rendered 3.2.6
Example5.1
Individual over 59 years of age: exemption 6.3(v)
Lump sum from fund: generally: see chapter 16
Lump sum from fund: example 10.7
War, old age, etc.: exemption 6.3(iv)
Pension: contributions
See generally chapter 16
Pension: payable
By employer to former employee and dependants: deductibility 8.12
Pension fund
See generally chapter 16
Accruals of: exemption 6.2(b), 10.22
Exemption: from non-resident shareholders’ tax 19.2
Exemption: from non-residents’ tax on interest 19.4
Exemption: from resident shareholders’ tax 19.3
Exemption: from residents’ tax on interest 19.8
Lump sum payment from: example 10.7
Unapproved
Contributions to 7.7(g), 16.2
Lump sum payments from 16.7.4
Person
Definition 2.6
In definition of “gross income” 2.1
“Petroleum operations” and “operators” 5.16, 8.24, 8.27(b), 19.11
Plantations: see Timber, Orchards, etc.
Police force
Allowances: exemption 6.2(f)
Pollution
Works for the prevention of: “industrial building” definition 9.2.3

209
Post Office Savings Bank
Accrual of: exemption 6.2(a)
Interest from: exemption 6.3(ix)
Practice notes, practices
Commissioner’s: see Departmental practices
Preliminary expenses
Deductibility 7.6.1(a)
Premium (for lease, patents, “know-how”, etc.)
Accruals: “gross income” definition 5.4
Deductions 8.4
Meaning5.4
Prepayments
Deductibility of taxpayer’s expenditure 7.3(e)
Prescription
Additional and reduced assessments 21.2.6
President of Zimbabwe
Emoluments: exemption 6.2(f)
Private business corporation
“Company”: includes 1.4
Conversion from/to company; reconstruction election 9.6
Income tax: rate 10.12
Non-resident shareholders’ tax 19.2
Resident shareholders’ tax 19.3
Private expenditure
Deductibility 7.4(f),(g), 7.7(a),(b)
Prizes
Capital and revenue accruals 4.12, 5.2(a)
Production of the income
Meaning7.5
Profit
Taxable income contrasted with 1.4
Prohibited deductions 7.7
Proof: burden of 4.1, 21.4.2
Prospecting 12.7
Protection of assets
Expenditure on 7.6.1(b)
Provident fund
May constitute a “benefit fund” 16.2
Provisional tax: see Annual payment dates
Provisions
Deductibility 7.3(g), 7.7(e)
Public officer
Company required to appoint, etc. 21.3.1, 21.3.4
Submission of company returns 21.1.1
“Purpose”
Tax avoidance: see chapter 17
Purposes of trade
Meaning7.4
R
Railway lines
Capital allowances 9.2.6
Rates of tax: see appropriate taxes
Received/receipts
“Accrued”: compared with2.5
“Gross income” definition 2.1
Meaning2.3

210
Reconstructions: group of companies: transfer of assets
: Capital allowances 9.5
: Life insurance 13.5.2
: Mining 12.9
Recoupment from capital expenditure (mining)
Capital redemption allowance 12.4.3
Recoupments
See generally 5.10
Capital allowances
Generally 9.3.4
Partnership: change in membership 14.4.1
Deemed source in Zimbabwe 3.2.10
Lease improvements, applied against purchase price 5.12
Pension fund: recovery by employer 16.7.5
Rentals payable, applied against purchase price 5.12
Recovered: see Recoupment
Reduced assessments: see chapter 21
Religious institutions
Accruals of: exemption 6.2(b)
Remittances: non-residents’ tax on
Double taxation agreements: restrictions on rate of tax: see chapter 20
Generally 19.7
Removal expenses
Plant: capital allowances 9.3.2(g)
Trading stock: deductibility 7.4(a)
Remuneration
See appropriate titles
See also Benefits
Double taxation agreements: general rule modified 20.2.6, 20.5.6
Pay as you earn 21.5.3
Rentals payable
Applied against purchase price: see Recoupments
Incurred 7.3(a)
Non-deductible 7.7(i)
“Passenger motor vehicle”: restriction on deductibility 7.7(k)
Rentals receivable
Loss on 7.6.2(a)
Source in Zimbabwe 3.1.9
Repairs
Deductibility 7.7(i), 8.2
Meaning8.2
Whether constitute “manufacturing” 9.2.3
Replacements election
Miners: capital redemption allowance 12.4.1
Representative taxpayers: see chapter 21
Research
Expenditure: deductibility 8.10
Reserve Bank of Zimbabwe
Accruals of: exemption 6.2(a)
Non-residents’ interest from: exemption 6.2(e)
Reserves
Provisions for: deductibility 7.3(g), 7.7(e)
Reservoir
Farmer’s: capital allowances 11.7.2
Resident: see Ordinarily resident
Resident shareholders’ tax: see generally 19.3

211
Residents’ tax on interest
See generally 19.8
Exemption of interest from income tax 6.3(xv)
Restocking deduction: farmers 11.6
Restraint of trade
Capital or revenue accruals 4.8
Capital or revenue expenditure 7.6.1(b)
Expenditure securing sole selling rights 7.7(j)
Retention money
Whether “accrued” 2.4(b)
Retirement annuity
See generally chapter 16
Commutation: taxability 16.6.2
Individual over 59 years of age: exemption 6.3(v)
Retirement annuity fund
See generally chapter 16
Retrenchment
Compensation: taxability 5.2(c), 6.3(iii)
Returns: see Income tax returns
Rights
Cost of acquisition: deductibility 7.6.1(b)
Roads, farm
Whether “farm improvement” 9.2.2
Royalties
Deemed source in Zimbabwe 3.2.9
Double taxation agreements: restrictions on rate of tax: see chapter 20
Non-residents’ tax on 19.6
Source in Zimbabwe 3.1.10
S
“Salary sacrifice” schemes
Validity of 6.3(vii)
Savings certificates
Interest: exemption 6.3(ix)
Scholarships
Accruals: exemption 6.3(i)
Expenditure: technical education: deductibility 8.11
School fees
Accruals: exemption 6.3(i)
Schools and related housing
Capital allowances: farmers 9.2.2
Capital redemption allowance: miners 12.2
Scrapping allowance
Generally 9.3.3
Secrecy
Tax officials bound by oath 1.1
Services rendered: business or profession
Deemed source in Zimbabwe 3.2.3(a), (b)
Remuneration for 5.2
Source in Zimbabwe 3.1.11(b)
Services rendered: employment
Benefits in respect of 5.6
Deemed source in Zimbabwe 3.2.3(a), (b), 3.2.4, 3.2.5
Remuneration for 5.2
Source in Zimbabwe 3.1.11(a)
Sham transactions 17.1
Share options 5.2(e)

212
Shares
Awarded to employee: employee’s taxability5.2(a)
Awarded to employee: employer’s deduction 7.7(l)
Sales: capital or revenue nature: see chapter 4
Sales: capital or revenue nature: source in Zimbabwe 3.1.3(a), 3.1.12
Shops
“Commercial building” definition 9.2.1
“Industrial building” definition 9.2.3
Showrooms
“Commercial building” definition 9.2.1
“Industrial building” definition 9.2.3
Soil erosion prevention: farmer’s 11.7.2
Source in Zimbabwe
See in particular chapter 3
See also appropriate titles
“Gross income” definition 2.1
Special initial allowance
See in particular 9.3.1, 9.3.2
“Special mining lease” holders 5.16, 8.24, 8.27(b), 19.11
Staff housing: capital allowances
See generally (other than miners) 9.2.7
Farmers: general 9.2.7, 11.7.1
Farmers: schools, hospitals, etc. 9.2.2, 11.7.1
Miners: general 12.2(b)
Miners: schools, hospitals, etc. 12.2(c),(d),(f)
Statutory corporations
Accruals of: exemption 6.2(b)
Stock in trade: see Trading stock
Stolen money: whether gross income 2.3
Storage buildings
“Industrial building” definition 9.2.3
Stud livestock: see Livestock
Stumping and clearing: farmers 11.7.2
Subscriptions
Professional, etc.: deductibility 8.14
Subsidies
“Gross income” definition 5.13
Suspensive sales
Method of assessment, and example 8.29
Purchase of fixed assets under: capital allowances 9.1
T
Tax: see appropriate taxes
Tax avoidance
See in particular chapter 17
Cession of right to income 2.4(c)
Disposals/donations of trading stock 11.4, 17.3.3
Fair market price 17.3.1, 17.3.2
Trusts, etc.: power over 2.7.5, 2.7.6
Tax evasion
See generally chapter 17
Commissioner’s powers 21.1.2
Offences: punishment 21.6
Tax avoidance distinguished/test unnecessary 17.1, 17.4
Tax reserve certificates
Issue of 21.5.6
Interest: exemption 6.3(x)

213
Taxable income
Accounting profit contrasted with 1.4, 7.1
Determination of: general rules 1.4, 7.1
“Temporary absence”
Deemed source: employee’s services 3.2.4
Term assurance: see Insurance
Termination of employee’s service
Accrual: taxability 5.2(c)
Payments by employer: deductibility 7.4(h)
Theft
“Gross income”: whether stolen money constitutes 2.3
Loss by: whether deductible 7.4(b)
Timber
Farmer’s 11.8
Tobacco barn
Equated with “farm improvement” 9.2.2
Tobacco levy
See generally 19.9
Deductibility 7.7(p), 11.10.3
Toll-bridge
“Industrial building” definition 9.2.3
Toll-road
“Industrial building” definition 9.2.3
Tourist Development Zones 10.26
Trade
See also Purposes of trade
Meaning7.4
Trade conventions and missions
Cost of attendance: deductibility 8.18
Trade marks: accruals from
Deemed source in Zimbabwe 3.2.9
Disposal of: capital 3.2.9
Source in Zimbabwe 3.1.13
Trade marks: expenditure on
Acquisition 7.6.1(a)
Premium 8.4
Trade unions
Accruals of: exemption 6.2(b)
Benefits from: exemption 6.3(vi)
Tradesman
Provision of own tools and clothing: deduction 7.4(l)
Trading stock
Acquired otherwise than in the ordinary course of trade: deductibility 8.17
Closing stock 5.8
Crops 11.3
Donated 5.8(a), 11.4, 17.3.3
Farmers’ consumable stores 11.3
Livestock 11.2
Meaning5.8
Opening stock 8.16
Valuation 5.8(c),(d)
Work in progress: professional persons’ 5.8(f)
Training investment allowance
Generally 9.3.5
Miners 12.5
“Transfer pricing” 17.3.2, 20.2.1, 20.2.8

214
Transferable credits (personal): between spouses 10.3.3
Travelling expenses
From home: deductibility 7.7(b)
Treasury bills 6.3(xv), 10.25, 19.8
Trusts
See in particular chapter 15
Income during settlor’s lifetime 2.7.5, 2.7.6
“Person”: trust may constitute 2.6, 15.1(c), 15.4
Public character: accruals: exemption 6.2(b)(iv)
Tax avoidance 2.7.5, 2.7.6
Trustee
Generally 21.3
Payment of tax 21.5.4
Submission of trust returns 21.1.1
U
Unapproved fund (pension or benefit)
Contributions to 7.7(g), 16.2
Lump sum payment from 16.7.4
Use: meaning 9.1
Usufruct 15.2.1(c), 15.2.2
Utensils
Capital allowances 9.2.4
V
“Venture”: meaning 7.4(a)
Venture capital companies
Accruals of: exemption 6.2(d)
Vineyard: farmer’s 11.9
W
Walls
“Industrial building” definition 9.2.3
Water conservation work: farmer’s: capital allowances 11.7.2
Water-furrow: “farm improvement” definition 9.2.2
Wear and tear allowance
See in particular 9.3.2, 9.3.3
Weir: farmer’s: capital allowances 11.7.2
Wells: farmer’s: capital allowances 11.7.2
Wills: see chapter 15
Withholding taxes
See generally chapter 19
Double taxation agreements: see chapter 20
Y
Year of assessment
Accounting year substituted 18.1
Ends on 31 December 1.1
“Gross income” definition 2.1
Period shorter than a year: credits (personal) 10.3.2
Z
Zambezi River Authority
Accruals of: exemption 6.2(a)
Zimbabwe Revenue Authority 1.1
Capital Gains Tax

A  B  C  D  E  F  G  H  I  J  K  L  M  N  O  P  Q  R  S  T  U  V  W  X  Y  Z

This index includes references to capital gains withholding tax


A

215
Accounting year: acceptance by Commissioner 22.2
Accrued
See also Deemed accrual
Meaning22.2
Administration
Department of Taxes 1.2, 22.21
Annual allowance
Deduction 22.2
Examples 22.22
Appeal
Legal expenses: deductibility 22.2
Right of, against assessment 22.21.1
Apportionment: expenditure 22.6.4
Assessed capital loss: generally 22.7
Assessments 22.21.1
Assignment: see Deed of sale
Associations, etc.: sale of assets exempt 22.5.2
Avoidance: see Tax avoidance
B
Bad debts: deductibility 22.2, 22.6.11
Benefit funds: sale of assets exempt22.5.2
Benefit societies: sale of assets exempt 22.5.2
Bond: "marketable security" definition 22.1
Building society
Liability of 22.4.7, 22.5.2
Sale of shares in 22.2
Business property: "rollover" relief 22.17
C
Capital amount: meaning and calculation of 22.2
Capital gain: meaning and calculation of 22.2
Capital Gains Tax Act: scope of 4.1, 22.1
Capital gains withholding tax 22.23
Cession: see Deed of sale: assignment of rights
Change in shareholding: company with assessed capital loss 22.7.2
Charitable institutions: sale of assets exempt 22.5.2
Claims: mining: immovable property 22.1
Clubs, etc.: sale of assets exempt 22.5.2
Collection of tax 22.21
Commissioner of Taxes: administration of Act 1.2
Compulsory improvement: see Leases
Court order: deemed sale of assets 22.4.3
D
Damage to asset
See generally 22.8
Examples 10 and 11 22.22
Repairs met out of recovery 22.8.2
Debenture: "marketable security" definition 22.1
Deceased estate: realisation and distribution exempt 22.5.1
Deductions: generally 22.2, 22.6
Deed of sale
Assignment of rights 22.12.2
Example 9 22.22
Deemed accruals 22.4.8
Deemed sales
Generally 22.4
Reduction in cost of asset or in share capital 22.14

216
Deemed source
Concept not applicable 22.2
“Depositary”: capital gains withholding tax 22.23
Destruction of asset
Amounts recovered 22.8.1
Deductibility of cost 22.6.10
Examples 10 and 11 22.22
Replacement cost met out of recovery 22.8.2
Disposal otherwise than by way of sale
"Gross capital amount" definition 22.4.1
Donation
By taxpayer: "Gross capital amount" definition 22.4.1
To taxpayer: deemed cost 22.6.3
Double taxation agreements
Immovable property and “marketable securities": gains on 22.20
Doubtful debts: deductibility 22.6.11
E
Ecclesiastical institutions: sale of assets exempt 22.5.2
Educational institutions: sale of assets exempt 22.5.2
Exchange
Example 5 22.22
Of assets generally 22.4.1, 22.10.2
Exchange rates
Losses 22.6.9
Profits 22.4.5
Exempt accruals
Deduction of cost prohibited 22.6.8
Generally 22.5
Export processing zones
Exemption of sales of assets 10.14, 22.5.1
Expropriation: deemed sale of assets 22.4.2
F
Fair market price: determination by Commissioner 22.9
Farm
Sale of: Example 1 22.22
Flats: rollover relief 22.16
Foreign company: winding up: transfer of assets 22.10.3
Foreign exchange: see Exchange rates
Friendly societies: sale of assets exempt 22.5.2
G
Government stock: exemption 22.5.1
Gross capital amount: meaning and calculation of 22.2, 22.4
H
Husband and wife
Joint ownership of asset: rollover relief 22.16
Transfer of assets between 22.15
I
Immovable property
Meaning and "specified assets" definition 22.1
Sale by individual to controlled company 22.11
Sale of: Example 1 22.22
Income tax
Assessed losses contrasted with assessed capital losses 22.7.1, 22.7.3
Comparative liability 4.1, 22.4.7
Definitions from Act re-used 22.2
Exclusion of relevant accruals and deductions 22.2

217
Incorporeals 22.1
Incurred: meaning 22.2
Individual
See also Principal private residence
Sale of business property to controlled company 22.11
Industrial park developer
Exemption of sales of assets 22.5.1
Inherited assets: deemed cost 22.6.2
Instalment sale: taxation of gain 22.13
Institutes, etc.: sale of assets exempt 22.5.2
Interest
Loan used to purchase asset: deductibility 22.6.1
L
Land and buildings:immovable property 22.1
Leases
Compulsory improvement to property: deductibility 22.6.7
Legal expenses: deductibility
Capital gains tax appeal 22.2
Cost of acquisition of asset 22.6.1
Licensed investor: see Export processing zones
Life insurance: sales of assets exempt 22.5.1
Local authorities
Investment in exempt 22.5.1
Own sales exempt 22.5.2
Losses: see Assessed capital loss
M
Marketable security: meaning 22.1
Married woman
Transfer of assets to or from husband 22.15
Maturity of stock: deemed sale 22.4.4
Medical aid societies: sale of assets exempt 22.5.2
Mergers: companies: transfer of assets 22.10
Mine: sale of: Example 3 22.22
"Minimum $5000"
Gain 22.2
Loss 22.7.1
Not applicable 22.12, 22.13
Mining claims
Immovable property 22.1
Sale of: Example 3 22.22
O
Objections 22.21.1
Offences 22.21.1
P
Partnership: accounting date: deemed accrual 22.4.8
Payment of tax 22.21.2
Penalties 22.21.1
Pension funds: sale of assets exempt 22.5.2
Principal private residence
Rate of tax 22.1
“Rollover” relief
Examples 6 and 722.22
Generally 22.16
Transfer to former wife or husband 22.15
Private business corporation 22.1
Conversion from/to company: reconstruction election 22.10.1

218
Member's interest equated to shares 22.1
Proof: burden of 22.6.10
Property: see Immovable property
R
Rates of tax: see Tax
Reconstructions
Example 4 22.22
Group of companies: transfer of assets 22.10
Redemption of stock: deemed sale 22.4.4
Reduction in cost of asset: treatment 22.14
Reduction in share capital
Example 12 22.22
Treatment 22.14
Representative taxpayers 22.21.1
Returns 22.21.1
“Rollover” relief
Business property22.17
Principal private residence 22.16
S
Selling expenses: deduction 22.2
Shares
Exchange of
Example 5 22.22
Generally 22.4.1, 22.10.2
In property-owning company: expenditure by shareholders on property 22.6.5
Issue of 22.4.6
"Marketable security" definition 22.1
Private business corporation: member's interest in 22.1
Sale of: Example 2 22.22
Societies, etc.: sale of assets exempt 22.5.2
Source: of gain 22.3
Specified assets: meaning 22.1
Statutory corporations
Investment in exempt 22.5.1
Own sales exempt 22.5.2
Stocks and shares
See also Shares
"Marketable security" definition 22.1
Suspensive conditions: sales under
Example 8 22.22
Taxation of gain 22.12
Swaps: see Exchange
T
Tax
Payment of 22.21.2
Rates of 4.1, 22.1
Tax avoidance 22.19
Town-house scheme
Expenditure by shareholder on property 22.6.5
Rollover relief 22.16
Trade unions: sale of assets exempt 22.5.2
Trusts: sales and distributions of assets 22.18
Trusts of a public character: sale of assets exempt 22.5.2
U
Unit trust: "marketable security" definition 22.1
Y

219
Year of assessment: meaning 22.2
Estate Duty

A  B  C  D  E  F  G  H  I  J  K  L  M  N  O  P  Q  R  S  T  U  V  W  X  Y  Z

A
Acquisition of property, date of 23.2.2(b), 23.2.3
Administration of Act 23.5
See also appropriate titles
Allowable deductions 23.3
See also appropriate titles
Annuities charged upon property, valuation 23.4(c)
Annuities not charged upon property, valuation 23.4(d)
Appeals 23.6.3
Assessment, objections 23.6.2
B
Benefit funds and pensions 23.3(m)
Books, statutory and objects of art 23.3(k)
C
Calculation of estate duty 23.9
Capital gains tax, deduction 23.5(g)
Charitable bequests and donations 23.3.(h)
Costs of administration as a deduction 23.3.(c)
D
Debts as a deduction 23.3.(b)
Debts due to non-residents 23.3(f)
Deductions
See allowable deductions and
appropriate titles
Deemed property 23.2.2(c)
Donations 23.2.2(c)(ii)
Insurance policies 23.2.2(c)(i)
Property not otherwise included 23.2.2(c)(iii)
Deemed property included in share valuations 23.3(l)
Donations defined 23.2.2(c)(ii)
Inter vivos 23.2.2(c)(ii)B-C
Mortis causa 23.2.2(c)(ii)A
Donation of use or occupation of property 23.3(j)
Donation of usufruct to deceased 23.3(g)
Donations, valuation 23.4.(e)
Double taxation 23.5(f), 23.7
E
Estate duty, calculations 23.9
Estate duty forms 23.10
Estate, what constitutes 23.2
See What constitutes an estate
Executor, liability for duty 23.6.1(c)
Exemption, South African assets 23.7.2
F
Family home 23.3(o)
Filing of accounts by Master 23.6.1(a)
Forms prescribed 23.10
Funeral expenses 23.3(a)
G
General provisions 23.6
See also appropriate titles

220
I
Immovable property, transfer of 23.6.1(e)
Improvements to property subject to life interest 23.3(i)
Introduction 23.1
Insurance policies, Master’s release 23.6.1(f)
Insurance policies where surviving spouse 23.3(n)
Interest on estate duty 23.5(d)
L
Legislation 23.1.2
Liability for payment of duty 23.5(b)
Liability of executor for duty 23.6.1(c)
Limited interests, valuation 23.4(b)
M
Master’s release 23.6.1
See also appropriate titles
Master’s requirements, expenditure thereon as a deduction 23.3(d)
N
National hero, estate not dutiable 23.2.5
Non-residents, debts due to 23.3(f)
O
Objection to assessment 23.6.2
Offences, regulations, forms and savings 23.8
Ownership of property subject to limited interest, valuation 23.4(f)
P
Pension and benefit funds 23.3(m)
Property deemed to be property 23.2.3(c)
See also Deemed property
Property of non-resident 23.2.4
Property not situate in Zimbabwe 23.3(e)
Property, when acquired 23.2.2(b)
Before 1 January 1967 23.2.3(a)
After 1 January 1967 23.2.3(b)
Property, valuation of 23.4 - 23.4(h)
R
Rates of estate duty 23.9
Release of property by Master 23.6.1(b)
Resident, property of non-resident 23.2.4
Returns, forms prescribed 23.5(a)
S
Sale of property, value 23.4(a)
Shares, transfer of 23.6.1(d)
Share valuations, deemed property included in 23.3(l)
South African assets, exemption 23.7.2
Surviving spouse, insurance policy deduction 23.3(n)
T
Transfer of immovable property, Master’s release 23.6.1(e)
Transfer of shares, Master’s consent 23.6.1(d)
Trust, value of interest in 23.4(b) Note 3
U
Unquoted shares, valuation 23.4(g)
Use or occupation of property, donation of 23.3(j)
Usufructs donated to deceased 23.3(g)
Usufruct, valuation 23.4(b)
V
Valuation of property 23.4
See also appropriate titles

221
Valuers, panel of 23.4(h)
Values, adjustment by Master 23.5(b)
W
What constitutes an estate 23.2
Administration purposes 23.2.1
Estate duty purposes 23.2.2
Principles to be applied 23.2.2
Addendums

• The Finance (No. 3) Act, 1996 •


• The Finance Act, 1997 and The Finance (No. 2) Act, 1997 •
• The Finance Act, 1998 •
• The Finance Act, 1999 and The Finance (No. 4) Act, 1997 •

THE FINANCE (NO. 3) ACT, 1996


1. Introduction
The footnote to the preface refers. The Act was promulgated on 1 November 1996.
The Income Tax Act, Capital Gains Tax Act and principal Finance Act are amended as indicated below.
References to 1 April 1996 are to the year of assessment which commenced on that date.
Cross-references in the following text are to the main paragraphs, etc., of the book.
The contents of this addendum have not been included in the index.
2. Income Tax
2.1 Rates of income tax and personal credits (see chapter 10)
(a) General rates
There are no changes. Further, the drought surcharge of 5% of income tax payable has effectively
been continued, it appears indefinitely, through the medium of a 5% development levy.
For individuals, however, there is a slight increase in the “zero rate band” resulting in the
following, though it is unclear, from conflicting provisions, whether this is to be first effective for the year
ending 31 March 1997 or 1998:

Cumu-
Taxable Amount Rate Amount lative
Income Bands Within Band of Tax of Tax Total
Tax
$ $ % $ $
1 to 9 360 9 360 0 0 0
9 361 to 15 000 5 640 20 1 128 1 128
15 001 to 30 000 15 000 25 3 750 4 878
30 001 to 45 000 15 000 30 4 500 9 378
45 001 to 60 000 15 000 35 5 250 14 628
Over 60 000 40

(b) New projects in growth point areas (see 9.7.4)


Special rates of tax, for the first five years of operations, are extended to:
(i) persons other than companies: 10%, in relation to the commencement of manufacturing
operations, with effect from 1 April 1996;
(ii) persons who commence operations for the provision of roads, bridges, sanitation or water
reticulation: 15% backdated to 1 April 1991.
(c) Personal credits (see chapter 10)
The credit of $360 for a family taxpayer is to be abolished with first effect for the year ending 31
March 1998.
2.2 Individuals’ benefits, exemptions and business losses
(a) Motor vehicle benefit (see 5.6.3)
With effect from 1 April 1997 the target figures are removed from the legislation.
(b) Interest exemption (see 6.3(xiv))

222
With effect from 1 April 1996 the exemption of the “first $1 000” is repealed.
(c) Business losses (see 8.27)
With effect from 1 April 1996 any such assessed loss is “ring fenced” in that it may not be set off
against income accruing under a contract of employment.
2.3 Pension funds: contributions to and amounts from
(a) The ceilings on deductibility of annual contributions are amended with effect from 1 April 1996 as
follows (see 16.4):
(i) in relation to employers: to $15 000 for each member;
(ii) in relation to each member of a pension fund or funds: to $10 000;
(iii) in relation to each contributor to a retirement annuity fund or funds: to $12 000;
(iv) aggregate in relation to an individual contributing as in (ii) and (iii): the limit as in (ii).
(b) Member’s receipt on his withdrawal from, or the winding up of, a fund (see 16.7.2)
With effect from 1 April 1996, where a member’s contributions have been wholly or partially
disallowed at the time they were paid, any return of such disallowed amount is not subject to tax.
2.4 Other expenditure: donations to charitable trusts (see 8.13)
With effect from 1 April 1996 donations to trusts administered by either the Minister responsible for social
welfare or the Minister responsible for health are deductible.
2.5 Capital allowances (see 9.2.3)
With effect from 1 April 1996 the definition of “industrial building” is expanded to include any toll-road or
toll-bridge declared in terms of the Toll-roads Act [Chapter 13:13].
2.6 Venture capital companies (see 6.2(d))
With effect from 1 April 1996 exemption applies to all such companies as are approved by the Minister by
statutory instrument. (This means, as the law stands, that dividends payable by such companies are not
exempt in the hands of shareholders.)
3. Capital gains tax
3.1 Deductible annual allowance (see 22.2)
With effect from 1 April 1996 this is increased to 15%. The increased deduction applies from the dates of
acquisition, construction, etc., of, or additions to, any specified asset now sold.
3.2 Ceilings on taxable gains and assessed capital losses
With effect from 1 April 1996 such ceilings on gains (see 22.2) and losses (see 22.7.1) are increased to $5
000.
4. Residents’ tax on interest (see 10.9 and 19.8)
With effect from 1 April 1996 this effectively becomes a “final” tax, in that is levied only at the
withholding stage and is not subject to further assessment (against which the tax withheld has in the past
been set off).
Small amounts of interest on deposits with financial institutions are brought into the withholding
requirements.
5. Automated financial transactions tax
With effect from 1 September 1996 financial institutions have been required to withhold and pay such tax
to the Commissioner. The rate is 15c on each stipulated transaction, namely withdrawals from and debits to
any customer’s account by means of an automated teller machine.
This is not a tax on income though the Income Tax Act is being used as the vehicle for its imposition.
THE FINANCE ACT, 1997
and
THE FINANCE (No. 2) ACT, 1997
1. Introduction
1.1
Cross-references in the following text are to the main paragraphs, etc., of the book. It will be necessary for
readers to refer also to the footnote to the preface to the book, and to the Addendum.
1.2
The Finance Act, 1997, to which this addendum almost entirely relates, was promulgated on 28 November
1997. References below:
(i) to 1 April 1997 are to the nine-month “year” of assessment (explained below)
commencing on that date and ending on 31 December 1997; and
(ii) to 1 January 1998 and 1 January 1999 are to years of assessment commencing on those
dates.

223
1.3
The Finance (No. 2) Act, 1997 was promulgated on 20 February 1998; only paragraph 2.11, below, relates
to it.
2. Income Tax
2.1 General
(a) “Year” of assessment (see 1.1, 18.1)
As indicated in the introduction, the tax year end has been changed from 31 March to 31
December, with the nine-month period commencing 1 April 1997 being deemed to be a year.
(b) Prescription periods
(i) Assessments (see 21.2.6)
The period within which the Commissioner may issue an additional assessment has been
extended from three to six years save in cases of fraud, etc., where he is no longer bound by a time limit
(see 21.2.6(a)).
Similarly the period within which a taxpayer may claim that an assessment be reduced (or
that an assessment of loss be increased) has been extended from three to six years (see 21.2.6(b)).
(ii) Withholding taxes (see chapter 19)
The period within which refunds of overpayments of the various taxes may be claimed is
extended from three to six years.
2.2 Rates of tax and personal credits (see chapter 10 and Addendum paragraph 2.1)
(a) Rates of tax : individuals
(i) Employment : sliding scale rates
The rates enacted result in the following.

Cumu-
Taxable Amount Rate Amount lative
Income Bands Within Band of Tax of Tax Total
Tax
$ $ % $ $
1 April 1996
1 to 9 000 9 000 0 0 0
9 001 to 15 000 6 000 20 1 200 1 200
15 001 to 30 000 15 000 25 3 750 4 950
30 001 to 45 000 15 000 30 4 500 9 450
45 001 to 60 000 15 000 35 5 250 14 700
60 001 and over 40
1 April 1997 and 1 January 1998
1 to 9 360 9 360 0 0 0
9 361 to 15 000 5 640 20 1 128 1 128
15 001 to 30 000 15 000 25 3 750 4 878
30 001 to 45 000 15 000 30 4 500 9 378
45 001 to 60 000 15 000 35 5 250 14 628
60 001 and over 40
1 January 1999
1 to 12 000 12 000 0 0 0
12 001 to 20 000 8 000 20 1 600 1 600
20 001 to 35 000 15 000 25 3 750 5 350
35 001 to 55 000 20 000 30 6 000 11 350
55 001 to 80 000 25 000 35 8 750 20 100
80 001 and over 40

The 5% development levy remains unchanged.


(ii) Trade or investment
Initially the above sliding scale rates continue to apply.
With effect from 1 January 1999, however, an individual’s taxable income from trade or
investment is taxable at a flat rate of 35%, i.e. without the application of the sliding scale. Such flat rate is
similarly subject to the 5% development levy.

224
Despite this enactment in relation to taxable income from trade or investment, specified
rates (i) continue to apply in relation to export processing zones (see chapter 18) and (ii) will apply to (see
below) “BOOT” arrangements and Treasury bills.
(b) Rate of tax: companies (see chapter 10 and Addendum)
The company (and trust) rate remains at 37,5% from 1 April 1996, 1 April 1997 and 1 January
1998, but is reduced to 35% from 1 January 1999.
The 5% development levy remains unchanged.
Specified rates (i) continue to apply to export processing zones (see chapter 18) and (ii) will apply
to (see below) “BOOT” arrangements and Treasury bills.
(c) Personal credits : individuals (see 10.3.2 and Addendum paragraph 2.1(c) )
With effect from 1 April 1997, and 1 January 1998 and 1999, the only remaining personal credits
are as follows.

1997 1998 1999


Elderly person $200 $200 $200
Blind person $500 $500 $ 1 500
Medical
Invalid appliances $1 for every $5 of cost
Expenses on treatment, etc.,
in excess of $250 $1 for every $5 of excess
Medical aid society $1 for every $5 of contributions
Disabled persons
Disabled taxpayer $500 $500 $1 500
Disabled child $500 $500 $1 500

2.3 Individuals’ benefits : motor vehicles (see 5.6.3 and Addendum paragraph 2.2(a))
The application of the cost basis for the calculation of the benefit (or, in terms of the
Commissioner of Taxes’ practice note No. 42, the applicable AA rate) is staggered as follows:
1 April 1997: 50% of the benefit
1 January 1998: 75% of the benefit
Subsequent years 100% of the benefit
2.4 Capital allowances : general (including farmers’) staff housing (see 9.2.7)
The cost limitation in respect of expenditure incurred on or after 1 January 1999 on each
residential unit is increased to $125 000 but the limit of the “first $50 000” ranking for allowances has not
so far been changed.
2.5 Capital allowances: farmers: schools, hospitals, etc. (see 9.2)
In relation to each school or hospital, etc., the allowance limitation in respect of aggregate
expenditure is with effect from 1 January 1999 increased to $750 000 (see 9.2.2(b)(i)).
In relation to each building constituting housing for staff employed at the school, etc., the
allowance limitation in respect of expenditure is with effect from 1 January 1999 increased to $70 000 (see
9.2.2(b)(ii)).
2.6 Capital redemption allowance: miners: schools, hospitals, etc. (see 12.2)
In relation to each school, hospital, etc., the allowance limitation in respect of aggregate
expenditure is with effect from 1 January 1999 increased to $750 000 (see 12.2(c)).
In relation to each building constituting housing for staff employed at the school, etc., the
allowance limitation in respect of expenditure is with effect from 1 January 1999 increased to $70 000 (see
12.2(d)).
2.7 Wear and tear allowance for the (nine-month) “year” of assessment commencing 1 April 1997
(a) “Accelerated” wear and tear allowance (see 9.3.2(a))
The 25% allowance is not apportionable.
(b) “Long-term” wear and tear allowance (see 9.3.2(c) and (f))
It is understood that the Commissioner accepts such non-apportionment under the
concept of the “year” of assessment.
2.8 Build, own, operate and transfer (“BOOT”) arrangements
These are likely to affect only a very limited number of contractors who enter into special
agreements with Government or statutory corporations for the construction of items of infrastructure.

225
Attributable taxable income is stipulated as to be taxed as follows with effect from 1 January
1999:
First five years 0%
Second five years 15%
Thereafter at normal rates
The 5% development levy does not apply
2.9 Treasury bills
With effect from 1 January 1998 residents’ tax on interest (see 10.9 and 19.8) applies to income
(deemed to be “interest”) from Treasury bills in the hands of persons, other than financial institutions, who
purchase them from financial institutions (including purchases from the Reserve Bank). The rate of tax,
apparently being applied administratively, is 15%. Such income is then exempt from income tax (see
Addendum paragraph 4).
No withholding tax applies to financial institutions’ own income from Treasury bills; income tax
at 15% arises in their hands at the assessment stage.
It is understood (i) to be the Commissioner’s approach that no applicable expenditure is
deductible, presumably on the grounds that the Act imposes liability in this case on “income” as opposed to
“taxable income” and (ii) that the subject may be under some form of review.
2.10 Pension funds : removal of exemption (see 6.2)
With effect from 1 January 1998 pension funds became liable to income tax at the rate of 15% of
their taxable income. No special rules regarding the determination of the latter have so far been enacted.
2.11 War veterans’ pensions and gratuities
With effect from 1 December 1997 amounts receivable in terms of the War Veterans Act, and
regulations thereunder, are exempt.
3. Informal traders tax
With effect from 1 January 1998 an obligation, subject to penalties, is placed on any landlord letting
property to an individual whose annual turnover from trade on his own account is less than $100 000, to
recover an amount of 10% of the rent receivable (i.e. in addition to the rent) and to pay this amount to the
Commissioner within 30 days of recovery. Such obligation arises, however, only in the circumstances laid
down in the new 26th Schedule to the Income Tax Act.
In the trader’s hands the amount withheld ranks as a credit against his income tax payable on assessment in
due course.
4. Capital Gains Tax
4.1 General
(a) “Year of assessment” (see 22.2)
As in the case of income tax, referred to at 2.1(a) above, the tax year end has been changed from
31 March to 31 December, with the nine-month period commencing 1 April 1997 being deemed to be a
year.
(b) Prescription periods (see 22.21.1)
As in the case of income tax, referred to at 2.1(b)(i) above, the periods within which the issue of
additional and reduced assessments is permitted have been increased.
4.2 Rates of tax (see 4.1 and 22.1)
The rates remain unchanged at 20% and 10% respectively.
5. Estate duty
There are no changes to the law.
THE FINANCE ACT, 1998
1. Introduction
1.1
At the time when the Fourth Edition of the book went to print, the Finance (No. 3) Act, 1996, had not been
promulgated. A commentary on its contents was included as an addendum which was bound into the book.
Early last year a second addendum was published as a separate document covering the Finance Act, 1997,
and the Finance (No. 2) Act, 1997.
The present addendum covers the Finance Act, 1998, which was promulgated on 31 December 1998.
Although most of the Act’s provisions introduce new law, some amend, with effect from 1 January 1999,
provisions which had been introduced by the Finance Act, 1997, and which thus never came into force.
The latter aspects are also noted in the text below.
1.2

226
Cross-references below are to the paragraphs, etc., of the book.
1.3
References to 1 January and 1 April are to years of assessment which began on those dates.
2. Income Tax
2.1 General
(a) Rates of tax: individuals
(i) Employment
Sliding scale rates
The rates enacted result in the following with effect from 1 January, 1999.

Taxable Amount Rate Amount Cumu


lative
Income Bands Within Band of Tax of Tax Total Tax
$ $ % $ $
1 April 1996
1 to 24 000 24 000 0 0 0
24 001 to 48 000 24 000 20 4 800 4 800
48 001 to 72 000 24 000 25 6 000 10 800
72 001 to 96 000 24 000 30 7 200 18 000
96 001 to 150 000 24 000 35 18 900 36 900
150 001 to 720 000 570 000 40 228 000 264 900
720 001 and over 50*

*This is the effective rate, being 40% plus a 25% surcharge.


The 5% development levy is discontinued.
The “$12 000 to $80 000” table enacted in the Finance Act, 1997, this never came into
effect.
(ii) Trade or investment
With effect from 1 January 1999 an individual’s taxable income, or portion of his taxable
income, from trade or investment, is subject to tax at a flat rate of 35%. This is thus the effective rate on the
whole of such income since neither the sliding scale nor the 25% surcharge apply.
(iii) Example of the above, showing a calculation of the amount payable for the year ending
31 December 1999.

Salary} from own 270 100


Bonus} company 18 000
Total from employment 288 100
Director’s fees from other company15 000
Interest from own company 3 500
Interest from bank (after deducting withholding tax) 840
Dividend from own company 600
Total $308 040
Doctor’s fees paid $2 400
Calculation of taxable income at normal rates:
Total from employment 288 100
Less exempt portion of bonus 2 000
$286 100
At flat rate:
Director’s fees 15 000
Interest 3 500
$18 500
(Bank interest and company dividend exempt)
Calculation of tax payable:
Tax on $286 100 at normal rates 91 340,00
Tax on $18 500 at 35% 6 475,00

227
97 815,00
Less: Credit for medical expenses:
50% of $2 400 1 200,00
Tax payable $96 615,00
(Less credit for PAYE withheld).

(iv) Effective rates of income tax which continue to apply to other operations area:

Export processing zones (see chapter 18)


Year of commencement and the following four years Exempt
Thereafter 15%
“BOOT” arrangements
First five years 0%
Second five years 15%
Thereafter 35%
Treasury bills
Financial institutions and short-term insurers15%
Other recipients (but liable to 15% withholding tax:
see also 2.12 below) Exempt

(v) A new category of taxpayer is recognised, namely an industrial park developer. The rates
of tax with effect from 1 January 1999 are the same as those for operations in export processing zones as
above.
(b) Rates of tax: companies (see chapter 10)
(i) The rate remains at 35% with effect from 1 January 1999, the 5% development levy
being discontinued.
The 25% surcharge on the portion of taxable income above $720 000 does not apply.
(ii) Effective rates of income tax which continue to apply to export processing zone
operations, “BOOT” arrangements and Treasury bills, and those which apply to industrial park developers,
are as above for individuals.
(iii) With effect from 1 January 1999 export incentive rates have been introduced for
companies conducting manufacturing operations.
Any such company which is incorporated on or after 1 January 1999 and which, in any of
its five years of assessment, exports 40% or more of its manufacturing output (calculated by quantity or
volume) is taxable at an effective 25% (being 10 percentage points less than the standard 35%) on its
taxable income from manufacturing for that year.
Any company, regardless of its date of incorporation, which in any year of assessment,
exports 50% or more of its manufacturing output, is taxable at an effective 27% (being 8 percentage points
less than the standard rate) on such taxable income for that year.
Obviously these effective rates will change whenever the future standard rate changes.
(c) Rates of tax: trusts (see 10.14)
(i) The rate remains at 35% with effect from 1 January 1999, the 5% development levy
being discontinued.
The 25% surcharge on the portion of taxable income above $720 000 does not apply.
(ii) Effective rates of income tax which continue to apply to export processing zone
operations, “BOOT” arrangements and Treasury bills, and those which apply to industrial park developers,
are as above for individuals.
(d) Annual payment dates (see 21.5.2)
“APDs” applicable to all taxpayers (other than regarding “PAYE-type” income of individuals)
which had previously been announced administratively, have now been enacted, namely:
28 February
30 June
30 November.
The effective date is stated to be 1 January 1998 but this must be questionable. (Bearing in mind
the date of enactment of the new legislation it should in practical terms apply to the year of assessment
which began on 1 January 1998).

228
These dates thus now apply automatically and the Commissioner is no longer required to issue
written notices. He may, however, by any such individual notice, vary them. Further, the Minister is
empowered to prescribe, by notice in the Government Gazette, lower limits of taxable income to which
APDs will not apply. By the date this addendum went to print, no notice had been gazetted.
2.2 Cash in lieu of leave (see 5.2(b))
The election is withdrawn with effect from 1 January 1999.
2.3 Motor vehicle benefits (see 5.6.3)
With effect from 1 January 1999 the cost basis for the calculation of the benefit (or, in terms of the
Commissioner’s practice note No. 42, the AA rate) is withdrawn. The application of 100% of the benefit
under such basis thus never arose. The calculation has been replaced by fixed deemed amounts relating to
the engine-size of the vehicle. The impact 1 April 1997 onwards is as follows:

1 April 1997 (the nine-month “year”on the change of the tax year-end from March to December)

50% of the benefit


1 January 1998: 75% of the benefit
1 January 1999and subsequent years
Up to 1500 cc $25 000
1501 cc to 2000 cc $40 000
2001 cc to 3000 cc $70 000
Over 3000 cc $100 000

These are fixed amounts, as indicated above, but are subject to reduction if the period of use is
shorter than the year of assessment.
2.4 Capital allowances: general (including farmers’) staff housing (see 9.2.7)
The cost limitation in respect of expenditure incurred on or after 1 January 1999 on each
residential unit is increased as is the amount ranking for allowances.
The resulting limits, depending on date of commencement of construction of the building, are:

Years commencing 1 April 1995 to 1 January 1998 $100 000*


Years commencing 1 January 1999 and subsequent years $200 000**
(The $125 000 cost limitation enacted in the Finance Act, 1997, thus never came into effect).
*In which event the first $50 000 ranks for allowances.
**In which event the first $100 000 ranks for allowances.

2.5 Capital allowances: farmers: schools, hospitals, etc. (see 9.2)


In relation to each school or hospital, etc. the allowance limitation in respect of aggregate
expenditure (see 9.2.2(b)(i)) is increased as follows, any excess expenditure being disregarded:

Years commencing 1 April 1995 to 1 January 1998: $500 000


Years commencing 1 January 1999 and subsequent years $1 500 000
(The $75 000 cost limitation enacted in the Finance Act, 1997, thus never came into effect).

In relation to each building constituting housing for staff employed at the school, etc., the
allowance limitation in respect of expenditure (see 9.2.2(b)(ii)) is increased as follows, any excess
expenditure being disregarded:

Years commencing 1 April 1995 to 1 January 1998: $50 000


Years commencing 1 January 1999 and subsequent years $100 000
(The $70 000 cost limitation enacted in the Finance Act, 1997, thus never came into effect).

2.6 Capital redemption allowance: miners: schools, hospitals, etc. (see 12.2)


In relation to each school hospital, etc., the allowance limitation in respect of aggregate
expenditure (see 12.2(c)) is increased as follows, any excess expenditure being disregarded:

Years commencing 1 April 1995 to 1 January 1998 $500 000

229
Years commencing 1 January 1999 and subsequent years $1 500 000
(The $750 000 cost limitation enacted in the Finance Act, 1997, thus never came into effect).

In relation to each building constituting housing for staff employed at the school, etc., the
allowance limitation in respect of expenditure (see 12.2(d)) is increased as follows, any excess expenditure
being disregarded:

Years commencing 1 April 1995 to 1 January 1998 $50 000


Years commencing 1 January 1999 and subsequent years $100 000
(The $750 000 cost limitation enacted in the Finance Act, 1997, thus never came into effect).

2.7 Pension fund contributions (see 16.4)


(a) Employers (see 16.4.3)
The annual limitation on deductibility in respect of contributions for each employee
remains, with effect from 1 April 1996, at $15 000.
(b) Employees: pension and retirement annuity funds (see 16.4.5)
The annual limitations on deductibility are increased as follows:

1 April 1996 to 1 January 1998


Pension funds only (see 16.4.5(a)) $10 000
Retirement annuity funds only (see 16.4.5(b)$12 000
Both such funds (see 16.4.5(c)) $10 000
1 January 1999
All cases $15 000

2.8 Bonus exemption (see 6.3(ii))


The annual exemption limit is increased to $2 000 with effect from 1 January 1998, subject to the
alternative test of 10% of other remuneration.
2.9 Passenger motor vehicles
(a) Capital allowances (see 9.2.5 and 12.2)
The allowance limitation is increased with effect from 1 January 1999 to $200 000.
(b) Rentals (see 7.7(k))
The limitation on deductibility is increased with effect from 1 January 1999 to $200 000.
2.10 Livestock epidemic relief (see 11.6 re drought relief)
With effect from 1 January 1999 the provisions relating to (i) deferment of liability on forced
sales, and special rates of tax thereon, and (ii) additional deductions upon restocking, are extended to
periods during which an “epidemic disease” occurs in an “epidemic area”.
The definition of “livestock”, which previously stipulated only cattle, sheep and goats, is
expanded to include “pigs”, crocodiles, ostriches, fowls and any other animals or birds that are raised or
possessed by a farmer as livestock in the course of his farming operations”.
2.11 Deductions: employment creation incentive (see 8.26 for comparable deduction re export market
development expenditure)
With effect from 1 January 1999, where in any year of assessment manufacturers either engage
fifty additional employees or increase their number of employees by 25% they are entitled to a deduction of
twice the additional employees’ remuneration. Such deduction may not, however, result in an employer
having an assessed loss.
Various conditions regarding additional employees have to be satisfied for their number to qualify
for the purposes of the employer’s claim. Such persons must constitute permanent staff. “Managerial”
employees (as defined) do not qualify; nor do persons previously employed by the taxpayer or by, in the
case of a company, another company in the same group.
2.12 Treasury bills
The rate of resident’s tax on interest (see 10.9 and 19.8) which was imposed on income from such
bills with effect from 1 January 1998 has now been stipulated at 15%. Such withholding tax applies to most
recipients of such income though not to, for example, financial institutions and insurers. Since the income
subject to withholding tax is exempt from income tax no deduction for applicable expenditure, such as
interest on funds borrowed to invest in bills, is allowable (see 7.7(f)).

230
Financial institutions and insurers do not suffer the above withholding tax on income from
Treasury bills which they themselves hold. Such income is liable to income tax in the hands of financial
institutions and short-term insurers at 15%.
2.13 Pension funds: removal of exemption (see 6.2)
Although, with effect from 1 January 1998, pension funds became liable to income tax at the rate
of 15% of their taxable income, special rules regarding the determination of the latter have not been
enacted and the Commissioner of Taxes has not so far enforced any liability.
3. Branch Profits Tax (see 10.13)
This is discontinued with effect from 1 January 1999.
4. Capital Gains Withholding Tax
(a) With effect from 1 January 1999 such tax is to be withheld, and paid to the Commissioner, at the
rate of:
• 10% of the sale price in the case of immovable property;
• 5% of the sale price in the case of a marketable security listed on the Zimbabwe Stock Exchange;
and
• 10% of the sale price in the case of any other marketable security.
Persons, known as “depositaries”, responsible for such withholding and payment, include in
particular:
• in the case of immovable property: conveyancers, legal practitioners and estate agents;
• in the case of marketable securities: stockbrokers and financial institutions.
Such payment is due by the end of the month following that in which payment of, commonly, the
proceeds of sale is made by the above parties to the seller.
There are provisions also whereby:
(i) liability to withhold falls on any agent receiving an amount from which a depositary has
failed to withhold;
(ii) liability falls on the seller where neither the depositary nor an agent has withheld;
(iii) personal liability falls on the depositary or agent;
(iv) liability for a 15% penalty arises.
(b) The withholding tax is not a final tax on the seller. Once the seller’s actual liability is assessed in
accordance with the general provisions of the Capital Gains Tax Act (i.e. using all the “normal” rules and
rates: see chapter 22) the capital gains withholding tax already paid is allowed as a credit; any excess is
refundable and any shortfall payable.
(c) At the time when this addendum went to print the Commissioner had suspended the operation of
the above provisions regarding locally-listed securities but his approach to the other affected assets was
unknown.
5. Estate duty
There are no changes to the law.
THE FINANCE ACT, 1999
and
THE FINANCE (No. 4) ACT, 1997
1. Introduction
1.1
At the time when the Fourth Edition of the book went to print, the Finance (No. 3) Act 1996 had not been
promulgated. A commentary on its contents was included as an addendum which was bound into the book.
A second addendum, covering the Finance Act, 1997, and the Finance (No. 2) Act, 1997, was published as
a separate document early in 1998 and a third addendum, covering the Finance Act, 1998, was published,
also as a separate document, early in 1999.
1.2
Cross-references below are to the paragraphs, etc., of the book.
1.3
In the context of income tax and capital gains tax references to 1 January are to years of assessment which
began on that date.
2. Income Tax
2.1 Rates of tax and personal credits (see chapter 10)
(a) Rates of tax: individuals
(i) Taxable income from employment

231
Sliding scale rates
The rates enacted result in the following with effect from 1 January 2000.

Taxable Amount Rate Amount Cumu-


lative
Income Bands Within Band of Tax of Tax Total
Tax
$ $ % $ $
1 to 30 000 30 000 0 0 0
30 001 to 60 000 30 000 20,60 6 180 6 180
60 001 to 90 000 30 000 25, 75 7 725 13 905
90 001 to 120 000 30 000 30,90 9 270 23 175
120 001 to 150 000 30 000 36,05 10 815 33 990
150 001 to 720 000 570 000 41,20 234 840 268 830
720 001 and over 51,20

The rates of tax shown are the effective rates, including (i) an AIDS levy of 3%, of the
basic tax, which was introduced with effect from 1 January 2000 and (ii) a surcharge of 25% of the basic
tax applicable to the portion of the individual’s annual taxable income in excess of $720 000. The AIDS
levy does not apply to the amount of the surcharge. Thus, for example:
• the rate of basic tax on annual taxable income in the $30 001 to $60 000 band is 20%,
plus 3% of 20%, giving 20,6% as above;
• the tax payable on annual taxable income of, say, $750 000 is:

On $720 000, as above i.e. basic tax plus surcharge plus AIDS levy 268 830
On the further $30 000, at 40%, plus surcharge and AIDS levy on 40% but not subject to AIDS
levy on the surcharge 15 360
$284 190

(ii) Taxable income from trade or investment


An individual’s taxable income, or portion of his taxable income, from trade or
investment, continues to be subject to tax at a flat rate of 35%. With effect from 1 January 2000 an AIDS
levy of 3% is to be added to the tax payable on such income. Thus the effective rate on the whole of such
income is 36,05% since neither the sliding scale nor the 25% surcharge apply.
(iii) Example of the above, showing a calculation of the amount payable for the year ending
31 December 2000.

Salary }from own 271 100


Bonus }company 17 000
288 100
Director’s fees from other company15 000
Interest from own company 3 500
Interest from bank (after deducting withholding tax) 840
Dividend from own company (after withholding tax) 600
Total $308 040
Doctor’s fees paid $2 400
Calculation of taxable income:
At normalrates:
Salary 271 100
Bonus 17 000
Less exempt 3 000 14 000
At normal rates: $285 100
At flat rate:
Director’s fees 15 000
Interest 3 500

232
$18 500
(Bank interest and company dividend exempt)
Calculation of tax payable:
Tax on $285 100 at normal rates 87 040,00
Tax on $18 500 at 35% 6 475,00
93 515,00
Less: Credit for medical expenses:
50% of $2 400 1 200,00
92 315,00
Add: AIDS levy 2 769,45
Tax payable $95 084,45
(Less credit for PAYE withheld)

(iv) Effective rates of income tax which continue to apply to other operations are:

Export processing zones (see chapter 18)


Year of commencement and the following four years Exempt
Thereafter 15%
Industrial park developers
As for operations in export processing zones as above.
“BOOT” arrangements
First five years 0%
Second five years 15%
Thereafter 35%

(v) As regards tax rates on Treasury bills see 5 below.


(b) Rates of tax: companies (see chapter 10)
(i) The basic rate remains at 35% but, with effect from 1 January 1999, an AIDS levy of 3%
of such tax is introduced resulting in an effective rate of 36,05%. The 25% surcharge on the portion of
taxable income above $720 000 does not apply.
(ii) Effective rates of income tax, which continue to apply to export processing zones,
industrial park developers and “BOOT” arrangements are as above for individuals.
(iii) As regards Treasury bills see 5 below.
(iv) An export incentive system, which was introduced with effect from 1 January 1999, has
been discontinued. Under it, the application of favourable tax rates depended on the percentage of a
manufacturer’s output (calculated by quantity or volume) as was exported.
With effect from 1 January 2000 it is replaced by a new system which recognises an
increase in exports. The taxable income derived from an increase in value of a company’s exports of
manufactured output will be taxed at ten percentage points below the corporate rate of tax, provided that
such increase is 20% or more as compared with the immediately-preceding year of assessment. The
increase is to be calculated in US dollars or some other (approved) convertible currency.
(c) Rates of tax: trusts (see chapter 10)
(i) The basic rate remains unchanged but with effect from 1 January 1999 an AIDS levy of
3% of such tax is introduced resulting in an effective rate of 36,05%.
The 25% surcharge on the portion of taxable income above $720 000 does not apply.
(ii) Effective rates of income tax, which continue to apply to export processing zones,
industrial park developers and “BOOT” arrangements are as above for individuals.
(iii) As regards tax rates on Treasury bills see 5 below.
(d) Personal credits: individuals (see 10.3.2)
The only remaining personal credits are as follows:

1 January
1999 2000
Blind persons credit $1 500 $3 000
Disabled persons credit $1 500 $3 000
Disabled child credit $1 500 $3 000

233
Medical expenses credit
(1) Invalid appliances 50% 50%
(1) Expenses on treatment etc., 50% 50%
Medical aid society contributions 50% 50%
Elderly persons credit
(2) Persons aged 59 or more $2 000 $3 000
(1) Credit not available if the taxpayer is ordinarily resident outside Zimbabwe throughout
the year.
(2) Reduced proportionately if the period of assessment is less than a year.

2.2 Exemptions: individuals


(a) Bonuses (see 6.3(ii))
With effect from 1 November 1999 the annual exemption limit is increased to $3 000, subject to
the alternative test of 10% of other “remuneration”.
(b) Retrenchment (see 6.3(iii))
With effect from 1 January 2000 “the first $10 000” is increased to “the first $20 000”.
(c) Pensions (see 6.3(v))
With effect from 1 January 2000 the annual exemption in respect of a pension accruing to a person
aged 59 or more is increased to “the first $12 000”.
(d) Pension commutation (see 16.6.1(b)(ii) and 16.7.1)
With effect from 1 January 2000 the non-taxable portion of a pension commutation is increased to
the greater of one-third of the amount due or $20 000.
2.3 Deductible expenditure: donations (see 7.4(j))
With effect from 1 January 2000 donations made to the State or any fund for the purpose of the following,
approved by the Minister of Health, are deductible: the purchase of medical equipment or drugs, or for the
construction, extension or maintenance of a hospital.
(Deductibility is limited to $10 000 000).
2.4 Pension funds: exemption (see 6.2(b)(ii))
The exemption of pension funds, which had been withdrawn with effect from 1 January 1998, has been
restored until such time as the Minister publishes a notice in the Government Gazette terminating the
exemption.
3. PAYE (see 21.5.3)
(a) With effect from 1 January 2000 the maximum penalty which may be imposed for late or
underpaid PAYE is increased from 15% to 100%.
(b) With effect from 1 January 2000 a “final deduction system” is being introduced with a view to
rendering unnecessary the submission of annual returns by a large proportion of taxable employees. The
system is to be operated by employers, selected by the Commissioner, in terms of directives issued by him.
4. Tobacco Levy (see 7.7(o), 11.10.3 and 19.9)
With effect from 1 January 2000 the levy of 5% of the price of auctioned tobacco, payable both by sellers
and by buyers, is reduced to 2,5% respectively.
5. Treasury Bills
Income from such bills is deemed to be interest and is subject to residents’ (withholding) tax on interest
(see 10.9 and 19.8).
With effect from 1 January 2000 the rate of withholding tax on such interest is increased from 15% to 30%.
This tax applies to most recipients though not to, for example, financial institutions and insurers. Since the
income subject to withholding tax is exempt from income tax no deduction for applicable expenditure, such
as interest on funds borrowed to invest in bills, is allowable (see 7.7(f)).
Financial institutions and insurers do not suffer the above withholding tax on Treasury bills which they
themselves hold. With effect from 1 January 2000 taxable income from Treasury bills in the hands of
financial institutions and short-term insurers is liable to income tax at 30%.
6. Demutualisation Levy
With effect from 1 January 1999 any issue of free shares under a scheme whereby, for example, a mutual
society is converted into an insurance company, is subject to a demutualisation levy of 2,5%. Although
liability is imposed on members the insurer is required to meet this on their behalf. In these circumstances
no further liability in this respect falls upon members.
(Liability on any sale of the shares will depend on the individual circumstances of each member).

234
7. Capital Gains Tax: Elections
With regard to “rollover” and other elections (see 22.10.1, 22.10.2, 22.10.3, 22.11, 22.15, 22.16 and 22.17)
these must, with effect from 1 January 2000, be made by the time the related return is submitted.
8. Capital Gains Withholding Tax
Paragraph 4(a) and (b) to the third addendum to the book refers. Suspension of the tax in relation to
marketable securities has been formalised through legislation.
Further provisions enable a seller (of immovable property, in view of the above suspension) to apply for a
clearance certificate under which liability to the tax may be eliminated or reduced.
9. Estate Duty
The maximum rate of duty remains unchanged at 20%.
In the case, however, of the estates of persons who die on or after 1 January 2000, duty applies to the total
net value of estates in excess of:
(i) $400 000 (previously $250 000) where there is a surviving spouse; or
(ii) $250 000 (previously $125 000) where there is no surviving spouse.

235

You might also like